Download as pdf or txt
Download as pdf or txt
You are on page 1of 207

CONTENTS

Quantitative Aptitude

1. Simplification, Approximation and Number Series . . . . . . . . . . . . . . . . . . . . . . . . . . . . . . . . . . 3

2. Ratio Proportion and Mixture & Alligation . . . . . . . . . . . . . . . . . . . . . . . . . . . . . . . . . . . . . . . . . . . 11

3. Partnership . . . . . . . . . . . . . . . . . . . . . . . . . . . . . . . . . . . . . . . . . . . . . . . . . . . . . . . . . . . . . . . . . . . . . . . . . 21

4. Average and Ages . . . . . . . . . . . . . . . . . . . . . . . . . . . . . . . . . . . . . . . . . . . . . . . . . . . . . . . . . . . . . . . . . . . . 28

5. Percentage and Profit & Loss . . . . . . . . . . . . . . . . . . . . . . . . . . . . . . . . . . . . . . . . . . . . . . . . . . . . . . . . 36

6. Simple Interest and Compound Interest . . . . . . . . . . . . . . . . . . . . . . . . . . . . . . . . . . . . . . . . . . . . . 47

7. Time & Work and Pipe & Cistern . . . . . . . . . . . . . . . . . . . . . . . . . . . . . . . . . . . . . . . . . . . . . . . . 57

8. Speed Time and Distance . . . . . . . . . . . . . . . . . . . . . . . . . . . . . . . . . . . . . . . . . . . . . . . . . . . . . . . . . 70

9. Boat and Stream . . . . . . . . . . . . . . . . . . . . . . . . . . . . . . . . . . . . . . . . . . . . . . . . . . . . . . . . . . . . . . . . . . . 81

10. Mensuration . . . . . . . . . . . . . . . . . . . . . . . . . . . . . . . . . . . . . . . . . . . . . . . . . . . . . . . . . . . . . . . . . . . . . . . 88

11. Permutation, Combination and Probability . . . . . . . . . . . . . . . . . . . . . . . . . . . . . . . . . . . . . . . . 97

12. Inequality . . . . . . . . . . . . . . . . . . . . . . . . . . . . . . . . . . . . . . . . . . . . . . . . . . . . . . . . . . . . . . . . . . . . . . . . . . . 108

13. Data Interpretation . . . . . . . . . . . . . . . . . . . . . . . . . . . . . . . . . . . . . . . . . . . . . . . . . . . . . . . . . . . . . . . . . . 127

14. Arithmetic Data Interpretation . . . . . . . . . . . . . . . . . . . . . . . . . . . . . . . . . . . . . . . . . . . . . . . . . . . . . . 150

15. Data Sufficiency . . . . . . . . . . . . . . . . . . . . . . . . . . . . . . . . . . . . . . . . . . . . . . . . . . . . . . . . . . . . . . . . . . . . . 174

16. Caselets . . . . . . . . . . . . . . . . . . . . . . . . . . . . . . . . . . . . . . . . . . . . . . . . . . . . . . . . . . . . . . . . . . . . . . . . . . . . . . . 191


Cracker Book for Bank (IBPS | SBI | RRB PO | Clerk) Mains Exams

Chapter
Simplification, Approximation
1 and Number Series

BEST APPROACH TO SOLVE THE QUESTIONS

Simplification: Simplification simply means simplifying the complex expressions. There are ways available always
which are simpler, easier and faster to calculate a particular complex expression. Questions related to simplification can
be solved in variety of ways. But in examination where one needs to calculate as fast as possible, applying good
simplification techniques may serve a good purpose. The most interesting about numbers is that they can be expressed
in an innumerable number of ways and thus comes the concept of breaking the expression in such a way that one
reaches the answer in less time.
Following are some examples citing the ways to simplifying an equation or expression.
9 11 17 12 16
1. 2
+ 3
+
6
=? + 5
+
10
+ 27× 33
27+22+17 24+16
⇒ 6
=? + 10
+ (30-3)(30+3)
66
⇒ 6
=? +4 + 900 − 9
⇒ ?=11+9-900-4= –884
2. ? × 65 ÷ 72 = 195 × 312 + 260 × 6
?×65
⇒ =65× 6(3× 52 + 4 × 1)
72
⇒ ?= 72× 6 × 160 = 432 × 160 = 69120
Approximation Questions based on approximation are usually easy to solve but turn out to be a bit difficult when
approximate values are not clearly visible. In case where a value like 17.97 is given, it is easy to replace it with 18. But
120 101
cases where values like or are given, then approximate values can be used by making appropriate adjustments to
13 11
other terms of the expression.
Let’s, for example, try to find out the approximate value of expression given below:
66 50
+ + 41% of 97 =?
16 17
66 50 66 50
The value of 16
is a bit more than 4 and that of 17 is a bit less than 3. So we may take value of 16
and 17
to be equal to 4
and 3 respectively.
We know 40% of 100 is equal to 40. As 41 near to and greater than 40 and 97 is near to and smaller than 100, we may
take the approximate value of 41% 𝑜𝑓 97 to be equal to 40 itself.
This way, the approximate value of the above expression= 4+3+40=47
A few sample problems:
178 217
− + 33 × 41 =? −32% 𝑜𝑓 207
12 18
⇒ 15 + 12 + 33(40+1) = ? – 33% of 200
⇒ 27 + 1355 = ? – 66 ⇒ ? = 1450
19.95 68.12
4
+ 17
− 12.5% 𝑜𝑓 127.9 =?
20 68 1
⇒ 4
+ 17 − 8 × 128 =?
⇒ 5 + 4 − 16 =? ⇒ ? = −7

3 Adda247 Publications For More Study Material


Visit: adda247.com
Cracker Book for Bank (IBPS | SBI | RRB PO | Clerk) Mains Exams

Number Series When different numbers are arranged in any mathematical and logical pattern, a number series is said
to be formed. There may be infinite such patterns. So it’s really difficult to find out one out of them. But there is one
element that gives us the idea of any pattern.
That element is the difference of the terms of a number series. By finding the difference of consecutive terms of a series
we can make out the pattern on which the number series is based. Some patterns are clearly visible but others can be
found out by cracking the pattern of the differences.
A few examples follow:
1. 45, 44, 48, 39, 55, 30, 66

2. 3, 9, 23, 97, 479, 2881, 20159

3. 10, 12, 15, 22, 38, 70, 127

Practice Exercise Based on new Pattern


1
Directions (1-15): What will come in place of the 6. 29% of 193 – 20% of 204.85 = (3375)3 − (? )2
question mark (?) in the following questions ?
(a) 1 (b) 4 (c) 16
1. (4 × 4)3 ÷ (512 ÷ 8)4 × (32 × 8)4 = (2 × 2)?+4
(a) 8 (b) 12 (c) 6 (d) 64 (e) None of these
(d) 14 (e) None of these 1 3 1 1 1
7. 3 3 ÷ 6 7 × 1 2 × 3 7 = (? )2
2
2. (2√392 − 21) + (√8 − 7) = (? )2 (a)
22
(b)
484
(c)
22
(a) 4 (b) –4 (c) 12 3 3 9
484
(d) 2 (e) 6 (d) 81
(e) None of these
1 1 1 1 8. 60% of 68.05 × 15% of 8 × 1.02 = ?
3. 1 4 + 1 6 − 1 8 =? +1 12
5 7 5 (a) 49.97592 (b) 49.73482 (c) 54.77442
(a) (b) (c)
24
7
24 12 (d) 41.84822 (e) None of these
(d) 12 (e) None of these
(112.6 ×114.4 )
3
9. 117.5 ÷115 ×√11
= (? )2
4. of 30% of 3420 = (? )2 ×2
19 (a)√11 (b) 11 (c) 121
(a) (81)2 (b) 7 (c) 9
(d) 1331 (e) 14641
(d) 81 (e) 49
1
5. 65% of √3136 × 5 =? +154 10. √3481 ÷ 7 × (441)2 = 12.5% of 1200 + ?
(a) 56 (b) 28 (c) 35 (a) 3 (b) 9 (c) 27
(d) 32 (e) None of these (d) 81 (e) None of these

4 Adda247 Publications For More Study Material


Visit: adda247.com
Cracker Book for Bank (IBPS | SBI | RRB PO | Clerk) Mains Exams
19 1 359.93
11. 3 27 % of 675 + 25 4 % of 184 = x 8. ?
= (8.89)3 – 14.5 × 39.89 + (1.95)2 – 34
(a) 70.64 (b) 71.64 (c) 71.46 (a) 8 (b) 2 (c) 3
(d) 70.46 (e) None of these (d) 9 (e) 5
2
12. [2.5 ÷ 0.1)3 ÷ (6.25)2 ] × [(1.25)2 ÷ (5)2 ] = (25)3+x 9. √429.87 + 520.23 + √120.97 = (? )2 + √35.98
(a) 1 (b) 2/3 (c) 1/3 (a) 2 (b) 3 (c) 8
(d) 2 (e) None of these (d) 5 (e) 9
1
13.
161
×
1595
÷
7
= (x)3 10. (23.89)² + √3598.97 – 171.93 + (?)² = (27.98)² + 4
377 253 13 (a) 18 (b) 24 (c) 28
(a) 343 (b) 216 (c) 64
(d) 22 (e) 26
(d) 125 (e) None of these
11. (13.97)² – 39.87% of 239.97 + (29.87)² + 330.97 =
2 4 7 2145 169
14. 3 of 5 of 11 of 1288 ÷ √529 = x (?)³
(a) 18 (b) 19 (c) 17
(a) 1 (b) 0.5 (c) 2
(d) 11 (e) 15
(d) 4 (e) None of these
?+134.5
√8649
3
√15625 2
12. + 209.87 + (69.87)² – 1999.83 = (54.87)² +
15. × 3 =x 24
√961 √1728 99.85
(a) 0.5 (b) 1.5 (c) 2.5 (a) 235.5 (b) 245.5 (c) 225.5
(d) 3.5 (e) None of these (d) 215.5 (e) 205.5

APPROXIMATION 13. ? × 12.97 + (24.97)² – (19.89)² + 363.83 = (27.93)²


(a) 10 (b) 20 (c) 18
Directions (1-20): What approximate value will come in (d) 12 (e) 15
place of (x or ?) in the following questions ?
593.89 1259.81
𝑥 14. ?
+ 14.87 × 35.88 + 17.93
= (25.89)2
1. 294.01 × − 19.99% 𝑜𝑓 119.99𝑥 = 254.9 ÷ 1.9
8.01 (a) 12 (b) 15 (c) 13
(a) 10 (b) 16 (c) 8 (d) 9 (e) 11
(d) 5 (e) 12
15. 48.01% of 249.99 + 𝑥% 499.99 = (11.01)2 + (2.11)2
410
2. 55.2 × + 499.9 – 127.9 =? % of 5600 (a) 2 (b) 3 (c) 5
69
(a) 17.5 (b) 12.5 (c) 10 (d) 1 (e) 6
(d) 15 (e) 8 2
16. 49.11 × ÷ 14.09 + √𝑥 = 1025 ÷ 257
7
3. 630 × ? + 599.85 – 55% of 12000 = 37 ½ % of 9200 (a) 9 (b) 8 (c) 27
(a) 12 (b) 18 (c) 15 (d) 16 (e) 25
(d) 20 (e) 8 3
17. × (15.99) + 31.9% 𝑜𝑓 3199 + 294.9 = 𝑥 3
4
11999.87 1
4. ?
+ 54.9% of 1800 – 389.9 = 11 9 % of 9900 (a) 10 (b) 11 (c) 12
(a) 24 (b) 28 (c) 20 (d) 15 (e) 8
(d) 18 (e) 32 18. ? % of 6519.97 + (21.97)2 – 29.98% 𝑜𝑓 9139.93 =
31 5 1
5. 8099.9 × 27 + 2699.8 × 3 – 1799.8 = 62 2% of ? (25.91)2
(a) 22500 (b) 15200 (c) 16200 (a) 25 (b) 15 (c) 30
(d) 18200 (e) 19200 (d) 45 (e) 40
577.93+?
6. 119.89% of 2000.23 + (35.96)² – 59.49% of 2999.89 = 19. (15.97)
+ 61.87% of 1449.87= –√2024.87 +
(49.89)² – ? 13.89 % 𝑜𝑓 7099.97
(a) 559 (b) 569 (c) 589 (a) 222 (b) 282 (c) 262
(d) 549 (e) 539 (d) 200 (e) 308
7. (11.87)² + 12.493 × 15.89 – √13224.98 – (?)² = 20. 32.87% of ? – 17.87 × 44.86 – √399.81 = (24.98)2 +
(14.96)² 29.89
(a) 9 (b) 2 (c) 8 (a) 4450 (b) 4500 (c) 4350
(d) 5 (e) 10 (d) 4600 (e) 4900
5 Adda247 Publications For More Study Material
Visit: adda247.com
Cracker Book for Bank (IBPS | SBI | RRB PO | Clerk) Mains Exams

NUMBER SERIES MISSING SERIES


Wrong Series Directions (1-15): What should come in place of the
Directions (1-15): Find the wrong number in the given question mark (?) in the following series ?
series that does not follow the pattern? 1. 10, 15, 45, 270, 3240, ?
1. 480, 960, 320, 1280, 272 , 1536 (a) 72250 (b) 77760 (c) 75260
(a) 960 (b) 272 (c) 1280 (d) 85280 (e) 762560
(d) 320 (e) 1536 2. 12, 26, 81, 328, 1645, ?
2. 210, 197, 171, 135, 80, 15 (a) 9981 (b) 8281 (c) 7321
(a) 197 (b) 15 (c) 80 (d) 9876 (e) 9978
(d) 171 (e) 135 3. 1024, 512, ? , 16, 1, 0.03125
3. 4, 3, 5, 14, 60, 528 (a) 64 (b) 128 (c) 256
(a) 60 (b) 4 (c) 5 (d) 216 (e) 324
(d) 14 (e) 528 4. 193, 97, 49, 25, ? , 7
4. 9, 63, 25, 216, 49, 512 (a) 15 (b) 12 (c) 17
(a) 25 (b) 216 (c) 63 (d) 13 (e) 11
(d) 512 (e) 49
5. 17, 24, 13, 26, 9, ?
5. 4498, 4888, 5336, 5846, 6422, 7070 (a) 30 (b) 32 (c) 29
(a) 5846 (b) 4498 (c) 5336 (d) 28 (e) 26
(d) 7070 (e) 6422
6. 18, 6, 2, −2 , − 10, ?
6. 122, 125.2, 131.6, 141.2, 152, 170 (a) −36 (b) – 48 (c) − 40
(a) 125.2 (b) 131.6 (c) 141.2 (d) −32 (e) − 52
(d) 170 (e) 152
7. 56, 1144, 2104, 2944, 3672, ?
7. 3671, 2341, 4537, 1163, 6072, − 783 (a) 4556 (b) 4456 (c) 4256
(a) 6072 (b) 2341 (c) − 783 (d) 4296 (e) 4552
(d) 3671 (e) 1163
8. 15, 91, 457, 1831, 5497, 10997 8. 21, 1750, 2752, 3267, 3487, ?
(a) 457 (b) 91 (c) 5497 (a) 3556 (b) 3456 (c) 3536
(d) 15 (e) 10997 (d) 3544 (e) 3558
9. 10, 17, 45, 108, 220, 390 9. 9, 15, 23, 33, 45, ?
(a) 220 (b) 390 (c) 10 (a) 57 (b) 59 (c) 53
(d) 108 (e) 45 (d) 51 (e) 49
10. 10, 30, 90, 450, 3150, 34650 10. 141, 197, 269, 359, 469, ?
(a) 10 (b) 34650 (c) 3150 (a) 607 (b) 609 (c) 601
(d) 3150 (e) 30 (d) 603 (e) 605
11. 325, 546, 754, 936, 1078, 1170 11. 25, 33, 46, 69, 112, ?
(a) 936 (b) 546 (c) 325 (a) 175 (b) 180 (c) 185
(d) 1078 (e) 1170 (d) 190 (e) 195
12. 192, 202, 210, 216, 225, 243, 288 12. ? , 5, 12, 39, 160, 805
(a) 210 (b) 202 (c) 288 (a) 2 (b) 4 (c) 3
(d) 192 (e) 243 (d) 5 (e) 8
13. 3, 11, 49, 191, 569, 1135, 1134 13. ?, 10, 20, 46, 96, 178
(a) 1135 (b) 1134 (c) 3 (a) 6 (b) 7 (c) 8
(d) 49 (e) 11 (d) 9 (e) 5
14. 23, 30, 42, 63, 95, 140, 200 14. 2160, ? , 360, 180, 60, 30
(a) 42 (b) 140 (c) 30 (a) 1080 (b) 720 (c) 900
(d) 200 (e) 23 (d) 810 (e) 1440
15. 8, 30, 60, 120, 180, 180, 90 15. 2, 35, 156, 477, ? , 959
(a) 30 (b) 120 (c) 90 (a) 858 (b) 564 (c) 958
(d) 8 (e) 60
(d) 727 (e) 676
6 Adda247 Publications For More Study Material
Visit: adda247.com
Cracker Book for Bank (IBPS | SBI | RRB PO | Clerk) Mains Exams

Solutions
2
SIMPLIFICATION 12. (c); [(25)3 ÷ (6.25)2 ] × ((1.25)2 ÷ 52 ) = (25)3+𝑥
2
253 1252 1
1. (c); (4 × 4)3 ÷ (512 ÷ 8)4 × (32 × 8)4 = (2 × 2)?+4 [254 × 1002 ] × [1002 × 52] = (25)3+𝑥
(42 )3 ×(44 )4 2
or, (4 3 )4
= (4)?+4 [
1
×
100×100
×
125×125
] = (25)3+𝑥
25 100×100 5×5
46 ×416 2
or, 4 12
= (4)?+4 25 = (25) +𝑥
3
2 2
or, 410 = 4?+4 ⇒ +𝑥 = 1⇒ 𝑥 = 1−
3 3
or, ? = 6 1
𝑥=3
2. (e); ?2 = 2√14 × 14 × 2 − 21 + 8 + 49 − 28√2 1
161 1595 13
= 28√2 − 21 + 57 − 28√2 = 36 = 62 13. (d); × × = (𝑥)3
377 253 7
∴?=6 Explanation
1 1 1 1 161 = 23 × 7
3. (a); 1 + 4 + 1 + 6 − 1 − 8 =? +1 12 377 = 29 × 13
1 1 1 1
⇒? = 1 + 4 + 1 + 6 − 1 − 8 − 1 − 12 1595 = 29 × 55
1 1 1 1 253 = 23 × 11
1
= + − − 23×7
×
29×55
×
13
= (𝑥)3
4 6 8 12 29×13 23×11 7
6+4−3−2 5 1
= 24 = 24 5 = (𝑥) ⇒ 𝑥 = 125 3

30 3
4. (c); 3420 × 100 × 19 = (? )2 ×2 2 4 7 2145 529
14. (a); × × × ×√
⇒ 162 = (? )2 × 2 3 5 11 1288 169
13 23
⇒ (? )2 =
162
= 81 × =1
2 23 13

∴ ? = √81 = 9 √8649
3
√15625
15. (c); × 3 = 𝑥2
65 √961 √1728
5. (b); √3136 × 100 × 5 = ? +154 93 25
× = 6.25 = 𝑥 2
65 31 12
⇒ 56 × × 5 =? +154 𝑥 = 2.5
100
⇒ 182 = ? +154 APPROXIMATION
⇒ ? = 182 − 154 = 28
𝑥 1 255
1. (a); ≈ 294 × 8 − 5 × 120𝑥 =
6. (e); (55.97 – 40.97) = 15 – (?)2 2
(?)2 = 0 ≈ 36.75𝑥 − 24𝑥 = 127.5
?=0 ≈ 12.75𝑥 = 127.5
1
≈ 𝑥 = 10
10 7 3 22
7. (d); × 45 × 2 × = (? )2 X
3
1
7 2. (b); × 5600 = 8 × 41 + 500 – 128
22 100
(? ) =2
700
9
484 X=
?= 56
81 X = 12.5
8. (a); 40.83 × 1.02 ×1.2 = ? 9200 55
? = 49.97592 3. (c); 630X + 600 = 37.5 × 100 + 100 × 12000
630X + 600 = 3450 + 6600
117
9. (c); = (? )2 630X = 10050 – 600
113
(? )2 = 14641 9450
X=
? = 121 360
X = 15
10. (c); 59 × 3 = 150 + ? 12000 55 100 1
? = 27 4. (a); = + × 1800 = × × 9900 + 390
X
100 9 100
100 101 12000
11. (c); % of 675 + of 184 = = 1490 – 990
27 100×4 X
675
+ 46.46 12000
27 X=
500
25 + 46.46 = 71.46 X = 24

7 Adda247 Publications For More Study Material


Visit: adda247.com
Cracker Book for Bank (IBPS | SBI | RRB PO | Clerk) Mains Exams
62.5X 31 5 594
5. (e); 100
= 8100 × 27 + 2700 × 3 – 1800 ?= 66
5x ?=9
= 9300 + 4500 – 1800
8 48 𝑥
12000 × 8 15. (d); × 250 + 100 × 500 ≈ 112 + 22
X= 100
5 120 + 5𝑥 ≈ 121 + 4
X = 19200
5𝑥 ≈ 5
120 59.5 𝑥≈1
6. (c); 100
× 2000 + (36)2 – 100 × 3000 = (50)2 – ?
2 1 1024
2400 + 1296 – 1785 = 2500 – ? 16. (a); 49 × × + √𝑥 ≈
7 14 256
2500 – 1911 = ?
1 + √𝑥 ≈ 4
? = 589
√𝑥 ≈ 3
7. (b); (12)² + 12.5 × 16 – √13225 – (?)² 𝑥≈9
= (15)² 17. (b);
3
× 16 +
32
× 3200 + 295 ≈ 𝑥 3
144 + 200 – 115 – (?)² = 225 4 100
(?)² = 229 – 225 12 + 1024 + 295 ≈ 𝑥 3
?=2 𝑥 3 ≈ 1331
𝑥 ≈ 11
360
8. (e); = (9)3 – 14.5 × 40 + (2)2 – 81 ? 30
?
360
18. (d); 100
× 6520 + (22)2 – 100 × 9140 = (26)2
= 729 – 580 + 4 – 81 ?
?
360 100
× 6520 = 676 + 2741 – 484
? = 72 2934×100
?=
?=5 6520
? = 45
9. (d); √430 + 520 + √121 = (? )2 + √36 19. (a);
578+?
+
62
× 1450 =– √2025 +
14
× 7100
16 100 100
√430 + 520 + 11 = (? )2 + 6 578+?
31 – 6 = (?)² + 899 =– 45 + 994
16
578+?
?=5 = 949 – 899
16
10. (a); (23.89)² + √3600 – 172 + (?)² = (28)² + 4 578 + ? = 50 × 16
576 + 60 – 172 + (?)² = 784 + 4 ? = 800 – 578
464 + (?)² = 788 ? = 222
(?)² = 788 – 464 20. (b);
33
×? = 625 + 30 + 810 + 20
? = 18 100
1485×100
?=
33
11. (d); (14)² – 40% of 240 + (30)² + 331 = (?)³
40×240 ? = 4500
196 – + 900 + 331 = (? )3
100
WRONG SERIES
1000 + 331 = (?)³
? = 11 1. (b);
?+134.5
12. (c); + 210 + (70)2 – 2000 = (55)2 + 100
24
?+134.5
+ 3110 = 3125
24
? + 134.5 = 15 × 24
So wrong number is 272
? = 360 – 134.5
? = 225.5 2. (e);
13. (e); ? × 13 + (25)² – (20)² + 364 = (28)²
? × 13 + 225 + 364 = 784
? × 13 = 784 – 589 So wrong number is 135
195
?=
13 3. (a);
? = 15
594 1260
14. (d); ?
+ 15 × 36 + 18
= (26)2
594
?
+ 540 + 70 = 676
So wrong number is 60
594 = ? (676 – 610)

8 Adda247 Publications For More Study Material


Visit: adda247.com
Cracker Book for Bank (IBPS | SBI | RRB PO | Clerk) Mains Exams

4. (c); 12. (b);

So wrong number is 63
So wrong number is 202
5. (d); Wrong number = 7070
13. (b);

So wrong number is 1134


14. (e);

5. (e); Wrong number = 152

So wrong number is 23
15. (a);
6. (a); Wrong number = 6072
Pattern of series —
So wrong number is 30

MISSING SERIES

8. (e); 1. (b); Series is ×1.5, ×3, ×6, ×12, ×24


So, 3240 × 24 = 77760
2. (d); Series is ×2+2, ×3+3, ×4+4, ×5+5, ×6+6
So, 1645 × 6 + 6 = 9876
So wrong number is 10997
3. (b); Series is
9. (b); ÷2, ÷4, ÷8, ÷16, ÷32
So, ? = 512 ÷ 4
= 128
4. (d); −96, −48, − 24, − 12, −6
So, 25 – 12 = 13
5. (d); Series
+7, −11, +13, −17, +19
So wrong number is 390 So, 9 + 19 = 28
10. (a); 6. (d); Pattern of series –
18 × 0.5 − 3 = 6
6×1−4=2
2 × 1.5 − 5 = −2
So wrong number is 10 − 2 × 2 − 6 = − 10
? = − 10 × 2.5 −7 = − 32
11. (d);
7. (d);

So wrong number is 1078

9 Adda247 Publications For More Study Material


Visit: adda247.com
Cracker Book for Bank (IBPS | SBI | RRB PO | Clerk) Mains Exams

8. (a); 21+ (123 + 1) = 1750 12. (b);


1750 + (103 + 2) = 2752
2752 + (83 + 3)= 3267
3267 + (63 + 4) = 3487
? = 3487 + (43 + 5) = 3556 13. (c);
9. (b);

14. (a);
10. (c); 141 + 7 × 8 = 197
197 + 8 × 9 = 269
269 + 9 × 10 = 359
359 + 10 × 11 = 469
15. (c); (2 + 5) × 5 = 35
? = 469 + 11 × 12 = 601
(35 + 4) × 4 = 156
11. (e); (156 + 3) × 3 = 477
(477 + 2) × 2 = 958
(958 + 1) × 1 = 959

10 Adda247 Publications For More Study Material


Visit: adda247.com
Cracker Book for Bank (IBPS | SBI | RRB PO | Clerk) Mains Exams

Chapter
Ratio Proportion and Mixture
2 & Alligation
BEST APPROACH TO SOLVE THE QUESTIONS

Alligation is a method used to solve problems concerned with weightage average. Or, in other words, alligation is
just a way of writing equations of weightage average for the easy calculation. Problems involving mixtures or any form
of weightage average can be solved easily using the method of alligation.
Questions related to replacement, addition or omission of components of mixture may be easily solved using
fractions. Be it in form of ratio or by taking volume into consideration. A very useful formula for problems based on
mixtures is given below:
Final ratio= Initial ratio (fraction of component remained)
Final volume= initial volume (fraction of component remained)
A few sample problems explaining the concept of alligation are given below.
2
Example 1: ‘An alloy containing 50% gold is formed by melting two metals A and B. Metal A contains 66 % gold while
3
metal B contains 40% gold. Find the ratio in which metal A and B are mixed to form the alloy.’
Solution: Let’s take the fractional equivalents of given percentages and calculate using alligation.

Hence, required ratio is 3:5.


Example 2: ‘Average weight of boys in a class is 55kg while average age of girls in the same class is 35. If the average of
all the students in the class is 40 kg, find the ratio of boys and girls in the class.’
Solutions:

Required ratio is 1:3.


Example 3: ‘A can contains mixture of two liquids A and B in the ratio 7 : 5. When 9 litres of mixture are drawn off and
the can is filled with B, the ratio of A and B becomes 7 : 9. How many litres of liquid A was contained by the
can initially?’

11 Adda247 Publications For More Study Material


Visit: adda247.com
Cracker Book for Bank (IBPS | SBI | RRB PO | Clerk) Mains Exams
Solution: Since B is filled by replacing some amount of mixture, we should take A into consideration
Initial ratio of A to the whole mixture = 7/12
7
Final ratio of A to the whole mixture = 16
7 7 12 3
Using formula, = ×𝑦 Or, 𝑦 = =
16 12 16 4
3 1
This shows the mixture remained 4 th of itself which means 4th part was drawn off.
1
ATQ, 4 of total = 9
Or, total mixture = 9 × 4 = 36 litres
7
Initial amount of liquid A = × 36 = 21 litres
12

Practice Exercise Based on new Pattern

1. 150 kg of alloy containing copper and zinc in the ratio 6. Two liquid containing (X and Y) mixture mixed, in the
3 : 2 mixed with ‘X’ kg of alloy containing copper and ratio 2 : 3. X contain 40% sugar in water, while Y
zinc in the ratio 2 : 3. If the overall alloy should contain alcohol in water. After adding 20 gram sugar
contain copper between 45% to 55%, what can be in the final mixture ratio of Sugar, alcohol and water
minimum value of X? becomes 8 : 5 : 16. Given that all ratio is calculate on
(a) 450 kg weight. Find the weight alcohol in mixture ‘Y’.
(b) 100 kg (a) 25 grams (b) 50 grams
(c) 60 grams (d) 80 grams
(c) 50 kg
(e) 75 grams
(d) Cannot be determined
(e) None of these 7. Two vessels A and B of equal capacities contain
mixtures of milk and water in the ratio 4:1 and 3:1,
2. Three different liquids which have 10% water, 20% respectively. 25% of the mixture from A is taken out
water and x% of water are mixed in the ratio of their and added to B. After mixing it thoroughly, an equal
quantity 2 : 3 : 4 respectively. If 12% of water is amount is taken out from B and added back to A. The
present in final mixture. Calculate value of x. ratio of milk to water in vessel A after the second
(a) 9% (b) 20% (c) 7% operation is
(d) 15% (e) 17% (a) 79: 21 (b) 83: 17 (c) 77: 23
(d) 81: 19 (e) None of these
3. ‘x’ liters of a 30% alcohol solution is mixed with 40
liters of 60% alcohol solution & a resultant of 50% 8. A 280 ml mix real juice pack contains mixture of
alcohol solution is formed. Now ‘3x’ liters of y% Mango juice and orange juice in the ratio of 9 : 5. If x
alcohol solution is added to 30 liters of 50% alcohol ml juice taken out from pack and 20 ml of orange juice
solution which resulted in 45% alcohol solution. The mixed in pack so the new ratio of Mango juice and
ratio of y : x is orange juice become 7 : 5, then find what quantity of
mango juice taken out from pack initially?
(a) 17 : 6 (b) 16 : 15 (c) 7 : 15
(a) 48 ml (b) 42 ml (c) 54 ml
(d) 14 : 5 (e) 17 : 8
(d) 60 ml (e) 64 ml
4. There are three varities of sugar with their quantity in 9. There are two vessels A and B which contains mixture
the ratio of 3 : 4 : 5. If 9 kg of first variety and 4 kg of of sulphuric acid and nitrous oxide in the ratio of 7 : 2
second variety are added to their respective quantity and 3 : 4 respectively. Mixture of both vessels are
and x kg of 3rd type is removed from it, then final ratio mixed to obtain a mixture of 390 ml, in which
becomes 9 : 10 : 10. Find the sum of initial quantities quantity of nitrous oxide is 160 ml. Find ratio of
of these varieties. quantity of mixture in vessel A quantity of mixture in
(a) 120 kg (b) 96 kg (c) 84 kg vessel B?
(d) 108 kg (e) None of these (a) 7 : 6 (b) 6 : 7 (c) 5 : 7
(d) 7 : 9 (e) 4 : 7
5. A jeweler mixed gold and copper in 2 proportion. In
type ‘A’ alloy, 6 gm gold is mixed with 5 gm copper 10. There is two vessels A and B. Vessel A & B contains
and in type ‘B’ alloy, 5 gm gold in mixed with 3 gm the mixture of milk, mango juice and water in the
copper. If jeweler have 122 gm gold and 90 gm ratio of 8 : 5 : 3 & 6 : 5 : 2 respectively. If both vessels
copper, then find the weight of type ‘B’ alloy. are mixed respectively in the ratio of 16 : 13 into
another vessel C then the total quantities of mango
(a) 60 gm (b) 80 gm (c) 70 gm
juice in vessel C will be 20 liters. Find the difference
(d) 100 gm (e) 90 gm
between the quantity of milk and water in vessel C?

12 Adda247 Publications For More Study Material


Visit: adda247.com
Cracker Book for Bank (IBPS | SBI | RRB PO | Clerk) Mains Exams

(a) 15 liters (b) 10 liters (c) 18 liters then find the difference between mixture taken out
(d) 25 liters (e) 20 liters from vessel A & B?
(a) 16 l (b) 12 l (c) 8 l
11. Two container P and Q have mixture. In P three liquid
(d) 10 l (e) 18 l
A, B and C are mixed in a ratio of 2 : 1 : 1 and in Q two
liquid B and C are mixed in ratio of 3 : 2. Mixture of 16. Mixture of petrol and water in vessel A are in the ratio
both container P and Q poured into third container X of 4:1 and in vessel B are in the ratio of 5:3. If 20 l of
which show the quantity ratio of A, B and C is 8 : 13 : mixture taken out from vessel A and proud in vessel B
10. Find the quantity in Q is what % less or more than so new ratio of petrol to water in vessel B becomes
the quantity in P. 19:10, then find initially quantity of petrol in vessel A,
(a) 3.25% (b) 12.5% (c) 6.25% if new quantity of mixture in vessel B is 26 l more
(d) 11% (e) 25% than initial quantity of mixture in vessel A?
(a) 64l (b) 72l (c) 68l
12. In vessel A 80% is milk and rest is water. Vessel B also
1 (d) 56l (e) 48L
contain same mixture in which milk is 622% of total
volume. 25ℓ of mixture taken out from vessel A and 17. Two container P and Q have mixture of different
poured in vessel B so that milk becomes 100% more liquids. In P three liquid A, B and C is mixed in a ratio
of 2 : 1 : 1 and in Q two liquid B and C is mixed in ratio
than water in vessel B. If new quantity of mixture in
of 3 : 2. Mixture of both container P and Q poured into
vessel B is 16% less than initial quantity of mixture in
vessel A then find initial quantity of milk in vessel A ? third container X which show the quantity ratio of A,
B and C is 8 : 13 : 10. Find the quantity in Q is what %
(a) 100l (b) 125l (c) 115l
less or more than the quantity in P.
(d) 105l (e) 112l
(a) 3.25% (b) 12.5% (c) 6.25%
Directions (13-14): Three milkmanP, Q and R have (d) 11% (e) 25%
mixture of milk and water in the quantity of (X + 24)
liters, (X+54) liters and (X + 84) liters and milk and water 18. Ratio of Vodka and Wine in vessel A is 5 : 3 and same
mixture in vessel B in the ratio of 3 : 2, 16ℓ of mixture
in the ratio of 2 : 3, 8 : 7 and 13 : 5 respectively.
from vessel A taken out and poured in vessel B new
13. If P sold 50 liters of his mixture and 8 liters of milk ratio of Vodka to wine becomes 29 : 19. If new
added in remaining mixture, new ratio of milk and quantity of mixture in vessel B is equal to initial
water becomes 6 : 7 . What quantity of water should quantity of mixture in vessel A, then find quantity of
be added by Q and R, so new ratio of their mixture Vodka after 16 ℓ of mixture has been taken out from
becomes 5 : 7 & 5 : 4 respectively ?(1 marks) vessel A ?
(a) 56 liters& 40 liters (a) 50𝑙 (b) 48𝑙 (c) 54𝑙
(b) 54 liters& 42 liters (d) 80𝑙 (e) 84𝑙
(c) 50 liters& 30 liters
(d) 60 liters& 50 liters 19. Sameer and Veer entered into partnership business
with the capital of Rs. x and Rs. (x + 12000)
(e) 64 liters& 30 liters
respectively. After One year Divyaraj joined them with
14. Q sold 40% of his mixture and R sold 50% of his capital of Rs. (x + 8000). At the end of 2nd year Sameer
mixture, remaining mixture of both became equal. If and Divyaraj withdraw their capital and Veer invested
all three charge Rs. 2 for one liter of water and for one more year. If Sameer, Divyaraj and Veer gets
production cost of milk for P, Q and R in the ratio of 5 : profit in the ratio of 8 : 6 : 21 respectively. Find sum of
6 : 7 and total mixture, which all three milkman have capital invested by all three ?
cost Rs. 3644. Find production cost of milk per liter (a) 66000 Rs. (b) 54000 Rs.
for P, Q and R respectively?(1 marks) (c) 64000 Rs (d) 68000 Rs.
(a) 5 Rs. 6 Rs & 7 Rs. (e) 70000 Rs.
(b) 25 Rs. 30 Rs & 36 Rs.
(c) 10 Rs. 12 Rs & 16 Rs. 20. Wheat of x Rs./kg is mixed with wheat of y Rs./kg in
the ratio 2 : 3. On selling the mixture at 49 Rs./kg
(d) 10 Rs. 12 Rs & 14 Rs. 2
(e)10 Rs. 14 Rs& 15s. there is a gain of 16 %. If the quantity in which they
3
were mixed is reversed and selling price remains
15. Vessel A & B contains mixture of orange juice and 550
mango juice in the ratio of 5 : 4 and 5 : 2 respectively. same, then gain percent becomes %. What is the
19
Some quantity of mixture taken out from vessel A and volume of ‘x’.
B and mixed in another vessel C, if vessel C contains (a) 25 Rs./kg (b) 30 Rs./kg
mixture of orange juice and mango juice in the ratio of (c) 28 Rs./kg (d) 32 Rs./kg
5 : 3 and total quantity of mixture in vessel C is 96 l. (e) 20 Rs./kg

13 Adda247 Publications For More Study Material


Visit: adda247.com
Cracker Book for Bank (IBPS | SBI | RRB PO | Clerk) Mains Exams

21. Three vessels A, B and C of same capacity are all full. 26. Amount invested in business respectively by Rana,
Vessel A and B contain mixture of milk and water in Nikhil and Yash, was in AP respectively, and Yash
ratios 4 : 1 and 7 : 3 while vessel C contains only pure invested thrice amount as that of Rana. Rana was
milk. The contents of all the vessels are poured into a manager and Nikhil was a programmer, and they both
larger vessel D. After drawing 30 liters of the mixture get salary of 8% and 5% of total profit respectively. If
from vessel D, the final quantity of water in Vessel D is Rana invested for a month, Nikhil for 8 months and
Yash invested for 4 months and Nikhil got Rs.6000
40 liters. Find the capacity of each vessel
more than both of them together, find their total
(a) 80 (b) 85 (c) 95 profit.
(d) 70 (e) 90 (a) Rs. 100000/- (b) Rs. 150000/-
22. Two container P and Q have mixture. In P three liquid (c) Rs. 174000/- (d) Rs. 180000/-
A, B and C is mixed in a ratio of 2 : 1 : 1 and in Q two (e) Rs. 20000/-
liquid B and C is mixed in ratio of 3 : 2. Mixture of 27. There are three vessels A, B & C having Milk to water
both container P and Q poured into third container X ratio as 3 : 5, 1 : 3 and 7 : y. After adding mixture of
which show the quantity ratio of A, B and C is 8 : 13 : vessel B to A, the milk to water ratio becomes 1 : 2 in
10. Find the quantity in Q is what % less or more than A . Now 60% of mixture is taken out from vessel A and
the quantity in P. mixture of vessel C is mixed with remaining mixture
(a) 3.25% (b) 12.5% (c) 6.25% of vessel A which results in final Milk to Water ratio of
3 : 4. Find the value of y given that volume of B is 20
(d) 11% (e) 25%
ltrs and volume of C is (7 + y) ltrs.
23. Vessel A contains (X + 24) 𝑙 of mixture of milk and (a) 1 (b) 4 (c) 5
water in the ratio of 7 : 8, while vessel B contains (X + (d) 3 (e) 2
54) 𝑙 of same mixture of milk and water in the ratio of 28. There are three mixtures A, B and C. Mixture A
1
3 : 2. If 37 % of mixture from vessel A and 40% of contains 10% of water and B contains 15% of water.
2
mixture from vessel B taken out and mixed in vessel If A and B are mixed together than resultant mixture
C, then the remaining mixture in vessel B is 15𝑙 more contains 12% water. while If B is mixed with C, which
contains 22% water so the resultant mixture contains
than that of in vessel A. find quantity of milk in vessel
of 18% water. All these mixture are mixed in a jar and
C?
then 30 ml water is also added. Find total volume of
(a) 49 litre (b) 57 litre (c) 55 litre water preset in jar, if initially mixture C contains 66
(d) 53 litre (e) 51 litre ml water.
24. Three container (A, B and C) have capacity in the ratio (a) 144 ml (b) 216 ml (c) 184 ml
of 6 : 8 : 7 respectively. These three container are (d) 196 ml (e) 200 ml
pour with three type of liquid X, Y and Z in the ratio 2 : 29. There are two containers A and B filled with oil with
3 : 1 (in A), 1 : 2 : 5 (in B) and 3 : 1 : 3 (in C) different prices and their volumes are 140 litres and
respectively. Equal quantity of mixture is taken out 60 litres respectively. Equal quantities are drawn
from these containers and poured in container D. from both A and B in such a manner that the oil drawn
Quantity of liquid Y is 1/8 liter more than quantity of from A is poured into B and the oil drawn from B is
liquid X in container D. Find out the quantity taken poured into A. The price per litre becomes equal in
both A and B. How much oil is drawn from each of A
out from each container.
and B:
(a) 24 (b) 35 (c) 21 (a) 40 litre (b) 80 litre (c) 30 litre
(d) 20 (e) None of these (d) 21 litre (e) 42 litre
25. A shopkeeper has mixtures of nitrous oxide and water 30. A shopkeeper has two types Wheat i.e Type A & Type
in two vessels A and B. Vessel A and vessel B contains B and cost price per kg of Type A wheat is 10 times of
mixtures of nitrous oxide and water in the ratio of 7 : cost price per kg of type B wheat. Shopkeeper cheated
2 & 5 : 3 respectively. The shopkeeper has taken out a costumer by saying that he will give him 200 kg of
the mixtures from vessels A and B in the ratio of 9 : 8 Type A wheat but shopkeeper gives 200 kg of mixture
and mixed it in vessel C. If shopkeeper sold 68 gm of of Type A & Type B wheat on 10% above the cost
mixture from vessel C on the cost price of nitrous price per kg of Type A wheat. If shopkeeper made a
7
oxide, which is Rs. 80 per gm, find profit of total profit of 71 % in this transaction, then find
8
shopkeeper? ratio between quantity of Type B wheat to Type A
2 2 2 wheat in sold mixture?
(a) 39 3 % (b) 41 3 % (c) 45 3 %
2 2 (a) 2 : 5 (b) 2 : 3 (c) 1 : 3
(d) 37 3 % (e) 35 3 % (d) 2 : 7 (e) 1: 5
14 Adda247 Publications For More Study Material
Visit: adda247.com
Cracker Book for Bank (IBPS | SBI | RRB PO | Clerk) Mains Exams

31. Two farmers A & B have mixture of urea & Potassium the largest vessel. If 5 times the juice is served by
in the quantity of 120 kg and 80 kg respectively. The largest vessel, 8 times juice is served by second
1 1
mixture of farmer A has 37 2
% potassium and the largest vessel and finally when 3 3
times juice is
mixture of farmer B has 40% potassium. If farmer A served by the smallest vessel, the tank gets emptied
used 60% of his mixture & B used 50% of his mixture completely. Find the total quantity of juice served by
and both gives their remaining mixture to farmer C, second largest vessel is what percent of volume of the
who have already an equal quantity of urea & tank?
Potassium. If ratio of urea & Potassium in resulting 4 4 4
(a) 42 % (b) 44 % (c) 48 %
9 9 9
mixture of farmer C is 7 : 5, find total initial quantity 4 4
(d) 40 % (e) 36 %
of mixture farmer C had? 9 9

(a) 24 kg (b) 36 kg (c) 32 kg 34. There are three fruit sellers Ramesh, Suresh and
(d) 42 kg (e) 48 kg Vikash, all of them have a mixture of two types of
apple. Ramesh has (x+18) kg of apple, Suresh has 12
32. Vessel A, B and C contains mixture of milk and water.
kg of apple more than that of Ramesh while Vikash
Ratio of the quantity of A, B and C is 25 : 20 : 18 has 3 kg of apple more than that of Suresh. The two
respectively. In vessel A milk to water ratio is 2 : 3 types of Apple are mixed in the ratio of 5 : 3, 2 : 3 and
and in vessel B water to milk ratio is 2 : 3. If vessel C 4 : 3 for them respectively. If Ramesh sold 16 kg of
contain milk to water ratio is 4 : 5 and all the mixture mixed apple and added 12 kg of second type of apple
of 3 vessel poured to another vessel D then in vessel D then the new ratio becomes 5 : 6 for him. Vikash
quantity of water exceeds by 6 litres than the quantity added y kg of second type of apple to his mixture and
thus the new ratio of both types for him is 1 : 1. Then
of milk in that vessel. Find out the quantity of mixture
find the value of y.
contained in vessel A, B and C. (a) 8 kg (b) 9 kg (c) 10 kg
(a) 50 Ltr., 40 Ltr., 36 Ltr. (d) 6 kg (e) 7 kg
(b) 25 Ltr., 20 Ltr., 18 Ltr.
35. There are two vessels, vessel A and vessel B. Vessel A
(c) 75 Ltr., 60 Ltr., 54 Ltr.
contains milk and water in the ratio 5 : 4 and vessel B
(d) 100 Ltr., 80 Ltr., 72 Ltr. contains Y litre of wine. 18 litre of mixture from vessel
(e) None of these A is taken out and poured in vessel B. Now, 10 litre of
mixture from vessel B is taken out and poured in
33. In a party a tank is filled with some quantities of juice. vessel A. If quantity of wine in vessel A is found to be
Three vessels of different volume are used to serve 4 litre then, find the initial quantity of wine in vessel
the juice. Volume of the smallest vessel is 25% less B.
than volume of second largest vessel and the volume (a) 8 L (b) 9 L (c) 10 L
1 (d) 12 L (e) 6 L
of second largest vessel is 33 % less than volume of
3

Solutions

1. (c); If overall alloy contain copper as 45% 10% of water in first type means 20a water
3 2x 45
(150 × 5 + 5 ) = 100 (150 + x) 20% of water in second type means 60a water
2x 9 x% of water in third type means 4xa water
(90 + ) = (150 + x) ∴ ATQ,
5 20
20a+60a+4xa 12
1800 + 8x = 1350 + 9x = 100
900a
x = 450 kg 28a
⇒ 4xa = 108a – 80a ⇒ x= =7
If overall alloy contain copper as 55% 4a
2x 55
(90 + )= (150 + x) 3. (e); From statement I
5 100 30 60
x× +40× 1 (2400+30x) 1
1800 + 8x = 1650 + 11x 100 100
= ⇒ =
(x+40) 2 100(x+40) 2
3x = 150 ⇒ x = 50 kg ⇒ x = 20
Minimum value of X is 50 kg Now from statement II
y 50
2. (c); Let the quantity of three liquids is 200a, 300a and (3x× +30× ) 45
400a
100
3x+30
100
= 100

15 Adda247 Publications For More Study Material


Visit: adda247.com
Cracker Book for Bank (IBPS | SBI | RRB PO | Clerk) Mains Exams

Here 3X = 3 × 20 = 60 litres = 100 ml


60y
( )+15 45 60y 81 ATQ—
⇒ 90 = 100 ⇒ 100 =
100
2
– 15 180–
9x
7
⇒ y = 42.5 5x
14
=
100– +20 5
14
∴ y ∶ x = 42.5 : 20 = 17 : 8 5(2520 – 9x) = 7(1400 – 5x + 280)
4. (d);Let the initial quantity is 3y, 4y & 5y of these 45x – 35x = 12600 – 11760
varities 10x = 840 ⇒ x = 84 ml
According to condition Quantity of mango juice taken out from pack
9
3y + 9 ∶ 4y + 4 ∶ 5y – x = 9 ∶ 10 ∶ 10 initially = 84 × = 54ml
14
From 1st 2 ratios
3y+9 9
⇒ 4y+4 = 10 9. (b);Let total mixture of sulphuric acid and nitrous
oxide in vessel A and B be P mℓ and Q mℓ
⇒ y = 9, Hence sum of initial quantities is (3 + 4 + respectively
5) × 9 = 108 kg ATQ,
NOTE: No need calculate Value of x P + Q = 390 … (i)
2P 4Q
5. (b);Let gold and copper in Type A alloy be 6a and 5a + = 160
9 7
Let gold and copper in type B alloy 5b and 3b 14P + 36Q = 10080 … (ii)
⇒ 6a + 5b = 122 … (i) From 14 × (i) – (ii)
5a + 3b = 90 … (ii) Q = 210 mℓ
Solving equation (i) and (ii) we get P = 390 – 210 = 180 mℓ
a = 12 ⇒ b = 10 180
Required ratio = =6:7
Weight of type ‘B’ alloy = (5 + 3) × 10 210
= 80 gm 10 (c); Let quantity of mixture mixed in vessel C from
6. (a); Let weight of alcohol in Y = a gram vessel A & B be 16x and 13x respectively.
Let weight of mixture X = 100x gram ATQ—
5 5
So, sugar weight in X = 40x gram 16x × + 13x × (6+5+2) = 20 liters
(8+5+3)
Water weight in X = 60x gram 5x + 5x = 20 liters
So, weight of Y = 150x gram x = 2 liter
Weight of water in Y = (150x – a) gram Required difference =
ATQ, 8 6 3
40x+20 : a : 150x–a+60x = 8 : 5 : 16… (i) [(16 × 2) × + (13 × 2) × ] – [(16 × 2) × +
16 13 16
a 5 2
⇒ 210x–a = 16 (13 × 2) × ] 13
⇒ a = 50x … (ii) = (16 + 12) – (6 + 4) = 18 liters
Put value of ‘a’ in eqn. (i) 11. (c); Let in P, Quantity of A, B and C is 2x, x and x
40x+20 8 40x+20 8 1
a
= 5 ⇒ 50x = 5 ⇒ x = 2 respectively
1 In Q → B and C is 3y and 2y
weight of alcohol in Y = 50 × 2 = 25 gram.
7. (a); Assume there is 20litres of the mixture in both the Now in final mixture → (X) A : B : C
vessels. 2x : x+3y : x+2y = 8 : 13 : 10
In vessel A, milk = 16litres and water Solving →
= 4litres x = 4 unit
If 25% mixture from A added to B, y = 3 unit
milk in B = 15 + 4 = 19litres Quantity in P → (2 + 1 + 1) × 4 = 16 unit
water in B = 5 + 1 = 6litres Quantity in Q → (3 + 2) × 3 = 15unit
1
Ratio = 19: 6 Required % = 16 × 100 = 6.25%
Equal amount from vessel B to vessel A
19 79 12. (a); Let milk and water in vessel A
milk in A = 12 + 5 = 5
4a and a respectively
6 21
= water in A = 3 + 5 = 5 Let Milk and water in vessel B
Hence, the ratio is 79: 21 5b and 3b respectively
ATQ—
8. (c); Mango juice in mixture 5b+25×
4
9 2
= 280 × (9+5) = 180 ml
5
1 =1
3b+25×
5
5 5b+20 2
Orange juice in mixture = 280 × (9+5) =1
3b+5

16 Adda247 Publications For More Study Material


Visit: adda247.com
Cracker Book for Bank (IBPS | SBI | RRB PO | Clerk) Mains Exams

6b – 5b = 20 – 10 5
15. (b);Part of Orange juice in vessel A = 9
b = 10 5
New quantity of mixture in vessel B Part of Orange juice in vessel B = 7
= (5 × 10 + 20) + (3 × 10 + 5) = 105ℓ 5
Part of Orange juice in vessel C =
8
Initial quantity of mixture in vessel A
100 By allegation —
= 105 × 84 = 125ℓ
Quantity of milk in vessel A
4
= 125 × 5 = 100ℓ
13. (b);In 50 liters sold mixture of P
2
Milk = 50 × = 20liters
5
3
Water = 50 × 5 = 30 liters
ATQ - =9:7
2(P+24)
−20+8 6 2P+48−60 6 (9–7)
5
3(P+24) =7 ⇒ =7 Required difference = 96 × = 12ℓ
−30 3P+72−150 16
5
7P – 42 = 9P = 234 16. (b);Lets petrol and water in vessel A be 4x and x
2P = 234 – 42 Petrol and water in vessel B by 5y and 3y
192
P= ⇒ P = 96 liters ATQ—
2 4
5y+20× 19 5y+16 19
Mixture, which Q have 5
1 = ⇒ =
3y+20× 10 3y+4 10
= (96 + 54) = 150 liter 5
Mixture which R have = (96 + 84) 50y + 160 = 57y + 76
= 180 liters 7y = 84 ⇒ y = 12
For Q – New mixture in vessel B
Milk =
8(96+54)
= 80 liters = (5 × 12 + 16) + (3 × 12 + 4)
15 = 76 + 40 = 116 ℓ
(96+54)
Water = 15 = 70 liters Initial quantity of vessel A = (116 – 26)
Lets Q added x liters of water = 90ℓ
80 5
=7 4
70+x Quantity of petrol in vessel A = 90 × 5
5x = 560 – 350
5x = 210 ⇒ x = 42 liters = 72 ℓ
for R- 17. (c); Let in P, Quantity of A, B and C is 2x, x and x
13(96+84)
Milk = = 130 liters respectively
18
5(96+84)
Water = = 50 liters In Q → B and C is 3y and 2y
18
Lets R added y liters of water Now in final mixture → (X) A : B : C
130 5 2x : x+3y : x+2y = 8 : 13 : 10
50+y
=4
5y = 520 – 250 Saving →
270 x = 4 unit ⇒ y = 3 unit
y = 5 ⇒ y= 54 liters
Quantity in P → (2 + 1 + 1) × 4 = 16 unit
14. (d);ATQ- Quantity in Q → (3 + 2) × 3 = 15unit
(100−40) (100−50) 1
(P + 54) × = (P + 84) × 100 Required % = 16 × 100 = 6.25%
100
60P + 3240 = 50P + 4200
10P = 960 ⇒ P = 96 liters 18. (a); Let ratio of Vodka and wine in vessel A be 5x and
Let production cost be Rs 5x, Rs 6x and Rs 7x 3x
ATQ_ and ratio of Vodka and wine in vessel B be 3y and
(48 × 5x + 72 × 2) + (80 × 6x + 70 × 2) + (130 × 7x
2y
+ 50 × 2) = 3644
240x + 480x + 910x = 3644 – (144 + 140 + 100) ATQ—
5
1630x = 3260 3y+16× 29 3y+10 29
8
3 = 19 ⇒ = 19
x = 3260 ⇒ x = 2 Rs 2y+16×
8
2y+6
production cost for P = 5 ×2 = 10 Rs 58y – 57y = 190 – 174
production cost for Q = 6 × 2 = 12 Rs y = 16
production cost for R = 7 × 2 = 14 Rs

17 Adda247 Publications For More Study Material


Visit: adda247.com
Cracker Book for Bank (IBPS | SBI | RRB PO | Clerk) Mains Exams
1
New quantity of mixture in vessel B ⟹ 5x − 6 × 30 = 40
= (16 × 3 + 10) + (16 × 2 + 6) ⟹ 5x = 45
= 96 ℓ ⟹x=9
= initial quantity of mixture in vessel A Capacity of each vessel = 10x = 90 liters
Quantity of Vodka remaining in vessel A
5 16×5
22. (c); Let in P, Quantity of A, B and C is 2x, x and x
= 96 × – = 60– 10 = 50ℓ respectively
8 8
In Q → B and C is 3y and 2y
19. (d);Investment Ratio of Sameer : Divyaraj : Veer Now in final mixture → (X) A : B : C
= x × 2 : Rs. (x + 8000) × 1 : Rs. (x + 12000) × 3 2x : x+3y : x+2y = 8 : 13 : 10
= 8 : 6 : 21 Saving →
ATQ— x = 4 unit
2x 8 y = 3 unit
(x+8000)
=
6
Quantity in P → (2 + 1 + 1) × 4 = 16 unit
6x – 4x = 32000 Quantity in Q → (3 + 2) × 3 = 15unit
x = 16000 Rs. 1
Required % = × 100 = 6.25%
Required sum of capital (Sameer + Divyaraj + 16

Veer) 23. (b);ATQ,


(100−37.5) (100−40)
= 16000 + (16000 + 8000) + (16000 + 12000) (x + 24) × + 15 = (x + 54) ×
100 100
= 68000 Rs. 5 3
(x + 24) × + 15 = (x + 54) ×
8 5
6
20. (b);C.P. of mixture = 49 × = 42Rs/kg 5(5x + 120 + 120) = 8(3x + 162)
7
2 y –42
= 42 –x 25x – 24x = 1296 – 1200
3 x = 96 liters
84 – 2x = 3y – 126
total mixture take out from vessel
3y + 2x = 210 … (i) 3
New cost price of mixture after reverting the A = (96 + 24) × = 45 litre
8
quantity in which they are mixed. Total mixture taken out from vessel B
11 2
(1 + )  new CP = 49 = (96 + 54) × = 96 liter 5
38
38 7 3
New CP = 49 × = 38 Rs./kg Milk in vessel C = 45 × + 60 ×
49 15 5
So, = (21 + 36) = 57 litre
24. (c); Let P ltr. of mixture are taken out from all 3
containers.
Quantity of X, Y and Z from A container
2P 3P P
→ , ,
3 y –38 6 6 6
2
= 38 –x From B container
114 – 3x = 2y – 76 P 2P 5P
→ , ,
8 8 8
190 = 2y + 3x ... (ii)
From C container
Solving (i) and (ii) 3P P 3P
x = 30 Rs/kg → 7 ,7, 7
3P 2P P
21. (e); Let the capacity of each vessel be 10x liters. Quantity of Y → 6
+8
+7
84P+42P+24P 150P
Then, vessel A contains 8x liters of milk and 2x = =
168 168
liters of water. 2P P 3P
Vessel B contains 7x liters of milk and 3x liters of Quantity of X → 6
+ +
8 7
56P+21P+72P 149P
water. = =
168 168
Vessel C contains 10x liters of milk. 150P 149P 1
Diff. = – = 8 ⇒ P = 21 liter
Hence, Vessel D contains 25x liters of milk and 5x 168 168
liters of water i.e. in the ratio 5 : 1. 25. (b);In vessel A
According to the question, Nitrous oxide : Water = 7 : 2
30 liters of the mixture is drawn from vessel D. In vessel B
Final quantity of water in Vessel D = 40 liters Nitrous oxide : Water = 5 : 3

18 Adda247 Publications For More Study Material


Visit: adda247.com
Cracker Book for Bank (IBPS | SBI | RRB PO | Clerk) Mains Exams

In 68 gm of mixture 28. (b);ATQ –


Mixture taken from vessel A
9
= 68 × 17 = 36 gm
8
Mixture taken from vessel B = 68 × 17
= 32 gm Therefore,
Total nitrous oxide in 68gm of mixture Ratio of mixture, A to B is 3:2.
7 5
= 36 × 9 + 32 × 8 = 28 + 20 = 48 gm And ratio of mixture B to C is 4:3.
Total cost of 68 gm mixture = 68 × 80 Hence, ratio of volume of mixtures A, B & C is
= 5440 Rs A:B:C=6:4:3
It is given that contains 22% water, which is 66
Cost of nitrous oxide = 48 × 80 = 3840
5440−3840 2 ml. Hence total volume of mixture C is
Required profit = × 100 = 41 % 100
3840 3 66 × = 300 ml
22
26. (a); Let Rana invested Y Rs. B is 400 ml
∴ Yash invested 3Y Rs. A is 600 ml
15
And these are in AP. Water in B = 100 × 400 = 60 ml
Y+3Y
∴ middle term is 2 = 2Y. 10
Water in A = 100 ×600=60 ml
2Y is amount invested by Nikhil. Total water in jar after adding 30 ml water
Let total profit be 100x. = 60 + 66 + 60 + 30 = 216 ml.
Then as a salary Rana got 8X.
And Nikhil got 5X. 29. (e); Let the price of oil per litre in container 'A' be 'x'
Remaining 87X was divided according to share. and that in 'B' be 'y' and the litres mixed be
Y × 1 : 2Y × 8 : 3Y × 4 'a' Initial price of A is 140x.
Y : 16Y : 12Y When 'a' litres are removed,it should be 140x-ax.
1 : 16 : 12 When 'a' litres from B are added,it is 140x-ax+ay
Out of 87X Nikhil got. , similarly for B is 60y-ay+ax.
16
29
× 87X = 48X. Now, the overall litres of both mixtures do not
Total money Nikhil got is 53X change as ‘a’ litres are taken away and are
Hence ATQ, replaced, So price per litre after mixing for :-
53X – 47X = 6000 ⇒ X = 1000 1. Solution A :- (140x-ax+ay)/140
∴ 100X = 100000/- Which is total profit. 2. Solution B :- (60y-ay+ax)/60.
27. (b);ATQ, Volume of B is 20 liters Equating them,
1 8400(x − y) = 200a(x − y)
Milk (vessel B)= × 20 = 5 liters.
4 Or, a = 8400/200
3
Water (vessel B) = 4 × 20 = 15 liters. Or, a = 42 litres
When this is added to vessel A.
3x+5 1
30. (b);Let cost price per kg of Type B wheat = x Rs.
= ⇒ 6x + 10 = 5x + 15
5x+15 2 And, Cost price per kg of type B wheat
x= 5 liters = 10x Rs.
Hence, milk in vessel A = 3x + 5 = 3 × 5 + 5 Selling price of mixture of Type A & Type B wheat
= 20 liters = 11x Rs.
Water in vessal A = 5x + 15 = 5 × 5 + 15 Cost price per kg of mixture of Type A & Type B
= 40 liters 32
60 wheat = 11x × = 6.4x Rs.
55
60% of mixture of A = 100 × (40 + 20)
ATQ –
= 36 liters.
Let N kg of type B wheat in mixture of 200 kg of
Remaining mixture = 24 liters.
Type A & Type B of wheat
Milk remaining = 8 liters.
Water remaining = 16 liters. 6.4x × 200 = N × x + 10x (200 − N)
After adding the content of Vessel C. 1280 = N + 2000 – 10N
8+7
=
3 12
⇒ 60 = 48 + 3x ⇒ x = 3 = 4 9N = 920 ⇒ N = 80 kg
16+x 4 80 2
Required ratio = 120 = 3

19 Adda247 Publications For More Study Material


Visit: adda247.com
Cracker Book for Bank (IBPS | SBI | RRB PO | Clerk) Mains Exams

31. (c); Farmer A has ratio of urea & Potassium in mixture ATQ—
10
=5:3 Q = 5 × 3x + 8 × 2x + 3 × 1.5x
Farmer B has ratio of urea & Potassium in mixture Q = 15x + 16x + 5x
=3:2 Q = 36x liter
Let farmer C have x kg of urea & x kg of potassium 16x 4
Required percentage = × 100 = 44 %
Farmer C has Urea in resulting mixture 36x 9
40 5 50 3
= 120 × × + 80 × × +x 34. (b);Ramesh has = (x + 18) kg
100 8 100 5
= (54 + x ) kg Suresh = (x + 18) + 12 = (x + 30) kg
Farmer C has Potassium in resulting mixture = Vikash = (x + 30) + 3 = (x + 33) kg
40 3 50 2
120 × × + 80 × × +x For Ramesh,
100 8 100 5
= (34 + x) kg 5
First type of Apple = (x + 18) × kg
8
ATQ – 3
(54 + X ) 7
= 2nd type of Apple = (x + 18) × 18 kg
(34
+ x) 5
270 + 5x = 238 + 7x ATQ,
5 5 5
2x = 32 ⇒ x = 16 kg (x+18)× −16×
8 8
=6 ⇒
5 (x+18)× −10
8
=6
5
3 3 3
initial quantity of mixture farmer C had = 16 (x+18)× −16× +12
8 8
(x+18)× +6
8
× 2 = 32 kg ⇒ (x + 18) ×
30
− 60 = (x + 18) ×
15
+ 30
8 8
32. (a); Let quantity of vessels A, B and C is 25x, 20x and 15
⇒ (x + 18) × = 90 ⇒ x = 30 kg
18x respectively 8

Now in vessel A → Milk : Water For Vikash,


2 : 3 Total quantity = 63 kg
4
10x milk and 15x water First type = 63 × 7 = 36 kg
In vessel B → Milk : Water 3
3 : 2 Second type = 63 × 7 = 27 kg
12x milk and 8x water ATQ,
In vessel C → Milk : Water 36 1
= ⇒ y = 9 kg
27+y 1
4 : 5
8x milk and 10x water 35. (d);Let vessel A has 5x L and 4x L of milk and water
ATQ→ After first process ratio of wine, milk and water in
33x– 30x = 6 ⇒ 3x = 6 ⇒ x = 2 vessel B will be
Quantity in A, B and C 5 4
= Y ∶ 18 × 9 ∶ 18 × 9 = Y : 10 : 8
50 liter, 40 liter and 36 liter
Now 10 litre of mixture from B is poured in A
33. (b);Let volume of largest vessel = 3x liter y
Then, wine will be = y+10+8 × 10
2
Volume of second largest vessel = 3x × 3 y
× 10 = 4
= 2x liter y+18
75
Volume of smallest vessel = 2x × 10y = 4y + 72 ⇒ 6y = 72
100
= 1.5x liter y = 12 litre

20 Adda247 Publications For More Study Material


Visit: adda247.com
Cracker Book for Bank (IBPS | SBI | RRB PO | Clerk) Mains Exams

Chapter

3 Partnership

BEST APPROACH TO SOLVE THE QUESTIONS

Any partnership in business is based on two components. One is the money or capital that is invested and the other is
the time for which the money is invested.
Mathematically,
Profit ∝ investment
Profit ∝ time
Profit ∝ investment × time
Profit share of an individual is directly proportional to investment and time.
Hence, any profit share of a person out of total profit is decided by the net effect of these two components.
If ratio of investment by two persons is X:Y and ratio of time for which they invested is A:B respectively, then their profit
share is in the ratio of 𝑋 × 𝐴 ∶ 𝑌 × 𝐵= XA : YB

Example 1: A, B and C entered into a business by investing their intial sum of Rs. 12600, Rs. 14400 and Rs. 13200
respectively. After 6 months C left the partnership and B also left the partnership after 8 months. If after a
year total profit was Rs. 69600, then find profit share of C.’
Solution. Here, instead of taking absolute values of money invested, we may use their simplest values i.e. ratio.
Ratio of investment of A, B and C is 21:24:22 respectively. And ratio of time for which they invested is 12:
8:6=6:4:3
Ratio of their profit share = (21× 6): (24 × 4): (22 × 3)= 21:16:11
11
Now, profit share of C= × 69600 = 15950.
48

Example: ‘Aman started a business investing Rs. 70,000. Rakhi Joined him after six months with an amount of Rs.
1,05,000 and Sagar Joined them with Rs. 1.4 lakhs after another six months. The amount of profit earned
should be distributed in what ratio among Aman, Rakhi and Sagar respectively, 3 years after Aman started
the business?’
Solution. Ratio of profit share is given by the ratio of (investment×time)
Or ratio of their investment is 70000 : 105000 : 140000 = 2 : 3 : 4
Ratio of the time for which they invested = 36 : 30 : 24 = 6 : 5 : 4.
Hence, ratio of profit share among them = (2× 6): (3 × 5) ∶ (4 × 4)
= 12:15:16

Practice Exercise Based on New Pattern


1. A and B started a business with a capital of Rs. 32,000 2. If ratio between A’s share in profit to B’s share in
and Rs. 56,000. C join the business on a condition that profit is 5 : 6 and difference between C’s share in
they all will share the profit equally. For the loss of A profit to B’s share in profit is 2340, then find the total
and B, C gives Rs. 19800 to A and B to compensate profit earned by all three together? (2 Marks)
their loss. How much amount A get out of Rs. 19800? (a) 7700 (b) 5390 (c) 6160
(a) 1200 (b) 1600 (c) 1800
(d) 6930 (e) Cannot be determined
(d) Data inadequate (e) None of these
Direction (2-3): A, B and C start a business. Ratio 3. Find the value of ‘x’. If out of total profit of Rs 2460,
between investment of A and B is 5 : x while ratio A’s share in profit is Rs 600. (1 Mark)
between investment of B and C is 9 : 11. A, B and C (a) 8 (b) 7 (c) 6
invested for 8 month, 6 month and 12 month respectively. (d) 5 (e) 4

21 Adda247 Publications For More Study Material


Visit: adda247.com
Cracker Book for Bank (IBPS | SBI | RRB PO | Clerk) Mains Exams
4. Rahul and Rohit start a business by investing Rs.4900 10. Three partners A, B and C invested their amounts in
and Rs.3600 respectively. After 4 months, Rahul and ratio of 2 : 5 : 8. At the end of four months, A invests
2 1
Rohit withdraw 7th and 3 of their initial investment some amount such that, his total investment will be
and after 6 months of starting of business they equal to C’s initial investment and C withdraw some
invested 60% of what they withdraw. Find the share amount that its total investment will be equal to B’s
of Rohit in total profit of Rs. 8490 initial investment. If 10% of B’s share in profit of one
(a) Rs.3555 (b) Rs.4935 (c) Rs.1855 year is Rs 2550. Then share of A and B in total annual
(d) Rs.4550 (e) Rs.4850 profit will be?
(a) 81500 (b) 61200 (c) 56100
5. ‘P’ and ‘Q’ entered into a partnership with 50,000 and (d) 75000 (e) None of these
60,000 for 8 and 6 months respectively. ‘P’ buy a car
from its whole profit and ‘Q’ invested his whole profit 11. A and B enter into a partnership business and decided
in a scheme which offer 10% interest on S.I. P’s car to distribute 60% of profit as per (investment× 𝑡𝑖𝑚𝑒)
1
value depreciated 11 % every year. If after 2-year ratio and rest in the ratio of 7:10. If the amount
9 invested by A is 25 % more than B and B withdraw
difference between their money (excluding initial half of his total investment after 9 months. Find the
investment) is 23,480 then find P’s profit. profit distribution ratio of A and B according to given
(a) Rs. 72000 (b) Rs. 35000 (c) Rs. 64000 condition at the end of a year.
(d) Rs. 54000 (e) Rs. 81000 (a) 43:41 (b) 44:41 (c) 48:43
6. A and B started a business in partnership by investing (d) 49:47 (e) None of these
in the ratio of 7 : 9 . After 3 months A withdraw 2/3 of Directions (12-13): Three partners Abhishek, Neeraj and
its investment and after 4 months from the beginning
1 Aniket enter into a partnership business with the capitals
B withdraw 33 3 % of its investment. If a total earned in the ratio of 3 : 5 : 8. After four month Abhishek, Neeraj
profit is Rs. 10201 at the end of 9 months, find the and Aniket added Rs. 6000, Rs. 9000 and Rs. 12000
share of each in profit. respectively, after next four months Abhishek and Aniket
(a) Rs. 3535, Rs.6666 (b) Rs. 3055, Rs. 5555 withdraw Rs. 5000 and Rs. 8000 respectively and Neeraj
(c) Rs. 4503, Rs. 1345 (d) Rs. 3545, Rs. 3333 added additional Rs. 6000.
(e) None of these
12. Two persons Ankur and Saurabh enter a business.
7. Satish and Saurbh entered into a partnership business Ankur invested capital 6000 more than what
for eight months. Satish invested Rs. 40x and Saurbh Abhishek invested for first four month and Saurabh
invested 40% more than Satish, after four months invested what Aniket invested for last four months. If
from starting of business Rituraj joined the both with Ankur and Saurabh got profit share after one year in
the equal capital of Satish and Saurbh invested the ratio of 15 :28 then find theinvestment of Neeraj
together initially. If they got a total profit of Rs. 7200, for last four months?(1 marks)
then find profit share of Rituraj? (a) 24000 Rs. (b) 18000 Rs. (c) 16000 Rs.
(a) 1600 Rs. (b) 1800 Rs. (c) 2000 Rs. (d) 30000 Rs. (e) 36000 Rs.
(d) 2200 Rs. (e) 2400 Rs.
13. Amir and Mayank started a business in partnership.
8. Veer and Sameer enter into a business by making the Amir investmentfor six months isRs. 6000 more
investment in the ratio of 3 : 4. After six months Veer thanwhat Neeraj invested for first fourmonthsand
added Rs. 2000 more in his initial investment, while Mayank invested for eight months, Rs. 2000 lessthan
Sameer withdraw Rs. 4000 from his initial whatAniket invested for first four monthsIf Mayank
investment. If at the end of one year Veer and Sameer got profit share of Rs. 15400, out of total profit Rs.
shared profit in the ratio of 7 : 9, then find initial 22750 thenfind the investment of Amir is how much
investment of both? less than investment of Mayank? (2 marks)
(a) 69000 Rs. 92000 Rs (a) 218000 Rs (b) 216000 Rs. (c) 210000 Rs.
(b) 36000 Rs. 48000 Rs. (d) 220000 Rs. (e) 232000 Rs.
(c) 27000 Rs. 36000 Rs.
(d) 45000 Rs. 60000 Rs. 14. Veer, Sameer and Gopal enter into a business by
(e) 24000 Rs. 32000 Rs. making investment in the ratio of 5 : 6 : 7 respectively.
1
9. P and R entered into partnership business with the After six months Veer and Sameer withdraw 5th and
capital of Rs. x and Rs. (x + 12000), after One year Q 25% of his initial investment respectively. After eight
1
joined them with capital of Rs. (x + 8000) at the end of months from the starting of business Gopal added 𝑡ℎ
7
2nd year P and Q with draw their capital and R invest
of his initial investment. If at the end of one year
for one more year, if P, Q and R gets profit in the ratio
profit share of Gopal is Rs. 2000 more than Sameer,
of 8 : 6 : 21 respectively. Find sum of capital invested
then find profit share of Veer?
by all three?
(a) 66000 Rs. (b) 54000 Rs. (c) 64000 Rs (a) 4420 Rs. (b) 4520 Rs. (c) 4820 Rs.
(d) 68000 Rs. (e) 70000 Rs. (d) 4320 Rs. (e) 4720 Rs.

22 Adda247 Publications For More Study Material


Visit: adda247.com
Cracker Book for Bank (IBPS | SBI | RRB PO | Clerk) Mains Exams

15. Manish and Rituraj invested Rs. 12000 and Rs. 16000 19. A, B and C enter into a partnership and invested some
in a business. After four months Manish and Rituraj amount. After one year A double its investment, B
both added Rs. 4000 in their initial investment. At the 1
increase its investment by 33 % and C increase its
end of one year the total profit was Rs. 172500, if 3

Manish and Rituraj invested their profit share on investment by 20%. In the third year A and B
compound interest at the rate of 20% and 10% withdraw their investments and D joins the
respectively then find difference between interests partnership with C. After three year they got profit in
got by both at the end of two years? the ratio of 12 : 14 : 17 : 8 (A : B : C : D). If difference
(a) Rs.10250 (b) Rs.11520 (c) Rs.12210 between initial investment of B and C is 1150. Then
(d) Rs.13110 (e) Rs.12660 Find out the total initial investment made by A and D
Direction (16 – 17): Data given below about investment together?
of three partners in a business. Read the data carefully (a) 12100 (b) 14400 (c) 13800
and answer the questions: (d) 15000 (e) None of these
Swati, Monika and Anshika three business partners enter 5
into a business by making investment Rs. 4000, Rs. 6000 20. Three partners A, B and C invested in the ratio of 4

4 6
and Rs. 8000 respectively. After first quarter Swati, ∶ 5 in a business. After 3 months A increased his
5
Monika and Anshika added some amount in the ratio of 2 : capital by 40%, after 3 months more A again
3 : 5. After second quarter Swati, Monika and Anshika
2 1 3 increased his capital by 20%. B increased his capital
withdrew 𝑡ℎ , 𝑟𝑑 and 𝑡ℎ respectively of capital by 25% after every 4 months. C increased his capital
5 3 7
invested in second quarter. At the end of nine month by 25% after 3 months and after 4 months more C
profit share divided among Swati, Monika and Anshika in increased his capital by 20%. If the total profit of Rs.
the ratio of 24 : 37 : 49 respectively. 30,130 is earned at the end of the year, find what was
16. Satish and Veer enter into a business, Satish invested the A’s share of profit?
200% more than Swati capital for second quarter, (a) Rs. 12,420 (b) Rs. 14,420 (c) Rs. 13,240
while Veer invested 300% more than Anshika capital (d) Rs. 15,340 (e) Rs. 12,340
for third quarter. And profit share of Satish at end of
two year is Rs. 7250 out of total profit of Rs. 15950. 21. A, B and C entered into a partnership business, A
Then find for how many months Veer invested his invested Rs. x, B invested 25% more than A and C
capital? invested 20% more than B for first 6 months. After
(a) 14 months (b) 12 months (c) 18 months that A left the business and B withdraw 50% of his
(d) 10 months (e) 8 months investment. B left the business after 3 more months
2
17. Monika and Anshika enter into a business with whereas C increased his investment by 16 % . If at
3
another partner Shikha. Monika invested 144% of the end of year difference between profit share of C
what she invested in second quarter, while Anshika and (A + B) together is Rs. 12375, then find profit
invested 160% of what she invested in second share of C ?
quarter. If all three invested their capital for a year (a) 58500 Rs. (b) 56500 Rs. (c) 55680 Rs
and Shikha gets a profit of Rs. 9600 out of total profit
(d) 55580 Rs. (e) 52680 Rs.
of Rs. 28000, then find investment of Shikha?
(a) 12,800 Rs. (b) 11,600 Rs. (c) 14,400 Rs. 22. Veer, Sameer and Satish entered into a business by
(d) 14,200 Rs. (e) 13,600 Rs. investing equal amount each. After four months
18. Satish, Veer and Neeraj entered into a business by Sameer added half of his initial investment, while
investing in the ratio of 4 : 6 : 3. After two years, Veer and Satish withdrew half of their initial
1
Satish and Veer withdrew 25% and 33 3 % of their investment. At end of one year the total profit is Rs.
8000 and Veer and Sameer invested their profit share
respective initial investment, while Neeraj added 50%
of his initial investment more. If at the end of three at the rate of 10% compounded annually for some
years they received total profit of Rs. 93750, out of time in a scheme. If the difference of the amount
which 22.5% are used for maintaining business and received by Veer and Sameer from the scheme at the
the remaining profit was divided among them end is Rs. 2420, find for how much time Veer and
according to their investment ratio. Find difference Sameer invested in the scheme?
1
between profit share of Satish and Neeraj? (a) 4 yr (b) 3 yr (c) 3 yr
2
(a) 978.75 Rs. (b) 976.75 Rs. (c) 974.25 Rs.
(d) 2 yr (e) 1yr
(d) 968.75 Rs. (e) 966.75 Rs.

23 Adda247 Publications For More Study Material


Visit: adda247.com
Cracker Book for Bank (IBPS | SBI | RRB PO | Clerk) Mains Exams

23. P, Q and R entered into a partnership with their initial investment and Divyaraj added Rs. 1800 respectively.
capital in the ratio of 3 : 5 : 8 respectively. After third If after one year profit share of Veer is Rs. 10500 out
1
month P withdrew rd of his initial capital and Q of total profit of Rs. 20850, then find difference
3
between initial investment of Veer and Sameer?
withdraw half of his initial capital. After next six
(a) 6400 Rs. (b) 7200 Rs. (c) 7800 Rs.
months P increased his capital by 100% more, Q
increased by 300% more and R increased by 25% (d) 5600 Rs. (e) 8400 Rs.
more. If at the end of an year, difference between 25. A, B & C, three friends started the business in which A
profit share of Q and R was Rs. 7000, then find profit invested for 6 months, B initially didn’t invest &
share of P? started as working partner, while C invested for 4
(a) Rs. 6600 (b) Rs. 7700 (c) Rs. 5500 months. They decided to donate 1% of total profit and
(d) Rs. 5750 (e) Rs. 5950 to give 22% of total profit to B as salary. Find the ratio
of their profit shares (A:B:C) if A and C invested in the
24. Veer, Sameer and Divyaraj enter into a business by
1 1 1 ratio of 2 : 3 and B also invested an amount which is
making an investment in the ratio of 129 ∶ 301 ∶ 602 80% of total amount invested by A and C together for
respectively. After six months from starting of a single month.
1
business Veer withdrew 33 3 % of his initial (a) 1 : 3 : 1 (b) 1 : 1 : 3 (c) 3 : 1 : 3
investment, while Sameer added 50% of his initial (d) 1 : 1 : 1 (e) None of these

Solutions

A B ⇒ 246 = 60 + 31𝑥
1. (c); Ratio of share of profit = 32 : 56 186
⇒ 𝑥 = 31 = 6
= 4 : 7 = 11
Let, total profit = Rs. 11 4. (a); Investment of Rahul
Now, A, B and C will distribute Rs. 11 equally For first four month = 4900
i.e. For fifth and sixth month =
5
× 4900 = 3500
A : B : C 7
3 2
11 11 11
For Last 6 month = 3500 + × × 4900
5 7
3
: 3
: 3 = 4340 Rs.
11 1
Loss of A = 4 – 3
=3 Investment of Rohit
Loss of B = 7 – =
11 10 For first 4 month = 3600 Rs.
3 3 2
Next 2 month = 3 × 3600 = 2400 Rs.
Ratio of Loss = 1 : 10
3 1
Amount that got to compensate the loss Last 6 month = 2400 + 5 × 3 × 5600 = 3120
1
= × 19800 = 1800 Profit ratio
11
Rahul Rohit
Solution (2-3): Ratio between A’s, B and C’s profit share 4900 × 4 + 3500 × 2 + 4340 × 6 : 3600 × 4 + 2400 × 2 + 3120 × 6
329 : 237
= 45 × 8 : 9x × 6 : 11x × 12
Total profit = 8490 Rs.
= 60 : 9x : 22x
Share of Rohit out of total profit
8490
2. (d);Let profit sharing of A, B and C be 60z, 9xz and = 566 × 237 = 3555 Rs.
22xz respectively.
ATQ, 5. (e); Profit ratio of P and Q
22xz – 9xz = 2340 50,000 × 8 : 60,000 × 6
⇒ 13xz =2340 10 : 9
⇒ xz = 180 Let P’s and Q’s profit → 10𝑥 and 9𝑥
B’s and C’s share in profit is 9 × 180 and 22 × 180 Value of P’s money after 2 years
8 8 640𝑥
respectively. = 10𝑥 × 9 × 9 = 81
9×180
A’s share in profit = 6 × 5 = 1350 Value of Q’s money after 2 years
9𝑥×2×10 120 108
Total profit earned by all three together =9𝑥 + = 9𝑥 × = 𝑥
100 100 10
= 1350 + 1620 + 3960 = 6930 108𝑥 640𝑥
Difference = 10 – 81 = 23,480
3. (c); ATQ, 𝑥 = 8100
60 600
= P’s profit = Rs. 81,000
60+9𝑥+22𝑥 2460

24 Adda247 Publications For More Study Material


Visit: adda247.com
Cracker Book for Bank (IBPS | SBI | RRB PO | Clerk) Mains Exams

6. (a); ATQ—
A : B 2𝑥 8
=6
(𝑥+8000)
Capital → 7×3 : 9×3 6x – 4x = 32000
New Ratio → A : B x = 16000
Required sum of capital (P + Q + R)
21x : 27x = 16000 + (16000 + 8000) + (16000 + 12000)
Total capital invested by A in 9 months = 68000 Rs.
= 21x × 3 + 7x × 6 = 105x
Total capital of B invested in 9 months 10. (c); Let amount invested by A, B and C are 2x, 5x and
= 27x × 4 + 18x × 5 8x respectively.
= 108x + 90x = 198x ⇒ At the end of 4th month A’s investment 2x + 6x
A : B = 8x = c’s initial invested
Capital → 105x : 198x ⇒ C’s investment = 8x – 3x = 5x = B’s initial
According to question, investment
(105x + 198x) = Rs. 10201 Ratio of investment
303x = 10201 A : B : C
10201 = (2x × 4 + 8x × 8) : (5x × 12) : (8x × 4 + 5x × 8)
x = Rs. 303 6 : 5 : 6
Hence, Let B’s profit = 5y
Share of A =
10201
× 105 = Rs. 3535 Now ATQ
303 5𝑦×10
10201 = = 2550
Share of B = 198 × 303
= Rs. 6666 100
y = 5100
7. (e); Satish capital = 40x Rs. share of A and B = 5y + 6y = 11y
40 = 11 × 5100 = 56100
Saurbh capital = 40x + 40x ×
100
= 56x Rs. 11. (b);Let the total investment of B = 100
Rituraj capital = 40x + 56x = 96x Rs. Investment of A = 100×125/100
Ratio of profit (Satish : Saurbh : Rituraj) Ratio of 60% profit
= 8 × 40x : 8 × 56x : 96x × 4 = 125×12 : 100×9+50×3= 10:7
= 320x : 448x : 384x Let the total profit = 170
=5:7:6 Profit for A for only 60 % of profit
6 = 170×0.6×(10/17)=60
Rituraj profit share = 7200 × (5+7+6)
Profit of B for only 60 % of profit
= 2400 Rs.
= 102-60 = 42
8. (a); Lets Veer and Sameer invested Rs. 3x and Rs. 4x Total profit of A = 60+ 68×7/17= 88
respectively. Total Profit of B = 42+68×10/17 = 82
Ratio of investment of Veer and Sameer Required ratio = 88:82 = 44:41
= [3x × 6 + (3x + 2000) × 6] : [4x × 6 + (4x – 4000)
Solutions (12-13): Let investment of Abhishek, Neeraj
× 6]
and Ankit is Rs. 3x, Rs. 5x and Rs. 8x respectively.
= (36x + 12000) : (48x – 24000)
ATQ—
ATQ—
36𝑥+12000 7 Investment ratio of Abhishek, Neeraj and Aniket
=9
48𝑥 –24000 = [3x × 4 + (3x + 6000) × 4 + (3x + 6000 – 5000)
36𝑥 × 9 + 12000 × 9 = 7 × 48𝑥 − 24000 × 7 × 4] :
12000(9 + 14) = 12(7 × 4𝑥 − 3 × 9𝑥) [5x × 4 + (5x + 9000) × 4 + (5x + 9000 + 6000) × 4] :
1000 × 23 = 28𝑥 − 27𝑥 [8x × 4 + (8x + 12000) × 4 + (8x + 12000 – 8000)
x = 23000 Rs. × 4]
Investment of Veer = 3 × 23000 = 69000 Rs. = (36x + 28000) : (60x + 96000) : (96x + 64000)
Investment of Sameer = 4 × 23000
= 92000 Rs. 12. (d);Investment of Ankur = Investment of Abhishek for
first four months and next four months
9. (d);P : Q : R = (3x + 6000) Rs.
Rs. x × 2 : Rs. (x + 8000) × 1 : Rs. (x + 12000)
Investment of Saurabh = Investment of Aniket for
×2
last four months
= 8 : 6 : 21
= (8x + 12000 – 8000) Rs.
25 Adda247 Publications For More Study Material
Visit: adda247.com
Cracker Book for Bank (IBPS | SBI | RRB PO | Clerk) Mains Exams

ATQ— Solutions (16-17): Let total amount added by Sawti and


(3𝑥+6000)×12 15 Monika after first quarter be Rs. 2x and Rs. 3x
(8𝑥+4000)×12
=
28 3
4000 ×3+(4000+2𝑥)×3+(4000+2𝑥)× ×3 24
120x - 84x = 168000 – 60000 5
2 = 37
6000×3+(6000+3𝑥)×3+(6000+3𝑥)× ×3
36x = 108000 1
3
12000+12000+6𝑥+(36000+18𝑥)× 24
x = 3000 Rs. 5
= 37
18000+18000+9𝑥+12000+6𝑥
Neeraj invested for last four months 156000+48𝑥 24
= (5x + 9000 + 6000) Rs. 240000+75𝑥
= 37
= (5×3000 + 9000 + 6000) 240500 + 74x = 240000 + 75x
= 30000 Rs x = 500 Rs.
13. (e); Investment of Amir = Investment of Neeraj for 16. (c); Total amount invested by Sawti in second quarter
first four months +6000 = (4000 + 2× 500) = 5000 Rs.
= 5x + 6000 Total amount invested by Anshika in third quarter
Investment of Mayank = Investment of Aniket for = (8000 + 500 × 5 − 4500)
first four months – 2000 = 6000 Rs.
300
= 8x -2000 Satish investment = 5000 × 100
(5x+6000)×6 22750 –15400
(8x−2000)×8
= 15400
= 15000 𝑅𝑠.
400
(5𝑥+6000)
=
7 Veer investment = 6000 × 100 = 24000 𝑅𝑠.
(8𝑥−2000) 11
Let Veer invested for T months
56x – 55x = 80000
ATQ –
x = 80000 15000 ×24 7250
required difference = 8x-2000-5x-6000 24000×𝑇
= (15950−7250)
3x – 8000 120
= 174
145
240000 – 8000= 232000 8×𝑇
T = 18 months
14. (d);Let Veer, Sameer and Gopal invested Rs. 5x, Rs. 6x
17. (c); Total amount invested by Monika in second
and Rs. 7x respectively
quarter = 6000 + 3 × 500 = 7500 𝑅𝑠.
Ratio between profit share of Veer, Sameer and 144
Gopal Monika investment = 7500 ×
100
4 3 = 10800 𝑅𝑠.
= (5𝑥 × 6 + 5𝑥 × × 6) : (6𝑥 × 6 + 6𝑥 × ×
5 4
8 Total amount invested by Anshika in second
6) : (7𝑥 × 8 + 7𝑥 × × 4) quarter = 8000 + 5 × 500 = 10500 𝑅𝑠.
7
160
= 54𝑥 ∶ 63𝑥 ∶ 88𝑥 Anshika investment = 10500 ×
100
= 54 : 63 : 88 = 16800 𝑅𝑠.
2000
Profit share of Veer = 54 × (88 –63) Let Shikha invested = P Rs.
= 4320 Rs. ATQ –
𝑃×12 9600
= (28000−9600)
15. (d);Ratio of profit of Manish and Rituraj (10800+16800)×12
𝑃 9600
= [(12000 × 4)+ (12000 + 4000) × 8] : [(16000 × =
27600 18400
4) + (16000 + 4000)× 8] P = 14,400 Rs.
= 17600 : 224000 = 11 : 14
Profit share of manish = 172500 × 25
11 18. (d); Let the amount invested by Satish, Veer and
Neeraj initially be 4x, 6x and 3x respectively.
= Rs.75900 Ratio of profit share of Satish, Veer and Neeraj
14
Profit share of Rituraj = 172500 × 25 3
= 4x × 24 + 4x × 4 × 12) ∶ (6x × 24 +
= 96600 Rs 2 3
Equivalent CI of two year at the rate of 20% = 6x × × 12) : (3x × 24 + 3x × × 12)
3 2
20×20
20 + 20 + = 44% = 22 : 32 : 21
100
Equivalent CI of two year at 10% Total remaining profit after excluding the amount
10×10 spend on maintaining business
= 10 + 10 + = 21% (100–225)
100 = 93750 × = 72656.25 Rs.
Required difference between interest 100
(22–21)
= 75900 ×
44
− 96600 ×
21 Required difference = 72656.25 ×
100 100 75
= 13110 = 968.75 Rs.

26 Adda247 Publications For More Study Material


Visit: adda247.com
Cracker Book for Bank (IBPS | SBI | RRB PO | Clerk) Mains Exams
n
19. (c); Let investment of A, B, C and D is a, b, c and d (1 +
10
) = 1.21
respectively. 100
A B C 𝐷 n = 2 years
23. (c); Lets P, Q and Q investment be 3x, 5x and 8x profit
Now in firt year → a × 12 : b × 12 : c × 12 share of P : Q : R
= (3x × 3 + 2x × 6 + 4x × 3) : (5x × 3 + 2.5x × 6 +
4b 6c
In 2nd year → 2a × 12 : × 12 : × 12 10x × 3) : (8x × 9 + 10x × 3)
3 5
= 33x : 60x : 102x
6c
In 3rd year × 12 : 𝑑 × 12 = 11 : 20 : 34
5
4 ATQ—
A : B : C : D ⇒ (a × 12 + 2a × 12) : (b × 12 + b ×
3 34 – 20 => 7000
6
12) : c × 12 + 2 c × 12 : d × 12 14 => 7000
5
7𝑏 17 1 => 500
3a : : 𝑐 : d = 12 : 14 : 17 : 8
3 5 Profit share of P = 11 × 500
⇒a:b:c:d=4:6:5:8 = Rs. 5500
Difference between B and C initial investment =
1150 24. (b);Initial investment of Veer, Sameer and Divyaraj =
1806 1806 1806
Total Investment of A and D together 129
∶ 301 ∶ 602
1150
= 1 × 12 = 13800 = 14 : 6 : 3
Let Veer, Sameer and Divyaraj initial investment
20. (a); Ratio of investment of A, B and C be 14x Rs., 6x Rs. & 3x Rs. respectivelyProfit ratio
5 4 6
= 4 ∶ 5 ∶ 5 = 25 ∶ 16 ∶ 24 of Veer, Sameer and Divyaraj
2
Ratio of profit after one year = (14x × 6 + 14𝑥 × × 6 ) : (6x × 6 + 9𝑥 × 6) ∶
3
A : B : C
[3𝑥 × 6 + (3𝑥 + 1800) × 6]
25  3 + 35  3 + 42  6 : 16  4 + 20  4 + 25  4 : 24  3 + 30  4 + 36  5
= (84x + 56x) : (36x + 54x) : (18x + 18x + 10800)
= 432 : 244 : 372
ATQ –
= 108 : 61 : 93 140𝑥 10500
30130 =
A’s share in profit = 262 × 108 = 12,420 90𝑥+36𝑥+10800
𝑥 1
20850−10500

(21𝑥+1800)
=
23
21. (a); Let investment of A, B and C
2x = 1800
= x : 1.25x : 1.5x
x = 900 Rs.
= 4x : 5x : 6x
Required difference = 14× 900 − 6 × 900
ATQ—
= 7200 Rs.
Profit share of A : B : C = 4x × 6 : (5x × 6 + 2.5x ×
3) : (6x × 6 +7x × 6) 25. (d);Let A invested 2x and C invested 3x.
= 24x : 37.5x : 78x 80
Then B invested = 100 × (3x + 2x)
Given→ C – (A + B) = 12375 80
78x – (24x + 37.5x) = 12375 = 100 × 5x = 4x
x = 750 Let 100Y be their total profit.
Profit share of C = 750 × 78 = 58500 Rs. According to question,
1% is given as donation and B gets 22% as salary
22. (d);Let Veer, Sameer and Satish invested Rs. x initially
of total profits.
Ratio of profit of Veer, Sameer and Satish Now ratio of their shares is
x 3x x
=(x × 4 + 2 × 8) : (x × 4 + 2 × 8) : (x × 4 + 2 × 8) 2x × 6 : 4x × 1 : 3x × 4
= 8x ∶ 16x ∶ 8x 12x : 4x : 12x
=1:2:1 3 : 1 : 3
3
1
Profit share of Veer = 8000 × = 2000 Rs. A gets × [100Y– 23Y]
7
4 3
Profit share of Sameer = × 77Y = 33Y
7
2 3
= 8000 × 4 = 4000 Rs. C gets = × 77Y = 33Y
7
Let both invested for n years 1
B gets = × 77Y + 22Y = 33Y
10 n 10 n 7
4000 (1 + 100) – 2000 (1 + 100) = 2420 Hence, ratio of their profit shares is
n
2000 (1 + 100) = 2420
10 33Y : 33Y : 33Y
=1:1:1

27 Adda247 Publications For More Study Material


Visit: adda247.com
Cracker Book for Bank (IBPS | SBI | RRB PO | Clerk) Mains Exams

Chapter

4 Average and Ages


BEST APPROACH TO SOLVE THE QUESTIONS

INTRODUCTION:
We all know that definition of average is sum of magnitude (weight, age, marks or any measurable quantity) of all
quantities divided by total number of all quantities. Always remember, if you are calculating average you are
making all the quantities equal.
For eg. three brothers have number of candies with them is 9, 11 and 16. But their mom took all the candies and
distribute 12 to each. Average in some questions referred as ‘Arithmetic mean’.
Questions from this topic are extremally easy and doesn’t require as much time or sharp concept, mostly questions are
asked in the form of average of ages.
Example: Marks obtained by seven students are 70, 85, 95, 86, 74, 83 and 67. What is average marks obtained by
them.
Solution: Sum of magnitude of all the observations is 70 + 85 + 95 + 86 + 74 + 83 + 67 = 560.
560
Required average = 7 = 80.

NOTE: Please avoid short cut tricks mentioned in many books for such small calculations. Those tricks are more
time consumable. There is other very simple and only trick that you must apply. Look at next question.
Example: Average age of 4 members of a family is 28 years and average increases by 2 years when one of their relative
joins them. Calculate the age of relative.
Solution: Easy one! Let age of relative is x years. Total age of family before joining the relative = 4×28=112
112+x
After joining the relative = 28+2, which gives us x = 38 years.
5
Right approach? Or wrong?
Try this approach, Imagine, they all have 28 rupees/candies/marbles before their relative joins them. When
he comes, they all have Rs 30 each. What does it mean? It means their relative gave Rs 2 to all 4 of them and
even then, he was left with Rs 30, as average means amount is distributed equally. How much he had given
to them? Rs. 8, Right? Therefore, he initially had total of Rs 38 with him.
Why to waste paper and time if we can do it mentally. Try next question.
Example: Average marks scored by a student in 4 subjects is 75. But when marks of English are added to it, overall
average became 70. How much did he score in English?
Solution: Let 4 men have 75 gold coins each and when 5th one joins them, each of those 4 men/exams gave 5
coins/marks to 5th one. And after taking 5 coins from 4 persons, 5th have total 70 coins. It means he initially
had 50 coins and that the answer.
Example: Average marks scored by a student in 5 subjects was 79. But when average is calculated for 4 subjects
(without Hindi) it was 4 lesser than the average of 5 subjects. What was the marks scored by him in Hindi?
Solution: Again let 5 men have 79 candies each and when one of them leave, he took all candies belonging to him. It
means each of these 4 had borrowed 4 candies from him. He took 79 candies that was with him, plus these 4
persons returned 16 of his. Therefore, he had 95 candies or marks whatever you assume.
Example: Average run scored by Sachin in 19 innings of year 2007 is 49 runs while total run scored run scored by
Afridi in 19 matches were 760. In 20th match, average of Afridi decreased by 2 runs/ match and average
run/match increased by 1 for Sachin. What is the difference b/w the scored runs in 20th match by them?

28 Adda247 Publications For More Study Material


Visit: adda247.com
Cracker Book for Bank (IBPS | SBI | RRB PO | Clerk) Mains Exams
760
Solution: Average runs by Afridi in 19 innings 19 = 40 runs.
His average decreased by 2, which means all the 19 innings/persons have given 2 runs/coins to 20 th
inning/person. New inning/person borrowed 19 × 2 = 38 runs/coins to become equal with others.
Therefore, he himself have no money and we can conclude Afridi scored 0 in that match.
While for Sachin, his 19 previous innings got 1 run each and even then, his 20 th inning remains with 50 runs.
Therefore, total runs by Sachin in 20th inning are 69 runs.
Required difference is 69 runs.
This may look longer when you read it, but once you get the concept you can solve these questions within 30
seconds.
AGES
The questions on ages are of two types either the average age of a group of people is asked or we are given the ratios of
ages of friends and we are asked to calculate their individual age or average age. See it through a question.
Example: Ratio of present age of Rahul and his father is 5:8 and 14 years earlier this ratio was 1:2. What is present age
of Rahul
Solution: 1 way to solve this is assume present age of Rahul and his father 5x and 8x. According to question 14 years
earlier ratio of their age was
5x−14 1
=
8x−14 2
 (5x − 14) × 2 = 8x − 14
 2x = 14
 X=7
 Present age of Rahul = 7× 5= 35 years.
Or
If you want to solve it mentally, or in a shorter time, observe carefully
Present age ratio 5 : 8
14 years earlier 1 : 2
Note that as difference b/w ages of two persons is always same, so what we do? We will make difference
b/w two ratio’s same.
Multiply first ratio by difference of second and second ratio bye difference of first.
First ratio when multiplied by difference of other i.e. (2-1=1) it becomes 5 : 8
When second ratio is multiplied by difference of first i.e. (8 - 5 = 3) we will get
1×3 : 2× 3
3 : 6
Now think mentally, that 14 years ago age of Rahul was 3 units and now it is 5 unit.
Therefore 2 units are equal to 14 years, which means 5 unit (age of Rahul) is 35 years.
NOTE: Concept behind multiplying by difference is that, we must make both differences (b/w the ratios) equal. To
make two numbers equal what we do? We take their LCM as common. If difference b/w two ratios are 4 and
6, we will multiply first ratio by 3 and second ratio by 2 to make their difference equal. Cross check it by
your self for some quantities for better confidence.
Example: Ratio of age of two friends is 7:9 while 10 years later it will be 19:23. Calculate their average age after 7
years from present.
Solution: Difference b/w the ratios are 2 and 4. So to make them equal multiply first ratio by 2. After multiplying we
will get
14 : 18.
And second ratio we have is 19 : 23.
Hence conclude mentally, that 14 units is present age of younger friend and 19 units is his 10 years later age.
It means that
19 – 1 4 =5 units = 10 year
1 unit = 2 years.
And their present average age is (28 + 36)/2 = 32 years.
And 7 years later mean 7 is added to both of their age, which means average age is 32 + 7 = 39 years.

29 Adda247 Publications For More Study Material


Visit: adda247.com
Cracker Book for Bank (IBPS | SBI | RRB PO | Clerk) Mains Exams

 Key Points to Remember for New Pattern Questions


➢ Calculating average means making each quantity equal.
➢ To make difference of two ratios equal, try to think through ‘LCM of difference’ approach.
➢ If the average age of a group of people is X years, Y years later it will be X + Y years.

Practice Exercise Based on new Pattern

1. In a group of 4 friends, ratio of present age of A and D (a) 20 (b) 18 (c) 24


is 4:5 and that B to C is 3:4. Calculate the present (d) 22 (e) 26
average age of A, B and C, if 4 years ago, A was 20
years younger than C and at present C is twice of age 7. Ratio of Ayush Age to his first son’s age is 20 : 9 and
of A. Ayush wife’s age to Ayush 2nd son’s age is 3 : 1. If elder
(a) 20 years (b) 50 years (c) 40 years son of Ayush is 6 year elder than another son (ayush
(d) 25 years (e) 30 years has only two son and no daughter) and average age of
family is 26.5 year then find the age of Ayush’s wife.
2. Ratio of age of Vikas to age of Rohit 2: 3 and average (a) 24 (b) 36 (c) 40
age of Rohit, Vikas and Rahul is 23. When Arjun joins (d) 38 (e) 28
the group average age of these 4 persons is 25 years.
Calculate sum of age Arjun and Rahul, if Rahul is 13 8. Ritu take four, two-digit numbers and take average of
years older than Vikas ? these number which is 52.5. If she reversed all the
(a) 40 years (b) 47 years (c) 60 years digits of 4 numbers she found that their average is
(d) can’t be determined now 22.5 less than the previous one and they formed
(e) None of these A.P having common difference 4. Find the largest
3. S₁ is a series of five consecutive multiple of three, number.
whose sum is 180 and S₂ is the series of four (a) 82 (b) 80 (c) 36
consecutive multiple of four whose second smallest (d) cannot be determined (e) 64
number is 13 more than second highest number of S₁ 9. The average age of a group of six children is 15 years.
series. Find the average of smallest number of S₁ From the group, two children, whose ages were 3
series and highest number of S₂ series. years more and 5 years more than the average age,
(a) 51 (b) 49 (c) 47 left. 4 new children, whose average age is 4 years
(d) 45 (e) 43 more than the given average age, join the group. Find
4. A student finds the average of five two digits the new average age.
numbers. If One number is reversed and the average (a) 15 years (b) 16 years (c) 17 years
is taken again then the average increase by 5.4. If all (d) 18 years (e) 12 years
five digits are consecutive multiple of four, then find 10. If age of P and R are added to twice the age of Q, the
the number which is reversed? total becomes 59. If the ages of Q and R are added to
(a) 58 (b) 36 (c) 74 thrice the age of P, the total become 68. And if the age
(d) 48 (e) None of these
of P is added to thrice the age of Q and thrice age of R,
5. Average weight of a group is 20 kg. When 2 people the total becomes 108, what is the total of ages of P, Q
include in the group average weight becomes 22 kg and R?
after that when 4 another people removed from the (a) 45 years (b) 47 years (c) 49 years
group, having weight half of the weight of 2 person (d) 42 years (e) 44 years
included previously, average weight become 25. Find
the strength of group initially 11. Sweta and Neha profess to tell their present ages as
(a) 18 (b) 20 (c) 22 25 and 20 years respectively. (Not original age). Ratio
(d) 16 (e) 14 of their original ages 5 year ago is 5 : 4. Sum of ages of
400
both 5 years hence is % more than the sum of
6. Four years ago, Ratio of Ram’s to Shyam’s age is 9 : 11 9
and Shyam’s age four years ago is same as Ram’s present ages of both professed by them. Find the ratio
present age. If Rahul’s present age is average of of their present original age.
present age of Ram and Shyam, then find Rahul’s age (a) 4:7 (b) 5:6 (c) 6:5
2 year ago. (d) 4:5 (e) 5:7

30 Adda247 Publications For More Study Material


Visit: adda247.com
Cracker Book for Bank (IBPS | SBI | RRB PO | Clerk) Mains Exams
12. Average age of a man, woman and their son is 30 (a) 20 years (b) 40 years (c) 24 years
years. Man’s age is two year more than his wife and (d) 30 years (e) 34 years
1
age of son is th the sum of age of his mother and 19. ‘2n’ years ago ratio of Amit’s age to Inder’s age is 5 : 4.
4
father.When two other family members were added, ‘n’ years ago ratio of Inder’s age to Satish’s age is 9 : 7.
new average becomes 27 years. If difference between Difference between present age of Amit to Satish’s
age of two new member are one year then find present age is 12 years. Find the sum of present ages
difference between son and the new member who is of all three if ratio of Amit’s age to Satish’s age after ‘n’
elder. year will be 13 : 9.
(a) 7 years (b) 8 years (c) 4 years (a) 81 (b) 84 (c) 87
(d) 2 years (e) 5 years (d) 90 (e) Cannot be determined
13. Present average age of A, B, C and D is 25 years. Sum 20. P1 is the series of four consecutive numbers divisible
of age of A and B is 150% of sum of age of C and D. by 4 with an average of 34, while P2 is the series of
Ratio of age of B to age of C is 3 : 5. Calculate 10 years
five consecutive numbers divisible by 9, and whose
later age of A, if B and D are of same age.
(a) 45 years (b) 40 years (c) 35 years second lowest number is equal to second highest
(d) 55 years (e) 50 years number of P1 series. If P3 is the series of seven
consecutive numbers, whose lowest number is
14. Ratio of present age of A and B is 4 : 5, while that of C 1
𝑟𝑑 𝑜𝑓 lowest number of P2 series, then find the sum
and D is 6 : 7. If 10 years ago Ratio of age of A and C is 3
1 : 2 and that of B & D is 3 : 5, Find average of present of average of P2 & P3 series?
age of A, B and C together. (a)53 (b)51 (c)57
(a) 27.5 (b) 25 (c) 27 (d)55 (e)49
(d) 26.5 (e) 24.5 21. Average of five two digit numbers, in which 2
15. The ratio of ages of three friends Veer, Sandeep and numbers say A and B are reverse of each other, is
1 1 1
Satish is : : . The average age of Veer and his wife is 25.4. If all three numbers except A and B multiplied by
3 2 4 (–2) then the average of five numbers decreased by
33 years whereas the average age of Sandeep and his
3 43.2. What could be the greatest value among A and B.
wife is 39 years. The age of Satish’s wife is th of the (a) 32 (b) 23 (c) 50
8
sum of age of Veer’s wife and Sandeep’s wife and the (d) 41 (e) 14
ratio of Veer’s age to that of age of Satish’s wife is 4 : 3 22. Average of any 200 consecutive natural numbers is
then find the sum of ages of all the three friends. 499.5. If next 1000 numbers more add in it then find
(a) 104 yr (b) 112 yr (c) 117 yr the new average.
(d) 91 yr (e) 120 yr (a) 1035.5 (b) 1299.5 (c) 1199.5
16. There are some teachers in a college with average age (d) 1099.5 (e) 999.5
of 45 years. Sum of maximum age and minimum age is 23. Average age of a group of people is four times of the
108 years and difference of maximum age and number of people in the group. Sakshi leaves the
minimum age is 27 years. If fourteen teachers left the group and the average age is still four times of the
college with the average age of 32.5 years and six new number of people in the group. After that Sheetal
teachers joined college. Out of six new teachers four leaves the group and the average age is still four times
ages are equal to maximum age teacher and two of the number of people in the group. If ratio between
teachers age equal to minimum age teacher. If new Sakshi’s age to Sheetal’s age is 21 : 19, then find the
4
average becomes 49 years, then find the number of average age of the group if Ritu leaves the group
7
teachers initially? whose age is 20 years
(a) 54 (b) 60 (c) 64 (a) 36 years (b) 37 years (c) 38 years
(d) 68 (e) 72 (d) 39 years (e) 40 years
17. The ratio between present age of Veer & Saurbh is 3: Directions (24-25): Rakesh distributed Rs.50000 into (X,
4 and between present age of Divyaraj & Gopal is 2: 3. Y, Z) 3 groups having 50 people in total. Number of people
If sum of present age of Veer, Saurbh, Divyaraj & in group ‘X’ is equal to the number of people in group ‘Z’
Gopal is 128 years and ratio between sum of age of and total money got by group ‘Y’ is equal to the total
Veer & Saurbh to sum of age of Divyaraj & Gopal is 63: money got by group ‘Z’. Average money got by group X
3500
65, then find the sum of age of Veer & Gopal after and Y together is Rs. and average money got by group
3
eight years.
Y and Z is Rs.1000
(a) 90 years (b) 78 years (c) 86 years
(d) 84 years (e) 82 years 24. Find the number of people in group Y?
18. 2n years ago, the age of Raju was four times that of his (a) 20 (b) 16 (c) 10
son and n years ago, the age of Raju was thrice that of (d) 18 (e) 24
his son. If n years later, the sum of the ages of Raju
and his son will be 80 years, then the difference in the 25. Find the amount got by each person in group Z.
ages of Raju and his son is (a) 800 Rs. (b) 600 Rs. (c) 1000 Rs.
(d) 750 Rs. (e) 500 Rs.
31 Adda247 Publications For More Study Material
Visit: adda247.com
Cracker Book for Bank (IBPS | SBI | RRB PO | Clerk) Mains Exams

Solutions
2x+44
1. (e); Let the present age of B is 3K and that of C is 4K. = 2 = x + 22
Also assume present age of A is 4m and D is 5m. ATQ,
Note that age difference between A and C will 22 (x + 2) – (x + 22) = (x + 2 – 4) × 25
always be 20 years. x = 18
4K – 4m = 20
K – m = 5 …(i) 6. (d); Let age of Ram and Shyam 4 years age 9x and 11x
4K respectively
Also 4m = 2 So ATQ,
K = 2m …(ii) 11x = 9x + 4
m = 5 years x=2
K = 10 years their present age is 22 and 26 years
20+30+40
∴ Average of A, B and C = = 30 years Rahul’s age 2 year ago
3 22+26
= 2 – 2 = 22
2. (c); Let age of Arjun, Vikas, Rohit and Rahul is a, b, c
and d respectively 7. (b); Let Ayush Age, his wife age, his first son’s age and
Now, his second son’s age is 20x, 3y, 9x and y
b : c = 2 : 3 …(i) respectively.
b + c + d = 23 × 3 = 69 …(ii) ATQ,
a + b + c + d = 25 × 4 = 100 9x – y = 6 …(i)
∴ a = 31 years. 20x + 3y + 9x + y = 26.5 × 4 …(ii)
Also d – b = 13 Solving (i) & (ii)
d=b+13 x=2
Put this value in eq. (ii) y = 12
2b + c = 56 his wife’s age = 12 × 3 = 36
From (i) b = 16 years, d = 29 years 8. (a); Let four number is w, x, y and z after reversing
c = 24 years they formed A.P.
∴ a + d = 31 + 29 = 60 years. Sum of all number = (52.5 – 22.5) × 4 = 120
3. (d); Let S₁ is a series consists 3x – 6, 3x – 3, 3x, 3x + n=4
3, 3x + 6 d=4
n
ATQ, Sn = [2a + (n– 1)d]
2
3x – 6 + 3x – 3 + 3x + 3x + 3 + 3x + 6 = 180 4
120 = [2a + (4 – 1)4]
⇒ x = 12 2
S₁ series = 30, 33, 36, 39, 42 a = 24
Second smallest no. of S₂ = 39 + 13 = 52 Reversed numbers
S₂ series = 48, 52, 56, 60 24, 28, 32, 36
30+60 90 Original number
Required average = 2 = 2 = 45 42, 82, 23, 63
4. (b); Let the number which is reversed is “10a+b” Largest number → 82
If it is reversed then the number becomes
“10b+a” 9. (b); Sum of the ages of 6 children
ATQ, = 15 × 6 = 90
10𝑏 + 𝑎 − 10𝑎 − 𝑏 = 5 × 5.4 When two children left, sum of the ages of 4
children = 90 − (18 + 20) = 52
⇒ 9𝑏 − 9𝑎 = 27
Sum of the ages of New children
⇒𝑏−𝑎 = 3
= (15 + 4) × 4 = 76
Numbers can be 14, 25, 36, 47, 58 and 69 but all 76+52
the five digits are multiple of 4 ∴ Required average = 8
128
⇒ The number should be 36 = = 16 years
8
5. (a); Let initially strength = x 10. (e); P + 2Q + R = 59
ATQ, 3P + Q + R = 68
Total weight → 20x P + 3Q + 3R = 108
When two persons include Solving the equation, P = 12 years, Q = 15 years, R
Total weight = 22 (x + 2) = 17 years.
Weight of two persons included = 2x + 44 Sum of their ages = 44 years.
4 persons excluded weight
32 Adda247 Publications For More Study Material
Visit: adda247.com
Cracker Book for Bank (IBPS | SBI | RRB PO | Clerk) Mains Exams

11. (c); Let age of Sweta 5 years ago = 5x ⇒ 8x – 6y = 10 …(i)


Let age of Neha 5 years ago = 4x Similarly
According to question 5x–10 3
= 5 ⇒ 25x – 50 = 21y – 30
400 7y–10
(5x + 10) + (4x + 10) = (100% + %) (25 +
9 ⇒ 25x – 21y = 20 …(ii)
20) Solving (i) and (ii)
13
9x + 20 = × 45 25𝑥 – 21𝑦 = 20
9
x=5 16𝑥 – 12𝑦 = 20
– + –
Sum of their present age = (5+ 4)×5+10
= 45 year + 10 years = 55 years 9𝑥 – 9𝑦 = 0
Required ratio = 6 : 5 𝑥=𝑦
Put this value in equation (i), we will get x=y=5.
12. (e); Let age of man is x years Therefore, average age of A, B and C together is
Age of woman = (x – 2) years 15 ×5 75
(x+x–2) (x–1) = = 25 𝑦𝑒𝑎𝑟𝑠.
3 3
Age of son = 4 = 2 years
15. (a); Ratio of ages of Veer, Sandeep and Satish be 4 : 6 :
ATQ— 3
(x–1)
x+(x –2)+
2
= 30 Let their age be 4x years, 6x years and 3x years
3
2x+2x–4+x –1 respectively
2
= 90 Age of veer’s wife = (66 – 4x) years
5x = 185 Age of Sandeep’s wife = (78 – 6x) years
x = 37 years Atq,
(37–1)
son age = 2 = 18 years Age of Satish’s wife
3
let age of two new members be y years and (y – = 8 × (66 − 4𝑥 + 78 − 6𝑥)
1) years 3
= 8 × (144 − 10𝑥)𝑦𝑒𝑎𝑟s
ATQ—
90+(y+y–1) Now,
= 27 4𝑥 4
5
3 =
2y – 1 = 135 – 90 8
(144−10𝑥) 3
46
y= ⇒ 8x = 144 – 10x
2
y = 23 ⇒ 18x = 144
Required difference = 23 – 18 = 5 years ⇒ x = 8 years
Required sum = 8 × (4 +6 + 3) = 104 years
13. (d); If average age of A, B, C and D is 25 years, then
total age of A, B, C and D is 100 years. 16. (c); Let total teachers in a college = N
Also if C + D = 2x Let maximum age = a year
Then A + B = 3x [A + B is 150% of C + D] Minimum age = b years
5x = 100 ATQ –
x = 20 a + b = 108 ------- (i)
A + B = 60 …(i) a – b = 27 --------(ii)
C + D = 40 …(ii) From (i) & (ii)
Now, in question ratio of B : C is given as 3 : 5. 2a = 135 year
a = 67.5 years
This is also ratio of D : C, as B and D are of same
Ans, b = 40.5 years
age. From (ii)
N× 45 − 14 × 32.5 + 4 × 67.5 + 2 × 40.5 + = (N
We can calculate 347
– 14 + 6) 7
C = 25 years
D = 15 years 7(45N – 455 + 270 + 81) = 347N – 2776
315N − 728 = 347N – 2776
& Hence B = 15 years
32N = 2048
∴ A = 45 years 2048
N = 32
10 years later age of A is 45 + 10 = 55 years.
N = 64
14. (b); Let present age of A, B, C and D is 4x, 5x, 6y and
7y respectively. 17. (e); Let present age of Veer & Saurbh be 3x and 4x
Then, ATQ, respectively and present age of Divyaraj & Gopal
4𝑥–10 1
= ⇒ 8x – 20 = 6y – 10 be 2y and 3y respectively
6𝑦–10 2

33 Adda247 Publications For More Study Material


Visit: adda247.com
Cracker Book for Bank (IBPS | SBI | RRB PO | Clerk) Mains Exams
3𝑥+4𝑥 63
= 65 or 11z – 7y = 54 …(iii)
2𝑦+3𝑦
and 10y – 8x = 9x – 13z
x : y = 9: 13=9k:13k
17x = 10y + 13z
ATQ –
17 (12 + z) = 10y + 13z
3x+4x+2y+3y=128
204 + 17z = 10y + 13z
3× 9𝑘+4× 9𝑘+2×13k+3×13k=128k=128
4z + 204 = 10y …(iv)
K=1.
Therefore, age of Veer is 27 years. On solving (iii) and (iv)
Similarly, age of Gopal is 39 years. y = 30, z = 24, x =36
Sum of their present ages=66years Required sum = 30 + 24 + 36 = 90
After 8 years, sum of their ages = 66+16 = 20. (c); Let four consecutive numbers of P₁
82years. Series be a, (a + 4), (a + 8) and (a + 12)
18. (d); Let, the present ages of Raju and his son be 𝑥 and ATQ—
𝑎+(𝑎+4)+(𝑎+8)+(𝑎+12)
𝑦 respectively. = 34
4
2n years ago,
4a + 24 = 136
𝑥 − 2𝑛 = 4(𝑦 − 2𝑛)
4a = 112
𝑥 = 4𝑦 − 6𝑛 ……….(i)
𝑛 years ago, 𝑎 = 28
𝑥 − 𝑛 = 3(𝑦 − 𝑛) P₁ series = 28, 32, 36, 40
⇒ 𝑥 = 3𝑦 − 2𝑛 …………..(ii) Let P₂ series be b, (b + 9), (b + 18), (b +
Solving (i) and (ii), 27), (b + 36)
𝑦 = 4𝑛 Given,
And, 𝑥 = 4 × 4𝑛 − 6𝑛 = 10n 𝑏 + 9 = 36
N years later, 𝑏 = 27
𝑥 + 𝑛 + 𝑦 + 𝑛 = 80 P₂ series = 27, 36, 45, 54, 63
⇒ 4𝑛 + 𝑛 + 10𝑛 + 𝑛 = 80 Let P₃ series be, 𝑐, (𝑐 + 1), (𝑐 + 2), (𝑐 + 3), (𝑐 +
⇒ 16𝑛 = 80 4), (𝑐 + 5) and (𝑐 + 6)
⇒𝑛=5 Given,
Difference in their ages = 10𝑛 − 4𝑛 = 50 − 20 = 27
𝑐=
30 3
𝑐=9
19. (d); Let, Present age of Amit, Inder and Satish be x, y
P₃ series
and z respectively.
9, 10, 11, 12, 13, 14, 15
ATQ,
x –2n 5 Required average
y –2n
=4 (27+36+45+54+63) (9+10+11+12+13+14+15)
= +
⇒ 4x – 8n = 5y – 10n 5 7
225 84
⇒ 5y – 4x = 2n …(i) = + = 45 + 12 = 57
5 7
y –n 9
=7
z –n 21. (d); Let A = 10a + b
⇒ 7y – 7n = 9z – 9n and B = 10b + a
⇒ 2n = 9z – 7y …(ii)
Let sum of other three number is = X
Equating (i) & (ii)
X + 10a + b + 10b + a = 5 × (25.4) = 127 …(i)
5y – 4x = 9z – 7y
12y = 9z + 4x And
And, x – z = 12 –2X+10a+b+10b+a= 127 – 5 (43.2) …(ii)
x+n 13 Solving (i) and (ii)
z+n
= 9
We get a + b = 5
⇒ 9x + 9n = 13z + 13n
If 𝑎 would be → 0, 1, 2, 3, 4, 5
⇒ 4n = 9x – 13z
Then b would be → 5, 4, 3, 2, 1, 0
9x – 13z = 18z – 14y
9x + 14y = 31z as 10a + b can’t be 50 because its reverse is 05
Now x = 12 + z which is not a two digit number.
9 (12 + z) + 14y = 31z Greatest 10a + b = 41
⇒ 22z – 14y = 108 10a + b = 41

34 Adda247 Publications For More Study Material


Visit: adda247.com
Cracker Book for Bank (IBPS | SBI | RRB PO | Clerk) Mains Exams

22. (e); Let 200 numbers are → a₁, a₂, a₅……, a₂₀₀ Solution (24-25)
ATQ, Let number of people in X, Y and Z group is a 𝑎, 𝑏 and 𝑎
a1 +a2 +a3 +...+a200 respectively.
499.5 = 200
⇒ 99900 = a1 + a2 + a3 +. . . +a200 Let total money got by group X, Y and Z is h, k and k
respectively.
Now,
This form an A.P. Average amount got by X and Y together
ℎ+𝑘 3500
a = a1 𝑎+𝑏
= 3 …(i)
d=1 Average amount got by Y and Z together
𝑘+𝑘
n = 200 = 1000
𝑎+𝑏
200
⇒ 99900 = [2a1 + (200– 1)1] k = 500 (𝑎 + 𝑏)…(ii)
2
a1 = 400 Put value of (𝑎 + 𝑏)from eq (ii) in eq (i)
ℎ+𝑘 7
Required sum for 1200 terms whose a1 = 400 =
𝑘 3
1200 ℎ 7
= [2 × 400 + (1200– 1)1] +1 = 3
2 𝑘
⇒ 1199400 ℎ 4
= 3…(iii)
1199400 𝑘
Required Average = = 999.5
1200 We know
23. (c); Let, initial Number of people in the group be ‘n’. ℎ + 𝑘 + 𝑘 = 50,000
Let 21𝑥 𝑎𝑛𝑑 19𝑥 be ages of Sakshi and Sheetal so, h = 20,000
respectively, and k = 15000
A.T.Q., now ratio (i) becomes
4𝑛2 − 21𝑥 = 4(𝑛 − 1)2 ……..(i) 𝑎 + 𝑏 = 30
And we know
And 4𝑛2 − 21𝑥 − 19𝑥 = 4(𝑛 − 2)2
𝑎 + 𝑏 + 𝑎 = 50
4𝑛2 − 40𝑥 = 4(𝑛 − 2)2 ………….(ii) So, 𝑎 = 20
Solving (i) and (ii), 𝑏 = 10
𝑥 = 4, 𝑛 = 11
24. (c); Number of people in Y group = 10
Average age of group after Ritu leaves the group
4×112 −21×4−19×4−20 304 25. (d); Amount got by each person in group Z
= = = 38 15000
11−3 8 = 20 = Rs. 750

35 Adda247 Publications For More Study Material


Visit: adda247.com
Cracker Book for Bank (IBPS | SBI | RRB PO | Clerk) Mains Exams

Chapter
Percentage and Profit & Loss
5
BEST APPROACH TO SOLVE THE QUESTIONS

1
A percentage is a number or ratio that represents a fraction of 100 i.e. 100
. For faster and easy calculation one should
2 2
use fractional equivalent of percentage. For example, it’s always easier to use 3 instead of 663%.

It becomes easy to understand a question when one talks in terms of percentage, but while calculating, it is always the
fractional equivalent that comes in hand.
While solving the question one must try to stay away from unnecessary calculation. This happens in many of the
questions related to profit and loss. Many times one starts calculating the absolute values of CP, SP etc. even when it’s
not required.
Let’s take a couple of sample questions.
Example 1 : 'The CP of an item is Rs. 270. After giving a discount of Rs. 27 a shopkeeper earns a profit of 20%. What
will be the profit percentage if he does not give the discount?’
Sol. Here, instead of finding out SP or MP, one should look at it the other way round. If discount of Rs. 27 is not
given, then Rs. 27 will become further profit for the shopkeeper.
27 is 10% percent of 270. Profit of 20% is already there, so new profit% would become 20% + 10% =
30%.
Example 2: ‘A man while buying cloth from a wholesaler uses a scale that measures 10% more than the actual length
and while selling the same cloth he uses a scale that measures 10% less than the actual length. Find his
overall profit or loss in this transaction.’
Sol. Here, one just needs to deal with the percentage the shopkeeper gains. While buying he gets 110 metre
cloth on the cost of 100 metre coth and while selling he gains the cost of 100metre just by selling 90 metre
cloth.
10
Multiplication factor while buying = 1+
100
11
=
10
100−90
Multiplication factor while selling = 1+
90
10
=
9
11 10
The final multiplying factor becomes = ×
10 9
11 2
= 9
=1+ 9
2
which clearly shows a profit of 229%.

Practice Exercise Based on New Pattern


1. Nishant bought an article at 20% discount on MRP, 2. A shopkeeper marked up the price of a mobile phone
and claims to sell it at profit of 10% of MRP. When by 40% of its cost price, if he increases the discount
Nikhil offered him Rs. 500 banknote, he cheated again from 5% to 10%, the profit would decrease by 1400
by giving him Rs. 125 instead of Rs. 225. Find overall Rs. How much profit shopkeeper would earn if he
profit% of Nishant. gives a discount of 20% on the marked price ? Assume
(a) 87.5% (b) 37.5% (c) 100% that, he calculates discount only on MRP.
(d) 62.5% (e) None of these (a) 1800 Rs. (b) 1200 Rs. (c) 2800 Rs.
(d) 2200 Rs. (e) 2400 Rs.

36 Adda247 Publications For More Study Material


Visit: adda247.com
Cracker Book for Bank (IBPS | SBI | RRB PO | Clerk) Mains Exams

3. A manufacturer of cricket balls wants to earn 25%


9. If shopkeeper cheat his customer by giving 20% less
profit on manufacturing cost after giving a discount of
quantity and reducing value of discount percentage
23 ⅓% on MRP marked by him. But due to some
by 20% then find the new profit percent.
reasons he lost 25% of balls & he decided to offer
9 (a) 60% (b) 75% (c) 62.5%
discount of 7 13 % on MRP of remaining balls. Find his (d) 80% (e) 70%
overall profit% or loss%.
10. Initially shopkeeper have 20 articles. Out of 20, 7
(a) 8% Profit (b) 12% Profit (c) 12% Loss
articles damaged and remains unsold. Marked Price
(d) 8% Loss (e) 5% Profit
should he labeled by how much percent more than
4. Rahul bought a cycle at a discount of 16⅔% on MRP. cost price so that his overall profit does not change
He earned half the amount of his CP by renting it for neither his discount percentage.
200 days. After that he resells it at half of MRP. In this (a) 156% (b) 146% (c) 136%
transaction he earned Rs. 200, find MRP of cycle (in (d) 120% (e) 125%
Rs).
(a) 1860 (b) 2490 (c) 2400 11. Thirty percent of total employees of Bankersadda are
(d) 2280 (e) 2310 females, and sixty percent of female employees earn
more than Rs. 17000 as monthly salary. If fifty five
5. A man purchased two items A & B and invested Rs 50 percent of total employees of Bankersadda earn more
& Rs 75 on their repairing respectively. If he earns than Rs. 17000 as monthly salary. What fraction of
profit of 10% on A and 12% on B, overall profit total male employees of Bankersadda earn Rs. 17000
earned by him is is Rs 84. But if he earns 20% on A & or less as monthly salary ?
10% on B, overall profit earned by him is 14% of total 13 11 51
(a) (b) (c)
price of items. Find initial total purchasing price of 20
33
20
69
100

both items. (d) (e)


100 100
(a) Rs 675 (b) Rs 725 (c) Rs 750
12. A shopkeeper raised the marked price of an article by
(d) Rs 625 (e) Rs 775 1
60% and allow three successive discounts of 12 2 %,
6. A shopkeeper sold two articles, if he marked up 2
1 14 % and 20% on new MP and make a profit of 20%.
second article at 11 9
% above first article’s selling 7
If shopkeeper would allow only two discounts of
price and gives a discount of 20% on that, then a loss 1
1 12 % and 20% on new MP, he made a profit of Rs
of 20% occur on first one and 33 3% profit on second 2
800. Find the cost price of article?
article. Find total selling price of both articles, if he
(a) 1800 Rs. (b) 2000 Rs. (c) 1600 Rs.
made total loss of Rs.75 on both articles.
(d) 2400 Rs. (e) 2800 Rs.
(a) 4000 Rs (b) 4500 Rs (c) 5100Rs.
(d) 4800 Rs (e) 5800Rs 13. Bankersadda started its publication with the 1500 DI
books. Printing cost per book is 125 Rs, packing cost
7. Satish buy two articles i.e. type A at Rs 500 and type B
per book is 15 Rs and shipping cost per book is 40 Rs.
at Rs 1500. He sold type A article at x% profit and
if 50% of books sold on half of total cost price, then
mark up type B article 2x% above the cost price and
find on how much percent above should be remaining
gave x% discount at the time of the sale. By this Satish
book sold to gets 20% profit on total outlay?
earn (x -6) % profit. Find the value of ‘x’.
(a) 50% (b)60% (c) 80%
(a) 15% (b) 18% (c) 25%
(d) 90% (e)70%
(d) 20% (e) 30%
14. Adda247 publication sold a puzzle book in Rs. 475
Directions (8-10): A article is mark up above cost price
and made profit of 25% on CP and sold a DI book in
such that markup percent is double of the profit percent.
If discount is 12.5%, then profit percent increased by Rs. 575 and made a profit of 15% on CP. If publication
1 sold puzzle book in Rs. 360, then find what price
33 3 %.
should DI book will be sold to made a profit of 30% on
8. On selling 20 such article at 12.5% discount, profit is both the books ?
Rs.300. Find the M.P. of each article. (a) Rs.784 (b) Rs.874 (c) Rs.847
(a) Rs.60 (b) Rs.160 (c) Rs.80 (d) Rs.748 (e) Rs.478
(d) Rs.240 (e) Rs.72

37 Adda247 Publications For More Study Material


Visit: adda247.com
Cracker Book for Bank (IBPS | SBI | RRB PO | Clerk) Mains Exams

Directions (15-16): Abhishek bought some chairs and 20. Neeraj purchased two mobile from a shop. He sold
tables from a shopkeeper. The marked price of a chair and first mobile at the price of Rs. 18750 and second
a table were in the ratio 5 : 8. The shopkeeper gave mobile at the price of Rs. 14250. If the profit percent
discounts of 20% and 25% on the chair & the table on first mobile is five times of the loss percent on
respectively. The ratio of number of chairs and tables second mobile, then find the overall profit made by
bought by Abhishek is 6 : 5. Neeraj after selling both the mobile phones. (C.P of
both mobile is same)?
15. If Abhishek sells each chair and table bought by him at
(a) 15% (b) 12% (c) 14%
discounts of 25% and 20% respectively after marking (d) 10% (e) 18%
up the prices of both by 50% and gives one table free
for every four chairs bought by a customer and only 21. There are three societies A, B and C. Ratio of total
2
rd of the total chairs are sold in bunch of four chairs, population of A, B and C is 2 : 3 : 5. In society A, 35%
3 of total population is female. In society B, number of
then what is the net profit /loss % made by Abhishek 2
after selling all of the items which he bought from the females is 66 3 % less than number of males of society
shopkeeper? B. In society C, ratio of male to female is 27 : 23. Find
2
(a) 6 3 %
1
(b) 3 3 %
1
(c) 2 2 % total number of females is what percent less than total
1
number of males in all societies together ?
(d) 4 4 % (e) 5 % 1
(a) 33 3% (b) 40% (c) 60%
2
16. If the marked price of a table set by the shopkeeper (d) 66 3 % (e) 75%
was Rs.300 more than that of a chair and the total
expenditure made by Abhishek in purchasing the 22. The marked price of a bike in two stores A and B is
chairs and table from the shopkeeper was Rs.108000, same.In store A bike is available at 30% discount and
then how many chairs were purchased by Abhishek? in store B same bike is available at two successive
(a) 150 (b) 60 (c) 120 discounts of d% and 8%. A man bought bike from
(d) 90 (e) 80 store A at Rs. 49000. If man would have paid Rs. 7672
more, he could have bought the same bike from store
Directions (17-18): A car ‘X’ is manufactured in a B. Find the discount ‘d’ allowed by store B on bike ?
company ‘A’. Cost of raw material on a car is 25% of total (a) 8% (b) 10% (c) 12%
cost and cost of labor is 20% of rest. 30% of total cost (d) 16% (e) 15%
occur due to four engineers working there and remaining
cost is due to machines and taxes which is in the ratio 7 : 23. A shopkeeper gives a discount of 19% on marked
3. Company sold the car ‘X’ at 28% profit. price of article A and selling price of article B is
1
11 9 % more than selling price at article A. If
17. If cost of raw material is increased by 30% and all
shopkeeper made 20% profit on article B and selling
their 4 engineers leave the company, so they hire 2
price of article A was Rs. 840 more than cost price of
new engineers by giving 50% more salary as they give
article B, then find the cost price of article A.
to previous engineer each. Find the % change in
Shopkeeper sold article A at profit of 25%?
selling price so overall profit must remain same.
(a) 9070 Rs. (b) 9060 Rs. (c) 9040 Rs.
(a) 0% (b) 100% (c) 2%
(d) 9020 Rs. (e) 9072 Rs.
(d) 50% (e) 3%
Directions (24-25)A sale conducted on Samsung mobiles
18. Company A, started making car ‘Y’ in which same raw
store where Cost price of three types Samsung mobiles A,
material used, labor cost is increased by 20%. If all
B and C in the ratio of 5 : 7 : 9. Store owner made mark
other cost is same then find the ratio of selling price of
price of mobile A and C, 30% above cost price and mobile
car ‘X’ to car ‘Y’, given that car ‘Y’ sold at 20% profit.
B, 40% above cost price. Discounts allowed by store on
(a) 331 : 309 (b) 320 : 314 (c) 313 : 309 5 3 1
(d) 320 : 309 (e) 309 : 320 mobiles A, B and C are 15 13 %, 21 7 % 𝑎𝑛𝑑 11 9 %
respectively.
19. Marked price of two articles A and B are in the ratio of
7 : 9, at the time of selling shopkeeper gives discount 24. A man purchased fourteen type A mobile, twenty type
of d% on article A and (d + 5)% on article B and made B mobile and fifteen type C mobile in sale. If store
a profit of 25% on each article, if cost price of article A owner made a total profit of Rs. 17500 on all type A
and B are in ratio 112 : 135. Then find percent of mobile, purchased by man than find total profit made
discount given by shopkeeper on both articles? by store owner on type B and type C mobiles, which
(a) 12.5%, 17.5% (b) 25%, 30% was purchased by man?
(c) 10%, 15% (d) 15 %, 20% (a) 82500 Rs. (b) 84500 Rs. (c) 78500 Rs.
(e) 20%, 25% (d) 87500 Rs. (e) 88500 Rs.

38 Adda247 Publications For More Study Material


Visit: adda247.com
Cracker Book for Bank (IBPS | SBI | RRB PO | Clerk) Mains Exams

25. Store decided to give no discount on purchase of any 30. In a city 60 percent are male and remaining are
type of mobile it will be given two mobiles free on females. Out of total males, 20% of male are
purchase of six type A mobiles, one mobile free on government employees. Out of remaining males, ratio
purchase of five type B mobiles and three mobiles free between private employees to unemployed male is 5
on purchase of ten type C mobiles. A retailer come to :7. Out of total females, 40% of female are in
purchase mobile and take away48 type A mobiles, 36 government jobs and out of remaining females 40% of
type B mobiles and 39 type C mobiles. Find overall female work for private firms and remaining are
loss percentage of store owner in this transaction?(2 unemployed. Find the total number of male and
marks) female, who are government employees, if difference
28 28 76 between male and female who are unemployed is
(a) 10 281 % (b) 8 281 % (c)4281 %
3400?
28 28
(d) 14 281 % (e) 9 281 % (a) 6000 (b) 5000 (c) 7000
(d) 4000 (e) 3000
26. Ratio between marked price of article A to article B is
4 : 5. Shopkeeper allowed d% discount on article ‘A’ 31. A person ‘A’, invested 37½% of his savings in scheme
and (d + 18)% discount on article ‘B’, so selling price XYZ, while B invested 62½% of his savings in the
of both articles become equal. If shopkeeper made a same scheme. If B invested 50% of his remaining
profit of 20% on article A and 25% on article B and savings in scheme PQR, and A thought to invest
profit made on article B is Rs. 384 more than that of double of the amount invested by B in PQR scheme
then find what percent of A’s remaining savings after
article A, then find the cost price of article ‘A’ and
investment in XYZ, A should invest in PQR. Given that
article ‘B’ respectively?
amount invested by B in XYZ is twice the amount
(a) 9000 Rs. 8400 Rs (b) 9600 Rs. 9216 Rs.
invested by A in same scheme.
(c) 9800 Rs. 9012 Rs. (d) 9600 Rs. 8488 Rs.
(a) 80% (b) 68% (c) 64%
(e) 9200 Rs. 9216 Rs.
(d) 72% (e) 60%
27. Aman started a shoes manufacturing company, 32. A man bought few kg’s of sugar of 3 variety (X, Y, Z) in
manufacturing cost of each shoes is Rs. 500 , Raw the ratio of their quantity 2 : 3 : 5 respectively and
material cost is Rs 300 and transport cost of Rs 100 If started to sell them at profit of 20%, loss of 10% and
company manufactured 150 Shoes in first order and profit of 32% respectively. He managed to sell only
sold 50% of shoes on half of its cost price. Then find 80% of total sugar and rest was return at the same
at what price of should the company sold the price as he bought. The price of these sugar is Rs.
remaining shoes to gets 50% profit on its total cost? 30/kg, Rs. 40/kg and Rs. 50/kg respectively. Calculate
(a) 100% (b) 150% (c) 75% his total profit amount, if total sugar bought was 50 kg
(d) 125% (e) 120% and he sold all the sugar of X and Y types.
28. The ratio between marked price of a same watch in (a) 320 (b) 300 (c) 0
two shops P and Q is 7 : 5 . In shop P watch available (d) 180 (e) None of these
at two successive discounts of 10% & 25% , while in 33. In an election survey, 83 ⅓% of total voters took part
shop Q watch available at two successive discounts of in survey, 50% then claims to votes for candidate A,
d% and 10%. If selling price of watch at shop Q is less 10% are uncertain & rest says to vote for B. If all of
than that of selling price of watch at shop P by them voted according to their commitments on the
44
𝑡ℎ of the selling price of watch at shop P, then find day of election and those who were not part of survey,
105 voted to A and B in ratio of 2 : 1. All the people who
the value of ‘d’ ?
are uncertain in survey, voted to A : B in the ratio of 1
(a) 39% (b) 29% (c) 31% : 4 in election. If A won by 640 votes then find total
(d) 35% (e) 37% number of votes in election.
29. Cost price of a Samsung mobile is 20% more than cost (a) 6000 (b) 5000 (c) 9000
price of a Mi mobile. Shopkeeper marked up Samsung (d) 7200 (e) 6750
mobile at 25% above its cost price and Mi mobile at 2
34. In a village 60% are males in which 6 % males are
40% above cost price. If shopkeeper allowed discount 2
3
of 20% on Mi mobile and 25% on Samsung mobile above 60 years, in remaining 147 % males are below
and difference between selling price of Samsung 1
18 years, out of remaining 622 % are above 18 years
mobile and Mi mobile was 160 Rs. then find sum of
but below 50 years and remaining males are above 50
marked price of one Samsung mobile and one Mi
years but below 60 years. If difference between males
mobile?
above 60 years and males above 18 years but below
(a) 92840 Rs (b) 92260 Rs (c) 98460 Rs 50 years are 7800, then find total female population of
(d) 92800 Rs (e) 98530 Rs village?

39 Adda247 Publications For More Study Material


Visit: adda247.com
Cracker Book for Bank (IBPS | SBI | RRB PO | Clerk) Mains Exams

(a) 16000 (b) 18000 (c) 20000 a costumer by saying that he will give him 200 kg of
(d) 12000 (e) 24000 Type A wheat but shopkeeper gives 200 kg of mixture
35. Divyaraj purchased jeans and shirt from a seller. of Type A & Type B wheat on 10% above the cost
Marked price of jeans and shirt are in the ratio of 9 : 7 price per kg of Type A wheat. If shopkeeper made a
2 1
and seller offered 14 7 % discount on shirt and 11 9 % total profit of 71
7
% in this transaction, then find
8
on jeans and number of jeans and shirt purchased by
ratio between quantity of Type B wheat to Type A
Divyaraj are in the ratio of 5 : 8. If Divyaraj marked up
jeans and shirt 50% above their cost price and wheat in sold mixture?
1
offered 25% discount on jeans and 11 9 % on shirt, (a) 2 : 5 (b) 2 : 3 (c) 1 : 3
Find overall profit of Divyaraj in this transaction. (d) 2 : 7 (e) 1: 5
19 19 19 1
(a) 22 22 % (b) 21 22 % (c) 19 22 % 39. In a store marked price of a shirt is 33 3% less than
19
(d) 2322 %
19
(e) 25 22 % marked price of a jeans. There are two discounts
schemes A & B. In Scheme A, if someone buy one jean
36. In 2016 a store sold some mobile phone at certain and one shirt together, then store gives overall
price. In 2017 store increase selling of mobile phone discount of 35%, while in scheme B if someone buy
by reducing the price of mobile phone by 25%. The one shirt and one jeans separately, store give 20%
total revenue generated by selling mobile phone in discount on shirt and 25% on jeans. If difference
2017 is increased by 5% compared to previous year. between selling price of one jeans and one shirt in
The total revenue generated by store in 2017 is 94.5 scheme A and B is Rs. 384 then find cost price of one
lakh and number of mobile phones sold by store in the 1
jeans & one shirt, given that store made of 33 3 % on
year 2017 is 90 more than that in 2016. Find initial
cost of one mobile. shirt and 25% on jeans, when it sold one jeans and
(a) 32000 Rs. (b) 40000 Rs. (c) 36000 Rs. one shirt in scheme B ?
(d) 24000 Rs. (e) 20000 Rs. (a) 720 Rs, 1150 Rs (b) 768 Rs. 1152 Rs.
(c) 786 Rs. 1168 Rs. (d) 796 Rs. 1144 Rs.
37. Veer bought some Shirts & some Jeans from a store. (e) 790 Rs. 1108 Rs.
The marked price of a shirt and a jeans is in the ratio 5
: 7 and store allows discounts of 20% and 25% on the 40. The Hero company manufactured cycles, 40% of the
shirt & the jeans respectively. The ratio between total manufactured cost is on raw material, 20% on
number of shirt and jeans bought by Veer is 9 : 8. Veer labour charges, 20% on fixed charges and the rest on
marking up the prices of both by 50% on the price at transportation. The cycle is sold at a profit of 25%.
which he bought and sells each shirt and jeans bought The price of the raw material increased by 15% and
by him at discounts of 25% and 20% respectively. If the labour charges increased by 20% and the cost on
Veer gives one 1 shirt free for every 4 jeans bought by the transportation increased by 50% while the fixed
a customer, then what is the net profit made by Veer costs remained unchanged. If the manufacturer wants
7
after selling all of the items which he bought from the a 13 % profit, then by what percentage should
11
store? company reduced its expenditure on raw materials (at
(a) 2% (b) 7% (c) 6% the increased price), the selling price remaining the
(d) 5% (e) 8% same ?
17 17 17
38. A shopkeeper has two types Wheat i.e Type A & Type (a) 19 23 % (b) 21 23 % (c) 19 23 %
B and cost price per kg of Type A wheat is 10 times of 17
(d) 23 23 %
17
(e) 27 23 %
cost price per kg of type B wheat. Shopkeeper cheated

Solutions

1. (a); Nikhil gave Rs. 500 note, & Nishant have to ∴ MRP is Rs. 250
actually pay him Rs. 225. Hence Nishant sold it at 80
And CP for Nishant is 100 × 25 = Rs. 200.
Rs. 500 – 225 = 275 to Nikhil.
Now, Let MRP is 100x. He got Rs. (500 – 125)
If he will get 10% profit, then SP will becomes = Rs. 375 from Nikhil.
110x. Hence Profit % = (
375–200
) × 100
110x = 275 200
7
x = 5.5 = 8 × 100 = 87.5%

40 Adda247 Publications For More Study Material


Visit: adda247.com
Cracker Book for Bank (IBPS | SBI | RRB PO | Clerk) Mains Exams

2. (e); Put this value in equation (i)


10 (2y) 12
CP of Mobile M.P. mobile S.P. of mobile 100 3
+ 100 y = 84
20y+36y
100% 140% 133%(1st discount) 300
= 84
84×300
126%(2nd discount) ⇒y= 56
= Rs 450
2
(133 – 126)% = 1400 x = 450 × = Rs 300
3
7% = 1400 Initial purchasing price is
1400
C. P. = 7 × 100 = (300 + 450) – (75 + 50)
= 20000 Rs. = Rs 625
112
S. P. after 20% discount = 20000 × 100 6. (c); Let marked price of second article be
= 22400 10x Rs.
Profit = 2400 Rs. Selling price of first article be 9x Rs.
Selling price of second article
3. (b);Let he manufacture 𝑛 balls and manufacturing 8
= 10x × = 8x Rs.
cost of each ball is 𝑦 10
And 𝑥 is marked price. C.P. of fisrt article
9x
Now, ATQ = × 5 = 11.25x Rs.
4
7 5 3
𝑥 [1– 30] = 𝑦 × 4 C.P. of second article = 8𝑥 × = 6x Rs.
4
23
⇒ 𝑥 × 30 = 𝑦 × 4
5 Total C.P. of both articles = 11.25x + 6x
= 17.25x Rs.
5 30 75
⇒𝑥 =𝑦× × =𝑦× …(i) Total S.P. of both articles
4 23 46
3 = 9x + 8x = 17x Rs.
After accident he sells 4
𝑛 balls on a price of ATQ—
1
𝑥 (1– ) = 𝑥 ×
12 17.25x – 17x = 75
13 13 75
3 3 12
x = 0.25 = 300
Total SP of 𝑛 balls = 𝑛. 𝑥. selling price of both articles = 17 × 300
4 4 13
3
= (4 × 𝑛) (𝑦 × 46) (13)
75 12 = 5100 Rs.
675 7. (d);ATQ
= 𝑛𝑦 × 598 𝑥 2𝑥 𝑥
675 500 [ 1 + 100] + 1500 [1 + 100] [1 − 100] =
𝑛𝑦× −𝑛𝑦
Hence profit % = 598
× 100 2000 [1 +
𝑥−6
]
𝑛𝑦
100
77 2𝑥 𝑥 2𝑥 2
= × 100 ≅ 12% profit 500 + 5𝑥 + 1500 [1 + − − ]=
598 100 100 100×100
𝑥−6
4. (c); Let the MRP of cycle is 6𝑥. 2000 [1 + ]
100
50 3𝑥 2
∴ discount is % of 6x = x 500 + 5𝑥 + 1500 + 15𝑥 − = 2000 + 20(𝑥 −
3 10
∴ Rahul bought this cycle for 5x 6)
He earns 2.5x by renting it and resells it at 3x. 3𝑥 2
ATQ 2000 + 20𝑥 − 10
= 2000 + 20𝑥 − 120
3x + 2.5x – 5x = 200 3𝑥 2
= 120
0.5x = 200 ⇒ x = 400 10
∴ MRP = 6x = 6 × 400 = Rs. 2400 x² = 400
⇒ x =20%
5. (d);Let the purchasing price of items A is B be Rs x –
50, Rs y – 75 respectively. Solutions (8-10): Let cost price = 100
When he applied Rs 50 & Rs 75 on them, their Profit percent = 𝑥%
actual CP becomes x & y So, selling price ⇒ 100 + 𝑥
ATQ M.P. ⇒ 100 + 2𝑥
10
x+
12
y = 84 …(i) Now if discount is 12.5%
100 100 1
Also, Then profit percentage increases by 33 3
% ⇒Profit
20 10 14 4 4
x + 100 y = 100 (x + y) percent = 𝑥 × = 𝑥
100 3 3
⇒ 20x + 10y = 14x + 14y ATQ,
4 7
⇒ 6x = 4y 100 + 3 𝑥 = 8 (100 + 2𝑥)
2
x= y 𝑥 = 30
3
41 Adda247 Publications For More Study Material
Visit: adda247.com
Cracker Book for Bank (IBPS | SBI | RRB PO | Clerk) Mains Exams

So, if cost price = 100 5


C.P. of article = 96x × 6
Selling price = 130
= 80x Rs.
M.P. = 160
2nd selling price
Profit = 30 7 4
30 = 160𝑥 × 8 × 5 = 112x Rs.
Discount % = × 100 = 18.75%
160 Given
8. (c); Profit on one articles = 30 112x – 80x = 800
Profit on 20 articles = 30 × 20 ⇒ 600 32x = 800
600 → 300 x = 25
1 → 0.5 cost price = 80 × 25 = 2000 Rs.
M.P. → 160 → 160 × 0.5 ⇒ Rs.80
13. (d);Total CP of one DI book = 125 + 15 + 40
9. (e); Let total quantity ⇒ 1000 = 180 Rs.
He gives → 800 → for → 100 C.P. of 1500 books = 1500 × 180
Cost price of → 800 → 80 750×180
SP of 750 book = 2
30
Now initial discount = × 100 = 18.75% Let required profit percent = x%
160
18.75×4 750×180 100+x
Reduced discount = = 15% ∴ + ( 100 ) × (750 × 180)
5 2
160×85 120
S.P. = = 136 = 100 × 180 × 1500
100
(136–80) 67500 + 135000 + 1350x = 324000
Profit = × 100 = 70%
80
1350x = 324000 – 202500
10. (b);Let cost price of 20 articles → 20 × 100 121500
x = 1350 = 90%
= 2000 475
Actual profit on 20 articles → 20 × 30 14. (a); CP of puzzle book = 125 × 100
= 600 = 380 Rs.
575
S.P. → 2600 CP of DI book = × 100
115
S.P. of each undamaged article
2600 = 500 Rs.
⇒ 20–7 = 200 For 30% profit on both
30 130
Initial discount = × 100 = 18.75% = (380 + 500) × = 1144
160 100
So M.P. of each article should be DI book should be sold
200 = 1144 – 360 = 784 Rs.
= 81.25 × 100 ≈ 246
Approximately markup% = 146% 15. (b);Let the MP of a chair and a table be Rs.5x and
Rs.8x respectively.
11. (d);Let total employee in Bankersadda = 100 And, the number of chairs and tables bought be 6y
Females employee = 30 and 5y respectively.
Male employee = 70 CP of a chair for Abhishek = (100 – 20)% of 5x
Employee who earned more than Rs. 17000 as = Rs.4x
monthly salary = 55 CP of a table for Abhishek = (100 – 25)% of 8x
Employee who earned less than Rs. 17000 as = Rs.6x
monthly salary = 45 Total CP for Abhishek = 4x × 6y + 6x × 5y
Total female employee earned less than = 24xy + 30xy = 54xy
Rs. 17000 as monthly salary SP of a chair for Abhishek = (100 – 25)% of
40 (100 + 50)% of 4x = 4.5x
= 30 × 100 = 12 SP of a table for Abhishek = (100 – 20)% of
Total male employee earned Rs. 17000 or less as (100 + 50)% of 6x = 7.2x
monthly salary = 45-12 = 33 Number of chairs sold in bunch of four by
33 2
Required fraction = 100 Abhishek = 3 rd of 6y = 4y
So, number of table sold for free by Abhishek
12. (b);Let original marked price be Rs 100x. 1
= 4 th of 4y = y
Then, New marked price of article
Total SP for Abhishek = 4.5x × 6y + 7.2x × (5y – y)
= Rs 160x.
= 27xy + 28.8xy = 55.8xy
Selling price of article 55.8xy – 54xy
7 6 4 Profit % = × 100
= 160𝑥 × 8 × 7 × 5 54xy
1.8xy 1
= 96x Rs. = × 100 = 3 %
54xy 3

42 Adda247 Publications For More Study Material


Visit: adda247.com
Cracker Book for Bank (IBPS | SBI | RRB PO | Clerk) Mains Exams

16. (c); According to the question, 20. (d);Let cost price of each mobile = 100x Rs.
MP of a table = 300 + MP of a chair Percent profit on first mobile
18750−100𝑥 18750−100𝑥
⟹ 8x = 300 + 5x ⟹ x = 100 = [( )] × 100 =
100𝑥 𝑥
Total CP for Abhishek = 108000 Loss percent on second mobile
⟹ 54xy = 108000 100𝑥−14250 100𝑋−14250
= [( )]× 100 =
⟹ 54 × 100 × y = 108000 ⟹ y = 20 100𝑥 𝑥
Number of chairs purchased by Abhishek ATQ –
18750−100𝑥 100𝑋−14250
= 6y = 120 =5×
𝑥 𝑥
Solutions (17-18): Let total cost of car X = 100𝑥 18750 − 100𝑥 = 500𝑥 − 71250
Cost of raw material = 25𝑥 600x = 90000 ⇒ x = 150 Rs.
(100–25𝑥)×20 Total cost price of both mobile = 2 × 15000 =
Cost of labor = = 15𝑥
100 30000 Rs.
Cost of engineers = 30𝑥 Total selling price of both mobile = 18750 +
Cost of machine = 21𝑥 14250 = 33000 Rs.
Cost of taxes = 9𝑥 33000−30000
Profit percentage = ×100 = 10%
Profit = 28𝑥 30000

So, proposed selling price = 128𝑥 21. (b);Let population of society A, B and C be 200x, 300x
25𝑥×13 and 500x respectively
17. (a); New cost of raw material = = 32.5𝑥 35
100
30𝑥
Female in society A = 100 × 200x = 70x
Now cost of each engineer = = 7.5𝑥
4 Male in society A = 200x – 70x = 130x
150
Cost on new engineer = 2 × 7.5𝑥 × In society B Let ‘a’ = Number of males
100
= 22.5 ATQ,
1 4𝑎
So new CP = 100𝑥 + 7.5𝑥 – 7.5𝑥 = 100𝑥 a + a = 300x ⇒ = 300𝑥
3 3
So, % change in S.P. = 0% ⇒ a = 225x = Number of males
15𝑥×120 Number of females = 300x – 225x = 75x
18. (d);Cost on labor for car Y = = 18𝑥
100 In society C,
Cost price of car Y = 103𝑥 23
Number of females = × 500x = 230x
128𝑥 320 50
Required ratio = 103𝑥×120 = 309 27
Number of males = 50 × 500x = 270x
19. (e); Let cost price of article A Rs. 112x and cost price Total number of female = 70x + 75x + 230x
of article B Rs. 135x = 375x
ATQ, Total number of male = 130x + 225x + 270x =
112𝑥
Selling price of article A = ×5 625x
4 625x –375x
= 140x Rs. Required% = 625x
× 100
135𝑥 250
Selling price of article B = 4 × 5 = 625 × 100 = 40%
= 168.75x Rs.
Let mark price of article A is 7y 22. (c); Let marked price of bike = 100x Rs.
And article B is 9y Selling price for store A
(100 –30)
100−𝑑 = 100x × = 70x
140x = 7y ×( ) ________(I) 100
100
100–(𝑑+5) Given, 70x = 49000 ⇒ x = 700
168.75x = 9y ×( 100
) ________(II) marked price of bike = 70000 Rs.
From (I) and (II) _________ ATQ—
100–𝑑
140𝑥 7𝑦×( ) Selling price of bike in store B=
= 100
95–𝑑 (100−𝑑) (100−8)
168.75𝑥 9𝑦×( ) Marked price of bike× ×
100
100 10
140×9 100–𝑑
= = 𝑠𝑒𝑙𝑙𝑖𝑛𝑔 𝑝𝑟𝑖𝑐𝑒 𝑜𝑓 𝑏𝑖𝑘𝑒 𝑖𝑛 𝑠𝑡𝑜𝑟𝑒 𝐴+7672
168.75×7 95–𝑑
16 100–𝑑 (100 –d) (100 –8)
= 70000 × 100
× 100
15 95–𝑑
16d – 1520 = 15d – 1500 = (49000 + 7672)
First discount d = 20% 644d = 64400 – 56672
7728
Second discount = (20 + 5) = 25% d = 644 ⇒ d = 12 %

43 Adda247 Publications For More Study Material


Visit: adda247.com
Cracker Book for Bank (IBPS | SBI | RRB PO | Clerk) Mains Exams
23. (e); Lets MP of article A = 100x Rs. 26. (b);Let marked price of article A and B be 400x and
Selling price of article A 500x respectively
(100−19)
= 100x × 100 = 81x Rs. ATQ—
10 (100–d) (100–d−18)
Selling price of article B = 81𝑥 × 9 400x × 100 = 500x × 100
= 90x Rs. 400 – 4d = 410 – 5d ⇒ d = 10%
90𝑥 90
Cost price of article B = 120 × 100 = 75x 400x×
Cost price of article A = 100
× 100
81𝑥 120
Cost price of article A = 125 × 100 = 300x Rs.
= 64.8x Rs. 500x×
(100–28)

Given, Cost price of article B = 125


100
× 100
81x – 75x = 840 = 288x Rs.
6x = 840 ⇒ x = 140 Rs. ATQ—
Cost price of article A = 64.8 × 140 (500x ×
72
– 288x) – (400x ×
90
– 300x) = 384
= 9072 Rs. 100 100
72x – 60x = 384 ⇒ x = 32
Solutions (24-25): Lets cost price of A, B and C type of Cost price of article A = 32 × 300 = Rs.9600
mobile be Rs. 5x, Rs. 7x and Rs. 9x respectively
130 Cost price of article B = 32 × 288 = Rs.9216
M.P. of type A mobile = 5𝑥 × = 𝑅𝑠. 6.5𝑥 27. (b);Total CP of one shoes = (500 + 300 + 100)
100
140
M.P. of type B mobile = 7𝑥 × 100 = 𝑅𝑠. 9.8𝑥 = 900
130
M.P. of type C mobile = 9𝑥 × 100 = 𝑅𝑠. 11.7𝑥 CP of 150 shoes = 900 × 150
900×75
S.P. of type A mobile CP of 75 shoes = 2
200 1 11
= 6.5𝑥 × (100– 13 ) × 100 = 6.5𝑥 × 13 = 𝑅𝑠. 5.5𝑥 Let’s required percentage =x
900×75 100+𝑥
S.P. of type B mobile ∴ +( ) × (900 × 75)
2 100
150 1 150
= 11.7𝑥 × (100– 7 ) × 100 = 𝑅𝑠. 7.7𝑥 = × 900 × 150
100
8
S.P. of type C mobile = 11.7 × = 𝑅𝑠. 10.4𝑥 = 33750 + 67500 + 675x = 202500
9
= 675x = 202500 – 101250
24. (d);Total cost price of Fourteen type A mobile
675x = 101250
= 14 × 5x = Rs. 70x 101250
Total selling price of fourteen type A mobile = 14 𝑥= ⇒ x = 150%
675
× 5.5x = 77x
Given total profit = Rs. 17500 28. (a); Let marked price of watch at shop P & Q be Rs.
7x = 17500 ⇒ x = Rs. 2500 140x and Rs. 100x respectively
Total profit made by store owner on type B and Selling price of watch at shop P
90 75
type C mobile = (7.7 × 20 – 7× 20) × 2500 + (10.4 = 140𝑥 × × = 94.5𝑥 𝑅𝑠.
100 100
× 15 – 9 × 15) × 2500 Selling price of watch at shop Q
= 14 × 2500 + 21 × 2500 (100−𝑑) 90
= 35000 + 52500 = Rs. 87500 = 100x × 100 × 100
25. (c); In the transaction — ATQ –
(100−𝑑) 90 44
Given, 94.5x - 100x × 100 × 100 = 94.5𝑥 × 105
In 48 type A mobile, 12 mobile are free.
9450 – 9000 + 90d = 3960
In 36 type B mobile, 6 mobile are free.
In 39 type C mobile, 9 mobile are free. 90d = 3510
3510
Total profit of store owner on selling of 48 type A d = 90 ⇒ d = 39%
mobile = 48 × 5x – 36 × 6.5x
= 240x – 234x = 6x 29. (d);Let cost price of one Mi mobile be Rs. 100x and
Total loss of store owner on selling of 36 type cost price of one Samsung mobile be Rs.120x
mobile = 36 × 7x – 30 × 9.8x Marked price of Samsung mobile
125
= 252x – 294x = 42x = 120x × 100 = 150x Rs.
Total loss of store owner on selling of 39 type C
Marked price of Mi mobile
mobile 140
= 9x × 39 – 11.7x × 30 = 351x – 351x = 0 = 100x × 100 = 140x Rs.
Total loss of store owner in this transaction = Selling price of Samsung mobile
42x- 6x = 36x 75
= 150x × 100 = 112.5 x Rs.
Total cost price = 48 × 5x + 36 × 7x + 39 × 9x
80
= 240x + 252x + 351x = 843x Selling price of Mi mobile = 140x × 100
36𝑥 76
Required% = 843𝑥 × 100 = 4 281 % = 112x Rs.
44 Adda247 Publications For More Study Material
Visit: adda247.com
Cracker Book for Bank (IBPS | SBI | RRB PO | Clerk) Mains Exams

ATQ– 33. (d);Let total votes be 6𝑥


112.5 x – 112x = 160 ⇒ x = 320 Rs. People who took part in survey
Marked price of one Samsung & one Mi mobile = 83⅓% × 6𝑥 = 5𝑥
= (150 × 320) + ( 140 × 320)
2.5𝑥 claims to vote for A,
= 48000 + 44800 = 92800 Rs.
30. (c); 0.5𝑥 are uncertain.
People who didn’t took part in survey
Male 60% Female 40% = 6𝑥 – 5𝑥 = 𝑥
2 2
And vote for A= × 𝑥 = 3𝑥
Government Employees Government employees 3
20
= 60 × 100 = 12%
40
= 40 × 100 = 16% Vote for A, from people who are uncertain.
1
= × 0.5𝑥 = 0.1𝑥
Private employees Private employees 5
5 40 2
= 48 × 12 = 20% = 24 × = 9.6% Total votes for A = 2.5x + 0.1x + 𝑥
100 3
25 1 20
Unemployed = 48 – 20 =28% Unemployed = 24 – 9.6 = 10 x + 10 x + 30 x
= 14.4% 75𝑥+3𝑥+20𝑥 98𝑥
Given — = =
30 30
28% – 14.4% = 3400 ⇒ 13.6% = 3400 98𝑥 82𝑥
Votes for B = 6𝑥– 30
= 30
Government (male + female) employees
= 12% + 16% = 28% ATQ,
98𝑥 82𝑥 16𝑥 16𝑥
Government (male + female) employees – = ⇒ = 640
3400 30 30 30 30
= 13.6 × 28 = 7000 ⇒ 𝑥 = 40 × 30 = 1200
31. (d);Let the savings of A & B be Rs 8 x and Rs 8y Hence total votes are 6 × 1200 = 7200
respectively. Then amount invested by A in XYZ 34. (d);Let total population of village = 100x
37.5
scheme = × 8𝑥 = 3x Male population = 60x
100
62.5 1
Amount invested by B in XYZ = 100 8y = 5y Male above 60 year = 60x × 15 = 4x
ATQ, 1
⇒ 6x = 5y Male below 18 years = (60x – 4x) × 7 = 8x
5 6 Male above 18 but below 50 years
x = y or y = x
6 5 5
Now B invested 50% of (8y – 5y) =
1
× 3y = (56x – 8x) × 8 = 30x
2
= 1.5y in scheme PQR. Male above 50 year but below 60 years
A wants to invest 2× 1.5y = 3y = (56x – 8x – 30x) = 18x
6 18 𝑥
=3×5x= 5 ATQ—
His remaining savings = 8x – 3x = 5x 30x – 4x = 7800
18x
26x = 7800 ⇒ x = 300
% he could invest in PQR = 5
× 100
1800
5x Female population in village
= 25
% = 72% = 300 × 40 = 12000
32. (e); He bought total of 50 kg sugar. 35. (d);Let marked price of Jeans be Rs. 9x and shirt be
And ratio of quantity of both is given as 2:3:5. Rs. 7x for Divyaraj
Therefore, he have 10 kg of X, 15 kg of Y and 25 8
Kg of Z. CP of jeans for Divyaraj = 9x × 9 = 8x Rs.
And cost price of X, Y & Z is Rs. 30/kg, Rs. 40/kg 6
CP of shirt for Divyaraj = 7x × 7 = 6x Rs.
and Rs. 50/kg
Profit earn on 1kg of X is
20
× 30 = Rs. 6 Let total number of jeans purchased by Divyaraj
100
10 be 5y and shirt be 8y
Loss earn on 1 kg of Y is 100 × 40 = Rs. 4
Total cost price for Divyaraj
Profit earn on 1 kg of Z is 32/100] × 50 = 8x × 5y + 6x × 8y
= Rs. 16
But he sold all of X & Y and 40 – (10 + 15) =15 kg = 88xy
of Z. SP of jeans on, which Divyaraj sold
3 3
∴ Total amount of profit earned is = 8x × × = 9x
2 4
6 × 10 – 4 × 15 + 16 × 15
SP of shirt on, which Divyaraj sold
= 60 – 60 + 240 = Rs. 240

45 Adda247 Publications For More Study Material


Visit: adda247.com
Cracker Book for Bank (IBPS | SBI | RRB PO | Clerk) Mains Exams
3 8 6.4𝑥 × 200 = N × 𝑥 + 10x (200 − N)
= 6x × 2 × 9 = 8x
1280 = N + 2000 – 10N
Total S.P. on which Divyaraj sold all items
9N = 920
= 9x × 5y + 8x × 8y
N = 80 kg
= 45xy + 64xy = 109xy 80 2
109xy –88xy Required ratio = =
Required% = × 100 120 3
88xy
= 23
19
% 39. (b);Let marked price of jeans is 300𝑥 Rs. while
22 marked price of shirt is 200𝑥.
36. (b);Let cost of each mobile be Rs. ‘a’ in 2016 and Selling price of a jeans & a shirt in scheme A
65
number of mobile sold in 2016 be ‘b’ = (300𝑥 + 200𝑥) × 100
Total revenue generated by store in 2016 = 325𝑥
= ab Rs. Selling price of one jeans & one shirt in scheme B
In 2017 — 75 80
Cost of each mobile = 0.75a Rs. = 300𝑥 × + 200𝑥 ×
100 100
Total revenue generated by store in 2017 = 225𝑥 + 160𝑥
105
= 100 × ab = 94.5 lakh = 385𝑥 Rs.
Given, 385𝑥 – 325𝑥 = 384 Rs.
ab = 90 lakh …(i) 60𝑥 = 384 Rs.
Total number of mobile sold by store in 2017 𝑥 = 6.4 Rs.
1.05ab
= 0.75a = 1.4b Cost price of a shirt = 160𝑥 ×
3
4
Given, 3
Number of mobile sold in 2017 – number of = (160 × 6.4 × 4)
mobile phone sold in 2016 = 90 = 768 rs.
4
1.4b – b = 90 Cost price of jeans = 225𝑥 × 5
b = 225 4
=225 × 6.4 ×
Cost price of each mobile in 2016 5
90,00,000
= 225 = 40000 Rs. = 1152 Rs.
40. (b);Let, total manufactured cost of a cycle
37. (d);Let the marked price of a Shirt = 500 Rs = 100x Rs.
And the marked price of a Jeans = 700 Rs Then S.P a cycle = 125x Rs.
Also, no. of shirt bought by Veer = 9x Expenditure on raw materials
And no. of jeans bought by Veer = 8x = 100x × 0.40 = 40𝑥 𝑅𝑠.
4
C.P. of shirt for Veer = 5 × 500 = 400 Rs. Expenditure on labour = 100𝑥 × .20
3
C.P. of jeans for Veer = × 700 = 525 𝑅𝑠. = 20𝑥 𝑅𝑠.
4 Fixed charge = 100𝑥 × .20 = 20𝑥 𝑅𝑠.
Total C.P. of shirt and jeans for Veer = 9𝑥 × 400 +
Expenditure on transportation
8𝑥 × 525 = 7800𝑥 Rs.
= 100x − (40𝑥 − 20𝑥 − 20𝑥) = 20𝑥 𝑅𝑠.
Total S.P. of shirt and jeans for Veer =
New expenditure on raw materials
7𝑥 × 400 × 1.5 × 0.75 + 8𝑥 × 525 × 1.5 × 0.8
= 1.15 × 40x = 46x Rs.
= 3150𝑥 + 5040𝑥
New expenditure on labour
= 8190𝑥 Rs.
8190𝑥−7800𝑥 = 1.2 × 20x = 24x Rs.
Profit % = 7800𝑥
× 100 New expenditure on transportation
390
= 7800 × 100 = 5% = 1.5 × 20x = 30x Rs.
7
To get a profit of 13 11 % at a selling price of Rs.
38. (b);Let cost price per kg of Type B wheat = x Rs.
125x, the C.P. of an item must be equal to
And, Cost price per kg of type B wheat 1100
= 10x Rs. 125𝑥 × 1250
Selling price of mixture of Type A & Type B wheat = 110𝑥 Rs.
= 11x Rs. We need to reduce C.P. (or expenditure) by 10,
Cost price per kg of mixture of Type A & Type B and this 10 has to be reduced from expenditure
32 on raw materials.
wheat = 11x × 55 = 6.4𝑥 𝑅𝑠.
10𝑥
ATQ – Required % = 46𝑥 × 100
17
Let N kg of type B wheat in mixture of 200 kg of = 21 23 %
Type A & Type B of wheat

46 Adda247 Publications For More Study Material


Visit: adda247.com
Cracker Book for Bank (IBPS | SBI | RRB PO | Clerk) Mains Exams

Chapter
Simple Interest and Compound Interest
6
BEST APPROACH TO SOLVE THE QUESTIONS

The interest can be defined as amount paid by the borrower to the lender in addition to the amount which he had
borrowed. In competitive exams, interest is classified into Simple interest (SI) and Compound Interest (CI).
Easiest example, when we borrow money from bank, it doesn’t say pay Rs. 100 as interest every year no matter whether
you took a loan of Rs 1000 or Rs 100000. They offer us a rate generally in % at which we have to pay interest. This rate
is known as Rate of interest (ROI).
For eg. If I borrowed Rs. 500 for 1 year at a rate of 8%/annum. Find the interest that I have to pay.
Always remember, this 8% we have to calculate on the amount we borrowed. 8% of Rs. 500 means Rs. 40, that I have to
pay as an interest.
Note: Different forms of ROI may be given or we have to change, as 3% for 3 months or 24% for 18 months. For easier
calculation we can normalize these in 12 month/year rate. In above rates, former means 12%/annum and later means
16%/annum.
Now, if I asked what would be amount of interest that I have to pay, If I borrow Rs. 500 for 3 years at same rate?
Here arise difference b/w SI and CI.
SIMPLE INTEREST:
Simple interest simply means that we have to pay same amount of interest every time. If I have to pay interest on Rs. 500
at ROI of 8%/annum for 3 years, it means I will pay
40 × 3 = RS 120.
Formula to calculate SI on an amount is
PRT
SI = 100
Here, P is the amount borrowed and generally it is referred as Principal.
R is ROI in %. (100 in equation is used to because of %)
And T is time in years.
NOTE: If it is asked in a question, that what is interest obtained at the ROI of 10%(or any value) for 5 years on amount of
Rs. Z.
In order to save time first calculate mentally 10 × 5 = 50%. Total interest will be 50 % of Principal amount.

COMPOUND INTEREST:
The one liner difference b/w SI and CI is that interest paid for every year is not same. Consider the above example.
Interest accrued in 1st year = Rs. 40
In CI principal amount changes, after 1year principal amount will be Rs. 540 not Rs. 500.
8
After second year interest obtained on Rs. 540 is 100 × 540 = Rs. 43.2
8
And interest after 3 years will be calculated on Rs 540+Rs 43.2= Rs 583.2 i.e 100 × 583.2= Rs. 46.65
And total interest will be 40 + 43.2 + 46.65= Rs 129.85
The formula for amount obtained after the given time period in CI
𝐑 𝐧
𝐀 = 𝐏 (𝟏 + 𝟏𝟎𝟎)

Here A is amount obtained after interest, P is principal, n is number of years R is ROI.


CI= A – P.

47 Adda247 Publications For More Study Material


Visit: adda247.com
Cracker Book for Bank (IBPS | SBI | RRB PO | Clerk) Mains Exams
In competitive exams, this approach is rarely used because it will be too calculative if time is 3 years and ROI is 17%.
17 3 117
A = P (1 + ) = P( )3 Imagine if you started calculating cube of 117 .
100 100

Best approach to use in CI is resultant ROI, it makes CI as easy as SI. Eg.


If ROI on Rs.10000 in a scheme is Y%, calculate total interest after two years. Then resultant interest is
Y2
Y + Y + (Y × Y)/100 = 2Y +
100
17×17
Resultant interest % on 17%= 2×17 + = 34+ 2.89= 36.89%.
100
It is more time saving than traditional method. Little use of mind will help you to solve all kind of problems.
Note that if we can calculate resultant interest % for two years, we can also calculate it for three years. Actually general
formula for calculating resultant % for two years, if ROI on first year is X% and second year is Y%, then resultant interest
% will be
X×Y
X+Y+
100
For calculating 3year resultant interest, first calculate resultant interest of first two years and then again use resultant
formulae for adding resultant of first two year and ROI of 3rd year.
REMARK: Check 3 years total interest % for ROI 0f 10% whether it comes 33.1%.
DIRECT FORMULAE TO CALCULATE 3 YEARS RESULTANT INTEREST %
3r. 3 r² r³ __
Here, r is ROI
Note that
4 underscores (_ _ _ _) represent 4 digits after decimal. Further, make it clear that _ _ (last two digits are for last two digits
of value of r³]
And starting two (_ _) are for value of 3r².
Consider eg.
3r. 3 r² r³_
When r = 2
r³ = 08, r² = 4, 3r² = 12
Resultant interest is for 3 years
6.1208
Now consider r = 8
r³ = 512, r² = 64, 3r² = 192
use last two digits of r³ for last two digits.
i.e. 3r. _ _ 12, and carry on 5 to 3r².
3r² = 192 + 5 = 197.
Here also, use only its last 2 digit and carry on 1.
3r. 9 7 1 2
3r = 24 + 1 → carry on from 197.
Resultant interest % = 25.9712%
Check for r = 10%.

While solving a question, student should always keep in mind resultant interest %.
Example: Amount of Rs 4000 becomes Rs 6600 in SI in 5 years. Calculate ROI.
PRT
Solution: Though general formula of SI i.e. SI = is an easy approach,
100
But try mentally to solve RS 2600 is what % Rs 4000. You can mentally calculate that
2600 13
4000
= 20 which means 65%.
And 65% in 5 years in SI means 13%/year.
Example: Difference b/w interest obtained in SI and CI on an amount at the ROI of 10% is Rs. 310. Find the principal
amount.
Solution:
Calculate resultant % for SI and CI for 3 years at ROI of 10%.
Its 30% for SI and 33.1% for CI. But we know that we calculate interest on principal. It means that 3.1% of principal is
equal to Rs 310. Hence principal can be calculated.
POINTS TO REMEMBER ALWAYS:
Difference b/w interest obtained on CI and SI for 2 years is PR2/100.
Resultant interest % in CI for 3 years at 10% ROI is 33.1% and 15.7625% at ROI of 5%.
48 Adda247 Publications For More Study Material
Visit: adda247.com
Cracker Book for Bank (IBPS | SBI | RRB PO | Clerk) Mains Exams

Practice Exercise Based on new Pattern

1. Ankur invested X Rs. at the rate of 15% per annum on 7. A person invested a certain amount at simple interest
compound interest for two years and gets total at the rate of 6 per cent per annum earning Rs. 900 as
interest of 5805 Rs. if Ankur invest (X + 7000) Rs. for an interest at the end of three years. Had the interest
another two year at additional rate of 5%, then what been compounded every year, how much more
will be compound interest on that investment ? interest would he have earned on the same amount
(a) 10000 Rs. (b) 11000 Rs. (c) 12000 Rs. with the same interest rate after three years?
(d) 15000 Rs. (e) 18000 Rs. (a) Rs. 38.13 (b) Rs. 25.33 (c)Rs. 55.08
(d) Rs. 35.30 (e) None of these
2. Shikha invested 32000 Rs. at simple interest for 2 1
years at the rate of R% and gets an interest of 8. Mayank borrowed Rs. 48000 from a bank at 12 2 %
Rs.8000. If he invested total amount (Principle + per annum compound interest at the end of 1st, 2nd
Interest) in a scheme, which offered compound and 3rd year, he paid 14000, 13000 and 16000
interest at the rate of (R%+2.5%) then find total respectively. If he wanted to clear his loan at the end
of 4th year, what would he pay at the end of the fourth
compound interest obtained by Shikha after 2 years ?
year to clear his loan ?
(a) 12600Rs. (b) 12800Rs. (c) 14400 Rs.
(a) 21000 (b) 22500 (c) 16800
(d) 12000 Rs. (e) 12900Rs.
(d) 26000 (e) None of these
3. Ankit borrowed Rs. 12000 from Veer on C.I. at 10% 9. Rakesh lent Rs. P to Rahul on an agreement that for
per annum for three years and added some extra first two year, interest will be calculated on SI at 15%
amount and lent to Arun on C.I. at 20% per annum for per annum and for next two years interest will be
two year. If Ankit got Rs. 3948 more interest than calculated on C.I at 5% additional rate. If Rahul paid
interest got by Veer.Find how much extra amount was total amount of Rs. 17971.2 at the end of four year,
added by Ankit ? Find the amount borrowed by Rahul?
(a) 6000 Rs. (b) 4000 Rs. (c) 5000 Rs. (a) 9000 Rs. (b) 9800 Rs. (c) 9900 Rs.
(d) 8000 Rs. (e) 9000 Rs. (d) 9600 Rs. (e) 9200 Rs.

4. Abhi invested some amount on scheme ‘P’ which offer 10. Veer invested Rs 22500 for 2 year at the Rate of x% in
CI at the rate of 15% p.a.. After 2 years he got Rs. 1032 scheme A at compound interest annually and gets a
as interest. Abhi invest the amount he got from total amount of Rs 32400 If he added Rs 2600 in
amount and invested total amount in scheme B at S.I.
scheme ‘P’ in scheme ‘Q’ which offer 15% p.a. at SI for
for 3 year at same rate . Then find the total simple
4 years. Find the total interest he earned from scheme
interest veer gets from scheme B?
P and scheme Q together?
(a) Rs 22500 (b) Rs 22000 (c) Rs 17500
(a) 3571.2 (b) 3715.2 (c) 4232 (d) Rs 20000 (e) Rs 21000
(d) 3148 (e) 3379.2
11. Veer Invested X Rs. in SBI at the rate of 18% for 2 year
5. A man invests Rs. 1,200 at 10% p.a. At the end of the and obtained a total simple interest of 6750 Rs. If he
year he withdraws 30% of total amount and pays Rs. invested 2250 Rs. more for same period time at the
24 as transaction fee. At the end of 2nd year he 2 2
rate of 14 % for first year and at 16 % for 2nd year
7 3
withdraws 30% of the amount and pays Rs. 93 as on compound annually. Then find the total compound
transaction fee. What is the balance at the end of the interest obtained by Veer after 2 year ?
third year? (a) 6000 Rs. (b) 5000 Rs. (c) 3000 Rs.
(a) Rs. 660 (b) Rs. 825 (c) Rs. 500 (d) 7000 Rs. (e) 5500 Rs.
(d) Rs. 770 (e) None of these
12. Satish invested 16000 Rs. in simple interest for 2
6. A sum of Rs. 1000 after 3 years at compound interest years on certain rate and gets an interest of 4800 Rs,
of 10% p.a. becomes a certain amount that is equal to if he invested total amount (Principle + Interest) in a
the amount that is the result of 3 year depreciation scheme, which offered compound interest on 5%
from Rs. 1728. Find the approximate difference more interest rate as earlier rate. Then find total
between the rate of C.I. and rate of depreciation. interest gets by Satish after 2 years ?
(a) 3.33% (b) 0.66% (c) 4% (a) 9252 Rs. (b) 9225 Rs. (c) 9512 Rs.
(d) 2% (e) 6% (d) 925 Rs. (e) 9152 Rs.

49 Adda247 Publications For More Study Material


Visit: adda247.com
Cracker Book for Bank (IBPS | SBI | RRB PO | Clerk) Mains Exams

13. Bhavya and Veer invested their principle in two 19. Rahul took a loan of Rs.80000/- at rate of of 5% p.a. at
different schemes, Bhavya invested X Rs. on SI & lent it at 5% pa at CI. After 3 years, he cleared all
compound interest for two year at rate of 20% his debts and invested the profit earned in a scheme
annually and Veer invested 4000 Rs. more than which offers C.I. After two years he got Rs.128.1/- as
Bhavya on simple interest for three year at 15% interest from the scheme. Find the rate of interest
annually, if both gets total interest of Rs. 9632, then offered by scheme.
Find the amount invested by Veer ? (a) 8% (b) 7% (c) 6%
(a) 12900 (b) 12400 (c) 8800 (d) 10% (e) 12%
(d) 12800 (e) 12600
20. A man invested his total savings in two parts. He
14. A man invested Rs. x, y and z in three difference invested Rs. 30000/- in a scheme which offers CI at
schemes which offers 10% p.a., 20% p.a. and 30% p.a. ROI of 10% per annum and remaining in SI at ROI of
rate respectively and after one year he earns Rs. 2000 5%per half year. After two years the amount obtained
interest. If x, y and z are in arithmetic progression and by him in former scheme is half of later, then calculate
sum of x, y and z is Rs. 9,000. Find the difference amount invested in later scheme.
between ‘x’ and ‘z’. (a) Rs. 242000 (b) Rs. 60500 (c) Rs. 12100
(a) 1500 (b) 2000 (c) 2500 (d) Rs. 80525 (e) None of these
(d) 2250 (e) 1750
21. Satish saved 55% of his monthly income, which he
15. P invested Rs. X in a scheme for 2 year which offered further invested in two different schemes A & B in the
simple at the rate of 15% per annum and Q invested ratio of 7 : 5 on CI compounded annually at the rate of
Rs. (X + 2500) in another scheme for same period of 10% p.a. and 20% p.a. respectively. If after two years,
time, which offered compound interest at the rate of Satish gets total interest of Rs. 8074 from both the
20% per annum. If from both scheme P and Q got total schemes, then find the total annual saving of Satish?
interest of Rs. 32550, then find the value of X ? (a)316,600 Rs. (b)316,400 Rs. (c)316,000 Rs.
(a) 41500 (b) 42500 (c) 40500 (d)316,200 Rs. (e)316,800 Rs.
(d) 40000 (e) 38250
22. Aman invest Rs. (x – 2000) in scheme ‘P’ which offers
16. Rajat have some money in his hand. He invested 20% 20% CI pa while Rs. (x + 2000) in scheme ‘Q’ which
of the money in scheme ‘A’ for 4 year at 6% p.a., 30% offers 8% SI pa. After 2 year difference between
of the amount in scheme ‘B’ for 6 year at 12% p.a., interest earn from scheme P and Q is 1600, then
remaining in scheme ‘C’ 2 year at 15% p.a. If total amount invested by Aman in scheme ‘Q’ is what
amount Rajat received from scheme ‘A’, ‘B’ and ‘C’ is percent more/less than amount invested by him in
11,355, then find the difference of sum invested in scheme ‘P’ ?
scheme ‘B’ and scheme ‘A’. (a) 75%
1
(b) 133 % (c) 150%
3
(a) 1800 (b) 1500 (c) 1200 1
(d) 750 (e) 270 (d) 50% (e) 33 %
3

17. Satish borrowed Rs. 24000 from a SBI at the rate of 23. Ritu invested her total saving in three different FD
1 schemes A, B and C in the ratio of 5 : 4 : 6 on CI for
12 % compounded annually. If at the end of 1st, 2nd
2
two years at the rate of 10%, 15% and 20%
and 3rd year, he paid Rs. 7000, Rs. 6500 and Rs. 8000
respectively.If interest is calculated annually and
respectively. If Satish wanted to clear his loan at the
interest from scheme B is Rs. 744 more than interest
end of 4th year, what would he pay at the end of the
from scheme A then, find difference between interest
fourth year to clear his loan ?
received from scheme C and scheme B by Ritu?
(a) 11500 Rs. (b) 11250 Rs. (c) 10500 Rs.
(a) Rs. 4185 (b) Rs. 4175 (c) Rs. 3840
(d) 9500 Rs. (e) 14500 Rs.
(d) Rs. 4580 (e) Rs. 3250
18. Divyaraj invested an amount into two parts in the
24. Raghv invested Rs. 8400 and Rs. 9600 at the rate of
ratio of 4 : 3 on compound interest for two years at
R% and (R + 5)% respectively on simple interest for
the rates of 20% & 15% respectively. If he exchange
two years and gets total interest of Rs. 6360. If Raghv
rate of interests, then he will get Rs. 705 less interest
invested Rs.(8400 + P) and Rs.(9600 + P) at the rate of
than earlier interest, then find how much Simple
R% and (R + 5)% respectively on compound interest
interest he will get, if he invest total amount at the
for two year, then he would get total interest of Rs.
rate of 17.5% for two years?
8153. Find value of P?
(a) 14500 Rs. (b) 14700 Rs. (c) 14900 Rs.
(a) 1800 Rs. (b) 1600 Rs. (c) 2200 Rs.
(d) 14800 Rs. (e) 14900 Rs.
(d) 2400 Rs. (e) 2600 Rs.

50 Adda247 Publications For More Study Material


Visit: adda247.com
Cracker Book for Bank (IBPS | SBI | RRB PO | Clerk) Mains Exams

Directions (25-26): A person invested Rs. 20000 in a 31. Veer invested Rs. x in SBI for three years on S.I. at the
bank which is offering 10% per annum simple interest. rate of 15% per annum and Sameer invested Rs. (x +
After two years he withdrew the money from the bank 4000) in BOB for two years on C.I. at the rate of 8%
and deposited the total amount in another bank which per annum. If Divyaraj invested equal to sum of what
gives an interest rate of r% p.a. compounded annually. Veer and Sameer invested in UBI for two years on C.I.
After 2 years he received an amount of Rs. 2460 more at the rate of 20% per annum. Find the sum invested
than what he had invested in that bank. by Divyaraj, if they all got total interest of Rs.
25. What is the value of r?
20382.4?
(a) 10% (b) 15% (c) 5%
(a) Rs. 20000 (b) Rs. 18000 (c) Rs. 28000
(d) 12% (e) None of these
(d) Rs. 30000 (e) Rs. 26000
26. If the person had invested Rs. 50,000 instead of
20000 in the bank that offered simple interest, what 32. Three employee Ankit, Veer and Sameer got Rs. P, Rs.
would have been his net profit after following the (P+2400) and Rs. (P+4400) Diwali bonus. Ankit and
same procedure as given above? Veer invested their half of bonus on CI at the rate of
(a) Rs. 16,800 (b) Rs. 16,150 (c) Rs. 16,350 10% and 20% respectively for two years. Sameer
(d) Rs. 16,000 (e) None of these invested 60% of his bonus on simple interest at the
rate of 15% p.a. for three years. If Sameer got Rs. 132
27. A bank offers 10% p.a. at CI in scheme ‘A’, 20% p.a. at
CI in scheme ‘B’ and 40% p.a. at CI in scheme ‘C’. Veer more as interest got by Ankit and Veer together, then
have same amount in his hand. He invested 32% find the bonus got by Sameer?
amount in scheme ‘A’, 20% in scheme ‘B’ and (a) 12000 Rs. (b) 10000 Rs. (c) 14400 Rs.
remaining in scheme ‘C’. After two year he received (d) 14000 Rs. (e) 18000 Rs.
6600 as interest. Find the difference between amount 33. Manoj lend Rs. P for three years on S.I. at the rate of
invested by Veer on scheme ‘C’ to that of in scheme 15% per annum and Rajesh lend Rs. (P + 8000) for
‘B’.
two years on C.I. at the rate of 8% per annum. Suresh
(a) 5,000 (b) 4,000 (c) 2,000
(d) 3,000 (e) 2,500 borrowed sum equal to of what Manoj and Rajesh
lend, for two years on C.I. at the rate of 20% per
28. Bhavya borrowed Rs. 37500 on compound interest annum. If Suresh paid interest Rs. 5352 more than,
with agreement that, he will not pay amount after 2 what Manoj and Rajesh got total interest on their
year with interest then the rate of interest increase by sums together. Find total sum borrowed by Suresh?
5% as earlier interest rate. If Bhavya paid Rs.
12093.75 as total compound interest after two year.
(a) Rs. 34000 (b) Rs. 44000 (c) Rs. 32000
Then find how much amount Bhavya have to pay after
two year for completing his entire borrowing? (d) Rs. 46000 (e) Rs. 30000
(a) Rs. 58000 (b) Rs. 52000 (c) Rs. 42000 34. A person invested his savings of Rs. 32000 in two
(d) Rs. 45000 (e) Rs. 54000 parts at equal rate of interest in CI and SI. After 3
29. A man invested ‘X’ amount in scheme A and ‘Y’ years, he withdrew amount invested in CI, and put the
amount in scheme B. Scheme A offers 8% p.a. at S.I.
cash in his almirah. After another 2 years, the amount
scheme B offer 20%. p.a. at C.I. Man invested both
invested in SI got doubled and he also withdrew them.
schemes for 2 years. The sum of interest he earns
after 2 years will be equal to the simple interest he If the final ratio of both the amount is 36:65, then find
can earn if he invested ‘Y’ amount in scheme A for 7 amount invested by him in SI (in Rs.)
years. Find the total amount he has initially if (a) 20000/- (b) 18050/- (c) 19500/-
difference between interest earned by him after 2 (d) 21500/- (e) can’t be determined
years is 2304. 35. A man invested an amount into three schemes P, Q
(a) 12,300 (b) 12,900 (c) 12,600 and R in the ratio of 5 : 6 : 9 at the rates of 20%, 15%
(d) 13,500 (e) 13,800
and 10% respectively. Schemes P and R offered
30. Shikha and Sameer have equal amount. Shikha compound interest annually, while scheme Q offered
invested on C.I. for two years at the rate of 10% p.a. simple interest annually. Total interest received by
2
and Sameer invested 66 3 % of amount at the rate of man from scheme P and Q together is Rs. 1899 more
R% p.a. on simple interest and remaining amount at than total interest received from scheme R after two
the rate of 6.5% p.a. on simple interest. If interest years. If man would invest total amount on C.I at the
received by both at the end of two years are equal, rate of 15% p.a., then find the total interest received
then find the value of ‘R’? by the man?
(a) 10% (b) 12.5% (c) 15% (a) 5800 Rs. (b) 5805 Rs. (c) 5580 Rs.
(d) 12% (e) 8% (d) 5590 Rs. (e) 5900 Rs
51 Adda247 Publications For More Study Material
Visit: adda247.com
Cracker Book for Bank (IBPS | SBI | RRB PO | Clerk) Mains Exams

Solutions
2
15 = 900 × 1.1 = Rs. 990
1. (b); ATQ, 5805 = X [(1 + 100) – 1] 30
Withdrawal = × 990 + 93 = Rs. 390
129X = 5805 × 400 100
∴ Amount at the end of 3 years
X = 18000 Rs.
= 600 × 1.1 = Rs. 660
Now
Total Interest=(18000+7000) 6. (d); The amount @ 10% C.I. could become Rs. 1331.
(15+5) 2 Also,
×[(1 + ) – 1]
100 Rs. 1728 depreciated at R% has to become Rs.
11
= 25000 × (25) 1331.
Thus,
= 11000 Rs.
100−R 3
1728 × [ ] =1331(approximately).
2. (e); Let shikha invested at the rate of R% 100
ATQ— The closest value of R = 8%
8000×100 Thus, the difference is 2%.
R= R = 12.5%
32000×2 900 ×100
New Rate = 12.5 + 2.5 = 15% 7. (c); P = 6 × 3
Total amount = 32000 + 8000 = 40000 Rs. P = 5000
Equivalent CI of two years at the rate of 15%
15×15
= 15 + 15 +
100
= 32.25%
32.25
Required compound interest = 40000 × 100 =
12900 Rs. Required value = 955.08 – 900 = 55.08
10
3. (a); Interest got by Veer=12000[(1 + 100)3 − 1] 8. (b); 1st year CI
1
= 3972 48000 × 8 = 6000
Two year CI on 20% = 20 + 20 +
20×20 Amount = 48000 + 6000 = 54000
100 2nd year principle = 54000 – 14000 = 40000
= 44% 2nd year CI =
Atq, 1
44 40000 × = 5000
8
(12000 + 𝑥) − 3972 = 3948 Amount = 40000 + 5000 = 45000
100
5280+.44x – 3972 = 3948 3rd year principle
.44x = (3948 + 3972) – 5280 = 45000 – 13000 = 32000
.44x = 2640 3rd year CI
1
x = 6000 Rs = 32000 × 8 = 4000
4. (a); Let, sum Abhi have initially = 100x Amount = 32000 + 4000 = 36000
ATQ, 4th year principle = 36000 – 16000 = 20000
100x ×
115
×
115
– 100x = 1032 At the end of 4th year amount
100 100 1
= 20000 × 8 + 20,000
132.25x – 100x = 1032
1032
⇒ x = 32.25 = 32 = 2500 + 20000
= 22500
Amount initially Abhi have = 3200
Interest earned from scheme Q =
4232×15×4 9. (d); For first two years total S.I = 15 × 2
100 = 30%
= 2539.2 For next two years total C.I on (15+5)%
Required amount = 2539.2 + 1032 20×20
= 3571.2 = 20+20+ 100
= 44%
5. (a); The man invests Rs. 1,200 at 10% p.a. Amount after two year
At the end of 1st year the amount = Rs. 1,320 𝑃×30
30 = P + 100
Withdrawal 100 × 1320 + 24 = Rs. 420 13𝑃
Amount at the end of second year = Rs.
10

52 Adda247 Publications For More Study Material


Visit: adda247.com
Cracker Book for Bank (IBPS | SBI | RRB PO | Clerk) Mains Exams
10 20 30
Amount after 4 years × x + 100 × 3,000 + 100 × z = 2000
13𝑃 144 100
10
× 100 = 17971.2 0.1x + 0.3z = 2000 – 600 = 1400
17971.2 and x + z = 6,000
P= 1.872
0.1x + 0.3z = 1400 …(i)
P = Rs. 9600
x + z = 6,000 …(ii)
10. (e); ATQ, on solving (i) & (ii), we get
𝑅 2 x = 2,000
32400 = 22500 (1 + ) z = 4,000
100
324 100 + 𝑅 2 Difference between x & z = 2,000
=( )
225 100 15. (b); ATQ
18 100 + 𝑅 15×2x 20 2
= + (x + 2500) [(1 + ) – 1] = 32550
15 100 100 100
15R = 300 0.3x + 0.44x + 1100 = 32550
R = 20% 0.74x = 31450
20×3 31450
Simple interest = (32400 + 2600) × 100 x = 0.74
= 21000 Rs x = 42500

11. (d); Principle =


X×18×2
= 6750 16. (d); Let the amount Rajat have = 100
100
6750×100 ATQ,
⇒ X = 18×2 20×4×6 30×6×12 50×2×15
100 + + + = 11,355
100 100 100
X = 18750 Rs.
151.4 = 11355
According to question —
1 ⇒ 100 = 7500
1st year CI = (18750 + 2250) × 7 7500[30 –20]
Required difference = 100
= 3000
1 = 750
2 year CI = (21000 + 3000) ×
6
1 17. (b); 1st year CI
= 24000 × 6 1
24000 × 8 = 3000 Rs.
= 4000
Total CI after 2 year = 3000 + 4000 Amount = 24000 + 3000 = 27000 Rs.
= 7000 Rs. 2nd year principle = 27000 – 7000 = 20000 Rs.
2nd year CI =
4800×100 1
12. (e); Rate = 16000×2 20000 × 8 = 2500 Rs.
R = 15% Amount = 20000 + 2500 = 22500 Rs.
New rate = 15 + 5 = 20% 3rd year principle
2 year CI on 20% = 22500 – 6500 = 16000 Rs.
20×20
= 20 + 20 + 3rd year CI
100 1
= 44% = 16000 × 8 = 2000
Interest gets Satish Amount = 16000 + 2000 = 18000 Rs
44
= (16000 + 4800) × 100 4th year principle = 18000 – 8000 = 10000 Rs.
= 9152 Rs. At the end of 4th year amount, which Satish have
to pay
13. (d); ATQ, 1
= 10000 × 8 + 10,000
(X+4000)45 44X
100
+ 100 = 9632 = 1250 + 10000 = 11250 Rs.
0.45X + 1800 + 0.44x = 9632
0.89X = 9632 – 1800 18. (b); Let Divyaraj invested total amount = 7x Rs.
7832 Equivalent CI of two year at the rate of 20% = 20
X= 20×20
0.89 + 20 + 100 = 44%
X = 8800
Veer principle = 8800 + 4000 Equivalent CI of two year at the rate of 15% = 15
15×15
= 12800 Rs. + 15 + = 32.25%
100

14. (b); x + y + z = 9,000 ATQ –


44 32.25 32.25 44
& 2y = x + z 4x × + 3𝑥 × − 4𝑥 × − 3𝑥 × =
100 100 100 100
⇒ y = 3,000 705

53 Adda247 Publications For More Study Material


Visit: adda247.com
Cracker Book for Bank (IBPS | SBI | RRB PO | Clerk) Mains Exams

(1.76x + .9675x) – (1.29𝑥 + 1.32𝑥) = 705 ATQ—


2.7275x – 2.61𝑥 = 705 385𝑥 21 275 44
× + × = 8074
12 100 12 100
0.1175x = 705 1617𝑥+2420𝑥
x =6000 Rs. = 8074
240
Total amount invested by Divyaraj = 6000 × 7 = 4037x = 8074 × 240
42000 𝑅𝑠. 𝑥 = 480 Rs.
Required simple interest = 42000 × Annually saving of Satish
17.5×2 55
=14700 Rs. = 48000 × 100 × 12
100
= 316,800 Rs.
19. (d); Profit earned in 3 years = Difference in interest
obtained 22. (d); Compound interest rate for 2 years
D=P r2 (300+r) 20+20
= 20 + 20 + 100 = 44%
(100)3
80000×25×305 ATQ,
= 1000000 44 (x+2000)×8×2
(x– 2000) × – = 1600
= Rs610 44𝑥 16𝑥
100 100
Let scheme offers R% rate of interest ⇒ – – 880 – 320 = 1600
100 100
𝑅 2 28𝑥
610 (1 + 100) − 610 = 128.1 ⇒ = 1600 + 880 + 320 = 2800
100
R = 10% ⇒ x = 10,000
12000−8000 4
Required % = × 100 = × 100 = 50%
20. (b); Let he invested Rs. Y in second scheme 8000 8
5% per half year means 10% per annum in SI 23. (a); Let total saving of Ritu be x Rs.
Therefore, amount obtained by him after two 5𝑥
Invested in Scheme A =
years 𝑥
15

=Y+
Y(2)(10) = 3 Rs.
100 4𝑥
6𝑌
= 5 Amount Invested in scheme B = 15 Rs.
2𝑥
If he invested Rs. 30000/- in a scheme, which Invested in scheme C = 𝑅𝑠.
5
offers ROI of 10% per annum 10×10
Two year C.I on 10% =10+10+
100
Amount obtained by him after two years is
2 = 21%
10 15×15
= 30000 (1 + ) Two year CI on 15% =15+15 + 100
100
11 11
= Rs. 30000 × 10 × 10 = 32.25 %
20×20
= Rs. 36300 Two year CI on 20% =20+20+
100
Now According to Q = 44%
1 (6𝑌)
⇒ 36300 = 2 ATQ,
5 4𝑥 32.25 𝑥 21
⇒Y=
36300×5×2 × − × = 744
6 15 100 3 100
129𝑥 21𝑥
= Rs. 60500/- 1500
− 300 = 744
21. (e); Let total monthly income of Satish = 100x Rs. 24x = 744 × 1500
744×1500
Amount invested in scheme A x = 24
55 7
= 100𝑥 × 100 × 12 x = 46500 Rs.
385𝑥 Required difference
= 12 Rs. 46500×6 44 46500×4 32.25
= × − ×
Amount invested in scheme B 15 100 15 100
55 5 = 8184 – 3999
= 100𝑥 × 100 × 12
= 4185 Rs.
275𝑥
= 12 Rs.
24. (b); ATQ—
Equivalent CI of two years on 10% 8400×𝑅×2 9600×(𝑅+5)×2
10×10 + = 6360
= 10 + 10 + 100 1000 100
168R + 192R + 960 = 6360
= 21% 360R + 960 = 6360
Equivalent CI of two years on 20% 360R = 5400
20×20
= 20 + 20 + 100 R = 15%
= 44% Rs. (8400 + P) invested at 15%

54 Adda247 Publications For More Study Material


Visit: adda247.com
Cracker Book for Bank (IBPS | SBI | RRB PO | Clerk) Mains Exams
144
And Rs. (9600 + P) invested at 20% = 37500 × 100 = 54000 Rs.
Equivalent CI of two years at the rate of 15%
15×15 29. (c); ATQ
= 15 + 15 + 100 𝑥×8×2 20 2 𝑦×8×7
+ 𝑦 [(1 + 100) − 1] =
= 32.25% 100 100
Equivalent CI of two year at the rate of 20% 16x + 44y = 56y
20×20
= 20 + 20 + 100 = 44% 16x = 12y
𝑦 4 3𝑦
ATQ— =3⇒𝑥=
32.25 44 𝑥 4
(8400 + 𝑃) × + (9600 + 𝑃) × 100 = 8153 44y – 0.16x = 2304
100
270900 + 32.25P + 422400 + 44P= 815300 44y – 0.16 × 𝑦= 2304
3
76.25P = 122000 4

P = 1600 Rs. 0.44y – 0.12y = 2304


y = 7200
25. (c); Amount withdrawn from bank which offered
20000×10×2 x = 5400
simple interest = 20000 + = 24000
100 Required amount = 12,600
Compound interest accrued from another bank =
2460 30. (b); Let Shikha and Sameer have Rs. 100𝑥
𝑅 2 Equivalent CI for two years at the rate of 10%
2460 = 24000 [(1 + 100) − 1] 10×10
= 10 + 10 + = 21%
⇒r=5% 100
ATQ—
26. (b); Amount withdrawn after 2 years from bank 21 2 R×2 1 6.5×2
5000×10×2 100x × = 100x × × + 100x × ×
offering S.I. = 50000 + 100
100 3 100 3 100
4x×R 13x
= 60000 21x = 3
+ 3
Amount withdrawn after another 2 years from 63x = 4x × R + 13x
5 2
another bank offering C.I. = 60000 (1 + ) = 4x × R = 50x
100 50x
66150 R=
4x
New profit = 66150 − 50000 = 16,150 R = 12.5%
27. (d); Let, Total amount Veer have = ‘x’
31. (c); 2 year C.I. on 8% per annum
ATQ, 8×8
11 11 12 12 = 8 + 8 + 100 = 16.64%
0.32x [10 × 10 – 1] + 0.20x [10 × 10 – 1] +
14 14 2 year C.I. on 20% per annum
.48x [ × – 1] 20×20
10 10 = 20 + 20 + = 44%
= 6600 100
21 44 96 ATQ—
0.32x [ ] + 0.20x [ ] + 0.48x [ ] = 6600
100 100 100 x×3×15 (x+4000)16.64 (2x+4000)44
6.72𝑥 8.8𝑥 46.08𝑥 + + = 20382.4
100
+ 100
+ 100
= 6600 100 100 100
6600×100 0.45x + 0.1664x + 665.6 + 0.88x + 1760 =
⇒x=
61.6
6600×100
20382.4
Required difference = 61.6
× [. 48 – .20] = 1.4964x = 20382.4 – 2425.6
3,000 x=
17956.8
= 12000 Rs.
1.4964
28. (e); ATQ, Sum of Divyaraj invested = (2 × 12000 + 4000) =
r 2
12093.75 = 37500 [(1 + ) – 1] 28000 Rs.
100


12093.75 r 32. (d); Bonus of Ankit = P Rs.
37500
+ 1 = 1 + 100
Bonus of Veer = (P + 2400) Rs.
49593.75 r Bonus of Sameer = (P + 4400) Rs.
√ =1+
37500 100
r Equivalent CI at 10% for two years
1.15 – 1 = 100 10×10
= 10 + 10 +
r = 15% 100

New rate = 15 + 5 = 20% = 21%


Amount paid by Bhavya after next two more Equivalent CI at 20% for two years
years

55 Adda247 Publications For More Study Material


Visit: adda247.com
Cracker Book for Bank (IBPS | SBI | RRB PO | Clerk) Mains Exams
20×20
= 20 + 20 + = 44%
100
ATQ— Now,
60 15×3 P 21 (P+2400) 120 3
(P + 4400) × × – 2 × 100 − × (32000–x)( ) 36
44
100 100 2
2x
100
= 65
100
= 132 (32000–x)216 36
27P+118800 65P+105600
⇒ =
2x×125 65
– = 132 (32000–x)3 1
100 200
⇒ =
54P + 237600 – 65P - 105600 = 26400 x×25 13
11P = 105600 ⇒ 32000 × 39 = 64x
P = 9600 x = 500 × 39 = 19500/-
Sameer bonus = (9600 + 4400) 35. (b); Let man invested in scheme P, Q and R be Rs. 5x,
= 14000Rs. Rs. 6x and Rs. 9x respectively
33. (c); Three years SI on 15% = 15×3=45% Equivalent CI of two years on 20% = 20 + 20 +
8×8 20×20
Equivalent two years CI on 8 %= 8 +8+ 100 = = 44%
100
16.64% Equivalent CI of two years on 10% = 10 + 10 +
Equivalent two years CI on 20% = 10×10
= 21%
20×20 100
20+20+ 100 = 44% ATQ –
ATQ – 44 15 ×2 21
44(2𝑃+8000) 45𝑃 16.64(𝑃+8000)
5𝑥 × + 6x × – 9x× = 1899
100 100 100
100
− ( 100 + 100
) = 5352 2.2x + 1.8x – 1.89x = 1899
.88P + 3520 - .45P – .1664P – 1331.2 2.11x = 1899
= 5352 1899
x = 2.11
.2636P = 3163.2
3163,2 x = 900 Rs.
P = .2636 = 12000 𝑅𝑠. Total amount = 900 × (5 + 6 + 9)
Suresh borrowed = 12000×2+8000 = 32000 Rs = 18000 Rs.
34. (c); ATQ, If man invested total amount on C.I at the rate of
The amount invested by him in SI, got doubled 15% p.a.
after 5 years. Therefore ROI was 20% per Equivalent CI of two years on 15% = 15 + 15 +
15×15
annum. 100
= 32.25
Let amount invested by him in SI was x, then Required interest = 18000×
32.25
= 5805 Rs.
amount invested by him in CI is (32000 – x) 100

56 Adda247 Publications For More Study Material


Visit: adda247.com
Cracker Book for Bank (IBPS | SBI | RRB PO | Clerk) Mains Exams

Chapter
Time & Work and Pipe Cistern
7
BEST APPROACH TO SOLVE THE QUESTIONS

• Questions asked from this topic generally to test mental ability of a student calculate the time in which a
man/team complete the given work while efficiency is given.
• While doing the questions from this topic, a most common approach that student must use is that
‘if a man completes 1 unit of work in N days, then amount/part of work done by him in 1 day is 1/𝑁,’
Assume that work of man is to eat 100 chocolates and he eats all of them in 10 days, then amount of work done
100
(chocolates eaten) in 1 day is = 10 𝑐ℎ𝑜𝑐𝑜𝑙𝑎𝑡𝑒𝑠 𝑒𝑎𝑡𝑒𝑛.
10
Or
10
If Rashmi can make 10 cakes in 5 hours, then amount of work done (Cake made) by her in 1 hour is 5 = 2 cakes.
Another easy way to solve questions from this topic is % approach i.e ‘if a man can complete a work in 20 days,
100
then amount of work done by him in 1 day is % = 5%.
20
Consider an example: Rahul can complete a work in 10 days and Arun can complete the same work in 20 days.
Find the time in which the work will be completed if both of them work together.
There are three approaches to solve this:

(i) TRADITIONAL APPROACH


If Rahul can complete the work in 10 days, then amount of work done by him in 1 day
1
= 10 𝑢𝑛𝑖𝑡𝑠.
Similarly amount of work done by Arun in 1 day
1
= 𝑢𝑛𝑖𝑡𝑠.
20
1 1 3
Hence when both of them work together, total work done is + = 𝑢𝑛𝑖𝑡𝑠.
10 20 20
3
Hence if 20 𝑢𝑛𝑖𝑡𝑠 𝑜𝑓 𝑤𝑜𝑟𝑘 𝑖𝑠 𝑑𝑜𝑛𝑒 𝑖𝑛 1 𝑑𝑎𝑦
Then 1 unit of work will be completed in 20/3 days.

(ii) LCM APPROACH


In this method, assume the LCM of days as a number of chocolates. LCM of 10 and 20 is 20. Now assume
that there were 20 chocolates, and If Rahul take 10 days to eat them all, it can be concluded that he ate 2
chocolates per day. Similarly, Arun can eat 1 chocolate in a day.
Therefore,
They both will eat 3 chocolates in 1 day.
20
Time taken to complete whole work(to eat all of them)= 3 𝑑𝑎𝑦𝑠.
NOTE: Our answer will be same, we consider any number of chocolates, but for easy calculation purpose
we use a number that is easily divisible by number of days given.

(iii) % APPROACH
This method is kind of same as (i). Consider the work unit as 100% work.
Now note that if Rahul take 10 days to complete 100% of work, then work done by him in 1 day is 10%.
Similarly work done by Arun in 1 day is 5%. Therefore, both of them working together will complete 15%
of work in 1 day.
100 20
100% of work will be completed in = 𝑑𝑎𝑦𝑠.
15 3

In all these approaches, 1 common thing that occurs is to calculate amount/part/percentage of work done in 1
day.
LCM approach is best used in the questions in which efficiency of pipe or volume of tank is given. Consider eg.

57 Adda247 Publications For More Study Material


Visit: adda247.com
Cracker Book for Bank (IBPS | SBI | RRB PO | Clerk) Mains Exams
Example 1: Pipe A and Pipe B when both opened together can fill a tank in 4 minutes. Pipe A alone can fill it in 16
minutes. Find the time in which pipe B alone will fill the tank.
In traditional way:
1
Sol. Tank filled by Pipe A alone in 1 minutes = 16 𝑢𝑛𝑖𝑡𝑠.
Let pipe B alone can fill it in x minutes.
1
Then tank filled by pipe B alone 1 minute = 𝑢𝑛𝑖𝑡.
𝑥
1 1 1
Therefore, 𝑥 + 16 = 4
1 3 16
𝑥
= 16 , 𝑥 = 3
LCM APPROACH
Let total units of tank are 16 liters. It means A can fill 1 liter in a minute. Assume that B fills x unit in a minute. Tank was
filled in 4 minutes’ means
4(𝑥 + 1) = 16
𝑥+1 = 4
x = 3 units per minute
16
time taken is minutes.
3
All this could be done was mentally through LCM method. Give it a try!

MAN-DAYS-HOUR-WORK THEOREM
According to this theorem if more than 1 man of equal efficiency are working together on a same work, then
amount/part of work done by them is directly proportional to the time given to work by number of man. In more simple
words work done depends on efficiency and total time given.
𝐦 𝐝 𝐡 𝐦 𝐝 𝐡
⇒ 𝟏𝐰𝟏 𝟏 = 𝟐𝐰𝟐 𝟐
𝟏 𝟐
Here m₁, m₂ are number of men
d₁, d₂ are number of days
h₁, h₂ are working hours in given days
w₁, w₂ are part/amount of work done.
For eg. 10 men in 5 days can do same amount of work as 5 men in 10 days or if we explore it more we can see 10 men in
6 days working 4 hours a day can do same amount of work as 30 men in 4 days working 2 hours a day.
Please verify above two examples, by assuming a man can eat 1 chocolate in 1 hour and try to solve given problem.
Example 2: 12 Men working 4 hours a day can complete 48% of work in 8 days. Calculate how many men are required
to complete 2 units of a work, if they have to complete it in 10 days, 8 hours a day.
Sol. In this type of problems, by default we have to assume that all men have equal efficiency and they do the
work with uniform speed.
Applying Man-day-hour formulae
m1 d1 h1 m d h
w1
= 2w2 2
2
Here,
m₁ = 12 m₂ = ?
d₁ = 8 d₂ = 10
h₁ = 4 h₂ = 8
w₁ = 48 w₂ = 200
Note that, here w₂ = 200 is used because, 2 units of work means 200% of work.
12×8×4×200
m2 = 48×8×10
m2 = 20
Another type of concept that is introduced here is when people with unequal efficiencies are working, for eg. 4 women
can do the same amount of work in a given time as 3 men. For these type of questions, we use ratio approach, we simply
write
4W= 3M
𝑊 3
= > 𝑀 = 4.
And then we consider efficiency of a men as 4 units and that of women as 3 units throughout the problem.
Few points to remember:
If the ratio of efficiencies of two persons is x:y then ratio of respective time taken by them is y:x.
1
If X is N times efficient than Y, time taken by X to complete a work is 𝑁 𝑡𝑖𝑚𝑒𝑠 𝑜𝑓 time taken by Y.
Total work done by a man of efficiency X in N days is 𝑁𝑋 units.

58 Adda247 Publications For More Study Material


Visit: adda247.com
Cracker Book for Bank (IBPS | SBI | RRB PO | Clerk) Mains Exams

Practice Exercise Based on New Pattern

1. Ratio between efficiency of Arun, Yash and Rana is 6 : 7. Ratio of efficiency of A and B in completing a work is 3
4 : 5. All three starts to work together with same : 4. Both started to work together but A left after 2
efficiency. But Rana, destroys his 60% of work in days. Another person C joins B and they together
complete the remaining work in 6 days. If A and B
every evening, due to which they have to work 20
together can complete the work in 8 days then C alone
more days then estimated time. Find the estimated can complete the work.
days by them to complete the work. 27
(a) 4 days
56
(b) 3 days
41
(c) 3 days
(a) 80 (b) 76 (c) 84 28 49
(d) 72 (e) 90 (d) 3
days (e) 3
days

2. Pipe P can fill a tank in 24 minutes, pipe Q can fill the 8. Pipe A can fill a tank in 45 hr, pipe B is 50% more
efficient than A and pipe C can fill the same tank in 7.5
same tank in 36 minutes and an outlet pipe can empty
hr less than B. A and B opened together for X hr and
the fully filled tank in 48 minutes. Find in how many closed after that and pipe C fill remaining tank in (X +
minutes tank will be filled, if for first minute only pipe 9) hr, if the ratio between tank filled by (A + B)
P & Q opened together and in second minute all three together to tank filled by pipe C is 1 : 2. Find the value
pipes opened in tank alternatively? of X ?
4 4 (a) 3 hr (b) 4 hr (c) 6 hr
(a) 15 𝑚𝑖𝑛𝑢𝑡𝑒𝑠 (b)13 𝑚𝑖𝑛𝑢𝑡𝑒𝑠
5 5 (d) 8 hr (e) 7 hr
4 4
(c)18 𝑚𝑖𝑛𝑢𝑡𝑒𝑠 (d) 16 𝑚𝑖𝑛𝑢𝑡𝑒𝑠 9. A and B can do a piece of work in 72 days and 64 days
5 5
4 2
(e)12 𝑚𝑖𝑛𝑢𝑡𝑒𝑠 respectively. C can do the same work in 2 more
5 17
days as A & B take together to complete. If first day A
Direction (3-4): One day earning of P, Q and R together is & B work together and second day B & C work
Rs.1026 to do a work. ‘P’ earns more than R which is same together alternatively, then in how many days work
as ‘R’ earns more than ‘Q’. Efficiency of R and Q is 19 : 18. will be completed?
13 13
(a) 22 𝑑𝑎𝑦𝑠 (b) 27 𝑑𝑎𝑦𝑠
3. Find the amount earned by R and P together in 5 days 25 25
13 13
to do the same work? (c) 32 𝑑𝑎𝑦𝑠 (d) 25 𝑑𝑎𝑦𝑠
25 25
13
(a) Rs. 3620 (b) Rs. 3430 (c) Rs. 3510 (e) 29 25 𝑑𝑎𝑦𝑠
(d) Rs. 3310 (e) Rs. 3710
10. Four persons started to do a work together. ‘A’ works
4. S, who’s efficiency is average of efficiency of P and Q, only in starting two days after that B, C and D works
can complete a work in 36 days. If P, Q and R work alternately starting from B. Ratio of time taken by A,
together, and complete that work then find the total B, C and D if they work alone is 4 : 3 : 2 : 5. If the work
wage of Q is completed in 12 days then in how many days A and
C can complete the work if they work together ?
(a) 3888 Rs. (b) 4104 Rs. (c) 4320 Rs. (a) 6 days (b) 12 days (c) 10 days
(d) 3666 Rs. (e) 4520 Rs. (d) 8 days (e) 4 days
5. (X+4) men can complete a work in 2X days while 11. A person C can complete 21% of work in 10 days
(X+12) women can complete same work in (X+8) while working with 233⅓% of his efficiency. B is 11
1
days. If ratio of efficiency of men to women is 5 : 4 9
% more efficient than C. A, while working with his
then find in how many days 12 men and 15 women half efficiency can complete the work in half time as
together can complete the same work? compared to time taken by B. Find the time taken by A
(a) 32 days (b) 16 days (c) 48 days & B together to complete the 50% of whole work.
(d) 64 days (e) 80 days (a) 15 days (b) 10 days (c) 20 days
(d) 25 days (e) 22 days
6. A cistern can be filled completely by pipe A and B
together in 12 hours. If pipe A works with twice speed 12. Two pipes A and B can fill a cistern in 15 hours and 10
while pipe B work with 50% more speed than cistern hours respectively. A tap can empty the full cistern in
30 hours. All the three taps were open for 2 hours,
can be filled completely in 7 hours. Find the capacity
when it was remembered that the emptying tap had
of cistern if flow of water through pipe A is 2.5ℓ/ been left open. It was then closed. How many hours
minute. more would it take for the cistern to be filled ?
(a) 2800 liter (b) 3150 liter (c) 3300 liter (a) 4hr 30 min. (b) 4hr 12 min. (c) 4 hr24 min.
(d) 3650 liter (e) 4200 liter (d) 4hr 35 min. (e) 4hr 54 min.

59 Adda247 Publications For More Study Material


Visit: adda247.com
Cracker Book for Bank (IBPS | SBI | RRB PO | Clerk) Mains Exams

Directions (13-15): ‘B’ and ‘C’ together started to do a 19. One day efficiency of A is 20% more than B and that of
work ‘X’. After 6 days ‘B’ is replaced by ‘A’ who can do C is 20% less than B. D can do a piece of work in twice
work ‘X’ alone in same time in which ‘B’ and ‘C’ together of time as A, B & C complete same work in together. If
can do. After 4 days more ‘A’ left the work and remaining first eight days D work alone after that he left the
1 work and A, B & C complete the remaining work in 12
work is completed by ‘C’ alone in 2 days. ‘B’ did 33 3 % of days. Find in what time work will be completed, if B
work ‘X’ in total. ‘C’ and ‘D’ can complete the same work and D work in rotation on each day, starting with D?
1 1 1
‘X’ in 20 days while ‘D’ is 60% less efficient than ‘B’. (a) 41 3 𝑑𝑎𝑦𝑠 (b) 38 3 𝑑𝑎𝑦𝑠 (c) 36 3 𝑑𝑎𝑦𝑠
Efficiency of A, B, C and D remain same for work ‘X’, ‘Y’ 1 1
(d) 44 𝑑𝑎𝑦𝑠 (e) 46 𝑑𝑎𝑦𝑠
and ‘Z’ 3 3

13. Find the ratio between work done by ‘A’ in 4 days to 20. 5 inlet pipes (same capacity) can fill a tank in same
time in which 3 outlet pipes (same capacity) can
work done by ‘B’ in 9 days?
empty it. If 2 inlet and one outlet pipe is opened for
(a) 3 ∶ 4 (b) 1 ∶ 2 (c) 1 ∶ 3 first minutes and 5 inlet and 2 outlet pipe open for 2nd
(d) 2 ∶ 3 (e) 3 ∶ 1 minute and process continues till tank is completely
14. Find in how many days ‘A’ can complete another work filled in 30 minutes. Find the time in which 2 outlet
‘Y’ if ‘C’ and ‘D’ together can complete work ‘Y’ in 26 pipe can empty the completely filled tank.
days? (2 Marks) (a) 9 minute (b) 12 minutes (c) 6 minutes
(d) 10 minutes (e) 15 minutes
(a) 14.4 days (b) 15.6 days (c) 18.2 days
(d) 16.6 days (e) 19.5 days Direction (21-22): P, Q, R and S four pipes can fill a
cistern ‘N’ in 7 minutes if all are opened together. P is
15. ‘E’ who is 60% more efficient than ‘B’ can complete 25% less efficient then Q which is 100% more efferent
1
another work ‘Z’ in 12 days. Find in how many days than R. ‘S’ filled 25% of the cistern. P and Q together can
2
‘A’ and ‘C’ together can complete work ‘Z’? (2 Marks) fill the cistern in ‘x’ minutes while P and S can fill half
(a) 20 days (b) 15 days (c) 12 days cistern in ‘y’ minutes.
(d) 10 days (e) 8 days 21. Pipe A and B together can fill another cistern ‘M’ in
1
(x+12) minutes while pipe B and C together can fill
16. Veer and Sameer alone can do 25% & 33 3 % of a task cistern ‘M’ in (y+5) minutes. If pipe A, B and C
6𝑥
in 16 days individually. If Sameer & Satish together together can fill cistern ‘M’ in ( 𝑦 ) minutes, then find
can do same task in 16 days, then find in how many efficiency of ‘B’ is what % more/less than efficiency of
days Veer, Sameer and Satish will complete the whole ‘A’ ?
task, if they do it on alternate days, starting with (a) 50% (b) 75% (c) 100%
Satish and followed by Sameer & Veer respectively? (d) 150% (e) 200%
(a)38 days (b)42 days (c)36 days
22. ‘D’ men can complete a work in (x+9) day while (y+7)
(d)32 days (e)30 days men can complete same work in ‘E’ days. If y men can
17. Working alone, A can complete a task in ‘a’ days and B complete same work in (E+12) days then find in how
in ‘b’ days. They take turns in doing the task with each many day (E–D) men can complete the same work?
working 2 days at a time. If A starts they finish the (a) 21 days (b) 42 days (c) 35 days
(d) 14 days (e) 84 days
task in exactly 10 days. If B starts, they take half a day
more. How long does it take to complete the task if Directions (23-24): These questions are based on the
they both work together? information given below.
1 1 5 Each of A, B, C and D need a unique time to do a certain
(a) 5 3 days (b) 5 7 days (c) 5 9 days
work. A can do the work in 𝑥 days and B can do the work
5 2
(d) 5 11 days (e) None of these in 2𝑥 days. A started the work and do it for 22 9 days then
he is replaced by B and B completed remaining work in
18. Monika can do 40% of a work in 24 days and Anshika
same time as C and D together can complete the whole
can do 12.5% of same work in 4.5 days. Shikha can
work.
complete the same work in equal time as Monika and The ratio of the efficiency of C and D is 4 : 5. If C and D
Anshika can do together. If all three starts work work for alternative days starting from C then they can do
alternatively, starting with Shikha and followed by 1
the total work in 44 2 days.
Monika and Anshika respectively, then find how many
days will be required to complete the work? 23. What is the value of 𝑥?
1 1 1 2 1 2
(a) 30 𝑑𝑎𝑦𝑠 (b) 24 𝑑𝑎𝑦𝑠 (c) 33 𝑑𝑎𝑦𝑠 (a) 66 (b) 33 (c) 16
3 3 2 3 3 3
1 1 2
(d) 28 𝑑𝑎𝑦𝑠 (e) 20 𝑑𝑎𝑦𝑠 (d) 147 (e) none of these
3 3

60 Adda247 Publications For More Study Material


Visit: adda247.com
Cracker Book for Bank (IBPS | SBI | RRB PO | Clerk) Mains Exams
24. A and B together can compete 225% of the work in to complete the work. If Ram and Shyam work for 16
how many days? days, after that both left the task then find in how
2
(a) 663 days (b) 60 days (c) 50 days many days Ghanshyam will complete remaining
(d) 25 days (e) none of these work?
25. P can do 50% of a work in 32 days, Q can do 37 2 % of
1 (a) 6 days (b) 8 days (c) 4 days
1 (d) 5 days (e) 10 days
same work in 27 days, while R can do 62 2%of same 1
work in 30 days. Q and R together started and worked 30. A child can do 5th of work in same time as his father
for x days. After x days Q left the work and P joined R can do the whole work and efficiency of child’s
and both completed remaining work in (x +4) days. If mother is equal to half the sum of father and child
ratio between work done by (Q + R) together to work efficiency. If they complete 70% of work in 24 days,
done by (P + R) is 5 : 7 then in 2x days R will complete when they work alternatively, starting with father and
what fraction of work? child together on first day followed by father and
1 1 2
(a) 3 (b) 4 (c) 3 mother together on second day. Find in how many
1 1 days mother can complete 85% of work alone?
(d) (e)
2 5 (a) 48 days (b) 68 days (c) 60 days
26. Veer and Sameer can do a work in 64 days and 72 (d) 56 days (e) 54 days
days respectively. Satish and Ankit together take 48 31. Ankit and Apoorv together can finish a work in
4
days to complete the same work. Ankit is 40% more 8 𝑑𝑎𝑦𝑠 while Ankit can do it individually in 15 days.
7
efficient than Satish. If Veer and Sameer work for 18 Both work on another task for (x + 4) days and (x +
days together, after that both left the work, then find 90
20) days respectively. If remaining 7 % of the second
in how many days remaining work will be completed task is completed by Bhavya in 18 more days with the
by Satish and Ankit if they work on alternated days efficiency of 2 units work/day. Then, find what
and Ankit starts the work? portion of the second task will be completed in x days,
6 6 6
(a) 40 𝑑𝑎𝑦𝑠 (b) 42 days (c) 48 𝑑𝑎𝑦𝑠 if all three work together?
7 7 7 29 26 31
6 4 (a) (b) (c)
(d) 44 7 𝑑𝑎𝑦𝑠 (e) 49 5 𝑑𝑎𝑦𝑠 35
23
35
27
35
(d) 35 (e) 35
27. Veer takes 50% more time to complete a task than
that of Satish takes to complete the same task, while 32. One day efficiency of Raj is equal to one day efficiency
2 of Divyaraj and Veer together. Divyaraj is 25% more
Mahendra takes 2 days less than to complete same efficient than Veer and starts with 20% more of his
5
task as Veer and Satish takes together. If Satish efficiency with Veer and works for x days, after that
1
complete 3 𝑟𝑑 of task in 8 day, then find in how many Divyaraj left the work and remaining work will be
completed by Veer and Sameer together in (x + 8)
days work will be completed, if Veer and Mahendra days. If ratio of work done by Divyaraj and Veer
work in rotation, starting with Mahendra? together to work done by Veer and Sameer is 5 : 7 and
(a) 16 days (b) 18 Days (c) 24 days efficiency of Sameer is 25% less than Veer. Then find
(d) 12 days (e) 22 days Raj and Sameer complete work together in how many
days?
28. There are 3 pipes A, B and C. A can fill a bucket in 8 (a) 14 days (b) 12 days (c) 16 days
18
minutes. C can fill buckets in 18 minutes & pipe B (d) 18 days (e) 20 days
5
3
can fill 20 buckets in a minute. These pipes are opened 33. Two pipes P and Q can fill tank A in 28 minutes and
56 minutes respectively and empty pipe M can empty
in a tank alternatively, 1 minute each starting from A,
same tank in 42 minutes. If all three pipes opened in
then C, then B. If tank is filled after 2 hours, then find tank B for (x – 24) minutes together they filled 90 liter
the volume of tank if the capacity of bucket is 5 liters. of the tank which is 25% of the quantity tank B. Find
(a) 88 liters (b) 95 liters (c) 90 liters in ‘x’ minutes what portion of tank B filled, if all pipe P
(d) 105 liters (e) 92 liters and Q and M opened alternatively starting with P,
followed by Q and M respectively?
29. Efficiency of Ram is 25% more than Shyam who 7 7 5
2 (a) (b) (c)
completes a task in 60 days. Ghanshyam takes 6 3days 12 36 36
3 7
(d) 35 (e) 41
less than the days taken by Ram and Shyam together

61 Adda247 Publications For More Study Material


Visit: adda247.com
Cracker Book for Bank (IBPS | SBI | RRB PO | Clerk) Mains Exams

34. There are two pipes A & B, pipe A is for filling the 37. A, C and D working simultaneously completed work
swimming pool and pipe B is to empty the swimming ‘X’ in ‘n’ days and A, B, C and D working
pool. Capacity of swimming pool is 5040 m 3 and simultaneously completed work ‘Y’ in ‘m’ days. Find
volume of pipe B is 8 m3/minute more than that of the value of (𝑚 + 𝑛).
1 (a) 15 days (b) 10 days (c) 12 days
pipe A. If pipe A takes 11 more minutes to fill same
4
(d) 8 days (e) 17 days
swimming pool, than time taken by B to empty the
same swimming pool. If pipe B can empty second 38. A person E starts the work ‘X’ and leave after 12 days,
swimming pool in 112.5 minutes, then find the then B and C complete the remaining work in 8 days.
What is the ratio of number of days taken by A and E
capacity of second swimming pool?
(a) 7200 m3 (b) 6400 m3 (c) 5600 m3 together to complete the work ‘X’ to the number of
days taken by D, B and C together to complete the
(d) 7800 m 3 (e) 8400 m 3
both work ‘X’ and ‘Y’ .
35. Two pipes P and Q can fill tank A in 28 minutes and (a) 3 : 5 (b) 5 : 3 (c) 8 : 7
56 minutes respectively and empty pipe M can empty (d) 1 : 2 (e) 2 : 3
the tank in 42 minutes. Tank A have the capacity of
Directions (39-40): Ankur & Shubham can do a piece of
168 liters. If all three pipes opened in tank B for (x –
24) minutes together they filled 90 liter of the tank work 64 days together and Shubham &Kartik can do the
same work in 72 days together. First 22 days Ankur &
which is 25% of the quantity of tank B. Find in x
Shubham work together, after that Ankur left the work
minutes what portion of tank B filled, if all pipe P and
and Kartik joined Shumbham. Kartik and Shubham work
Q and M opened alternatively in each minute starting
for next six days, after that Shubham left the work and
with P, followed by Q and M respectively?
5 7 9 remaining work complete by Kartik & Anup in 30 more
(a) 36 (b) 36 (c) 38 days. Efficiency of Anup is 75% more than efficiency of
7 7
(d) 38 (e) 39 Kartik.
39. Kartik and Anup work for xdays, Ankur and Shubham
Directions (36-38): Working efficiency of A is 20% more work for next
2
(x+2 3) days,If remaining
than that of B. B can complete a work ‘X’ in 36 days. 1
B and C together started to complete the work ‘X’ and 12 % 𝑜𝑓 𝑡𝑜𝑡𝑎𝑙 work complete by Prabhat in 6 more
2
after 10 days they both left the work and then remaining day. Prabhat will do, what portion of total work in
work is done by A alone in 15 days. (x+12) days?
A and C together started to complete another work ‘Y’ and (a)
2
(b)
1
(c)
3

after working for 12 days they both left the work. 3 4 4


2 2
Remaining work is done by B alone in 16 days. D first (d) (e)
3 5
completed work ‘X’ and then completed work ‘Y’ in total 40. Ankur and Shubham start work alternatively and
38 days.
work for y days. After that Kartik and Anup replace
It is given that efficiency of all, in completing work ‘X’ and 𝑦 75
work ‘Y’ is same. them and work for next days alternatively. If % of
2 8
1 total work still remain, which complete by Abhimanyu
36. A, B and C working together completed 3rd of work ‘X’, in 9 more days. Find if Abhimanyu work for (y+16)
and then A and C are replaced by D. Now remaining days, then in how many days remaining work will be
of work ‘X’ is completed by B and D together. For how completed by Anup and kartik?(y is divisible by 4)
48 34
many days B worked? (a)11days (b)11days (c) 20 days
(a) 12 days (b) 10 days (c) 15 days (d) 28 days (e) 32 days
(d) 4 days (e) 8 days

Solutions

1. (a); Ratio of efficiency of Arun, Yash and Rana is 6 : 4 : The work done by them in actual
5 = (6 + 4 + 2) (x + 20) = 12(x + 20)
Total work done by them in 1 day ∴ 15𝑥 = 12𝑥 + 240
= (6 + 4 + 5) units = (15) units. 3𝑥 = 240
Let they estimate 𝑥 days to complete the work.
𝑥 = 80
Then total work = 15𝑥
40 Hence, estimated days are 80.
But Rana’s 1-day work is only 40%, i.e. 100 × 5
= 2 unit
62 Adda247 Publications For More Study Material
Visit: adda247.com
Cracker Book for Bank (IBPS | SBI | RRB PO | Clerk) Mains Exams

2. (d); ⇒ (3𝑋 + 16)(𝑋 − 12) = 0


16
⇒ 𝑋 = 12, − 3
16×24
12 men can complete work in 12
= 32 𝑑𝑎𝑦𝑠
24×20
15 women can complete work in
15
First minutes (P + Q) = (6 + 4) = 10 unit = 32 𝑑𝑎𝑦𝑠
Second minutes (P + Q – R) 32×32
Required time = = 16 𝑑𝑎𝑦𝑠
32+32
= (6 + 4 – 3) = 7 unit
6. (e); Let, pipe A and pipe B alone can fill the tank in x
17 unit in 2 minutes
and y hours respectively.
So, ATQ,
17 × 8 = 136 unit in 16 minutes 1 1
+ =
1
… (i)
(144 –136) x y 12
Remaining by (P + Q) = 2 1.5 1
4
10 and, x + y = 7 … (ii)
= 5 minutes
On solving (i) and (ii)
4
Total time = 16 + 5 x = 28 hour ⇒ y = 21 hour
4 Capacity of cistern = 28 × 60 × 2.5
= 16 5 minutes = 4200 liter
Solution (3-4): Let R earns = y 7. (b);Let A and B can do 3𝑥 and 4𝑥 unit of work in one
And P earns = y + x day.
So, Q earns = y – x So,
ATQ, Total work = (3𝑥 + 4𝑥) × 8 = 56𝑥
y – x + y + y + x = 1026 (A + B) two day work = 7𝑥 × 2 = 14𝑥
y = 342 Remaining work = 42𝑥
In 6 days B will complete
Each person earns according to their efficiencies so = 6 × 4𝑥 = 24𝑥 units
342
Q earns → × 18 = 324 So, remaining 18𝑥 units are completed by C in 6
19
342
An P earns = 19 × 20 = 360 day So,
56𝑥 unit will be completed in
Ratio of efficiency of P, R and Q 56𝑥 56
= 18𝑥 = 3 days
= 20 : 19 : 18 6
Money per unit work = Rs.18
8. (c); A = 45 hr
3. (c); Amount earned by R and P together in 5 days A : B = 100 : 150 = 2 : 3
Work done = (20 + 19) × 5 = 195 Total capacity of tank = 45 × 2 = 90 liter
90
Money earned = 195 × 18 = Rs.3510 C = 3 – 7.5 = 22.5 hr
20+18 90 ℓ
4. (a); Efficiency of S = 2 = 19 C efficiency = =4
22.5 hr
Total work → 19 × 36 5X 1
Days taken by P, Q and R to do that work According to question ⇒ 4(X+9) = 2
19×36
⇒ (20+19+18) = 12 day ⇒ 10X – 4X = 36
X = 6 hr
Q’s earning ⇒ 12 × 324 = Rs.3888
5. (b);(X+4) men can complete work in 2X days 9. (b);
⇒ 1 man can complete same work in 2X(X+4)
days
(X+12) women can complete work in (X+8) days
⇒ 1 woman can complete same work in
(X+8)(X+12) days 576 36 612
Now ratio of efficiency of men to women is 5 : 4 C takes = (8+9) + 17 = 7
= 36 days
⇒ Ratio of days taken by men to women is Efficiency of C =
576
= 16 units/days
4:5 367
4 2X(X+4) When First day (A and B) and second day (B and
So, 5 = (X+8)(X+12) C) work alternatively
⇒ 2(𝑋 2 + 20𝑋 + 96) = 5𝑋(𝑋 + 4) Two day work = (A and B) one day work and (B
⇒ 2𝑋 2 + 40𝑋 + 192 = 5𝑋 2 + 20𝑋 and C) one day work
⇒ 3𝑋 2 − 20𝑋 − 192 = 0 = (8 + 9) + (9 + 16) = 17 + 25
⇒ 3𝑋 2 − 36𝑋 + 16𝑋 − 192 = 0 = 42 units
⇒ 3𝑋(𝑋 − 12) + 16(𝑋 − 12) = 0
63 Adda247 Publications For More Study Material
Visit: adda247.com
Cracker Book for Bank (IBPS | SBI | RRB PO | Clerk) Mains Exams
26
In 26 day = 2 × 42 = 546 units In 2 hours, part of the tank filled
= 2(4) = 8 units
(A and B) on 27th day = 17 units (30−8)
After 27 days remaining work Now required time =
5
= (576 – 546 – 17) = 13 units =
22
= 4hour 24mins.
13 5
13 units work done by (B and C) on 28th day =
25
13 13 Solution (13-15): Let, efficiency of A, B, C and D be ‘a’, ‘b’,
Total time = (27 + 25) = 27 25 days ‘c’ and ‘d’ respectively
Total work = 6b + 6c + 4(a +c) + 2c
10. (d);A, B, C and D worked for 2 days together after that
And also, a = b + c
A leave and B, C and D worked alternatively for 10
⇒ Total work = 10b + 16c
days starting from B
ATQ,
∴ B worked for 4 days, C for 3 days, and D for 3 1
days. B did 3 of work in 6 days
Total days A worked = 2 ⇒ B can complete whole work in 18 days
Total days B worked = 4 + 2 = 6 And,
Total days C worked = 3 + 2 = 5 (10b + 16c) = 18b ⇒ 16c = 8b
𝑏 2
Total days D worked = 3 + 2 = 5 ⇒𝑐 =1
Let, their alone time to complete the work is 4x,
⇒ ‘C’ can complete whole work ‘X’ in 36 days
3x, 2x and 5x respectively. 18×36
2 6 5 5 A can complete whole work ‘X’ in = 12 day.
∴ + + + =1 18+36
18
4x 3x 2x 5x
30+120+150+60 D can complete whole work ‘X’ in × 5 = 45 days.
⇒ =1 2
60𝑥
360 Ratio of efficiency of A, B, C and D
⇒ 𝑥 = 60 = 6 1 1 1 1
= 12 ∶ 18 ∶ 36 ∶ 45
‘A’ can complete the work in 4 × 6 = 24 days = 15 : 10 : 5 : 4
‘C’ can complete the work in 2 × 6 = 12 days
12×24 12×24 4×15 60 2
Required time = = = 8 days 13. (d);Required ratio = = =
12+24 36 9×10 90 3

11. (b);Let the efficiency of C [is C units/days] total work 14. (b);Let efficiency of A, B, C and D be 15x, 10x, 5x and
is 100a Units 4x respectively
ATQ, Total work ‘Y’ = (5x + 4x) × 26
7 = 9x × 26 =234x
c × 10 = 21a 234𝑥
3
1 7
‘A’ can complete work ‘Y’ in 15 = 15.6 day
[233 % = ]
3 3
9𝑎 15. (d);Efficiency of ‘E’ = 16x
C= 𝑈𝑛𝑖𝑡𝑠 /𝑑𝑎𝑦 25
10 Total work ‘Z’ = 16𝑥 × 2 = 200x
1
Also B is 11 % more efficient means ‘A’ and ‘C’ together can complete work ‘Z’ in
9
10 200𝑥 200𝑥
B= of C = (15+5)𝑥 = 20𝑥 = 10 days
9
10 9𝑎
B = 9 × 10 Units/day 16. (a); Veer can complete the whole task alone
= a Units / day = 16 × 4 = 64 𝑑𝑎𝑦𝑠
A which while working with half efficiency total Sameer can complete the same task alone
half time. = 16 × 3 = 48 𝑑𝑎𝑦𝑠
∴ A : B = 4 : 1 [efficiency ratio] Total work = 192 𝑢𝑛𝑖𝑡𝑠(LCM of 64 and 48)
Efficiency of A = 4a Efficiency of Satish
192 192
Work done by then together is 4a + a = 16 − 48 = 8 𝑢𝑛𝑖𝑡𝑠/𝑑𝑎𝑦
= 5a/day If all three work alternatively
Time take to complete 50 a Units First day by Satish = 8 units
50𝑎 Second day by Sameer = 4 units
= 5𝑎 = 10 days
Third day by Veer = 3 units
12. (c); Total work in three days = 8 + 4 + 3
= 15 units
36
In total 36 days = × 15 = 180 𝑢𝑛𝑖𝑡𝑠
3
Satish on 37 days = 8 units
Remaining work after 37 days
= 192 – 180 – 8 = 4 units

64 Adda247 Publications For More Study Material


Visit: adda247.com
Cracker Book for Bank (IBPS | SBI | RRB PO | Clerk) Mains Exams

On 38 days remaining work by Sameer Let one day work of A , B , C & D be 6x units, 5x
4
= 4 = 1𝑑𝑎𝑦𝑠 units, 4x units & 7.5x units respectively
Total time = 38 days ATQ –
Total work
17. (b);If each works 2 days at a time alternately starting = 7.5x × 8 + (5𝑥 + 6𝑥 + 4𝑥) × 12
with A, the work is completed in exactly 10 days. = 240x units
∴ A works for 6 days and B worked for 4 days. When B & D work alternatively
6 4
a
+b =1 ………….(i) First day by D = 7.5x units
If B starts, the work is completed in 10.5 days. Second day be B = 5x units
∴ B works for 6 days and A worked for 4.5 days. Two day work of = 7.5x + 5x = 12.5x units
6 4.5
+ =1 ………….(ii) In 38 days total work
b a 38
By solving (i) and (ii) = 2 × 12.5𝑥 = 237.5𝑥 units
a = 9 days Remaining work = 240x – 237.5x = 2.5x
And, b = 12 days 2.5𝑥
Remaining work by D on 39 days =
Time taken by A and B working together to 7.5𝑥
1
complete the work = 𝑑𝑎𝑦𝑠
1 1 3
=1 1 =1 1 1
Total time = 38 𝑑𝑎𝑦𝑠
+ + 3
a b 9 12
36 1
= 7
= 5 7 days 20. (a); Let efficiency of one inlet pipe = 3x
18. (c); Monika can do whole work alone Efficiency of 5 inlet pipes = 5 × 3x = 15x
24 15x
= 40 × 100 = 60 days Efficiency of 1 outlet pipe = 3
= –5x
Anshika can do whole work alone ATQ,
4.5
= × 100 = 36 days 15 (2 × 3x + 1 × (–5x)) + 15 (5 × 3x + 2 × (–5x)) =
12.5
Let Total work = 180 units (LCM of days taken by Tank capacity
Anshika and Monika) Tank capacity = 90x
180 90x
Efficiency of Monika = 60 = 3 units/day Required time = = 9 minutes
10x
180
Efficiency of Anshika = = 5 units/day
36 Solution (21-22): Let efficiency of R is 2 units/ minute
Shikha take to complete whole work alone ⇒ Efficiency of Q is 4 units/ minute
180
= (3+5) = 22.5 days So Efficiency of P is 3 units/ minute
Efficiency of Shikha = 8 units/day P + Q + R together completed 75% of work
when Shikha, Monika & Anshika work ⇒ 75% of work = 9 units
alternatively— ⇒ 100% of work = 12 units
Work done on First day by Shikha = 8 units
Efficiency of S is 3 units/minute
Work done on First day by Monika = 3 units
Work done on First day by Anshika Total work = 7 × (12) = 84 unit
= 5 units Number of days taken by P, Q, R and S individually: -
In three days = (8 + 3 + 5) = 16 units
33
In total 33 days = 3 × 16 = 176 units
Remaining work complete by Shikha
(180 –176) 1
= 8
= 2 days
1
Total time = 33 + 2
1
= 33 days
2

19. (b);Let efficiency of B = 100


So, efficiency of A = 120
80 28×21
And efficiency of C = 100 × 100 = 80 P and Q together can fill the cistern = 28+21
100+120+80 28×21
Efficiency of D = = 150 = 49
⇒ x = 12 min.
2
28×28
Ratio of efficiency of A, B, C and D P & S together can fill half cistern =
28+28
= 6 : 5 : 4 : 7.5 y = 7 min.

65 Adda247 Publications For More Study Material


Visit: adda247.com
Cracker Book for Bank (IBPS | SBI | RRB PO | Clerk) Mains Exams
400+200
21. (c); A and B together can fill cistern ‘M’ in (x+12) =1
18𝑥
minutes = 12+12=24 minutes 1
⇒ 𝑥 = 33 3
B and C together can fill cistern ‘M’ in (y+5)
1
minutes = 7 + 5 = 12minutes 24. (c); A – 33 3 𝑑𝑎𝑦𝑠
6𝑥 2
A, B and C together can fill cistern ‘M’ in ( 𝑦 ) B − 66 3 days
6×12 72 200
minutes = = minutes (A+𝐵)− 𝑑𝑎𝑦𝑠
7 7 9
200 9
Required Time − × = 50 𝑑𝑎𝑦𝑠
9 4

25. (d);P can do whole work alone = 64 days


Q can do whole work alone
27
= 3 × 8 = 72 days
R can do whole work alone
30
= 5 × 8 = 48 days

A’s efficiency = 49 – 42 = 7 units/minute


C’s efficiency = 49 – 21 = 28 units/ minute ATQ—
(8+12)𝑥 5
B’s efficiency = 49 – 7 – 28 =
(9+12)(𝑥+4) 7
= 14 units/minute 140𝑥 = 105𝑥 + 420
14−7 420
Required % = 7
× 100 𝑥 = 35
7
= × 100 = 100% 𝑥 = 12 days
7
In 2𝑥 days work done by R = 2 × 12 × 12
22. (b);ATQ, = 288 units
D × (12+9) = (7+7) × E 288 1
Required portion = 576 = 2
3
⇒D×2 = 𝐸
26. (d);
And (7+7) × E = 7 × (E+12)
⇒ 2E = E + 12
⇒ E = 12
⇒D=8
Let (E – D) men can complete same work in ‘a’
days Let efficiency of Satish is 5x unit/day and that of
ATQ, Ankit is 7x unit/day
(12−8) × a = 8 × (12+9)
ATQ—
a = 2 × 21 = 42 days
5x + 7x = 12 unit
23. (b);Let the C do in one day = 4𝑦 work x = 1 unit/day
Let the D do in one day = 5𝑦 work Efficiency of Satish = 5 unit/day
2 day work of C + D = 9𝑦
Efficiency of Ankit = 7 unit/day
In 44 day they will complete 9𝑦 × 22
= 198𝑦 Work done by Veer and Sameer in 18 days
1 = (9 + 8) × 18 = 306 unit
In another days 2𝑦 work will be done
2 Remaining work = 576 – 306Ru = 270 unit
200y
C will take = 4y
days = 50 day When Ankit and Satish work alternatively
D will take = 40 days. Then two days’ work = (7 + 5) = 12 units
44
C and D will complete work together in In 44 days total work done = 2 × 12
50×40 200
= 90 = 9 = 264 unit
According to condition On 45th days remaining work done by Ankit =
(270 –264) 6
200 200 = days
+ =1 7 7
9𝑥 9 × 2𝑥 6
Total time = 44 7 days

66 Adda247 Publications For More Study Material


Visit: adda247.com
Cracker Book for Bank (IBPS | SBI | RRB PO | Clerk) Mains Exams

27. (b);Let Veer take 3𝑥 day and Satish take 2𝑥 days to ATQ,
complete the task They start work alternatively—
So, efficiency of Veer and Satish be 2𝑥 units/day & First day efficiency =(5x + x) = 6x
3𝑥 units/day respectively. Second day efficiency= (5x + 3x) = 8x
6x×12+8x×12
Whole work completed by Satish in 24 days Total work = × 100
70
So, while task done by Veer alone 168x
24 = 70 × 100
= 2𝑥 × 3𝑥 = 36 days
= 240x
Total task = 72𝑥 units
72𝑥 Mother complete 85% of work in
Veer and Satish do it together = 240x×
85
5𝑥
2 = 100
= 14 days 3x
204x
5
Mahendra do same task alone =
3x
72 12
= – = 12 days = 68 days
5 5
72𝑥 36×100×7
Efficiency of Mahendra = 12 = 6𝑥 unit/day 31. (e); Total work = = 280 units
90
When Veer and Mahendra do it alternatively — ATQ -
First day task = 6𝑥 unit 4 (x + 4) + 3 (x + 20) + 18 × 2 = 280
Second day task = 2𝑥 units 4x + 16 + 3x + 60 = 280 – 36
In two day = 8𝑥 unit 7x = 168
Task completed =
72𝑥
× 2 = 18 days. x = 24 days
8𝑥 They all together in 24 days
28. (b);When these pipes are opened for two hours, = 24 (4 + 3 + 2)
means each of them open for 40 min. individually. = 216 units
216
Pipe A can fill 1 bucket in 8 minutes Required portion =
280
& 5 buckets in 40 minutes …(i) 27
18×40 =
Similarly pipe C can fill 35
5×18
= 8 bucket in 40 min. …(ii) 32. (c); Ratio of efficiency of Divyaraj : Veer
And Pipe B can fill
3
× 40 = 6 buckets in 40 = 125 : 100
20 =5:4
minutes …(iii) 3
Hence volume of tank = 5 + 6 + 8 Efficiency of Sameer = 4 ×
4
= 19 buckets = 3 units/day
= 19 × 5 ATQ,
120
= 95 liters (5× +4)x 5
100
=
(4+3)(x+8) 7
29. (b);Efficiency of Ram :Shyam = 125 : 100 10x 5
=5:4 =
7x+56 7
60
Ram takes = 5 × 4= 48 days 70x – 35x = 280
x = 8 days
Let total work = 60×4= 240 unit
Total work = 10 × 8 + 7 × 16
Ram and Shyam takes together = 80 + 112
240 80
= (5+4) = 3 days = 192
Ghanshyam takes =
80

20
= 20 days Raj efficiency = 9 units/day
3 3 Sameer and Raj efficiency = (3 + 9)
240
Efficiency of Ghanshyam = = 12 unit/day = 12 units/day
20
192
Rams and Shyam 16 days work (Sameer + Raj) =
12
16 × (5 + 4)= 144 unit = 16 days
240−144
Remaining work by Ghanshyam = 12 33. (b);ATQ
96 For tank A —
= 12 = 8 days

30. (b);Let efficiency of child and father be x unit/day and


5x unit/day
x+5x
Efficiency of mother = 2
ATQ
= 3x unit⁄day
67 Adda247 Publications For More Study Material
Visit: adda247.com
Cracker Book for Bank (IBPS | SBI | RRB PO | Clerk) Mains Exams

For tank B — = 70 liter


70 7
6 (x – 24) + 3 (x – 24) – 4 (x – 24) = 90 Filled portion = 360 = 36
6x – 144 + 3x – 72 – 4x + 96 = 90
Solutions (36-38):
5x = (90 + 144 + 72 – 96)
210
x= 5
= 42 minutes
Total quantity of tank B = 90 × 4 = 360 liter
Alternatively (P + Q – M) for 42 minutes, means
each work pipe for 14 minutes— Lets efficiency of Kartik is x unit/day and Anup is 1.75x
All three in 14 minutes unit/day
ATQ—
(P + Q – M) = 14 × 6 + 14 × 3 – 14 × 4
22 days (Ankur + Shubham) + 6 days (Shubham + Kartik)
= 70 liter + 30 day (Kartik + Anup) = 576
70
Filled portion = 360 22 × 9 + 6 × 8 + 30 (x + 1.75x) = 576
7 30 (2.75x) = 576 – 246
= 36 330
2.75x = 30
11
34. (a); Let capacity of pipe A = y m3 x = 2.75 ⇒ x = 4 unit/day
So, capacity of pipe B = y + 8 m3 efficiency of Anup = 4 × 1.75 = 7 unit/day
Required time to filled the swimming pool
5040 36. (a); Working efficiency of A = 120% of working
= 𝑦 minutes
efficiency of B
Required time to empty the swimming pool = B can complete ‘X’ work = 36 days
5040
minutes A can complete ‘X’ work = 30 days
(𝑦+8)
Let ‘X’ work = 180 unit
ATQ – A working efficiency = 6 unit/day
5040 5040 45
– (𝑦+8) = B working efficiency = 5 unit/day
𝑦 4
112 112
− (𝑦+8) = 4
1 15 × 6 + 10 × 5 + 𝐶 × 10 = 180
𝑦 so, C working efficiency = 4 unit/day
3584 = y2 + 8y For work ‘Y
y2 + 8y −3584 = 0 Total of work ‘Y’=(6 + 4) × 12 + 16 × 5
y2 + 64y – 56𝑦 − 3584 = 0
= 200 𝑢𝑛𝑖𝑡𝑠
y(y + 64) – 56(𝑦 − 64) = 0
(y + 64) (𝑦 − 56) = 0 Total units of both work ‘X’ and work ‘Y’
Y = 56 m3 = 180+200= 380 units
380
Capacity of second swimming pool so,D working efficiency =
38
= (56 + 8) × 112.5 = 7200 m3
= 10 unit/day
35. (b);ATQ Now:
For tank A — A, B and C work together in ‘x’ work
1
= × 180 unit = 60 unit
3
A + B + C = (6 + 4 + 5) unit per day
60
= 15 = 4 days
Remaining work = 120 unit
ATQ (B + D) ⇒ (5 + 10) unit per day
120
For tank B — = 15 = 8 days.
6 (x – 24) + 3 (x – 24) – 4 (x – 24) = 90 B work for = 4 + 8 = 12 days.
6x – 144 + 3x – 72 – 4x + 96 = 90 37. (e); ‘X’ work = 180 unit
5x = (90 + 144 + 72 – 96) A + C + D ⇒ 6 + 4 + 10 = 20 unit/day
210 180
x = 5 = 42 minutes Days = 20 = 9 days
Total quantity of tank B = 90 × 4 = 360 liter Work = 200 unit
Alternatively (P + Q – M) for 42 minutes, means (A + B + C + D) ⇒ (6 + 5 + 4 + 10)
each pipe for 14 minutes— = 25 unit/day
200
All three in 14 minutes = 25 = 8 days
(P + Q – M) = 14 × 6 + 14 × 3 – 14 × 4 Total time = (9 + 8) = 17 days

68 Adda247 Publications For More Study Material


Visit: adda247.com
Cracker Book for Bank (IBPS | SBI | RRB PO | Clerk) Mains Exams

38. (a); Let efficiency of E is Z unit/day = (24 + 12) × 12 = 432 unit


432 3
he work for 12 days Required portion = 576 = 4
work complete = 12Z unit
40. (a); If Ankur and Shubham work alternatively, then
B and C work for 8 days = (5 + 4) × 8 unit 𝑦
both work for 2 days individually
= 72 unit 𝑦
Remaining work = 180 - 72 = 108 unit Same, if Kartik and Anup work for day
2
108 𝑦
Efficiency of E = = 9 unit⁄day alternatively both work for day individually
12 4
Now, ATQ—
y y y
A and E completed work ‘X’ Ankur × 2 + Shubham × 2 + Kartik × 4 +
180 y 75 1
= = 12 days Anup × 4 = 576 × (100 – 8 ) × 100
15
D, B and C completed both work ‘X’ and ‘Y’ or
y y
200+180 (Ankur + Shubham) × + (Kartik + Anup) ×
= = 20 days 2 4
19
= 522
Ratio = 12 : 20 ⇒ 3 : 5 𝑦
9 × 𝑦 + (4 + 7) × 2 = 522 ∗ 2
39. (c); (Kartik + Anup) × x days + (Ankur + Shubham) × y = 72 days
2 87.5
(𝑥 + 2 3) day = 576 × 100 Efficiency of Abhimanyu
75 1
3x+8 7 576× ×
(4 + 7) × x + 9 × ( 3
) = 576 × 8 = 8 100
= 6 unit/day
9
11x + 9x + 24 = 504 Abhimanyu work did in (y + 16) days
20x = 480 = (72 + 16) × 6 = 528 unit
x = 24 days
1 Remaining work done by Anup and Kartik in
576×
efficiency of Prabhat = 6 8 = 12 unit/day =
576 –528
=
48
days
11 11
Prabhat will do in (x + 12) days

69 Adda247 Publications For More Study Material


Visit: adda247.com
Cracker Book for Bank (IBPS | SBI | RRB PO | Clerk) Mains Exams

Chapter
Speed Time and Distance
8
BEST APPROACH TO SOLVE THE QUESTIONS

𝐷𝑖𝑠𝑡𝑎𝑛𝑐𝑒
Speed is defined as the distance covered in per unit time. Mathematically, Speed= 𝑇𝑖𝑚𝑒
.

The whole concept of TSD (Time, speed and distance) lies within this formula. Three most logical points in TSD are:
1. When speed is constant,
Distance is directly proportional to time i.e. D ∝ 𝑇
2. When distance is constant,
1
Speed is inversely proportional to time i.e. S ∝
𝑇
3. When time is constant,
Speed is directly proportional to distance i.e. S ∝ 𝑇
Another important term that we must know while solving questions in this chapter is RELATIVE SPEED.
Relative speed is defined as the net speed of two entities when they are in motion.
If the two entities are moving in opposite direction to each other, net/relative/resultant speed is sum of the speeds. And
when they move in the same direction, their net/relative speed is the difference of their speeds.
Example: ‘A and B stay next to each other and study in the same school. A and B start walking towards their
school at speeds of 2 km/hr and 3 km/hr respectively. The faster of the two reaches the school first,
turns around and starts walking back. If A and B meet 200 m away from the home. What is the
distance between their residences and their school?’
Solution: Let the distance where they met be ‘x’ metres from the school.
Distance travelled by A = 200m
Distance travelled by B= 200+x+x=200+2x
As the ratio of their speed is 2:3, and time is same, hence the ratio of distance travelled by the them is also
2:3.
200 2
200+2𝑥
=3
Solving, x= 50
Distance between their residences to school= 200+ x= 250m
Example: A goods train leaves a station at a certain time. After 6 hours, an express train leaves the same station
and travels in the same direction at a speed of 90 km/hr. If the express train catches up the goods
train in 4 hours, what is the speed of the goods train?
Solution: Distance covered by express train after leaving the station = 90× 4 = 360𝑘𝑚
Goods train covers this distance in 6+4=10 hours.
360
Hence speed of goods train= 10 = 36 𝑘𝑚/ℎ𝑟

 Key Points to Remember for New Pattern Questions


➢ Average speed is calculated by dividing the total distance travelled by total time taken.
➢ Always be careful that units of speed, time and distance for two or more entities should be same
while solving a problem.
5 18
➢ To change speed of km/hr into m/second multiply the former by and by 𝑖𝑛 𝑣𝑖𝑐𝑒 𝑣𝑒𝑟𝑠𝑎 𝑐𝑎𝑠𝑒.
18 5

70 Adda247 Publications For More Study Material


Visit: adda247.com
Cracker Book for Bank (IBPS | SBI | RRB PO | Clerk) Mains Exams

Practice Exercise Based on new Pattern

1. In how much time a train can cover a distance of 600 8. Two cities Rampur and Dhamnagar are 300 km apart,
km if speed of train is 25% more than the speed of car Bhavya starts from Rampur at 8 : 24 am and an hour
and speed of car is 20 % more than the speed of a
truck which cover 640 km in 20 hours in which he later Abhi starts from same city. After travelling for
takes 40 minute break after every 100 km. one hour Abhi reaches at city which Bhavya had
(a) 9 hours (b) 8 hours 20 mins passed 40 min earlier. The city is in the way of
(c) 10 hours (d) 8 hours (e) None of these Rampur and Dhamnagar. If they reaches city
2. A train travelling at 144 km/hr crosses another train, Dhamnagar at same time. Find their speed.
having 30 meter less length and travelling in opposite (a) 50 km/hr, 75 km/hr
direction at 126 km/hr in 6 seconds. If longer train (b) 75 km/hr, 75 km/hr
cross a railway platform in 20 second then find
smaller train will cross same platform in how many (c) 100 km/hr, 100 km/hr
seconds? (d) 75 km/hr, 100 km/hr
(a) 22 seconds (b) 24 seconds (e) None of these
(c) 28 seconds (d) 32 seconds
(e) 30 seconds 9. A 240 meters long train crosses a 210 meters long train
3. Train A running at the speed of 108 km/hr crosses a running in opposite direction in 6 sec. Ratio between
man, who running in the opposite direction at the speed of longer train and smaller train is 7 : 8. If faster
speed of 12 km/hr in 7.2 sec. If speed of train A train crosses a platform in 9 sec, then find time taken
increased by 25% and it takes 48 seconds to cross by slower train to cross a bridge, which is 60 meters
another train B, which running at the speed of 90
more long than platform?
km/hr in same direction. Find the length of train B? 142 136 90
(a) 280 meters (b) 360 meters (a) 7
𝑠𝑒𝑐 (b) 7
𝑠𝑒𝑐 (c) 7
𝑠𝑒𝑐
(c) 180 meters (d) 160 meters 148 142
(e) 220 meters (d) 𝑠𝑒𝑐 (e) 𝑠𝑒𝑐
7 7
4. Train P leaves Delhi at 7 PM and is travelling at a 10. Train P leave place A by is travelling at a speed of 120
speed of 80 kmph, 3 hours later another train Q leaves
kmph. 4 hours later another train Q leaves place by
Delhi and is travelling in same direction as train P in
how many time (in minutes) train Q will be 18 km travelling in same direction as train P. Train Q will be
ahead train P, if speed of train Q is 120 kmph? 36 km ahead of train P in T hours and speed of train Q
(a) 383 minutes (b) 387 minutes is 160 kmph, then find a bus travel what distance in
(c) 380 minutes (d) 377 minutes (T + 3.1) hr, if speed of bus is 75% of speed of train P?
(e) 375 minutes
(a) 1360 km (b) 1440 km (c) 1540 km
5. A bus and a car start from a place A toward B (d) 1280 km (e) 1720 km
simultaneously at 10:00 am. Bus reach at B in 4 hours
and return for C, which is exactly in the middle of A 11. Satish started from point A in a boat to reach point B.
and B, and meet the car after 1 hour after leaving B. After 6.5 hours he covered only 20% of the distance
Find the time at which car reach C after returning
from B. and reach at point M. Now, Satish started from point
(a) 6:00 pm (b) 8:00 pm (c) 10:00 pm M reached at mid point of A and B and came back to M
(d) 8:00 am (e) 12:00 pm in 29.25 hours. In what time Satish can cover the
6. Two trains A and B running at the speed of 108km/hr distance between B and A if he started from B?
and 162km/hr crossed another train ‘P’, which is (a) 58.5 hours (b) 32.5 hours (c) 65 hours
standing stationary of length 240 meters in 14 sec and (d) Can’t be determined (e) 50 hours
16 sec respectively. How much time both train A and
B will take to cross each other, if both running in 12. Train X having length 130 m and train Y having length
opposite direction? 145 m moving in opposite direction. They enter into a
(a) 8.4 𝑠𝑒𝑐 (b) 8.6 𝑠𝑒𝑐 (c) 8.8 𝑠𝑒𝑐
tunnel which have length equal to the sum of length of
(d) 9 𝑠𝑒𝑐 (e) 8.2 𝑠𝑒𝑐
both trains. Trains meet after 10 second of entering in
7. There was a race of 3000 meters between A & B on a the tunnel. What percent of train X part is leave out
circular track of 750 meters. First time they meet
during the race is after 5 minutes of starting of race. the tunnel when it meet train Y if they have there
Find the time taken by B to complete the race, if he speed in the ratio of 5 :6.
runs at half the speed of A. 11
(a) 2 13 %
11
(b) 3 13 % (c) 4 3 %
1
(a) 20 minutes (b) 18 minutes (c) 15 minutes
(d) 10 minutes (e) None of these (d) 5% (e) 8%

71 Adda247 Publications For More Study Material


Visit: adda247.com
Cracker Book for Bank (IBPS | SBI | RRB PO | Clerk) Mains Exams

13. A car and a bus starts from point ‘A’. After ‘T’ hours 19. If initially, customer alto start moving toward
bus is 48 km ahead of car, while after ‘8’ hours Mahendra at speed of 60 km/hr and take his
distance between bus and car is same as distance consignment after 3 hours then find the total distance
covered by bus in one hour. If relative speed of car between Mahendra and Customer initially.
and bus if they move towards each other is 240, then (a) 256 km (b) 150 km (c) 500 km
1
how much distance can bus cover in (𝑇 – ) hour. (d) 450 km (e) 300 km
2
(a) 320 km (b) 280 km (c) 384 km Directions (20-21): A man decided to run 15 rounds of a
(d) 336 km (e) 256 km circular track of 400 m in certain time with certain speed.
He starts running but after completing some round
14. Point A to Point B is a downstream journey of 300 around the track he reduced his speed by 40% due to
km on a stream which flows at a speed of 5 km/hr. which he takes 4 min extra as scheduled. But if he reduced
Two boats P and Q starts from point A and Point B his speed by completing 3 more rounds he would have
respectively with speed of 25 km/hr and 15 km/hr in reached 2 min earlier than the time he actually reached.
still water. After reaching the opposite point they
return to their starting points, find after how much 20. Find the original speed of man?
time will they meet second time? (a) 240 m/min (b) 160 m/min (c) 200 m/min
(a) 7.5hour (b) 15hour (c) 20hour (d) 400 m/min (e) 350 m/min
(d) 10hour (e) None of these 21. Find the number of rounds at which he decided to
15. Distance between Delhi and Jaipur is 300 km. Aman reduce his speed?
starts from Delhi and Rajiv from Jaipur at same time. (a) 7 (b) 9 (c) 10
After two hours, Aman realized he was travelling slow (d) 8 (e) 11
and therefore increased his speed by 25% and meet 22. A cyclist left point A for point B and travelled at the
Rajiv at a point 108 km from Delhi. Find the increased constant speed of 25 km/h. When he covered the
speed of Aman, if Rajiv derived at a constant speed of distance of 25/3 km, he was overtaken by a car that
75 km/hr. left point A twelve minutes after the cyclist and
(a) 40 km/hr (b) 50 km/hr (c) 60 km/hr travelled at a constant speed too. When the cyclist
(d) 55 km/hr (e) 65 km/hr travelled another 30 km, he encountered the car
Directions (16-17): Time taken by train A to cover a returning from B. Assume that the car did not stop at
distance is 1.5 times the time taken by train B to cover the point B. Find the distance between A and B.
same distance. Train A and train C starts simultaneously (a) 39.5833 km (b) 41.0833 km (c) 60.833 km
from a station in same direction. Train B, start after 30 (d) 43.33 km (e) 50 km
min and overtake train C in 1.5 hour later than it cross 23. Two person left simultaneously two places A and B.
train A. One of them left A for B while the other left B for A.
16. If speed of train A is 80 km/hr, then what is the speed Both travel at uniform speed . The first person on
of train C. (2 Marks) reaching B returns to A and then again travels back to
(a) 60 km/hr (b) 100 km/hr (c) 125 km/hr B and so on. Similarly the second person on reaching
(d) 75 km/hr (e) 80 km/hr A returns to B and then travels back to A and so on.
What will be the distance covered by the first person
17. If train A double its speed, then speed of train A is when they meet for the third time given the ratio of
what % more than the speed of train C. (1 Mark) the speed of the first person to that of the second
(a) 60% (b) 100% (c) 80% person is 3 : 2 and the distance between A and B is
(d) 50% (e) None of these
500 m?
Directions (18-19): A consignment should deliver on (a) 1000 m (b) 1500 m (c) 2500 m
time to customer for which Mahendra start his journey (d) 1200 m (e) 1800 m
with uniform speed, after 2 hours customer call Mahendra
24. A train M running at the speed of 108 km/hr crosses a
to deliver his order 1 hour before the decided time.
man running at 12 km/hr in opposite direction of
Mahendra increased his speed by 50% to deliver it 1 hour
train in 12 second and crosses a platform in 32 sec. If
before the decided time.
a train N is Standing on same platform and length of
18. Find the total time taken by Mahendra to deliver the platform is 140 meter more than length of train N.
consignment? Find in what time train N will cross train M running
(a) 4 hours (b) 5 hours (c) 6 hours in same direction, if train N passes a pole in 12
(d) 7 hours (e) 3 hours second?

72 Adda247 Publications For More Study Material


Visit: adda247.com
Cracker Book for Bank (IBPS | SBI | RRB PO | Clerk) Mains Exams

(a) 168 sec (b) 164 sec (c) 154 sec (a) 20 sec (b) 10 sec (c) 15 sec
(d) 186 sec (e) 172 sec (d) 14 sec (e) 18 sec
Directions (25-26): Satish and Bhavya starts from point 30. Two trains P and Q started from two points A and B
P and Q respectively at same time to reach at point B. towards each other to cover 300 km distance. Speed
Satish overtakes Bhavya after 10 hours at point A. If of train Q is twice of speed of train P. If train P runs at
Bhavya starts 4 hours earlier than Satish then they will its normal speed and train Q reduces its speed to 50%
meet at point B after 16 hours when Satish starts. Ratio then it will take 2.5 hrs. more to meet each other. Find
between distance between Q and A to Q and B is 1 : 2. the time (in sec) in which train P can cover a platform
Point P, Q, A and B lies on a straight road. of 150m length if the length of train P is given as 50m.
(a) 18 seconds (b) 36 seconds (c) 72 seconds
25. If distance between point A and point B is 180km (d) 54 seconds (e) 24 seconds
then find time taken by Bhavya to reach point ‘C’ 27
which is 432km ahead of point ‘B’? 31. A train P, 180 meter long train passed a pole in sec
4
(a) 36 hours (b) 44 hours (c) 24 hours and also passed two trains Q and R in 9 sec and 39 sec
(d) 32 hours (e) 40 hours respectively, where train Q running in opposite
direction of train P and train R is running in same
26. If Satish and Bhavya started to move towards each direction of train P. If length of train Q and R is 240
other from P and Q at 8:00 a.m, then at what time they meter and 210 meter respectively, then in what time
will meet? train Q will pass train R, if both runs in opposite
(a) 9 : 00 a.m (b) 9 : 30 a.m (c) 10 : 00 a.m direction ?
(d) 10 : 30 a.m (e) 11 : 00 a.m (a) 35 sec
7
(b) 9 sec
3
(c) 12 sec
11 11
27. A bus (X) start its journey from P to Q simultaneously (d) 15 sec (e) 55 sec
another bus (Y) start from Q to P they reach their
32. Total distance between Delhi to Lucknow is 480 km. A
respective destination after 3 hour. Bus X travel with train starts running with an average speed of 60
a speed of 40 km/hr in first hour 50 km/hr in second km/hr from Delhi to Lucknow while another train
hour and 60 km/hr in 3rd hour. Bus Y travel with a starts Journey after 120 minutes of first train and
speed of 60 km/hr in first hour, 40 km/hr in second reaches Lucknow 30 minutes before first train. If first
and 50 km/hr in last hour. Find out the distance train stops for 5 minutes on each station and second
between Q and the point where the buses cross each train did not stop at any station ,then find the ratio
other. between speed of first train to speed of second train
(a)87 (b)88 (c)89 given that total number of station between Delhi and
2 7
(d)82 (e) 87 Lucknow are nine.?
9 9
(a) 25 : 33 (b) 25 : 32 (c) 35 : 25
175
28. Speed of a car is 𝑚/𝑠 and speed of bus is (d) 25 : 31 (e) 21 : 31
9
2
14 % less to speed of car, bus and car covered 33. The ratio between length of two trains is 3 : 2 and
7
speed of both trains is 72 km/hr & 90 km/hr
different distance in different interval of time and the
respectively. Faster train crosses slower trains in 60
sum of the time taken by them to cover their
sec running in same direction. If slower train crosses
respective distances is10 hours but distance covered a goods train, which is running at the speed of 108
by bus is 80 km more than distance covered by car. If km/hr in 7.2 sec running in opposite direction. Then
a train covered total distance which covered by car find the time taken by faster train to cross goods train
2
and bus together in 6 3 ℎ𝑜𝑢𝑟𝑠, then find speed of running in same direction?
train? (a) 40 sec (b) 64 sec (c) 60 sec
(a) 96km/hr (b) 84km/hr (c) 78km/hr (d) 56 sec (e) 50 sec
(d) 72km/hr (e)64km/hr 34. Two athletes ‘A’ and ‘B’ practice running on 800
29.Two trains running in opposite directions cross each meters’ track. If they start running at their usual
20 speeds, in same direction, at same time, from same
other in 3 sec. Longer train cross shorter train in 60 point, first time they meet is in 100 seconds. During
sec when both train running in same direction. If 2
warm-up, ‘A’ runs at 18 % of his usual speed and
length of shorter train increased by 50% than longer 11
completes 400 meters in 50 second. Find the time
train cross shorter train in 72 sec, when both running
taken by athlete ‘B’ to complete 9 rounds of 400-
in same direction. If difference between length of both 5
trains is 60 meters, then find the time in which longer meter track, if he runs at of his usual speed. Assume
4
train will cross a platform whose length is 77 %
7 athlete ‘A’ is faster than ‘B’.
9 (a) 100 second (b) 450 second (c) 90 second
more than length of longer train? (d) 80 second (e) 85 second

73 Adda247 Publications For More Study Material


Visit: adda247.com
Cracker Book for Bank (IBPS | SBI | RRB PO | Clerk) Mains Exams

35. The speed of car is 25% more than the speed of bus. express’ is 360 m and 180 m respectively, then find in
The time difference between them to cover a certain what time ‘Shatabdi express’ will cross ‘Duranto
distance D, is 1 hour. On particular day, driver noticed express’ while running in same direction?
that if they are (D- 40) km apart from each other then (a) 24 sec (b) 28 sec (c) 32 sec
they can meet in 2 hours driving in opposite direction (d) 36 sec (e) 48 sec
at their normal speed. Find 150% of the speed of bus ?
(a) 150 km/hr (b) 90 km/hr (c) 120 km/hr Directions (39–40): There are two trains A and B, both
(d) 105 km/hr (e) None of these train starts travelling to each other from stations P and Q
respectively. Train A reaches the station Q in total 4 hours
36. Time taken by car A to cover distance from point X to while train B reaches the station P in total 4 hours 48
point Y is 6½ hrs including ½ hour of stoppage. B also minutes. Speed of train A is 120 km/hours.
started from point X and stopped 90 km earlier from
point Y after travelling for 6½ hrs. Had they started 39. Train A and B running on two different railroads
from same point and at same time, the difference parallelly in opposite direction to two different trains
between their distance covered in 7.5 minutes will be C and D respectively. Train A passed train C, which
2.5 km. Find the distance between two cars, if they
running at the speed of 60 km/hr. in 6 sec. and train B
start a race for the distance which is twice the
distance between X and Y , assuming that both cars passed train D, which running at the speed of 110
stops when one of the car finish the race km/hr. in 9 sec. if ratio between length of train A to
(a) 360 km (b) 240 km (c) 180 km train C is 3: 2, and that of between train B to train D is
(d) 300 km (e) 280 km 4 : 3. Find in what time train D passed train C, if both
37. Two trains A and B with their length difference 17 m running in same direction ?(2 marks)
cross each other in 2.6 seconds while running in (a) 22.64 sec (b) 24.84 sec (c) 24.24 sec
opposite direction. Train A crosses train B in 13
(d) 28.84 sec (e) 20.84 sec
seconds while they are running in same direction. If
train A takes 2 seconds to cross a pole, then find the 40. Ratio between speed of train A to train E is 6: 5.Train
sum of speed of both the trains, assuming that train B E starts a journey from Kolkata to Delhi, which is 600
is longer than train A. km at 9.45pm and train A starts same journey after 30
(a) 34 ms¯¹ (b) 51 ms¯¹ (c) 68 ms¯¹ minutes of train E.Train A stop for x minutes on each
(d) 85 ms¯¹ (e) 102 ms¯¹ three stoppages and train E stop for (x+10) minutes
each on three stoppages between Kolkata to
38. ‘Rajdhani express’, 240 m long train crosses a pole in
Delhi.Train A reaches Delhione hour earlier than train
6 sec and it also crosses two trains ‘Shatabdi express’
120 E, which reaches at 4.30 am.Findinwhat time train B a
and ‘Duranto express’ in 𝑠𝑒𝑐 & 42 sec respectively, total distance of 720 km cover between Lucknow to
17
where ‘Shatabdi express’ is running in opposite Jaipur. If train B stop for (x+10) minutes on four
direction of ‘Rajdhani express’ and ‘Duranto express’ stoppages between Lucknow to Jaipur?(2 marks)
running in same direction as that of ‘Rajdhani (a) 4 hours (b) 8 hours (c) 8.2 hours
express’. If length of ‘Shatabdi express’ & Duranto (d) 10 hours (e) 12 hours

Solutions

1. (c); Stop of truck = 6 × 40 = 240 mint or 4 h. Let length of platform be P meters


640 640 ATQ,
Speed of truck = = = 40 𝑘𝑚/ℎ
20−4 ℎ. 16 5 240+𝑃
40×120 144 × =
Speed of car = 100
= 48 𝑘𝑚/ℎ 18 20
48×125 P = 800 – 240 = 560 meters
Speed of train = 100
= 60 𝑘𝑚/ℎ Let required time = T
600
Required time = = 10 ℎ𝑜𝑢𝑟𝑠 ATQ,
60 5 210+560
126 × 18 =
2. (a); Let length of both trains be L meters and (L – 30) 770
𝑇
meters respectively T = ⇒ T = 22 sec
35
ATQ,
5 𝐿+ (L – 30) 3. (b); Let length of train A be L meters
(144 + 126)× = 5 𝐿
18 6 (108 + 12) × 18 = 7.2
450 = 2L – 30
L = 240 meters
L = 240 125
Smaller train length = 210 meters New speed of train A = 108 × 100

74 Adda247 Publications For More Study Material


Visit: adda247.com
Cracker Book for Bank (IBPS | SBI | RRB PO | Clerk) Mains Exams
= 135 𝑘𝑚/ℎ𝑟 8. (d); Let the city in way of Rampur and Dhamnagar is
Let length of train B be S meters C.
5 240+𝑆
(135 – 90)× 18 = 48
S = 360 meters
4. (b); Relative speed of both train = (120 – 80) = 40
kmph
Distance covered by train P from Delhi when Time Speed
train Q was not moving = 80 × 3 = 240 km Bhavya 80 3x
Train Q should be covered (240 + 18) km from Abhi 60 4x
ATQ,
Delhi for ahead 18 km to train P from Delhi 300 300
Required time =
258
km – =1
3x 4x
40 25
= 6.45 = 6.45 × 60 = 1 ⇒ x = 25
x
= 387 minutes ∴ Bhavya Speed = 3 × 25 = 75 km/hr
5. (b); Abhi speed = 4 × 25 = 100 km/hr
9. (c); Let speed of longer train and smaller train be 7𝑥
and 8𝑥 respectively.
ATQ—
(240+210)
Car travel → 3/4 part of one side distance in 5 (7𝑥 + 8𝑥) =
6
hour. 90𝑥 = 450
20
1 part = hour 𝑥=5
3
Car can travel 1.5 part in Speed of longer train = 5 × 7 = 35 m/s
20
× 1.5 = 10 hour Speed of smaller train = 5 × 8 = 40 m/s
3
Let length of platform be l meter
Time to reach C after returning form point 𝑙+210
B → 10:00 + 10 = 8:00 pm 40 =
9
40 × 9 = 𝑙 + 210
6. (c); Let speed of train A and B be ‘x’ meters and ‘y’
meters respectively l=150
ATQ, Then, length of bridge = 150 + 60 = 210 meters
5 x+240 Let time taken by slower train to cross bridge be
108 × 18 = 14 m⁄s
T sec.
30 × 14 = x + 240 210+240
T=
x = 180 meters 35
And, 35T = 660
5 𝑦+240 90
162 × 18 = T = 7 sec.
16
45 × 16 = y + 240 10. (b); Relative speed of both trains
y = 480meters = (160 – 120) = 40 kmph
Let required time be T sec Distance covered by train P from place A when
5 480+180 train Q was not moving = 120 × 4 = 480 km
(108 + 162) × 18 = T
Train Q should cover (480 + 36) km from place A
75 𝑇 = 660
in T hrs
T = 8.8 sec 516
Required time, T = 40 = 12.9 hr
7. (a); Let speed of A is 2x & speed of B is x Bus speed = 120 × = 90𝑘𝑚𝑝ℎ
3

According to question, 4
750 Bus covered in ( T+3.1)hours = ( 12.9 + 3.1) × 90
a−b
= 5 min = 1440 km
⇒ a – b = 150 meters/min 11. (c); Let total distance from A to B = ‘D’
⇒ 2x – x = 150 meters/min ATQ, Satish cover 20% distance in 6.5 hours So,
⇒ x = 150 meters/min he can cover 30% distance (M to mid-point of A
It is speed of B hence and B) in
3000 3000 6.5
Time taken = = = 20 minutes 2
× 3 = 9.75 hr.
x 150

75 Adda247 Publications For More Study Material


Visit: adda247.com
Cracker Book for Bank (IBPS | SBI | RRB PO | Clerk) Mains Exams

Time taken by Satish to come back from mid- 15. (b); Let initial speed of Aman is x km/hr.
point to M = 29.25 – 9.75 = 19.5 hr Distance travelled by him in 2 hours is 2x km.
30% distance covered by Satish in 19.5 hr. While distance travelled by Rajiv in these 2 hours
19.5
100% distance covered by Satish in × 10 is 75 × 2 = 150 km
3
= 65 hr When both of them meet, Aman had travelled a
12. (b); Speed of train X and Y 5x and 6x respectively. distance of 108 km.
Length of tunnel → 130 + 145 = 275 m. Distance travelled by Aman with 25% increase in
Speed of trains per second his speed = (108 – 2x) km
275 125 5x
= = 27.5 𝑚/s And his increased speed = x( )= km/hr
10 100 4
5x + 6x = 27.5 If Aman had travelled 108 km, Rajiv had travelled
x = 2.5 192 km.
speed of train X → 12.5 m/s 192 – 150 = 42 km after 2 hours
Speed of train Y → 15 m/s
Time taken by Rajiv to travel 42 km
Distance cover by train X in tunnel 42 14
= 12.5 × 10 = 125 m. = = hours
75 25
Length of train X leaves out = 130 – 125 = 5m This is equal to time taken by Aman to travel (108
5
Required% = 130 × 100 – 2x) km
11 (108–2x) 14
=3 % 5 =
13 x 25
4
13. (a); Let, speed of car = x km/hr 108– 2x =
14×5x

Speed of bus = y km/hr 25×4


7
ATQ, 108 = 10 x + 2x
Distance covered by bus in one hour 27
x = 108 ⇒ x = 40 km/hr.
= Distance between bus & car in 8 hour 10
⇒ y = 8 (y – x) [ ∵ y > x] Therefore increased speed of Aman was
5
y = 8y – 8x = × 40 = 50 km/hr
4
8x = 7y …(i)
and x + y = 240 …(ii) Solutions (16-17): Time ratio between train A and train
On solving (i) & (ii) B = 1.5 : 1
x = 112, y = 128 So speed ratio between train A and train B is
After T hour bus is 48 km ahead of car = 1 : 1.5 or 2 : 3
⇒ (128 – 112) T = 48 Let speed of train A = 2x km/hr
48 Speed of train B = 3x km/hr
⇒ T = 16 = 3
Relative speed = 3x – 3x = x km/hr
Required distance = 2.5 × 128 = 320 km 2𝑥
Distance travel by train A in 30 m ⇒ 2
= x km
14. (c); Time when P and Q meet first time 𝑥
Train B cross train A ⇒ 𝑥 = 1 hour
300 300
= = = 7.5 hr So,
25+15 40
Distance travelled by P = 7.5 × (25 + 5) = 225 km Train B cross train C ⇒ 1 + 1.5 = 2.5 hour
Distance travelled by Q = 7.5 × (15 – 5) = 75 km Let speed of train C = y km/h
75 ATQ
P reach at point B in 30 i.e. 2.5 hour
2.5 × 3x = 3y
In 2.5hr Q travelled 2.5 × 10 = 25 km
x:y⇒2:5
Now Boat P returns and relative distance b/w P
Ratio of speed of A, B and C
and Q is 100 km and relative speed is 10 km i.e.
4:6:5
difference of speed of P (25 -5) = 20 km/hr
(upstream) and speed of Q (15 – 5) = 10 km/hr 16. (b); Speed of train A = 80 km/hr
80
(upstream) Speed of train C = × 5 = 100 km/hr
4
100
Time taken by P and Q to meet = = 10 hr
10 17. (a); Let speed of train A = 4x km/hr
Total time when P and Q meet second time = 7.5 So speed of train C = 5x km/hr
+2.5 + 10 = 20 hr (2×4𝑥–5𝑥)
Required % = 5𝑥
× 100 = 60%

76 Adda247 Publications For More Study Material


Visit: adda247.com
Cracker Book for Bank (IBPS | SBI | RRB PO | Clerk) Mains Exams

18. (b); Let speed of Mahendra = 2𝑥 km/hr Now, according to question,


So, Distance covered by him in 2 hours distance between first meeting and second
= 2𝑥 × 2 = 4𝑥 meeting is 30
Let he decide to deliver his consignment in 𝑡 hour So,
So total distance ⇒ 2𝑥 × 𝑡 = 2𝑥𝑡 distance between first meeting and point B will
ATQ 75+30
⇒ 2𝑥𝑡 = 2𝑥 × 2 + (𝑡– 3)3𝑥 be = = 52.5 km
2
3𝑥 → increase speed Required answer = 52.5 + 8.33 = 60.833 km
𝑡– 3 = time for which he travelled after speed is
23. (b); When they meet for the third time they together
increased
cover 5 × 𝐴𝐵 = 5 × 500 = 2500𝑚
On Solving equation →
5 ℎ𝑜𝑢𝑟𝑠 = 𝑡 The first person will cover
3
× 2500 = 1500 𝑚
19. (d); Distance covered by Mahendra in 3 hours → 2+3
2𝑥 × 3 = 6𝑥 24. (b); Lets length of train M = L meter
Total distance ⇒ 5 × 2𝑥 = 10𝑥 ATQ,
Remaining distance ⇒ 10𝑥 – 6𝑥 = 4𝑥 = Distance (108+12)×5 𝐿
covered by customer 18
= 12
and 4𝑥 = 60 × 3 L = 400 m
𝑥 = 45 km Let length of platform = Lp
108×5 𝐿𝑝 +400
Total distance = 45 × 10 = 450 km =
18 32
Solutions (20-21): Total distance = 15 × 400 = 6000 m 𝐿𝑝 = 960– 400
Let his original speed = x meter/min 𝐿𝑝 = 560 meter
Decided time = t min Length of train N = 560 – 140
Let after ‘n' round he reduced his speed 1 st decided = 420 meter
condition Speed of train N = 𝑆𝑛
6000
⇒t= …(i) 𝑆𝑛 =
420
= 35 m/s
x
3x 12
is the reduced speed after n rounds Relative speed of train M and N, when both
5
Now 2nd condition running in same direction
400×n 6000–400×n 5
+ 3x = t+4 …(ii) = 35 −108 × = 5 𝑚/𝑠𝑒𝑐
x 18
5
Let required time is t
3rd → supposed condition
400(n+3) 6000–400×(n+3) ATQ,
+ 3x = t + 2 …(iii) 400+420
x
5
5=
𝑡
Putting value of ‘t’ from (i) into (ii) and (iii) 820
t= 5
⇒ 3x + 200n = 3000 …(iv)
⇒ 3x + 400n = 4800 …(v) t = 164 sec
Solving (iv) and (v) Solution (25-26)
x = 400 m/min
n=9
20. (d); Original speed of man = 400 m/min
21. (b); Number of rounds after which he reduced his Let speed of Satish = ‘a’
speed = 9 And, speed of Bhavya = ‘b’
Let distance between P and Q = x
22. (c); Time taken by Cyclist to reach 25/3 km And, QA and QB be y and 2y respectively.
25 1
= 25 × 3 = 3 hr = 20 min After 10 hrs Satish overtakes Bhavya at point A.
Car has taken to reach 25/3 km = 20 – 12 = 8 min 𝐷𝑖𝑠𝑡𝑎𝑛𝑐𝑒
⇒ 𝑆𝑝𝑒𝑒𝑑 =
25 𝑇𝑖𝑚𝑒
y= 10 × b and,
Speed of Car = 3  60 = 62.5 km/h (a - b) × 10 = x …(i)
8
Now time taken by cyclist to go further 30 km If Bhavya starts 4 hour earlier than Satish, then Satish
30 6 overtakes Bhavya at point B after 16 hours
= = hr = 72 min
25 5 Distance travelled by Bhavya in 4 hours = 4b
72 So,
Car will go in 72 min =  62.5 = 75 km
60 (x + 4b) = (a – b) × 16 …(ii)
77 Adda247 Publications For More Study Material
Visit: adda247.com
Cracker Book for Bank (IBPS | SBI | RRB PO | Clerk) Mains Exams

On solving (i) & (ii) [280+(280+80)]×3


Speed of train = 20
20𝑏 2𝑦
𝑥= 3
⇒𝑥= 3
…(iii) = 96 km⁄hr
Satish can travel and reach point B in 16 hours 29. (a); Let length of shorter train be L meters and length
⇒ 16a = 2y + x …(iv) of longer train be (L + 60) meters
On solving (iii) & (iv) And also speed of shorter and longer train be V
y = 6a m/s and U m/s respectively.
3𝑥 𝑥 3𝑥
So, 𝑦 = 2 , 𝑎 = 4 , 𝑏 = 20 When both trains travelling in opposite direction
(L+L+60)3
=U+V
20
3L+180
20
= U + V …(i)
When both train running in same directions—
L+L+60
= U– V
60
(2L+60)
= U– V …(ii)
60
25. (b); ATQ, When length of shorter train increased and both
3𝑥
2
= 180 train running in same direction —
(1.5L+L+60)
⇒ x = 120 km/hr = U –V
3𝑥 3𝑥 72
+ +432 2.5L+60
Required time = 2 2
3𝑥
= U– V …(iii)
72
3×120+432
20
From (i) and (ii)
= 3L+180 2L+60
18 2U = +
792 20 60
= = 44 hours U=
11L+600
…(iv)
18
120
𝑥 7L+480
26. (d); Required time = 𝑥 3𝑥 U= …(v)
+ 120
4 20
𝑥 After solving (iii), (iv) and (v)
= 5𝑥+3𝑥 L = 120 m, U = 25 m/s and V = 20 m/s
20
20 Length of longer train = 120 + 60 = 180 meter
= 8
= 2 ℎ𝑜𝑢𝑟 30 𝑚𝑖𝑛𝑢𝑡𝑒 7
Length of platform = 180 + 180 × 9 = 320 meters
Satish & Bhavya meet at 10 : 30 am
Time taken by longer train to cross platform
27. (d); Distance between P and Q is 150 km. 320+180 500
= = = 20 sec
25 25
Now X bus cover 40 km in 1 hour
Y bus cover 60 km in 1st hour 30. (b); Let, speed of train P = x
Remaining distance = 50 km Speed of train Q = 2x
𝑅𝑒𝑚𝑎𝑖𝑛𝑖𝑛𝑔 𝑡ime to cross each other Let, Normal time taken by train to meet each
50 50 5 other = ‘y’
= 40+50 = 90 = 9 ℎ𝑟
5 ATQ,
Distance which is covered by ′Y ′ in 9 hr (x + 2x) y = (x + x) (y + 2.5)
5 200
= 40 × 9 = 9 km 3y = 2(y + 2.5)
Distance between Q and the point where buses y = 5 hr.
300
crosses each other Now, x + 2x = = 60
5
200 2 3x = 60
= 60 + = 82 km
9 9 x = 20 km/hr
50
28. (a); Speed of bus =
175
×7=
6 25×6 = 9 m/sec
9 9 (150+50)
=
50
m⁄s =
50
×
18
= 60 km⁄hr Required time = ×9
50
3 3 5
200
Let distance travel by car and bus be D and (D + = × 9 = 36 sec
80) respectively. 50
D (D+80) 31. (c); Lets speed of train P, Q and R be S₁, S₂ and S₃
+ = 10
70 60 respectively
6D+7D+560 180 80 m
= 10 Speed of train P (S1) = 27 m⁄s =
420 3 s
13D = 4200 – 560 4
Speed of train Q (S2 )
13D = 3640 80 240+180
D = 280 km + 𝑆2 =
3 9

78 Adda247 Publications For More Study Material


Visit: adda247.com
Cracker Book for Bank (IBPS | SBI | RRB PO | Clerk) Mains Exams
420 80
S2 = 9 – 3 From 2nd condition
2
S2 = 20 m⁄s When athlete ‘A’ will run at 18 % of his usual
11
Speed of train R (𝑆3 ) 200
speed = 1100 × 𝑎 = 11 𝑎
2
80 210+180
3
– S3 = 39 400
80 = 2 = 50 ⇒ 𝑎 = 44 ms–1
S3 = 3
– 10 11
𝑎

S3 = m⁄s
50 ∴ 𝑏 = 36 ms–1
3 5
Lets required time be T sec If 𝑏 runs at 4
of his speed, which means with 45
50 240+210
Required time = 20 + 3 = ms–1
T 4×400
110
=
450
⇒T =
450×3 Time taken by him = 45
= 80 seconds.
3 T 110
3
T = 12 sec 35. (c); Let speed of car is 5𝑥 𝑘𝑚/ℎ𝑟 and speed of bus is
11
4𝑥 𝑘𝑚/ℎ𝑟.
32. (b); Speed of first train = 60 km/h
Total distance = 480 km According to first condition,
𝐷 𝐷 5𝐷–4𝐷
Time taken by first train to cover that distance – = 1 ⇒ 20𝑥 = 1
4𝑥 5𝑥
480
without stoppage = 60 = 8ℎ ⇒ 𝐷 = 20𝑥 …(i)
9 station × 5 minute = Total stoppage time Also if they are travelling in opposite direction,
= 45 minute Then
Train took total time to reach Lucknow 𝐷–40
= 2 ⇒ D – 40 = 18x …(ii)
35 4𝑥+5𝑥
= 8 hr 45 minute = ℎ𝑜𝑢𝑟
4 From (i) & (ii)
2nd Train reach 30 min. before it start 2 hour late 18𝑥 + 40 = 20𝑥 ⇒ 𝑥 = 20
from Delhi Hence speed of bus = 4 × 20 = 80 km/hr
So, 150
Time taken by 2nd train = 8 h 45 min – 2h 150% of speed of bus = × 80 = 120 km/hr
100
– 30 min. 36. (b); Let the Distance b/w X to Y be D km and speed of
= 6 hour 15 minute car A and B be A km /hr & B km/hr respectively
1 25
= 6 4 = 4 hour 6A –
13
B = 90 – (i)
480×4 384 2
Speed of 2nd Train= = km/h & according to second condition
25 5
60
Required Speed ratio of train = 384 =25:32 7.5
A–
7.5
𝐵 = 2.5
5 60 60

33. (c); Let length of two trains be 3L meter and 2L meter A – B = 20 – (ii)
ATQ— Solving (i) & (ii)
5 3L+2L
(90– 72) × = A = 80 km, B = 60 km
18 60
300 = 5L Distance b/w X & Y is = 6 × 80 = 480 km
960
L = 60 meter Time take by A to cover 960 km is = 12 hours
80
Length of faster train = 120 meter
Distance covered by B in 12 hours = 12 × 60 =
Length of slower train = 180 meter
Let length of goods train be G meter 720 km
(108 + 72) × =
5 180+𝐺 Distance b/w them = (960–720) km = 240 km
18 7.2
50 × 7.2 = 180 + G 37. (d); Let the length of train A be L m and speed is a
G = 360 – 180 m/s.
G = 180 meter
& the length of train B will be L + 17 m and speed
Let faster train cross good train in T sec —
5 120+180 is b m/s.
(108 – 90) × =
18 T ATQ,
5T = 300 2𝐿+17
T = 60 sec 𝑎+𝑏
= 2.6 …(i)
2𝐿+17
34. (d); Let the speed of athletes ‘A’ and ‘B’ is 𝑎 and 𝑏 = 13 ...(ii)
𝑎–𝑏
respectively For train A
From 1st condition 𝐿
= 𝑎 ⇒ 𝐿 = 2𝑎
800 2
= 100 ⇒ 𝑎– 𝑏 = 8 ms–1
𝑎–𝑏 Put L=2a in (i) and (ii)

79 Adda247 Publications For More Study Material


Visit: adda247.com
Cracker Book for Bank (IBPS | SBI | RRB PO | Clerk) Mains Exams
4𝑎+17 Relative speed of train B and D, when both
𝑎+𝑏
= 2.6 …(iii)
5
4𝑎+17 running in opposite direction = (100 + 110) × 18 =
= 13 .....(iv) 175
𝑎–𝑏
m/sec
On solving (iii) and (iv), 3
Lets length of train B and D is 4L meter and 3L
a= 51m/s
meter respectively
and, b=34m/s
ATQ—
Hence sum of speed =51+34= 85 𝑚𝑠 –1 175 4L+3L
3
= 9
38. (d); Let speed of ‘Rajdhani express’ be S m/s 7L = 525
ATQ – L = 75
240 Length of train D = 75 × 3 = 225 meter
S= 6 Relative speed of train C and D when both train
S = 40 m/s running in same direction
5
Speed of ‘Shatabdi express’ be V m/s = (110 – 60) × 18
17(240+360) 125
(40 +V) = 120 = meter/sec
9
4800 + 120V = 10200 Required time =
(120+225)×9
= 24.84 sec
125
120V = 5400
V = 45 m/s 40. (c); Speed ratio of train A and Train E = 6 : 5
Speed of Duranto express’ be u m/s So, ratio of time taken by train A and train E
240+180 (without stoppage) = 5 : 6
(40 − U) =
42 Now → Train E start = 9 : 45
40 − U = 10 Halt → 3 × (x+ 10) ⇒ 3x + 30 min
U = 30 m/s Train A start →9 : 45 + 30 → 10 : 15
Let ‘Shatabdi express’ will cross ‘Duranto express’ Halt time ⇒ 3 × x = 3x min
running in same direction in T sec If halt time add, before starting journey
360+180 Train E ⇒9 : 45 + 30 + 3x
(45 − 30) =
𝑇
⇒10 : 15 + 3x
15T = 540
Train A ⇒10 : 15 + 3x
T = 36 sec So, time difference on reaching on Delhi is have
Solutions (39-40): Speed of train A = 120km/h same proportion
6 – 5 = 1 hour
Distance between P and Q = 120 × 4 = 480
480 ×5 Train A take → 5 hour
Speed of train B = = 100 km/hr Train E take→ 6 hour
24
Train A speed = 120 km/h
39. (b); Relative speed of train A and C, when both
Train E = 100 km/h
running in opposite direction
5
For x –
= (120 + 60) × 18 (3.30 am – 10 .15 pm) – 3x = 5hours
= 50 m/sec 3x = 5. 15 – 5
Lets length of train A and C is 3L meter and 2L X = 5minutes
meter respectively. Speed of train B = 100 km/hr
ATQ— Train cover total distance of 720 between
3L+2L Lucknow to Jaipur
50 = 720
6
= + 4(𝑥 + 10)
5L = 300 100
L = 60 meter = 7.2 hours + 4(5+10)
= 7.2 hour + 60 minutes
Length of train C = 60 × 2 = 120 meter
= 8.2 hours

80 Adda247 Publications For More Study Material


Visit: adda247.com
Cracker Book for Bank (IBPS | SBI | RRB PO | Clerk) Mains Exams

Chapter
Boat and Stream
9
BEST APPROACH TO SOLVE THE QUESTIONS

The questions based on boats and streams are simply based on the concept of relative speed. Generally, two cases arise
when a boat moves. Either the boat moves along the flow of the stream or the boat moves in the direction opposite to
that of stream.
1. If boat moves in the direction of stream,
The net speed is the addition of the speed of stream and boat.
2. If boat moves in the direction opposite to that of stream, the net speed is the difference of their speeds (speed of
boat being more than that of stream).

In questions related to boat and streams, usually it happens that one may need to solve complex calculations but using a
bit of intelligent guess work one may save a good amount of time.

A sample problem shows one way of approach:

Example : ‘A man can row 24 km in upstream and 36 km in downstream in 9 hours. The same man can row 8 km in
upstream and 48 km in downstream in 6 hours. Calculate the speed of the stream.’
Sol. The basic approach to this question involves formation of two equations, which are If we assume speed of
stream as r and speed of boat as x.
24 36
𝑥−𝑟
+ 𝑥+𝑟 =9
8 48
𝑥−𝑟
+ 𝑥+𝑟 =6

It’s cumbersome to solve these set of equations. So we need to look at distance travelled in downstream or
in upstream. In majority of cases it’s better to take HCF of distance travelled in downstream in both cases.
HCF of 36 and 48 is 12. Now we have to break 12 in two parts such that, adding or subtracting them,
completely divides all the four distance. Try (10,2) (9,3) (8,4) and check whether these fulfill the
condition. Or check from option.
In short we can only check this by hit and trial method.
Here, speed of stream=4km/hr and that of boat in still water is 8 km/hr.

81 Adda247 Publications For More Study Material


Visit: adda247.com
Cracker Book for Bank (IBPS | SBI | RRB PO | Clerk) Mains Exams

Practice Exercise Based on New Pattern

1. A boat goes 28 km downstream and while returning (a) 12 hours (b) 16 hours (c) 18 hours
covered only 75% of distance that covered in (d) 14 hours (e) 20 hours
downstream. If boat takes 3 hr more to cover 7. A boat has to travel from point P to point Q in
upstream than downstream then find the speed of upstream and reach Q by 5.00 PM. If boat starts form
5
boat in still water (km/hr) if speed of stream is point P at 2.00 PM with the speed of 20 km/hr and
9
m/sec ? after travelling one hour the speed of current is
increased by 25% of its initial speed. If distance from
(a) 8 km/hr (b) 2 km/hr
point P to Q is 36 km and speed of current is 8 km/hr,
(c) 5 km/hr (d) 4 km/hr then find by what percent boat has to increase its
(e) 3 km/hr speed to reach point Q in time?
2. The ratio of time taken by boat A and boat B to swim a (a) 5% (b) 15% (c) 12%
certain distance downstream in a river is 3 : 4 (d) 10% (e) 17.5%
respectively. The time taken by boat B to cover some Directions (8-9): A boat ‘A’ start from point X, toward
other distance in upstream is 50% more than the time point Y in upstream simultaneously second boat ‘B’ start
taken by it to cover the same distance in downstream. from point Y to X having speed more than first boat ‘A’.
What is the ratio of speed of boat A to that of boat B in Difference between speed of both boats in still water is
still water? equal to the speed of stream and they cross each other
(a) 7 : 5 (b) 7 : 4 (c) 7 : 3 first time after 2 hours. Given that both boats start their
respective return journey by reaching their destination
(d) 7 : 9 (e) 7 : 2
points.
3. There are three points P, Q and R in a straight line in a 8. If distance between X and Y is 56 km and speed of
river such that point Q is equidistant from point P and stream is 4 km/hr then find the distance from ‘Y’
Point R. Boat travelled from point P to R downstream when boats cross each other 2nd time.
in 8 hours and from point Q to P upstream in 12 (a) 2.8 km (b) 2.4 km (c) 5.6 km
hours. Find the ratio between speed of boat in still (d) 3.2 km (e) 4.8 km
water to speed of current?
9. Find the time taken by boats to cross each other on
(a) 3:1 (b) 2:1 (c) 1:2 2nd time from the first time if speed of second boat ‘B’
(d) 5:2 (e) 2:3 is 1.5 times of the speed of first boat ‘A’ in still water?
4. A boat cover 60 km upstream and 60 km downstream (a) 5 hours (b) 3 hours (c) 2 hours
in 22.5 hr with its usual speed. If boat double its speed (d) 6 hours (e) 4 hours
then new upstream speed is 150% more than the 10. Point A to Point B is a downstream journey of 300 km
usual upstream speed. Find the time taken by boat to on a stream which flows at a speed of 5 km/hr. Two
cover 80 km in downstream with usual speed. boats P and Q starts from point A and Point B
(a) 12 hr (b) 20 hr (c) 5 hr respectively with speed of 25 km/hr and 15 km/hr in
(d) 16 hr (e) 10 hr still water. After reaching the opposite point they
return to their starting points, find after how much
5. A boat covers total distance of 360km downstream in time will they meet second time?
three equal parts with the speed of boat be 4x km/hr, (a) 7.5hour (b) 15hour (c) 20hour
5𝑥 km/hr and 7𝑥 km/hr respectively. If speed of (d) 10hour (e) None of these
stream is x km/hr and boat takes total 29.5 hours, 11. Speed of current is 10 km/hr and speed of a motor
then find boat will cover total given distance
boat is 80% more than speed of current. Motor boat
upstream in three equal parts with three givens
travels 280 km downstream with its usual speed,
speeds?
(a) 36 hours (b) 40 hours (c) 45 hours after that it’s increased speed by ‘s’ kmph and
(d) 48 hours (e) 54 hours travelled for another 280 km then it returns and
6. The speed of two boats A and B in the still water is in covers 560 km in upstream. If boat complete whole
the ratio of 3 : 4 and the speed of current is 4km/hr. A journey downstream to upstream in 45 hr, then find
start from point P ,45 minutes earlier than B in the value of ‘s’?
downstream direction. If B catch boat A in three hours (a) 10 km/hr (b) 8 km/hr (c) 6 km/hr
then, find boat B takes how much time to cover 120
km distance each in downstream and in upstream. (d) 12 km/hr (e) 4 km/hr

82 Adda247 Publications For More Study Material


Visit: adda247.com
Cracker Book for Bank (IBPS | SBI | RRB PO | Clerk) Mains Exams

12. Speed of current is 5 km/hr and speed of boat in still find time take by C on third day to reach at point Q,
water is 80% more than speed of current.A boat (consider speed of stream same on all three days)?
1 1 1
travels 50% of total distance of 280 km, in (a) 4 7 ℎ𝑜𝑢𝑟𝑠 (b) 5 7 ℎ𝑜𝑢𝑟𝑠 (c) 3 7 ℎ𝑜𝑢𝑟𝑠
downstream. After that the speed increase by x kmph 1 1
(d) 2 7 ℎ𝑜𝑢𝑟𝑠 (e) 9 7 ℎ𝑜𝑢𝑟𝑠
because of a man who added an engine. If boat
complete whole journey downstream and upstream in 17. Usual speed of a boat is S km/hr. Boat rowing in a lake
45 hr, then find the value of x? (neglect the current speed) and covered distance of
(a) 6 km/hr (b) 8 km/hr (c) 9 km/hr 360 km in three equal parts with three different
(d) 4 km/hr (e) 5 km/hr speeds in total 7.5 hours. If boat covered first part
13. A lake in which speed of water current is zero has a with usual speed and second part with 20% increased
stretch of 144 km. Two boats, whose sum of speed in speed and third part with further increase of 25% of
still water is 28 km/hr, starts from two ends of lake. A latest speed. Find the speed of boat at which it
fish starts swimming simultaneously with one boat covered third part of journey?
toward another boat and reach the second boat in 4 (a) 56km/hr (b) 36km/hr (c) 40km/hr
hours and then returns toward first boat and touch it (d) 60km/hr (e) 48 km/hr
in 48 minutes. Find the speed of fish. 18. Ratio between speed of two boats A & B in still water
(a) 12 km/hr (b) 16 km/hr (c) 24 km/hr is 4 : 3 and speed of current is 4 km/hr .Boat B starts
(d) 40 km/hr (e) 8 km/hr from point P in downstream, two hours earlier than
14. A boat goes certain distance downstream and then boat A and it is found that boat A is16 km ahead of
3 3 boat B after 12 hours of its start. If boat A start from
return th of the distance upstream.It takes of the
4 2 point Q in upstream and boat B start from point P in
time in upstream than in downstream. If boat
1
downstream at same time and distance between P
increases its speed by 33 % and cover a distance of and Q is 108 km, then find in what time boat A & B
3
60 km in downstream and then return upstream in 16 meet each other?
6 6 6
hours, find increased speed of boat? (a) 47 ℎ𝑜𝑢𝑟𝑠 (b) 5 7 ℎ𝑜𝑢𝑟𝑠 (c) 9 7 ℎ𝑜𝑢𝑟𝑠
(a) 6 km/hr (b) 14 km/hr (c) 16 km/hr 6 6
(d) 3 ℎ𝑜𝑢𝑟𝑠 (e) 11 ℎ𝑜𝑢𝑟𝑠
(d) 12 km/hr (e) 8 km/hr 7 7

15. Vikas can swim in still water with twice the speed as 19. When two ships A & B are travelling in opposite
that of speed of water. Time difference to cover a direction crosses each other in 2 second. Speed of
certain distance in upstream and in downstream is 4 ship B is 50% more than the speed of ship A in still
hours at his usual speed. But if he had doubled his water. On a particular day, ship A was ahead of ship B
usual speed, then time difference would reduce by by 9 meters and both of them were travelling in
80% of its usual time difference to cover same downstream. If Ship B overtook Ship A in 11 seconds
distance. Calculate total time taken by Vikas to cover 6 and length of ship B is 54 meters then find the speed
km upstream and 2 km downstream, at his usual of ship A.
speed. (a) 56m/s (b) 36m/s (c) 40m/s
3 5 (d) 60m/s (e) 18m/s
(a) 1 hour (b) hour (c) hour
4 4
4
(d) 3 hour (e) Can’t be determined 20. A boat goes a certain distance downstream and then
returns and covers 40% of distance covered in
16. Speeds of three motor boats A, B and C are equal and downstream. Ratio of time taken in covering
all cover 8 km of distance upstream in 48 minutes. downstream and upstream distances is 3 : 2. If speed
Ratio between speed of motor boats and speed of of boat in still water is reduced by 50% then it covers
stream is 6 : 1. On first day A starts in downstream 60 km downstream in 10 hours. Find the speed of
from point P to Q, which shifts 9 km away from point boat in still water.
P each day. On second day B starts from point P in (a) 9 km/hr (b) 8 km/hr (c) 6 km/hr
downstream and reach at point Q in 4.5 hours, then (d) 10 km/hr (e) 12 km/hr

83 Adda247 Publications For More Study Material


Visit: adda247.com
Cracker Book for Bank (IBPS | SBI | RRB PO | Clerk) Mains Exams

Solutions
5 18 x = 3y
1. (c); Speed of current = 9 × 5
Now,
= 2 km⁄hr 60 60
Let’s still water speed = x km/hr + = 22.5
x –y x+y
ATQ, 60 60
+ 4y = 22.5
3 2y
28× 28
(x–2)
4
– (x+2) = 3 y = 2 km/hr
21x + 42 – 28x + 56 = 3x² – 12 x = 6 km/hr
80
–7x + 98 = 3x² – 12 Required time = 6+2
3x² + 7x – 110 = 0 = 10 hr
x = 5 km/hr
5. (c); ATQ—
2. (a); Let total distance be D km. 120 120 120
+ (5𝑥+𝑥) + (7𝑥+𝑥) = 29.5
And, speed of boat A in still water be (4𝑥+𝑥)
120 120 120
x km/hr 5𝑥 6𝑥
+ 8𝑥
+ = 29.5
Speed of boat B in still water be y km/hr 5760+4800+3600
=𝑥
Speed of stream be r km/hr 240×29.5

ATQ - 𝑥=2
120 120 120
𝐷

𝐷
=4
3 Required time = 3𝑥 + 4𝑥 + 6𝑥
𝑥+𝑟 𝑦+𝑟 1 1 1 3
𝑦+𝑟 3 = 120 (6 + 8 + 12) = 120 × 8 = 45 ℎ𝑜𝑢𝑟𝑠
Or, 𝑥+𝑟 = 4 ……..(i)
Also, for a distance D1, 6. (b);Let speed of boat A and B in still water be 3𝑥
𝐷1 3 𝐷1
= 2 (𝑦+𝑟 ) km/hr and 4𝑥 km/hr respectively
𝑦−𝑟
Downstream speed for boat A
or, 2𝑦 + 2𝑟 = 3𝑦 − 3𝑟
= (3𝑥 + 4) km/hr
or, 𝑦 = 5𝑟
Downstream speed for boat B
Putting this in (i),
6𝑟 3 = (4𝑥 + 4) km/hr
= Relative speed of boat A and B
𝑥+𝑟 4
or, 3𝑥 + 3𝑟 = 24𝑟 = (4𝑥 + 4) – (3𝑥 + 4) = 𝑥 km/hr
or, 𝑥 = 7𝑟 ATQ—
Ratio of speed of boat A and B in still water 45 (3𝑥+4)
× =3
60 𝑥
= 7r : 5r
9𝑥 + 12 = 12𝑥
=7:5
𝑥 = 4 km/hr
3. (b);Let still water speed x km/hr and Speed of boat B = 4 × 4 = 16 km/hr
Speed of current y km/hr — 120 120
Required times = (16+4) + (16–4)
Downstream speed = (x + y) km/hr
Upstream speed = (x – y) km/hr = 6 + 10 = 16 hours
Let distance between P to R is 2ℓ and Q to P is ℓ 7. (d);Boat travels total distance in the hours
km Upstream speed of boat = 20 – 8 = 12 km/hr
ATQ— In one hour, boat will cover a distance of
2ℓ
=8 … (i) = 12 × 1 = 12 𝑘𝑚
(x+y)
Remaining distance = 36 – 12 = 24 km
and
ℓ To reach Q at 5.00 PM boat has to cover
(x–y)
= 12 … (ii) remaining distance at speed
24
From (i) and (ii) = = 12 𝑘𝑚/ℎ𝑟
x 2 2
y
=1 New speed of stream = 8 × = 10 𝑘𝑚/ℎ𝑟
125
100
4. (e); Let usual speed of boat in still water = x New upstream speed of boat =20 – 10
River speed = y = 10 km/hr
ATQ, For 12 km/hr speed boat has to increase its speed
(𝑥−𝑦)250 by 2 km/hr
= (2𝑥 − 𝑦) 2
100 Required percentage = 20 × 100 = 10%
5𝑥 − 5𝑦 = 4𝑥 − 2𝑦
84 Adda247 Publications For More Study Material
Visit: adda247.com
Cracker Book for Bank (IBPS | SBI | RRB PO | Clerk) Mains Exams

Solutions (8-9): Let speed of first boat ‘A’ 5𝑎


⇒ 4 km
= a km/hr
Relative speed of boat ‘A’ and ‘B’ when they cross
Speed of stream = b km/hr
each other
So, 3𝑎 𝑎 𝑎 𝑎
Speed of 2nd boat ‘B’ = 𝑎 + 𝑏 km/hr = ( 2 – 2 – 2)= 2
5𝑎
Time taken to cross
4
5𝑎×2
Distance XY ⇒ 2 × (a – b + a + b + b) = 4a + 2b ⇒ 4×𝑎 = 2.5 hour
8. (d);Given XY = 56 = 4𝑎 + 2𝑏 So total time for 2nd crossing
2𝑎 + 𝑏 = 28 ⇒ 2.5 + 0.5 = 3 hours
Now speed of stream = 𝑏 = 4 km/hr 10. (c); Time when P and Q meet first time
2𝑎 + 4 = 28 300 300
𝑎 = 12 = 25+15 = 40 = 7.5 hr
Speed of first boat ‘A’ in still water = 12 Distance travelled by P = 7.5 × (25 + 5)
Speed of 2nd boat ‘B’ in still water = 16 = 225 km
First time crossing Distance travelled by Q = 7.5 × (15 – 5)
= 75 km
75
Time taken by 2nd boat ‘B’ to reach at X after first P reach at point B in i.e. 2.5 hour
30
crossing In 2.5hr Q travelled 2.5 × 10 = 25 km
16 4
= 20 = 5 hour Now Boat P returns and relative distance b/w P
4 and Q is 100 km and relative speed is 10 km i.e.
Distance travelled by first boat ‘A’ in 5 hour difference of speed of P (25 -5) = 20 km/hr
4 32 (upstream) and speed of Q
= 5 × 8 = 5 km
(15 – 5) = 10 km/hr (upstream)
Now,
Time taken by P and Q to meet
Distance between first boat ‘A’ and 2nd boat ‘B’ 100
32 = = 10 hr
16 + 5 ⇒ 22.4 km 10
Total time when P and Q meet second time = 7.5
On return journey of second boat ‘B’, speed of
+2.5 + 10 = 20 hr
boat = 12 km/hr
Time taken by first boat to reach Y 11. (d);speed of boat in still water
56–22.4 80
= 8 = 4.2 hours = (10 + 10 × 100) km⁄hr
Distance remaining between both boat after = 18 km⁄hr
reaching of first boat ‘A’ at Y
ATQ—
⇒ 56 – 4.2 × (12 – 8) = 5.6 km 280 280 560
Time taken by both boat to cover this distance (18+10)
+
(18+10)+s
+ (18–10)+s = 45
5.6 280 560
= 12+16 = 0.2 hours + = 35
28+s 8+s
Distance from Y = 0.2 × 16 = 3.2 km 8 16
28+s
+ 8+s = 1
9. (b);If speed of first boat ‘A’ = 𝑎 km/hr
3 64 + 8s + 448 + 16s = 224 + 28s + 8s + s 2
Speed of second boat ‘B’ ⇒ 𝑎 km/hr
2 s² + 12x – 288 = 0
3 𝑎
So speed of stream ⇒ 𝑎– 𝑎 = km/hr s = 12 km/hr
2 2
Distance travelled by first boat till first crossing 80
𝑎 12. (a); Still water speed = (5 + 5 × 100) km⁄hr
⇒ (𝑎– 2) × 2
⇒ 𝑎 km = 9 km⁄hr
𝑎
Speed of first boat in upstream = 2 km/hr ATQ—
140 140 280
Speed of 2nd boat in downstream (9+5)
+ (9+5)+x + (9–5)+x = 45
= 2𝑎 km/hr 140 280
+ 4+x = 35
14+x
4 8
𝑎 1 + 4+x = 1
2nd boat reached at X ⇒ = ℎ 14+x
2𝑎 2
1 1 𝑎 𝑎 12x + 128 = 56 + 14x + 4x + x²
In hour first boat cover ⇒ × = km
2 2 2 4
𝑎
x² + 6x – 72 = 0
Distance between both boats ⇒ 𝑎 + 4 x = 6 km/hr

85 Adda247 Publications For More Study Material


Visit: adda247.com
Cracker Book for Bank (IBPS | SBI | RRB PO | Clerk) Mains Exams
80 𝐷 𝐷
12. (a); Still water speed = (5 + 5 × ) km⁄hr 2𝑥–𝑥
– 2𝑥+𝑥 = 4
100
𝐷 𝐷
= 9 km⁄hr ⇒ – =4
𝑥 3𝑥
ATQ— ⇒ D = 6𝑥 …(i)
140 140 280
(9+5)
+ (9+5)+x + (9–5)+x = 45 From 2nd condition, when speed of Vikas is 4𝑥
140
+
280
= 35 km/hr
14+x 4+x 𝐷 𝐷 20
4 8 – = 100 × 4
14+x
+ 4+x = 1 4𝑥–𝑥 4𝑥+𝑥
𝐷 𝐷 4
12x + 128 = 56 + 14x + 4x + x² ⇒ 3𝑥 – 5𝑥 = 5
x² + 6x – 72 = 0 ⇒ D = 6𝑥 …(ii)
x = 6 km/hr Hence both equations are same so answer can’t be
determined.
13. (c);
16. (b); Speed of motor boats in upstream
60
= 8 × 48 = 10 km/hr
ATQ,
Let speed of boat 2 and speed of fish is x and y Let speed of motor boats be 6x km/hr and speed
respectively. of stream be x km/hr
Boat 2 and fish meet in 4 hour 6x – x = 10
So, x = 2 km/hr
(x + y) × 4 = 144 …(i) Downstream speed of all boats
Now = (6 × 2 + 2)
ATQ, = 14 km/hr
Speed of boat 1 = 28 – x Let distance between point P to Q on first day = y
In 4 hour boat 1 cover = 4 × (28 – x) km
48
× (𝑦 + (28 – 𝑥)) = 144 – [4𝑥 + 4(28 − 𝑥)] Second day distance = (y + 9)
60 𝑦+9
4 14 =
(𝑦 − 𝑥) + 22.4 = 144 − 112 4.5
5
y= 63 – 9
𝑦 − 𝑥 = 12 … … . (𝑖𝑖)
y = 63 – 9
Sovling (i) & (ii)
y= 54 km
x = 12 km/hr
Distance travelled on third day = 54 + 9 × 2
y = 24 km/hr
= 72 km
14. (e); Let distance travelled by boat in downstream be D Total time taken by boat C on third day to reach
and speed of boat in still water be x km/hr and point Q
72 1
speed of current be y km/hr = = 5 hours
14 7
ATQ,
3
×𝐷 3 𝐷 17. (d);Three equal parts of journey
4
= 2 × (𝑥+𝑦) 360
(𝑥−𝑦) = 3 = 120 km
1 1
2(𝑥−𝑦)
= 𝑥+𝑦 ATQ—
120 120 120
x = 3y + S + 6S 6S 1 = 7.5
S S+ + ×
1 5 5 5 4
New sped of boat = 3y + 3y × 3 120 600 120×10
+ + = 7.5
= 4 y km/hr S 6S 12S+3S
3600+3000+2400 15
60
+
60
= 16 30S
= 2
(4𝑦+𝑦) (4𝑦−𝑦) 9000 15
12 20 =
𝑦
+ 𝑦
= 16 30S 2
9000×2
32 S=
y= = 2 𝑘𝑚/ℎ𝑟 30×15
16 S = 40 km/hr
Increased speed of boat = 4×2 = 8 km/hr
Speed of which boat which it covered third part of
15. (e); Let the speed of water is x km/hr, then speed of Journey
vikas in still water is 2x km/hr 20 20 1
= (40 + 40 × ) + (40 + 40 × )×
100 100 4
From 1st condition,
= 60 km/hr

86 Adda247 Publications For More Study Material


Visit: adda247.com
Cracker Book for Bank (IBPS | SBI | RRB PO | Clerk) Mains Exams

18. (d);Let speed of boat A & B in still water be 4x km/hr = 3x + Y – 2x –Y= x m/s
and 3x km/hr respectively ATQ,
Downstream speed of boat A 54+9+ℓ
= 11
𝑥
= (4x + 4) km/hr
63 + ℓ = 11x …(ii)
Downstream speed of boat B
From (i)
= (3x + 4) km/hr
ℓ = 10x – 54
AT Q –
put this value in eqn. (ii)
12 (4x + 4) – 16 = 2 (3x + 4) + 12 (3x + 4)
= 63 + 10x – 54 = 11x
48x + 48 − 16 = 6x + 8 + 36x + 48
x = 9 m/s
6x = 24
Hence speed of ship A in still water
x = 4 km/hr
= 2 × 9 = 18 m/s
Speed of boat A in still water = 16 km/hr
Speed of boat B in still water= 12 km/hr 20. (b);Let speed of boat in still water and speed of
Relative speed = (16 – 4) + (12 + 4) current be respectively x km/hr and y km/hr
= 28 km/hr And Let time taken to cover given distance in
108 downstream and upstream be = 3m hours and 2m
Required time = 28
6 hours
= 3 ℎ𝑜𝑢𝑟𝑠 40% of distance in upstream is covered in 2 m
7

19. (e); Let speed of ship A in still water is 2x m/s , and its hour
length is ℓ m 100% of distance upstream is covered in 5m
And speed of ship B is 3x m/s and its length is 54 hours
and speed of water is Y m/s. According to question
x−y 3
ATQ, when both of them are travelling in opposite x+y
=5
direction. 5x - 5y = 3x + 3y
Downstream speed of ship A = (2x + Y) m/s x
⇒ =
4
[assume ship A is travelling in downstream and y 1

ship B in upstream] And


60
Upstream speed of ship B = (3x – Y) m/s x = 10
( +y)
Their relative speed = 2x + Y + 3x – Y x
2

= 5x m/s 2
+y= 6
4𝑦
ATQ, +𝑦=6
54+ℓ 2
5x
= 2 …(i) 6y = 12
Similarly when both are travelling in downstream y = 2 km/hr
their relative speed is so, x = 8 km/hr

87 Adda247 Publications For More Study Material


Visit: adda247.com
Cracker Book for Bank (IBPS | SBI | RRB PO | Clerk) Mains Exams

Chapter
Mensuration
10
BEST APPROACH TO SOLVE THE QUESTIONS

Questions related to mensuration usually don’t involve any short-cut methods for solving a problem. The reason
being how can one use short tricks when all one has to do is to deal with formulae. To solve questions faster and in a
lucid manner, one must avoid doing unnecessary and multiple calculations. Rather one must calculate only where it is
needed. In case, ratio or percentage is asked in the question, then it becomes more easy to do away with hefty
calculations and arrive at the answer by using basic elimination methods.
‘The length, breadth and height of a room are in the ratio 3:2:1. If the breadth and height are halved while
the length is doubled, then the percentage change in total area of the four walls of the room.’
Area of four walls of the room= 2(lh+bh), where l,b and h are length, breadth and height of the room respectively.
As we have been asked the percentage change, let’s take length, breadth and height of the room to be 6,4 and 2
respectively.
Then changed values of length, breadth and height would be 12, 2 and 1.
Initial area of four walls= 2 (6 × 2 + 4 × 2) = 40
Final area of four walls= 2 (12× 1 + 2 × 1)= 28
28 is 70% of 40, which means there has been a reduction of 30%.
‘A solid cylinder has height 10 meters and radius R meters. Some part of it is melted and casted into two
cones. One of the cones has radius 3cm and height 4cm. The other cone has height 8 cm and radius 6cm. If the
ratio of initial curved surface area of cylinder to the sum of curved surface areas of two cones is 4:1, then find
the radius of the cylinder.’
Slant height of cone having radius 3cm is 5cm (PLEASE SOLVE IT PROPERLY)
Slant height of cone having radius 6 cm is 10 cm
Curved surface area of a cone is πrl and that of cylinder is 2πrh
2𝜋×𝑅×10 4
ATQ, =
𝜋(3×5+6×10) 1
➢ R=15cm

Practice Exercise Based on new Pattern


360 410 470
1. The ratio of the radius and height of a cone is (a) cm (b) cm (c) cm
2 7 7 7
5 : 12. Its volume is 314 7 cm. Then its slant height is ? (d)
440
cm (e)
460
cm
7 7

(a) 9 cm (b) 10 cm (c) 11 cm 4. Total surface area of a cylinder mounted with a


(d) 12 cm (e) 13 cm hemispherical bowl on one end is 2552 cm². If height
of cylinder is 8 cm then find the volume of the solid
2. If the perimeter of a rectangle and a square are equal
body?
and the ratio of 2 adjacent sides of a rectangle is 2:1. 1 1 1
Then find the ratio of area of rectangle and area of (a) 10443 3 (b) 10677 3 (c) 10547 3
1 1
square. (d) 10977 3 (e) 10787 3
(a) 1:1 (b) 1:2 (c) 2:3
(d) 8:9 (e) 9:4 5. Sum of Area of circle and a rectangle is equal to 622
sq cm. The diameter of the circle is 14 cm, then what
3. The parameter of a square is equal to the parameter is the sum of circumference of the circle and the
of a rectangle having dimensions 24 cm × 16 cm. Find perimeter of the rectangle if the length of rectangle is
26 cm ?
the circumference of a circle whose diameter is equal
(a) 142cm (b)132cm (c)152cm
to the side of square ? (d) 140cm (e)134cm
88 Adda247 Publications For More Study Material
Visit: adda247.com
Cracker Book for Bank (IBPS | SBI | RRB PO | Clerk) Mains Exams

6. A copper sphere of diameter 42 cm is drawn into a 15. A solid cube is melted to form 3 spheres, with radius,
wire of diameter 70 mm. Then, the length of the wire 1 cm, 2 cm and 3 cm and additional irregular shape
is? was formed. Calculate the volume of irregular shape
(a) 243 cm (b) 343 cm (c) 443 cm (upto two decimal place) if diagonal of cube is
(d) 972 cm (e) None of these 6√3 cm.
(a) 65.14 𝑐𝑚3 (b) 70.35 𝑐𝑚3 (c) 54.75 𝑐𝑚3
7. A cylindrical roller rolls over a square field. It takes (d) 50.00 𝑐𝑚3 (e) 78.90 𝑐𝑚3
500 rounds to cover the complete field. If the radius of
the cylindrical roller is 0.63 meter and length is 7.7 m 16. 20% increase in both radius and height of the cylinder
then find out the side of square (approximately). increases the total surface area of cylinder by 677.6
(a) 135 m (b) 125 m (c) 100 m cm², If the ratio of radius to height is 1 : 4, then find
(d) 150 m (e) 110 m the radius of cylinder
(a) 21 cm (b) 10.5 cm (c) 3.5 cm
8. A cuboid is cut along its base such that, now it become (d) 14 cm (e) 7 cm
a cube. Cube again cut along its base, now ratio of
17. A Cone cylinder and hemisphere have equal radius
height to length becomes 1 : 𝑎. Find the value of ‘𝑎’.
and height. Find the ratio of total surface area of
(a) 𝑎 > 1 (b) 𝑎 = 1 (c) 𝑎 ≥ 1
cylinder, cone and hemisphere.
(d) 𝑎 < 1 (e) 𝑎 ≤ 1
(a) 4 ∶ √2: √6 (b) 4 ∶ √2 ∶ 3
9. Volume of a cone and cylinder, having same base, are (c) √2 + 1 ∶ 4 ∶ 3 (d) 3 ∶ 4 ∶ √2 + 1
in ratio of 4 : 3. If height and radius of cone is 4 : 3 (e) 4 ∶ √2 + 1 ∶ 3
then find the ratio of total surface area of cone and
that of cylinder. 18. Sum of circumference of a circle and perimeter of a
(a) 1 : 1 (b) 3 : 1 (c) 5 : 3 rectangle is 220 cm while area of circle is 1386 sq. cm.
1
(d) 2 : 1 (e) 4 : 1 If length of rectangle is 33 3 % more than radius of the
given circle then find the area of rectangle?
10. If the volume and curved surface area of a cylinder
(a) 408 𝑐𝑚2 (b) 418 𝑐𝑚2 (c) 428 𝑐𝑚2
616 m3 and 352m2 respectively, what is the total 2 2
(d) 448 𝑐𝑚 (e) 438 𝑐𝑚
surface area of the cylinder (in m2)?
(a) 429 (b) 419 (c) 435 19. Four identical coins are placed in a square. For each
(d) 421 (e) 417 coin, area of each coin is equal to circumference of
each coin. Then, find the area of the square that is not
11. Area of a rectangle is 252 cm². If length of the covered by the coins
rectangle is equal to side of a square, whose
perimeter 72 cm then find the volume of cone, whose
radius is equals to breadth of rectangle and height
equals to length of rectangle?
(a) 4752 cm2 (b) 3586 cm2 (c) 4026 cm2
(d) 4246 cm2 (e) 3696 cm2
(a) 16(π − 1) (b) 16(8 − π) (c) 16(4 − π)
12. Sum of length, breadth and height of cuboid is 12 cm π
(d) 16 (4 − 2 ) (e) None of these
and length of its diagonal is 5√2. Then find the total
surface area of cuboid. 20. The surface area of a sphere is 423.5 cm 2 less than
(a) 94 cm² (b) 84 cm² (c) 72 cm² total surface area of a hemisphere. If ratio between
(d) 64 cm² (e) 90 cm² radius of hemisphere and sphere is 3 : 2, then find the
radius of hemisphere?
13. Circumference of a circular garden is 66 cm and area (a) 5.5 cm (b) 5 cm (c) 4 cm
of circular garden is 25% more than perimeter of a (d) 7 cm (e) 10.5 cm
square hall. Find the area of square hall ?
(a) 4802.49 sq cm (b) 4704.48 sq. cm 21. A cylindrical jar, whose base has a radius of 15 cm is
(c) 4820.49 sq.cm (d) 4822.49 sq.cm filled with water up to a height of 20 cm. A solid iron
(e) none of these spherical ball of radius 10 cm is dropped in the jar to
submerge completely in water. Find the increase in
14. Internal radius of pipe is 3 cm and the external radius the level of water (in cm) is
of pipe is 5 cm and the length of pipe is 15 cm then (a) 5
17
(b) 5
5
(c) 5
8

find total surface area of pipe ? 27 7 9

(a) 278 π cm² (b) 275 π cm² (c) 268π cm² 25 7


(d) 5 (e) 5
27
(d) 272 π cm² (e) none of these 27
89 Adda247 Publications For More Study Material
Visit: adda247.com
Cracker Book for Bank (IBPS | SBI | RRB PO | Clerk) Mains Exams

22. The dimensions of a field are 20 m and 9 m. A pit 10 m (a) 98 m2 (b) 146 m2 (c) 84 m2
long, 4.5 m wide and 3 m deep is dug in one corner of (d) 168 m2 (e) 68 m2
the field and the earth (mud) removed has been 29. The area of a rectangle gets reduced by 9m 2 if its
evenly spread over the remaining area of the field. length is reduced by 5 m and breadth is increased by
What will be the rise the height of field as a result of 3m. If we increase the length by 3 m and breadth by 2
this operation ? m, the area is increased by 67 m2. The length of the
(a) 1.5 m (b) 2 m (c) 3 m rectangle is :
(d) 4m (e) 1 m (a) 9 m (b) 15.6 m (c) 17 m
(d) 18.5 m (e) 19 m
23. Difference between perimeter of two rectangles is 8
cm. If breadth of both rectangle is 16 cm and length 30. A toymaker makes a toy in which a cone is mounted
are in the ratio of 7 : 6 then find the total surface area on the base of a hemisphere. If the total surface area
of cylinder whose radius is half of the length of larger of the toy is 858 cm² then find the volume of the toy,
rectangle and height equal to length of smaller given that the diameter of the toy is 14 cm?
2 2
rectangle? (a) 1950 3 cm3 (b) 1250 3 cm3 (c) 1400 cm3
(a) 3344 cm2 (b) 3668 cm2 (c) 3444 cm2 (d) 1500 cm3 (e) 1200 cm3
(d) 3468 cm2 (e) 3462 cm2
31. If sum of circumference of a circle and perimeter of
Directions (24-25); There are three toys which are in rectangle is 204 cm and perimeter of rectangle is 50%
cylindrical, conical and spherical shape. Height of all the of magnitude of area of square and side of square is
toys is same. Ratio between the height of cylinder to 12 cm .If length of rectangle is 8 cm more than that of
radius of cylinder is 4 : 3. breadth, then find sum of area of circle and rectangle ?
(a) 1694sq cm (b) 1450sq cm (c) 1550sq cm
24. Find the ratio between total surface area of cylinder (d) 1750sq cm (e) 1460sq cm
to that of sphere.
(a) 17 : 15 (b) 21 : 16 (c) 28 : 23 32. A sphere is melted into ‘n’ number of small spheres.
Total surface area of these small spheres is what
(d) 21 : 8 (e) 15 : 4
percent more or less than total surface area of large
25. Volume of cone is what percent of the volume of sphere, if the ratio of radius of small sphere to radius
sphere if radius of cone is equal to that of cylinder of large sphere is 1 : 3.
(a) 112.5% (b) 100% (c) 125% (a) 50% (b) 100% (c) 150%
(d) 75%
8
(e) 88 % (d) 200% (e) 75%
9
33. Ratio of height of cylinder to that of diameter is 2: 3.
26. A well whose shape is of equilateral triangle having This cylinder is formed by melting a sphere having
side 8 cm has to be dug 44√3 cm deep. The earth same radius as the radius of circle. Ratio of magnitude
taken out of the well is used to fill a cylindrical tank of area of circle to magnitude of its circumference is
having diameter 8 cm. Find the height up to which 21: 2. Find height of cylinder.
earth is filled in the tank. (a) 14 units (b) 21 units (c) 42 units
(a) 10.5 cm (b) 21 cm (c) 42 cm (d) 28 units (e) 35 units
(d) 84 cm (e) 32 cm 34. If the curved surface area of first cone is thrice that of
27. A circle is inscribed in a square. If the difference second cone and slant height of the second cone is
between area of the square and circle is 262.5 cm², thrice that of the first, find the ratio of the area of
then find the area of the rectangle whose perimeter is their base.
same as that of circle while length of rectangle is 20% (a) 81 : 1 (b)9 : 1 (c)3 : 1
more than the breadth of rectangle (in cm²) (d)27 : 1 (e) None of these
(a) 1500 (b) 1400 (c) 700 35. There are two cylindrical vessels with same capacity
(d) 750 (e) 3000 and same dimensions. If radius of one vessel
increased by 7 cm and height of second vessel
28. Inside a square plot a circular garden is developed
increases by 15 cm, then capacity of both vessel
which exactly fits in the square plot and the diameter increased by equality ‘Q’ cm3. If actual height of both
of the garden is equal to the side of the square plot vessels is 12 cm, then find value of Q?
which is 28 metre. What is the area of the space left (a) 9840 cm3 (b) 9240 cm3 (c) 9460 cm3
out in the square plot after developing the garden? (d) 9160 cm3 (e) 9780 cm3

90 Adda247 Publications For More Study Material


Visit: adda247.com
Cracker Book for Bank (IBPS | SBI | RRB PO | Clerk) Mains Exams

36. There are four light poles in four corners of a park, 41. In a house, there is a semicircular corridor, a circular
four at midpoint of each side and one pole in the hall and a circular garden. Sum of perimeter of
middle of that square park. Area of park is 19600 cm 2. corridor and hall is 102 m.If radius of hall is 50%
If side of park is 50 times of radius of each light pole, more than radius of corridor then find area of circular
garden, given radius of garden is 300% more than
then find the total base area covered by these poles if
sum of radius of corridor and radius of hall together.
pole’s base is circular in shape? (a) 6300𝜋 sq.m (b) 5400𝜋 sq.m (c) 5700𝜋 sq.m
(a) 197.12 cm2 (b) 172.48 cm2 (c) 246.4 cm2 (d) 4200𝜋 sq.m (e) 4900𝜋 sq.m
(d) 220.76 cm2 (e) 221.76 cm2
42. Height of a cylindrical vessel is equal to side of a
37. Area of a square is 306.25 cm2 and breadth of square, whose area is 256 cm². Radius of cylindrical
rectangle is 3.5 cm less than side of square. Area of vessel is equal to average of radius of a sphere and
rectangle is 252 cm2 and height and radius of a hemisphere. If ratio between the radius of
hemisphere and that of the sphere is 2 : 1 and
cylindrical vessel equal to length & breadth of
difference between their total surface area is 4928
rectangle respectively. If vessel filled, contains cm², then find volume of cylindrical vessel ?
mixture of milk and water in which milk is 75%, then (a) 21,288 cm³ (b) 20,176 cm³ (c) 22,176 cm³
find the volume of water contained by cylindrical (d) 26,176 cm³ (e) 24,288 cm³
vessel?
43. Radius of a cylinder is half of the side of square,
(a) 2572 cm3 (b) 2472 cm3 (c) 2772 cm3 whose perimeter is 6 cm more than perimeter of
(d) 2872 cm2 (e) 2878 cm2 rectangle. If ratio between length to breadth of
Directions (38-39); A hemispherical bowl is filled with rectangle is 7 : 2 and between length of rectangle to
side of square is 3 : 2. Find volume of cylinder, given
hot water to the brim. The contents of the bowl are
height of cylinder is equal to breadth of rectangle?
transferred into a cylindrical vessel whose radius is 50% (a) 24,938 cm3 (b) 24,948 cm3 (c) 24,848 cm3
more than its height. (d) 24,996 cm3 (e) 24,918 cm3
38. If diameter of the bowl is the same as that of the 44. A solid sphere of some radius is melted and re-casted
cylindrical vessel, then the volume of the hot water in into a number of small cylinders and cones. The
the cylindrical vessel is height of the cone is 33⅓% less than the height of the
(a) 60% of the cylindrical vessel cylinder which is 6 cm and the ratio of radius of cone
(b) 80% of the cylindrical vessel to the radius of cylinder is √3 : 1 and the ratio of
number of cylinder to the number of cone is 2 : 3. Find
(c) 100% of the cylindrical vessel
the total surface area of the sphere if the number of
(d) 50% of the cylindrical vessel cylinders are 12 and the radius of cylinder is 4 cm.
(e)none of these 1 2 4
(a) 1810 7 cm² (b) 1810 7 cm² (c) 1810 7 cm²
39. This hemispherical bowl is joined at one end of the (d) 905
11
cm² (e) 905
9
cm²
14 14
cylindrical vessel and the solid obtained by combining
these figure is filled completely with water. If a drain 45. Radius of a cylindrical vessel is 1.5 cm less than
pipe is connected to it then pipe will empty 539 cm³ breadth of rectangle, while height of cylindrical vessel
is equal to length of rectangle, whose area is 168 cm 2.
of water is one minute. If radius of the solid is 21 cm,
If ratio between length & breadth of rectangle is 7 : 6,
then in what time whole of the water will be emptied then find the think ness of 1 disc, if these discs are
from the solid. Given that diameter of cylinder and kept one above another in cylindrical vessel and 10
hemisphere are same. discs get completely fit in cylindrical vessel.(Let
(a) 72 min (b) 60 min (c) 90 min radius of discs equal to radius of cylindrical vessel)?
(d) 75 min (e) None of these (a) 1.2 cm (b) 1.6 cm (c) 1.8 cm
(d) 1.4 cm (e) 1.0 cm
40. A cylinder whose height is equal to its radius is full of
milk. Its milk is poured into a hemi spherical bowl of 46. The ratio between radius of two hemispheres solid tin
pieces is 2 : 3 and difference between volume of both
same radius as of cylinder and remaining milk is
is 836/21 cm3. These two hemispheres are melted
poured into another hemi-spherical bowl of same 𝜋
into a cylindrical vessel and used 74 3 cm3 extra tin
volume as of previous one. What percentage of the
material for polishing the vessel. If ratio between
volume of second hemispherical bowl remained
height & radius of cylindrical vessel is 3 : 4, then find
empty? the total surface area of cylindrical vessel?
(a) 40% (b) 66⅔% (c) 33⅓% (a) 154 cm2 (b) 132 cm2 (c) 176 cm2
(d) 50% (e) 100% (d) 208 cm2 (e) 198 cm2

91 Adda247 Publications For More Study Material


Visit: adda247.com
Cracker Book for Bank (IBPS | SBI | RRB PO | Clerk) Mains Exams

Solutions

1. (e); Let, radius = 5X and height = 12X cm. 6. (e); Let length of the wire = h
2200 70
Volume = Radius = 2 = 35 mm = 3.5 cm
7
1 22 2200 4
= × × 12X × 5X × 5X = Volume of the wire = πr 2 h = π𝑟 3
3 7 7 3
4
X=1 and, volume of sphere = 3 π × (21)3
r = 5 and h = 12 4
π𝑟 2 h = π × 21 × 21 × 21
ℓ = √52 + 122 3
h = 1008 cm
= √25 + 144
= √169 7. (b); Radius of cylinder = 0.63 m
= 13 cm Length = 7.7 m
Let side of square is ‘a’ meter
2. (d); let side of a rectangle=a, 2a, side of a square=b ATQ,
2
2(a+2a)=4b , a= b Curved surface area of cylinder × 500 = Area of
3
8 field
Area of rectangle=a×2a=2a2 or ⌈ ⌉b2 22
9 2 × 7 × 0.63 × 7.7 × 500 = a2
Area of square=b2
a = 123.47 m
Ratio = 8:9
approximately (a) ≈ 125 𝑚
3. (d); Parameter of square = parameter of rectangle 8. (a); Ratio of length, breadth and height of cuboid →
= 2 (24 + 16) 𝑎∶𝑎∶1
4a = 80 cm As, a cuboid is cut into a cube this means after
Side of square = 20 cm cutting ratio become 1 : 1 : 1 or a : a : a
20
Radius of circle = = 10 cm In starting height is more than length and
2
Circumference of circle breadth. We conclude this because of the
22
2πr = × 20 =
440
cm condition that base is same, and cuboid is cut
7 7 along its base, so height is reduced.
4. (b); T.S.A = 2πr² + πr² + 2πrh = 2552 After cutting this cube again along its base a : a : a
3πr² + 2πr × 8 = 2552 becomes a : a : 1 ⇒ 𝑎 > 1
2552
3r² + 16r = 22 × 7 9. (a); Same base → Same radius
3r² + 16r = 812 Let radius = r
3r² + 16r – 812 = 0 ATQ—
1 2
πr H
3r² + 58r – 42r – 812 =0 3
=3
4
r(3r² + 58) – 14(3r + 58) = 0 πr2 h
58 H and h are the height of cone and cylinder
r = 14, − respectively.
3
2 H 4
Required volume = 3 πr 3 + πr 2 h h
=1
2 22 22 Ratio of height and radius of cone
= × × (14)3 + × (14)2 × 8
3 7 7
1 H:r=4:3
= 5749 3 + 4928 h=x
1
= 10677 3 h = 4x
r = 3x
14
5. (b); Radius of circle = = 7 slant height of cone = 5x
2 πrℓ+πr2 1
Area of circle Ratio of T. S. A = = ⇒1∶1
2πrh+2πr2 1
22
= × 7 × 7 = 154 sq. cm 𝜋𝑟2 ℎ 616
7 10. (a); =
Rectangle area = 622 – 154 = 468 sq. cm 2𝜋𝑟ℎ 352
Breadth of rectangle 𝑟 = 3.5 m
468 𝜋𝑟 2 ℎ = 616
= = 18 cm 616
26 ℎ = 11×3.5 = 16 m
Required sum
= 2πr + 2(ℓ + b) Total S.A. = 2𝜋𝑟ℎ + 2𝜋𝑟 2
22 = 2𝜋𝑟(ℎ + 𝑟)
= 2 × × 7 + 2(26 + 18) 22
7 = 2 × 7 × 3.5(3.5 + 16)
= 44 + (44) × 2 = 132 cm.
= 429 m2

92 Adda247 Publications For More Study Material


Visit: adda247.com
Cracker Book for Bank (IBPS | SBI | RRB PO | Clerk) Mains Exams

11. (e); Let, length and breadth of rectangle be ‘l’ and ‘b’ 20
𝑛𝑒𝑤 𝑟𝑎𝑑𝑖𝑢𝑠 = (1 + ) 𝑟 = 1.2 𝑟
respectively 100
Also 1.2h.
Given (ℓ × b) = 252 cm²
2π × 1.2 r(1.2r + 1.2h)
and, 4𝑙 = 72 cm
44 × 2πr (h + r) = 67760
𝑙 = 18 cm
r(h+r)= 245
length of rectangle = 18 cm
252 Let radius and height be x and 4x
breadth of rectangle, 𝑏 = = 14 𝑐𝑚 𝑥 × 5𝑥 = 245
18
Radius of cone = 14 cm x = 7 cm
Height of cone = 18 cm
1 1 22 17. (e); Let ratio of cone, cylinder and hemisphere = r
Volume of cone = 3 𝜋𝑟 2 ℎ = 3 × 7 × 14 × 14 × 18 Height = r (Because height of hemisphere is equal
= 3696 cm² to its radius)
12. (a); Length + breadth + height = 12 cm Required ratio →
and 2πr(r + h) : πr (ℓ +r) : 3πr²
√ℓ2 + 𝑏2 + ℎ2 = 5√2 ⇒ 4r : √2r+r : 3r
ℓ² + b² + h² = 50 cm 4 : √2 +1 : 3
𝑙 + 𝑏 + ℎ = 12 18. (d); 2πr + 2 (ℓ + b) = 220 cm
Square both sides πr² = 1386 sq. cm
(𝑙 + 𝑏 + ℎ)2 = 122 1386×7
r2 =
𝑙2 + 𝑏 2 + ℎ2 + 2 (ℓb + bh + hℓ) = 122 22
12² = 50 + 2 (ℓb + bh + hℓ) r = 21 cm
2 (ℓb + bh + hℓ) = 94 cm² Length of rectangle
4
= 21 × = 28 cm
13. (a); Radius of circular garden 22
3

=
66×7 21
= cm 2 × 7 × 21 + 2(28 + b) = 220
2×22 2
Area of circular garden (πr²) 132 + 56 + 2b = 220
32
22 21 21
= 7 × 2 × 2 = 346.5 sq cm b = = 16 cm
2
4 Area of rectangle = (28 × 16) = 448 𝑐𝑚2
Perimeter of square = 346.5 ×
5
=277.2 cm 19. (c); Let r be the radius of each circle.
4a = 277.2 cm Then by given condition,
a = 69.3 cm πR2 = 2πR ⇒ R = 2
Area of square hall ∴ The length of the side of the square = 8
= 69.3 × 69.3 Now the area covered by 4 coins = 4 × π (2)2 =
= 4802.49 sq cm 16 π
14. (d); r = 3 cm, R = 5 cm, h = 15 cm And area of the square = 64
Total surface area of pipe ∴ The area which is not covered by the coins
= 2πrh + 2πRh + 2(πR² – πr²) = 64 – 16π = 16 (4–π)
= (2π × 3 × 15) + (2π × 5 × 15) + 2(5²π – 3²π)
20. (e); Total surface area of sphere = 4πr²
= 90π + 150π + 32π
= 272π cm² Total surface area of hemisphere = 3πr²
Let radius of hemisphere and sphere be 3𝑥 cm
15. (a); If the diagonal of cube is 6√3 cm
And 2𝑥 cm respectively.
Then side of cube is 6cm.
Now let volume of irregular shape is x cubic ATQ—
centimeter. 3πr³ – 4πr² = 423.5 cm²
4 22 22
216 = 3 π((1)3 + (2)3 + (3)3 ) + x 3× 7
× (3𝑥)2 – 4 × 7
× (2𝑥)2 = 423.5
4 22
216 = 3 × × [36] + x 𝑥 = 3.5 cm
7
1056 21
𝑥 = 216 − Radius of hemisphere = 2
cm =10.5
7
456
𝑥= 7
= 65.14 cubic centimeter 21. (d); Let level of water will be increased by h.
4
16. (e); Let the radius and height of cylinder be are r & h π × (15)2 × h = 3 π(10)3
respectively 4 10×10×10 25
Now 20% income in both mean ⇒h=3× 15×15
= 5 27 cm

93 Adda247 Publications For More Study Material


Visit: adda247.com
Cracker Book for Bank (IBPS | SBI | RRB PO | Clerk) Mains Exams
22
22. (e); Volume of earth removed = 10 × 4.5 × 3 =2× × 17.5 = 110
7
= 135m3
Let, Breadth of rectangle = x
Remaining area = [20 × 9 − 10 × 4.5] = 180 −
45 = 135 ⇒ length of rectangle = 1.2x
Let, rise in height be h m ATQ,
Therefore, 2(1.2x + x) = 110
55
ℎ × 135 = 135 ⇒ 𝑥 = 2.2 = 25
ℎ = 1m
Required area = 1.2 × 25 × 25 = 750 cm²
23. (a); Lets length of two rectangles be 7x cm 6x cm
respectively 28. (d); Area of the space left out = Area of square – Area
ATQ of circle
2 (7x + 16) – 2 (6x + 16) = 8 cm
2x = 8 cm
x = 4 cm
4×7
Radius of cylinder = 2 = 14 cm
Height of cylinder = 4 × 6 = 24 cm
Total surface area of cylinder = 2πr (r + h) 22
22
= 2 × × 14(14 + 24) = (28)2 − × 14 × 14
7
7
11×14
= 88 × 38 = 3344 cm² = 28 (28 − )
7
Solutions (24-25): Let height of cylinder = 4𝑥 = 28(28 − 22)
So radius of cylinder = 3𝑥 = 28 × 6
Height of cone = 4𝑥 = 168 m2
4𝑥
Radius of sphere = = 2𝑥
2 29. (c); (ℓ − 5)(𝑏 + 3) = ℓ𝑏 − 9 ……..(i)
24. (d); T.S.A of cylinder = 2𝜋(𝑟𝑎𝑑𝑖𝑢𝑠)(𝑟𝑎𝑑𝑖𝑢𝑠 + ℎ𝑒𝑖𝑔ℎ𝑡) (ℓ + 3)(𝑏 + 2) = ℓ𝑏 + 67 ……..(ii)
T.S.A of sphere = 4𝜋(𝑟𝑎𝑑𝑖𝑢𝑠)2 Solving (i) and (ii),
2𝜋×3𝑥×(4𝑥+3𝑥) 21
Required ratio = = 8 = 21 : 8 ℓ𝑏 + 3ℓ − 5𝑏 − 15 = ℓ𝑏 − 9
4𝜋(2𝑥)2
or, 3ℓ − 5𝑏 = 6 ………….(iii)
1
25. (a); Volume of cone = 3 × 𝜋(3𝑥)2 × 4𝑥 ℓ𝑏 + 2ℓ + 3𝑏 + 6 = ℓ𝑏 + 67
Volume of sphere = 3 𝜋(2𝑥)3
4 or, 2ℓ + 3𝑏 = 61 ………(iv)
1
×𝜋×36×𝑥 3
Solving (iii) and (iv),
Required % = 34 × 100 = 112.5% ℓ = 17m
×𝜋×8×𝑥 3
3
30. (a); Total surface area of the toy = C.S.A of cone +
26. (c); Volume of Equilateral triangle well = volume of
C.S.A of Hemisphere
cylindrical tank filled.
Let, slant height of cone be l cm.
Let h = height of cylinder πrℓ + 2πr² = 858 cm2
ATQ, πr(ℓ + 2r) = 858 cm2
√3 22
× 8 × 8 × 44√3 = ×4×4×h ℓ =25 cm
4 7
height of cone = 24cm
⇒ h = 42 cm
volume of the toy
1 2
27. (d); Let, the side of square = 2a = πr 2 h + πr 3
3 3
⇒ Diameter of circle = 2a 1 2 (h
= πr + 2r)
ATQ, 3
2
22 = 1950 3 cm3
4a2 – 7
× a × a = 262.5

28a2 –22a2
= 262.5 31. (a); 2πr + 2(ℓ + b) = 204 cm
7 2 (ℓ + b) = 144 × ½
⇒ 6a² = 262.5 × 7 2 (ℓ + b) = 72
⇒ a² = 306.25 ℓ + b = 36
⇒ a = 17.5 cm (b + 8) + b = 36
22
Perimeter of circle = 2 × ×a b = 14
7
ℓ = 22

94 Adda247 Publications For More Study Material


Visit: adda247.com
Cracker Book for Bank (IBPS | SBI | RRB PO | Clerk) Mains Exams

Area of rectangle = 14 × 22 = 308 sq. cm Second Vessel, 𝑉2 = 𝜋(𝑟 + 7)2 12


Circle radius Second Vessel, 𝑉2 = 𝜋𝑟 2 (12 + 15)
2πr = 204 – 72 Given,
πr = 66 𝑉1 = (𝑉 + 𝑄) 𝑐𝑚3 _______(I)
r = 21 cm 𝑉2 = (𝑉 + 𝑄) 𝑐𝑚3 ________(II)
22
Area of circle = × 21 × 21 From (i) and (ii) We get
7 𝑉1 = 𝑉2
= 1386 sq. cm
𝜋(𝑟 + 7)2 12 = 𝜋𝑟 2 (27)
Required sum = (308 + 1386) sq. cm
r = 14 cm
= 1694 sq. cm
So increased capacity (Q)
22 22
32. (d); Let the radius of large sphere is 3𝑥 and small = 7 × (14 + 7)2 × 12 − 7 × 14 × 14 × 12
sphere is 𝑥. = 16632 – 7392
Then if large sphere is melted into n small = 9240 cm3
spheres.
4 4
𝜋(3𝑥)3 = 𝑛 3 𝜋(𝑥)3 36. (e); Given, area of square park = 19600 cm²
3 a² = 19600
27𝑥 3
𝑛 = 𝑥 3 = 27 𝑎 = 140 cm
140
Surface area of large sphere = 4𝜋(3𝑥)2 = 36𝜋𝑥 2 Radius of each light pole = = 2.8 cm
50
Surface area of 27 small spheres = 27 × 4 𝜋 (𝑥)2 Area covered by One pole in park
= 108𝜋𝑥 2 22
= × 2.8 × 2.8 = 24.64 cm²
108𝜋𝑥 2 –36𝜋𝑥 2 7
Required %= 36𝜋𝑥 2
× 100 = 200% Total area covered by these nine poles
= 9 × (24.64)
33. (d); Let the radius of circle is R = 221.76 cm²
Atq,
πR2 21 37. (c); Given, Area of square = 306.25 cm2
= ⇒ R = 21
2πR 2 Side of square = 17.5 cm
Volume of sphere of same radius Breadth of square = 17.5 − 3.5 = 14 cm
4
= πR3 252
Length of rectangle = 14 = 18 𝑐𝑚
3
Let, radius of cylinder is r and height is h Radius & height of cylindrical vessel be 14 cm &
4
3
πR3 = πr 2 h 18 cm respectively
4 22
r 2 h = 3 × 21 × 21 × 21 Volume of cylindrical vessel = × 14 × 14 ×
7
1
= 4 × 7 × 21 × 21 18 × = 2772 cm3
4
r 2 h = 28 × 21 × 21 …(i)
In question 38. (c); let height of vessel = h
3
h:D=2:3 [D is diameter of cylinder] Let radius of bowl = h = radius of vessel
2
h : r = 2 : 3/2 Volume of hot water = 3 𝜋ℎ3
2

= 4: 3
Required percentage
4x: 3x 2 3 3
𝜋( ℎ)
Put this in equation (i) = 3 2
× 100
3 2
36x3 = 28 × 21 × 21 𝜋( ℎ) ×ℎ
2
x³ = 7 × 7 × 7 = 100%
x=7
height = 28 units 39. (a); Let radius be 3r then height will be 2r
Value of figure formed
34. (a); Let radius of 1st cone be r1 and that of second 2
= 3 π × (3r)3 + π(3r)2 h
cone be r2. 2
𝜋𝑟1 ℓ = 3(𝜋𝑟2 . 3ℓ) = 3 π × 27r 3 + π9r 2 × 2r
or, 𝜋𝑟1 ℓ = 9𝜋𝑟2 ℓ = 36 πr³ = 3r = 21
𝑟 9
⇒ 𝑟1 = 1 r=7
2 22
Ratio of areas = 81 : 1 = 36 × 7 × 7 × 7 × 7
= 36 × 22 × 49 cm³
35. (b); Let initial radius of both vessels r cm and volume Required time
is V cm3 36×22×49
V = 𝜋𝑟 2 12 = 539
For first vessel, 𝑉1 = 𝜋(𝑟 + 7)2 12 = 72 minute

95 Adda247 Publications For More Study Material


Visit: adda247.com
Cracker Book for Bank (IBPS | SBI | RRB PO | Clerk) Mains Exams

40. (d); Let radius of cylinder and hemisphere is ‘r’ Height of cylinder = 6 × 3 = 18 𝑐𝑚
Also height of cylinder = ‘r’ Volume of cylinder = πr2h
22
Total milk = πr² × r = πr³ = 7 × 21 × 21 × 18
Milk poured in 1 hemi-sphere = 24948 cm3
2
= πr 3
3
1 44. (b); Let the radius of sphere be x cm.
Remaining milk = πr 3 4 1 2
3 ⇒ 𝜋(𝑥)3 = 18 ( 𝜋 × (4√3) × 4) +
When this milk poured in 2nd sphere then it 3 3

remained empty 12(𝜋 × (4)2 × 6)


4
2 1 1 ⇒ (𝑥)3 = 1152 + 1152 cm³
3
πr 3 – 3 πr 3 = 3 πr 3 3
⇒ x = 12 cm
Required% = 50%
Then, total surface area of sphere = 4πx²
41. (e); Perimeter of corridor and hall = 102 m 22
= 4 × × (12)2
7
Let radius of hall is R m and corridor is r m 2
ATQ— = 1810 7 cm²
2πR + πr + 2r = 102 m 45. (d); Let length & breadth of rectangle be 7x & 6x
22 3r 22r
2× × + + 2r = 102 m respectively
7 2 7
204r
= 102 Given area of rectangle = 168 cm 2
14 7x × 6𝑥= 168 cm2
r=7m
42x2 = 168
21
So, R = m x2 = 4
2 x = 2 cm
Radius of circular garden
21 400 Radius of cylindrical vessel = 6 × 2 − 1.5 =
= ( + 7) × 10.5 𝑐𝑚
2 100
35
= × 4 = 70 m Height of cylindrical vessel = 7 × 2 = 14 𝑐𝑚
2
Area of Garden = πr² Volume of cylindrical vessel = 𝜋 × 10.5 × 10.5 ×
= 70 × 70 𝜋 14
= 4900 𝜋 sq. m = 1543.5π cm3
Let think ness of each discs be x cm
42. (c); Given, height of cylindrical vessel = side of square So, 10 × 𝜋 × x × 10.5 × 10.5 = 1543.5π cm3
a2 = 256 cm² 1543.5
x = 10.5×10.5×10
a = 16 cm
x = 1.4cm
Let radius of hemisphere is 2r cm and sphere is r
cm. 46. (c); Let the radius of two hemisphere be 2x & 3x
ATQ— ATQ,
2 2 836
3π(2r)² – 4πr² = 4928 cm² π(3x)3 – 3 π(2x)3 = 21 cm3 [∵ volume of
3
12r² – 4r² = 1568 hemisphere = ⅔ π r³)
r² = 196 2 836
r = 14 ⇒ π[19x 3 ] =
3 21
Radius of cylindrical vessel ⇒x=1
28+14
= 2 = 21 cm ∴ radius are 2 cm & 3 cm.
Now,
Volume of cylindrical vessel 2 2 74
22 [3 π(2)3 + 3 π(3)3 ] + [ 3 πcm3 ] = πR2 H
= 7 × 21 × 21 × 16 2 2
= 22176 cm³ ⇒ 3 π[8 + 27] + 3 π 37 = πR2 H
2
43. (b); Ratio between length, breadth and side of square ⇒ 3 𝜋[35 + 37] = 𝜋𝑅2 𝐻
= 21 : 6 : 14 ⇒ R²H = 48
Let length and breadth of rectangle be 21x cm ⇒ R : H = 4 : 3 (given)
and 6x cm respectively. Let R = 4a, H = 3a
And, side of square be 14x cm ⇒ 16a². 3a = 48
ATQ – 𝑎=1
4(14x) – 2 (21x + 6x) = 6 ∴ R = 4, H = 3
56x – 54x = 6 T.S.A. of cylinder = 2πR(R + H)
22
x = 3 cm = 2 × × 4(7)
14×3 7
Radius of cylinder = = 21 cm = 176 cm²
2

96 Adda247 Publications For More Study Material


Visit: adda247.com
Cracker Book for Bank (IBPS | SBI | RRB PO | Clerk) Mains Exams

Chapter
Permutation, Combination and Probability
11
BEST APPROACH TO SOLVE THE QUESTIONS

The best way to define probability is number of favorable outcomes divided by total number of possible outcomes. Eg.
When we conduct an experiment of throwing a dice and we have to calculate probability of getting 2.

Now number of favorable outcome is 1 i.e. ‘2’


And total number of possible outcomes is 6 i.e it may appear ‘1, 2, 3, 4, 5 or 6’ on throwing a dice. Hence probability of
getting a ‘2’ is
1
=6

If we were asked what is the probability of getting an even number.


Number of possible outcome = 3 (2, 4 or 6)
3 1
Probability of getting an even number on throwing a dice = =
6 2

But the probability in bank exam is not this much easy, we encounter with way more complicated questions.
Example 1: A C++ program shuffles all the letters of a word. When the word INDIA was entered, a new word was given
as an output. What is the probability that new word formed is having all the vowels together?
Solution: We need to calculate two things
(i) Total number of possible outcomes
(ii) Total number of favorable outcomes
Imagine how many different words can be formed by re-arranging the words of INDIA.
And then how many different possible words can be formed when ‘IIA’ of ‘INDIA’ are together.

To solve these kind of questions we have PERMUTATION AND COMBINATION.


Permutation and Combination:
Basic definition: Permutation means different possible arrangement of things taken all or some of them together.
While Combination means different way of selection of things taken all or some of them together.
Eg. Of arrangement/permutation:
There were two chairs numbered 1 and 2. In how many ways A, B and C could sit on them?
SOLUTION: AB or BA, AC or CA and BC or CB. (First person denotes to sit on chair named A). Hence there are 6 possible
ways.
Eg. Of selection/ Combination
There were two chairs and three persons. In how many different ways they can sit on chairs?

97 Adda247 Publications For More Study Material


Visit: adda247.com
Cracker Book for Bank (IBPS | SBI | RRB PO | Clerk) Mains Exams

Solution: Either AB, BC or AC will sit on two chairs. Hence there are 3 possible ways.

The point is arrangement when sequence matters is known as permutation or selecting some or all the persons is called
combination.
Before further discussion, let us revise few terms of permutation and combination.
! is called factorial.
N! where N is a natural number is N × (N-1) × (N-2)….. ×1
0! = 1
N N!
Pr = (N–r)!, it is used in permutation cases

N N!
Cr = C!(N–r)!, it is used in combination cases.
E.g. In previous cases.
3 3! 3×2×1
P2 = (3–2)! = = 6 arrangements.
1
3 3!
C2 = = 3 cases of selection.
2!(3–2)!

NOTE:
Though permutation and combination in itself are too big, here we will discuss few of the types of questions that are
asked in exam. Look at the previous example of INDIA.

Example 2: What is the probability that after rearranging the word ‘INDIA’ a new word is formed in which all vowels
are together?
SOLUTION: When n things in which p are of one type, q are of one type , then total no. of arrangements of those n
n!
things is =
p!q!
Taking two Is and one A as a single entity, we have in all 3 entities(N,D and the group of Is
and A).
Total no. of ways of arranging these three entities is 3!.
3!
Two Is and one A can be rearranged among themselves in ways.
2!
3!
Hence, total no. of ways in which all vowels are together is equal to 3!× =18.
2!
5!
Total no. of ways in which all letters of word INDIA can be arranged = 2!=60.
18 3
Hence required probability= 60=10

Example: What is the probability that a word formed after rearranging “ARRANGEMENT’’ such that all vowels are
together?
SOLUTION: Following the same method as that of previous question,
11! 11!
Total no. of ways of rearranging the letters of word ARRANGEMENT= =
2!2!2!2! 16
8! 4! 8!4!
No. of ways of arranging letters such that all vowels are together= 2!2! × 2!2! = 16
8!4!
4
Required probability= 16
11! = 165
16

KEY POINTS: Number of permutation of n things taken all at a time is n!


N
Pr = NCr × r!

98 Adda247 Publications For More Study Material


Visit: adda247.com
Cracker Book for Bank (IBPS | SBI | RRB PO | Clerk) Mains Exams

Practice Exercise Based on New Pattern

1. There are five mangos and six oranges in a bucket. 9. In how many ways 9 hindi books are placed between
What will be probability of Picking up four fruits 10 English books such that no two Hindi book comes
which contains at least two orange? together?
53 43 59 (a) 60 (b) 55 (c) 45
(a) (b) (c)
66 66 66 (d) 96 (e) 75
49 3
(d) 66 (e) 5 10. Fourteen applications apply for a job out of which
1 2 there are 9 men and 5 women. If only three
2. Three shooter having probability of shot a target , applications selected for the job then find the
3 5
3
and 7 respectively. What is the probability that exactly probability that at least one of the selected
application is of a woman?
one of them did not hit the target? 60 10 80
12 29 19 (a) 91 (b) 13 (c) 91
(a) 35 (b) 105 (c) 35 40 50
41 1 (d) (e)
(d) (e) 91 91
105 5
11. In a bucket there are three type of fruits i.e, mango,
3. Rahul, Sachin and Sanjeev speaks 2 times, 5 times and banana and papaya. Probability of selecting one
3 times truth out of 5 times, 6 times and 8 times 1
mango out of total fruits in bucket is 3 and the
respectively. Find the probability that they contradict probability of selecting one papaya out of total fruits
each other when asked to speak on a fact? 3
3 5 7 in bucket is . If number of banana in the bucket is
8
(a) (b) (c)
16 16 16 seven, then find the number of papaya in the bucket.
13 9
(d) 16 (e) 16
(a) 4 (b) 5 (c) 11
4. Probability of choosing one boy from a group of 26 (d) 9 (e) 7
3
girls and ‘x’ boys is 5. If one more boy joined the group 12. A bucket contains seventeen rectangular boxes, from
then find the probability of choosing one boy and one number 1 – 17. If three boxes drawn at random from
girl from that group? bucket without repeated, then what will be
8 6 4 probability of that all three rectangular boxes
(a) 33 (b) 11 (c) 11
represent even number boxes?
14 16 9 6 7
(d) (e) (a) 85 (b) 85 (c) 85
33 33
11 21
5. In how many ways word “EATEN” can be arranged (d) (e)
85 85
with or without meaning in which at least two vowels 13. Two boxes contain 4 and 16 balls respectively. Two
will come together? balls in the first box and four in the second box are
(a) 48 (b) 54 (c) 72 black. If a box is chosen randomly and two balls are
(d) 108 (e) 90 drawn at random from it, what is the probability that
at least one ball is black ?
6. Sum of digits of a 5-digit number is 43. Find the 11 43 77
probability that such a number is divisible by 11? (a) 20 (b) 120 (c) 120
9 7
(a) 1/3 (b) 1/5 (c) 2/5 (d) 20 (e) 23
(d) 2/15 (e) 2/3 14. Probability of getting one Red ball from a bag full of
balls is 2/13 and Black balls in the bag is 5. If white
7. Three-unit squares are chosen at random on a
ball is 30% less than Brown ball and 40% more then
chessboard. What is the probability that two of them the black balls then find the number of Red balls.
are of same color while remaining one is of another (a) 5 (b) 4 (c) 6
color? (d) 10 (e) 13
(a) 16/21 (b) 8/21 (c) 5/21
15. Probability of choosing one boy from a group of 26
(d) 4/7 (e) 32/63 3
8. Two dice are thrown, find the probability of getting an girls and ‘x’ boys is 5. If one more boy joined the group
even number after multiplying numbers obtained then find the probability of choosing one boy and one
from the dice. girl from that group?
8 6 4
(a) ¼ (b) ¾ (c) ⅔ (a) 33 (b) 11 (c) 11
14 16
(d) ½ (e) ⅙ (d) 33 (e) 33

99 Adda247 Publications For More Study Material


Visit: adda247.com
Cracker Book for Bank (IBPS | SBI | RRB PO | Clerk) Mains Exams

16. In a meeting of HCL company, in the month of June, 23. A bag contains 5 red, 4 green and 3 black balls. If
there are 6 CEO’s , Y MD’s and 7 HR’s. Probability of three balls are drawn out of it at random, find the
5
calling one MD on stage is . When the meeting is probability of drawing exactly 2 red balls?
18 5 7 6
conducted in next month (Y + 1) CEO’s, (Y + 3) MD’s (a) 22 (b) 24 (c) 25
2 7
and (Y + 2) HR’s attended the meeting. If two (d) (e)
11 22
representatives are called together on stage, then
what will be probability of both being CEO’s? Directions (24-25): A man can send a message by using
1 1 1 six flags. He had 4 black flags, 3 blue flags and 1 green flag.
(a)16 (b)14 (c)12
1 1 24. In how many ways a message can be transmitted?
(d) (e)
18 20 (a) 108 (b) 185 (c) 20
17. There are three groups of children in a school i.e. A, B (d) 136 (e) 110
and C. In group A, there are (x + 2) girls and x boy. In 25. In how many ways a message can be transmitted by
group B, there are 2 boys and 2 girls, whereas in using all three colored flags?
group C, there are 1 girl and 2 boys respectively. One (a) 150 (b) 90 (c) 100
child is selected at random from each group. Find the (d) 70 (e) 125
value of x, if the probability of selecting 1 girl and 2
3 Directions (26-27): A box contains two types of ball i.e.,
boys is . red and Blue. Probability of choosing one red and one
8
(a) 2 (b) 3 (c) 1 1
blue ball is while probability of choosing two red balls is
(d) 4 (e) 5 3
2
.
18. In a room, 10 people sit in which 4 left hander and 6 20

right hander. Find the probability that left hander 26. Find the probability of selecting two balls without
shake hand with only left handers. replacement from the box such that at least one of
1
(a) 15
7
(b) 45
17
(c) 45 them is red?
9 11 13
5 2 (a) 20 (b) 20 (c) 20
(d) 9 (e) 15 17 3
(d) 20 (e) 4
19. In a cold drink cartoon 3 pepsi, 5 limca and 7 sprites
bottels. If one bottle is drawn at random from the 27. If 5 Red balls and 6 blue balls are added in the box,
cartoon what is the probability that it is either sprite then find the probability of choosing two blue balls
or pepsi ? from the box at present?
5 3 2 1 1 1
(a) 7 (b) 4 (c) 3 (a) (b) (c)
2 3 4
8 1 1
(d) (e) None of these (d) (e)
9 5 6

20. Find the difference between number of words formed Directions (28-30): There are three sections A, B and C
from ALAIS when all vowels are together, and words in a class. Every section has some boy and some girl
formed from ENGLISH when no vowels are together? students in it. Probability of a girl being selected when
2
(a) 3528 (b) 3618 (c) 3572 one student is selected randomly from section A is 5, that
(d) 3592 (e) 3582 4
from section B is 9 and that from section C is 9.
5

21. In a box some Munch chocolate, some Dairy milk


chocolate and Kit Kat chocolate. The probability of 28. If the ratio of total number of students in sections A, B
3 and C is 10 : 12 : 9, then what is the probability of a
selecting one Munch chocolate out of box is and the
10 girl being selected when one student is selected
probability of selecting one Kit Kat chocolate out of randomly from the students from all the three
2
box is . The number of Dairy milk chocolate in box is sections together? (2 Marks)
5 14 11 13
6. Find probability of selecting three chocolate from (a) 31 (b) 23 (c) 31
the box such that at most two chocolate are same? (d) 93
43
(e) Cannot be determined
64 64 42
(a) (b) (c)
85 95 85
87 67 29. If the number of girls in sections A is same as the
(d) 95 (e) 85
number of boys in section C, then what is the ratio of
22. Find the probability of forming the word from number of boys in section A to the number of boys in
ANANAB such that N never comes together. section C? (2 Marks)
2 4 3
(a) 3 (b) 5 (c) 5 (a) 2 : 3 (b) 3 : 4 (c) 3 : 2
2 4
(d) 5 (e) 5 (d) 4 : 3 (e) 3 : 5

100 Adda247 Publications For More Study Material


Visit: adda247.com
Cracker Book for Bank (IBPS | SBI | RRB PO | Clerk) Mains Exams

30. If 20 girls leaves section B, then the probability of a 37. Find the probability of winning England against India
boy being selected when one student is selected when Kohli make a century but cook does not.
5
randomly from this section will be 8. What is the (a)
9
(b)
16
(c)
2
250 250 5
number of boys in section B? 14 21
(2 Marks) (d) (e)
250 250
(a) 120 (b) 100 (c) 75
Direction (38-39): In a stationary shop there are four
(d) 60 (e) Cannot be determined
types of colored sheets of red, blue, green and white
Directions (31-32): There are some horses, hens and colors. The probability of selecting one red sheet out of
elephants in a park. Ratio between number of heads in 1
park to number of legs in park are 7 : 20. the total sheets is ,the probability of selecting one blue
3
2
31. Find the probability of choosing a hen over all the sheet out of the total sheets is 7
and the probability of
animals. 1
selecting one white sheet out of the total sheets is . The
3 2 3 4
(a) (b) (c)
7
4
7
2
8 number of green sheets in the bag is 22.
(d) 7 (e) 9
38. If all the sheets are numbered as 1,2,3,….and so on
32. If probability of choosing 2 horses over all the animals and one sheet is picked up at random, then find the
9
are , then find the number of elephant present in probability of picking up a sheet which is numbered
119
park. (Number of horses is double than that of the as a multiple of 13 or 17.
number of elephants in park). (2 marks)
(a) 5 (b) 10 (c) 20 1 1 5
(a) (b) (c)
(d) 15 (e) 25 8 7 8
9
Directions (33-34): There are two bags, one of which (d) 10 (e) none of these
contains 3 black and 4 white balls, while the other
39. What is the total number of sheets in the bag?
contains 4 black and 3 white balls. A dice is cast, if the face
1 or 3 turns up, a ball is taken from the first bag and if any (1 marks)
other face turns up, a ball is chosen from the second bag. (a) 117 (b) 168 (c) 154
(d) 120 (e) none of these
33. The probability of choosing a black ball is:
(2marks) Direction (40-41) : A coach invited 9 players of the team
11 11 11
(a) (b) (c) on dinner. All 10 persons seated in a circular table.
24 21 27
11 3
(d) 29
(e) 8 40. If in how many ways captain and vice-captain seat
34. The probability of choosing a white ball is: opposite to each other?
(2marks) (a) 80640 (b) 5040 (c) 3624
11
(a) 21
11
(b) 29
10
(c) 21 (d) 40320 (e) 10080
14 11
(d) (e) 23 41. In how many ways team can seat, if caption and vice-
23
captain seat either side of coach.
Directions (35-37) : India wins every 6 matches out of
(a) 80640 (b) 5040 (c) 3624
10 matches against England. Virat Kohli (Indian Player)
make a century in every 3 matches out of 5 matches and (d) 40320 (e) 10080
in which he makes century, India wins 80% of the Directions (42-44): A joker have 10 balls, 10 ribbon and
matches. Alastair cook (England’s player) make 1 century 10 plates of red, yellow and blue color in ratio 2 : 3 : 5, 5 :
in every 4 matches and England wins 60% of that
2 : 3 and 3 : 5 : 2 respectively. He does his performance by
matches.
using one item of each category.
35. Find the probability of India’s winning against
England when Kohli and Cook both make centuries. 42. What is the probability that he performs with only red
20 17 36 color?
(a) 623 (b) 625 (c) 625
24 18 (a) 0.3 (b) 0.02 (c) 0.25
(d) 625 (e) 625
(d) 0.03 (e) 0.15
36. Find the probability that India wins against England
43. What is the probability that he chooses yellow color
when Kohli failed to make a century.
8 3 6 for ribbons?
(a) 25 (b) 5 (c) 25
4 12
(a) 0.0333 (b) 0.025 (c) 0.2
(d) 25 (e) 25 (d) 0.25 (e) 0.03
101 Adda247 Publications For More Study Material
Visit: adda247.com
Cracker Book for Bank (IBPS | SBI | RRB PO | Clerk) Mains Exams

44. If he chooses 3 balls, 5 ribbons and 1 plate then find selected one fruits for eating, then the probability of
the probability that all items are of same color. 3 1
getting orange is 16. The value of x is 37 2 % less than y.
(a) 1 (b) 0 (c) 0.05
In bucket Q, number of Mangos is 40% more than that of
(d) 0.5 (e) 0.002
bucket P. If Satish selecting two fruits at random for
Direction (45-47): - Kishan have two trays of eggs i.e. 18
eating then the probability of getting both Bananas is .
203
tray A and tray B. Tray A have same number of fresh eggs
Total number of fruits in bucket Q is 29.
as tray B have rotten eggs. Rotten eggs in tray A is 40%
In bucket R, the ratio of number of mangos to number of
less than that of in tray B. Fresh eggs in tray B is 14 more
orange is 3 : 4. Total number of bananas & orange is 33 . If
than that in tray A. Total number of eggs Kishan have is
Veer selecting one fruit for eating the probability of
140. 3
getting mangos is .
14
45. Fresh eggs in tray B is what percent more than that in
tray A? 48. What is the total number of bananas present in all the
(a) 140% (b) 80% (c) 60% three buckets?
(d) 40% (e) 20% (a) 30 (b) 38 (c) 36
(d) 32 (e) 28
46. Find the probability of choosing two eggs from tray ‘A’
such that at least one egg should be fresh? 49. If all mangoes of bucket P is placed in the bucket Q
19 17 27 then find the probability of choosing a mango from
(a) (b) (c)
22 22 44
25 21 bucket Q?
(d) 44 (e) 22 5 9 3
(a) (b) (c)
17 17 17
47. Find the probability of choosing one rotten egg? 7 6
(d) 17 (e) 17
23 7 19
(a) 48 (b) 16 (c) 48
25 9 50. If 25% oranges of bucket R are spoiled and thrown
(d) 48 (e) 16 away then find the probability of choosing a good
Direction (48- 50)- There are three bucket P, Q & R orange from bucket R?
3 5 7
contains three fruits Mangos, Bananas & Oranges. (a) 13 (b) 13 (c) 13
In bucket P, contains x Mangos and y Bananas. Number of 6 2
(d) 13 (e) 13
bananas are 3 more than the number of Mangos. If Ram

Solutions
1. (a); Favorable case = (20, 2M) or (30, 1M) or 40 Probability that they all speak truth
∴ Probability 2 5 3 1
= × × =
6C × 5C 6C × 5C 6C 5 6 8 8
2 2 3 1
= 11C + 11C + 11 4 Probability that they all speak lie
4 4 C4
15×10 20×5 15 3 1 5 1
= + + 330 = × × =
330 330 5 6 8 16
1 1
265
= 330 = 66
53 Required probability = 1 – –
8 16
16 –2 –1
=
2. (b);Required probability 16
13
1 2 4 1 3
= 3×5×7+3×7×5+5×7×3
3 2 3 2 = 16
29
= 4. (e); ATQ,
105
x 3
2 26+x
=5
3. (d);Probability that Rahul speaks truth =
5 ⇒ 5x = 26 × 3 + 3x
5
Probability that Sachin speaks truth = ⇒ 2x = 26 × 3
6
3 ⇒ x = 39
Probability that Sanjeev speaks truth = 8
Total no. of students now in group
Rahul, Sachin and Sanjeev do not contradict with = 26 + 39 + 1 = 66
each other if they all are correct or they all are Required probability = 2 × ×
26 40
66 65
incorrect means if they all speak truth or they all 16
speak lie. = 33

102 Adda247 Publications For More Study Material


Visit: adda247.com
Cracker Book for Bank (IBPS | SBI | RRB PO | Clerk) Mains Exams
5!
5. (b);Total number of ways = 2! = 60 ways HEHEHEHEHEHEHEHEHEHEH
Selection of vowels can occur in three pair i.e. There are → English book, H can occupy 10 + 1
( AE, EE, EA) = places
Number of ways in which two vowels will come So no. of ways → ¹¹C₉
together =
11×10
= 55 ways
3×4! 2
= = 3 × 4 × 3 = 36
2!
Above, we had not multiplied 2 in numerator 10. (b);Total number of ways to select three persons
because further arrangement of vowels selected = 14C3
i.e. Required case = 2 men 1 women or 1 man 2
(AE, EE, EA) will give the same arrangement women or 3 women
Number of ways in which three vowels will come = ( 9C2 × 5C1 ) + ( 9C1 × 5C2 ) + ( 5C3 )
together
3!×3! = (36 × 5) + (9 × 10) + (10)
= 2! = 3 × 2 × 3 = 18
= 180 + 90 + 10
Required no. of ways = 36 + 18 = 54 = 280 14C3 = 14C3 = 364
Alternate, 280 10
No. of ways in which no vowel will came together Required probability = 364 = 13
3!
= 2 × 2! = 6
11. (d);Let number of papaya and mango in bucket be ‘P’
Required no. of ways = 60 – 6 = 54 and ‘M’ respectively
6. (b);Let, numbers in 5-digit number is a, b, c, d, e. Number of banana in bucket = 7
In order to get sum of 43, the combinations exist Probability of selecting one mango = 3
1
are
3
9, 9, 9, 9, 7 = 5 numbers and selecting one papaya = 8
9, 9, 9, 8, 8 = 10 numbers So,
Total 15 numbers can be formed having sum 43. M 1
For number which are divisible by 11 main (7+M+P)
=3
condition is 3M = 7 + M + P
(a + c + e) – (b + d) = 11 …(i) 2M – P = 7 …(i)
and (a + b + c + d + e) = 43 …(ii)
Similarly—
On solving (i) and (ii) P 3
a + c + e = 27, b + d = 16 (7+M+P)
=8
a, c and e can be 9, 9, 9 8P = 21 + 3M + 3P
while b, d can 7, 9 or 8, 8 or 9, 7 respectively
5P – 3M = 21 …(ii)
So, there are three numbers which are divisibly
by 11 having sum 43 i.e., 97999, 99979, 98989. Solving (i) and (ii) we get
Required probability = =
3 1 Number of Mango = 8, Banana
15 5
= 7, Papaya = 9
7. (a); Total number of ways = ⁶⁴C₃ = 41664 Total number of papaya in bucket = 9
Favorable ways = 2 × ³²C₂ × ³²C₁
= 2 × 496 × 32 12. (c); Total even number boxes between 1 to 17
= 31744 = 8 (2, 4, 6, 8, 10 …..16)
31744 16
Required probability = 41664 = 21 Total boxes = 17
(³²C₂ × ³²C₁ is taken because if two are black, then Probability of first box represent even number
8
one will be of white or if two are white, then one box = 17
will be of black)
Probability of second box represent even number
8. (b);Favourable case → 7
box = 16
(Dice 1 → even and dice 2 → odd)
(Dice 1 → even and dice 2 → even) Probability of third box represent even number
6
(Dice 1 → odd and dice 2 → even) box =
3 3 3 3 3 3 3 15
= × + × + × = 8 7 2 7
6 6 6 6 6 6 4 Required probability = 17 × 16 × 5 = 85
9. (b);Let us denote English book by E and Hindi books
by H 13. (c); At least one black can be chosen in three ways:
The possible arrangements is 1. first one is black, second is non-black
2. first one is non-black, second is black

103 Adda247 Publications For More Study Material


Visit: adda247.com
Cracker Book for Bank (IBPS | SBI | RRB PO | Clerk) Mains Exams

3. both are black. 17. (c); Possible cases


1 Girl Boy Boy or Boy
Probability of selecting a box is
2
Girls Boy or Boy Boy Girl
Now, probability of choosing at least one black 3 x+2 2 2 x 2 2
ball from first box 8
= (2x+2 × 4 × 3) + (2x+2 × 4 × 3) +
1 2 2 2 2 2 1 5 x 2 1
= ×( × + × + × )= (2x+2 × 4 × 3)
2 4 3 4 3 4 3 12
Probability of choosing at least one black ball (2x+2)×4×3 2(x+2) 2x x
⇒ =( + + )
2×8 3 3 3
from 2nd box (x+1)×9
1 4 12 12 4 4 3 ⇒ = 5x + 4
= 2 × (16 × 15 + 16 × 15 + 16 × 15) 2

27
⇒ 9x + 9 = 10x + 8
= 120 ⇒ x=1
5 27
Required probability = + 18. (e); Total no. of ways in which hand shakes
12 120
50+27 77 10(10 –1)
= 120
= 120 = = 45
2
No. of ways in which left hander shakes hand with
14. (b);Let number of red balls is 2x
left handers
Total balls = 13x 4(4 –1)
Now ATQ = 2
=6
6 2
Black balls = 5 Required probability = =
5×140 45 15
White balls = =7
100 19. (c); Total number of bottle in cartoon
7
Brown balls = 70 × 100 = 10 = 3 + 5 + 7 = 15
Now→ 2x + 5 + 7 + 10 = 13x ∴ Probability of either sprite bottle or pipe bottle
7C 3
x=2 = 1 + C1
15C
Red balls = 4 7+3
1
2
= 15
=3
15. (e); ATQ,
x
=5
3 20. (e); AAILS when all vowels are together
26+x 3!
⇒ 5x = 26 × 3 + 3x = 3! × = 18 words
2!
⇒ 2x = 26 × 3 Total words from ENGLISH = 7! = 5040
⇒ x = 39 Words from ENGLISH when no vowels are
Total no. of students now in group together
= 26 + 39 + 1 = 66 = 5040 – [when all vowels are together]
26 40 = 5040 – [6! × 2] = 3600
Required probability = 2 × ×
66 65 ∴ Required difference = 3600 – 18 = 3582
16
= 33
21. (d);Let total chocolate in box be 10c. Munch chocolate
16. (b);In June — be a and kit-kat chocolate be b .
a 3
Total CEO’s = 6 Probability of selecting Munch chocolate = =
10c 10
MD’s = y ⇒ HR’s = 7 a = 3c ... (i)
Now, Probability of selecting Kit Kat chocolate
ATQ, b
= 10c = 5
2
y 5
13+y
= 18 ⇒ y = 5 b = 4c … (ii)
In July — ATQ,
CEO’s ⇒ 5 + 1 = 6 a + b + 6 = 10c
MD’s ⇒ 5 + 3 = 8 ⇒ 3c + 4c + 6 = 10c ⇒ c = 2
HR’s ⇒ 5 + 2 = 7 So, total chocolate = 20
6C 6C 8 6
3 + C3 + C3
Required probability = 21 2 Required probability = 1 − 20 C
C2 3
6×5 1 (20+56+20) 1044 87
= 21×20 = 14 =1− = 1140 = 95
1140

104 Adda247 Publications For More Study Material


Visit: adda247.com
Cracker Book for Bank (IBPS | SBI | RRB PO | Clerk) Mains Exams

22. (a); Total number of words possible from ANANAB From (iii) and (iv)
6! 5x−5 5x−5+3x 5x−5
= 3!2! = 60 4x ( )=[ ] [x + − 1]
3 3 3
No. of word in which N is always together 20x
(x − 1) = [
8x−5 8x−8
5! ⇒ 3 3
][ 3
]
= 3! = 4 × 5 = 20
⇒ 60x² − 60x = 64x² − 64x – 40x + 40
No. of in which N is never together ⇒ −4x² + 44x – 40 = 0
= 60 – 20 = 40 ⇒ x² − 11x + 10 = 0
40 2
Required probability = 60 = 3 ⇒ x² − 10x – x + 10 = 0
⇒ x(x – 10) – 1 (x − 10) = 0
23. (e); Exactly 2 red balls can be selected in two ways.
(i) 2 red ball & 1 green ball ⇒ x = 1, 10
(ii) 2 red ball & 1 black ball x=1 x = 10
Total no. of possible outcome = ¹²C₃ ⇒y=0 ⇒ y = 15
12×10×11 But y can’t be zero.
= = 220 ways
3×2 ⇒ Number of red balls = 10
Total favourable outcome Number of blue balls = 15
= ⁵C₂ × ⁴C₁ + ⁵C₂ × ³C₁
5×4 5×4
= 2 × 4 + 2 × 3 = 40 + 30 = 70 ways. 26. (c); Required probability
10 C 10 C 15 C
2+ 1 1
Probability = 220 = 22
70 7 y= 25 C
2
45+150 195 13
= = =
24. (b);Case I: When all 4 black flag selected 300 300 20
Two ways could be 21 C
2
1 green flag + 1 blue flag or 2 blue flags 27. (b);Required probability = 25+11 C
2
6! 6! 21×20 1
No. of ways to arrange = 4! + 4!2! = 36×35 = 3
Case II:
When all 3 blue flag selected 28. (d);Probability of a girl being selected from a section
Total girls in the section
3 ways could be = Total students in the section
2 black flags + 1 green flag or 3 black flags Let the number of girls, number of boys and total
6! 6!
No. of ways = + number of students respectively:
3!2! 3!3!
Case III: When one green selected For section A: 2x, 3x and 5x.
⇒ 3 black + 2 blue [other cases already included] For section B: 4y, 5y and 9y.
6! For section C: 5z, 4z and 9z.
3!2! According to the question,
Adding all
Ratio of total number of students in the three
⇒ 30 + 15 + 60 + 20 + 60 = 185
sections:
25. (a); All colored used ⇒ ⟹ 5x : 9y : 9z = 10 : 12 : 9
1 green flag + 2 black flags + 3 blue flags ⟹x:y:z=6:4:3
1 green flags + 3 black flags + 2 blue flags Let the values of x, y and z be 6k, 4k and 3k
1 green flag + 4 black flags + 1 blue flag respectively.
6! 6! 6!
⇒ + + Total number of girls in all the three sections = 2x
2!3! 3!2! 4!
+ 4y + 5z = 12k + 16k + 15k
⇒ 60+ 60 + 30
= 150 = 43k
Total number of students in all the three sections
Solution (26-27): Let, Red balls = x = 5x + 9y + 9z = 30k + 36k + 27k
Blue balls =y = 93k
ATQ, Probability of a girl being selected from the
x y 1
2× × = ... (i) students from all the three sections together
(x+y) (x+y−1) 2 Total girls in all sections 43k 43
And, = Total students in all sections = 93k = 93
x×(x−1) 3
(x+y)(x+y−1)
= 20 ... (ii) 29. (c); According to the question,
Dividing (ii) by (i) Number of girls in sections A = Number of boys in

x−1
=
3 section C
y 5 ⟹ 2x = 4z
⇒ 5x – 5 = 3y …(iii) ⟹ x = 2z
And, from (i) Number of boys in section A : Number of boys in
4xy = (x + y) (x + y – 1) ... (iv) section C = 3x : 4z = 6z : 4z = 3 : 2
105 Adda247 Publications For More Study Material
Visit: adda247.com
Cracker Book for Bank (IBPS | SBI | RRB PO | Clerk) Mains Exams

30. (b);Probability of a boy being selected from this Probability that India win when Kohli makes
5 4
section B after 20 girls left the section = 8 century = 5
Number of boys in section B 5 1
⟹ Total number of students in section B−20 = 8 Probability of Cook’s century = 4
5y 5
⟹ = Probability of other country wining when Cook
9y − 20 8
⟹ 40y = 45y – 100 make century
3 2
⟹ y = 20 = 1– =
5 5
Number of boys in section B = 5y = 100 6 3 4 1 2
Combining all = 10 × 5 × 5 × 4 × 5
Solution (31-32): Let number of hens, horses and 18
elephant in park is — a, b and c respectively. = 625
Let number of heads and number of legs be 7x and 20x
respectively 36. (c); Probability when Kohli failed to make a century
2
So, =5
a + b + c = 7x So, Probability that India wins against England
b + c = 7x– a …(i) 6 2 6
2a + 4(b + c) = 20x …(ii) when Kohli failed to make a century = × =
10 5 25
Solving (i) and (ii)
2a + 4 × (7x– a) = 20x 37. (a); Probability that Kohli will make a century
3
2a + 28x – 4a = 20x =
5
a = 4x Probability that India lose when Kohli make
1
31. (d);Probability of choosing hen over all the animals = century = 5
4x 4 3
= Probability that Cook doesn’t make century = 4
7x 7
Probability that of Winning of England against
32. (a); No. of hens = 4x 4
No. of horses and elephants together India = 10
= 7x– 4x = 3x 4 3 3 1 9
Overall probability = × × × =
10 4 5 5 250
If number of horses is double than that of the
number of elephants then 38. (a); Let, total no. of sheets be 84x,
1
Number of horses = 2x Then no. of red sheets = × 84x = 28x
3
Number of elephant = x 2
ATQ no. of blue sheets = 7 × 84x = 24x
2x 1
C2
=
9 no. of white sheets = 4 × 84x = 21x
7xC 119
2
2x(2x–1) 9 Now,
= 28x + 24x + 21x + 22 = 84x
7x(7x–1) 119
x=5 or, 11x = 22
or, x = 2
33. (b);Probability of choosing a black ball from bag 1 Total no. of sheets = 84x = 168
2 3 1
=6×7=7 There are 12 multiples of 13 and 9 multiples of 17
Probability of choosing a black ball from bag 2 from 1 to 168.
4 4 8
= 6 × 7 = 21 Also, there is no multiple of 13 and 17 from 1 to
1 8 11 168. (because, 13× 17 = 221)
Total probability = 7 + 21 = 21 21 1
Hence, required probability= 168 = 8
34. (c); Probability of choosing a white ball from bag 1
2 4 8 39. (b);Total no. of sheets = 168
=6×7 = 42
40. (e);
Probability of choosing a white ball from bag 2 =
4 3
6
×7
12
= 42
10
Total probability = 21
6
35. (e); Probability that India wins against England = 10
When two places fixed so we can arrange other 8
Condition applied in → (8!) ways and captain and vice-captain
3
Probability that Kohli makes century = 5 Total ways ⇒ 8! ⇒ 40,320
106 Adda247 Publications For More Study Material
Visit: adda247.com
Cracker Book for Bank (IBPS | SBI | RRB PO | Clerk) Mains Exams

41. (e); 35 34 35×21


46. (a); Required probability = 56 × 55 + 2 × 56×55
17 21 19
= 44 + 44 = 22

47. (c); Required probability


1 21 1 35 3 5 19
= 2 × 56 + 2 × 84 = 16 + 24 = 48

3 places fixed ⇒ Direction (48– 50): In bucket P –


So total place ⇒ 10 – 3 = 7 Given, y = x + 3 … (i)
3y
Number of way to arrange ⇒ 7! x= 8 … (ii)
Captain and vice-captain change their place in two By solving (i) & (ii) we get
ways Mangos (x) = 5
So, total ways ⇒ 7 ! × 2 = 10,080 Bananas (y) = 8
Solutions (42-44) Let the number of Orange be z.
z 3
Balls = 10 Ribbon = 10 Plates = 10 Then, =
13+z 16
Red = 2 Red = 5 Red = 3 16z = 39 + 3z
Yellow = 3 Yellow = 2 Yellow = 5 z=3
Blue = 5 Blue = 3 Blue = 2 In bucket Q –
140
Mangos = 5 × 100 = 7
42. (d);Performance with only red color
Let the number of Bananas be ‘a’ and number of Orange
One ball of red color, one ribbon of red color and
be ‘b’
one plate of red color
2 5 3
7 + a + b = 29
Required Probability = 10 × 10 × 10 a + b = 22
3 a
C2 18
= 100 = 0.03 29C = 203
2
a×(a−1) 18
43. (c); Condition is yellow color for ribbon while plate 2×406
=
203
and ball can be of any color so, probability of a × (a − 1) = 72
choosing ball and plate is 1 while probability of Bananas (a) = 9
choosing one yellow color ribbon is Orange (b) = 12
2 In bucket R –
= = 0.2
10
Let Mangos & Orange be 3x & 4x respectively
3x
44. (b);Now C1
=
3
33+ 3xC1 14
Joker choose → 3 balls, 5 Ribbon and 1 plate 3x 3
Condition → all are of some color =
33+3x 14
⇒ 5 ribbons of same color is only of red color 14x = 33 + 3x
But, 3 red ball cannot be possible so x=3
Required Probability = 0 Mangos = 9
Solution (45-47) : - Let fresh eggs in tray A = 100x Orange = 12
= Rotten eggs in tray B Bananas = 33 – (9 + 12 ) = 21
Rotten eggs in tray A = 60x 48. (b); Total bananas in all three buckets
Fresh eggs in tray B = 100x + 14 = 8 + 9 + 21 = 38
Total eggs = 100x + 100x + 60x + 100x + 14 =
5+7
140 49. (e); Required probability = 29+5
360x = 126 6
= 17
⇒ x = 0.35
Tray A Tray B 3
50. (a); In bucket R Good orange = 12 × = 9
Fresh Rotten Fresh Rotten 4

35 21 49 35 Total fruits = 9 + 21 + 9
= 39
49−35
45. (d);Required % = 35
× 100 Probability =
9
39
14
= × 100 = 40% =
3
35
13

107 Adda247 Publications For More Study Material


Visit: adda247.com
Cracker Book for Bank (IBPS | SBI | RRB PO | Clerk) Mains Exams

Chapter

12 Inequality
BEST APPROACH TO SOLVE THE QUESTIONS
In examination, we come across questions in which we need to find two quantities and compare them to reach at the
correct answer. Usually such questions are asked in two ways. In one type of questions, a common data is provided
based on which we are supposed to find both quantities. In other type of questions each quantity comes up with some
data pertaining to it only.
In these questions, it’s important not to mingle or use data of one quantity in finding other quantity in cases where
separate data is given for both the quantities. Where common data is provided for both quantities, we have to use it for
calculating both the quantities. After calculating, we compare both quantities and choose the correct answer from the
options available.
Example: A and B starting a business together. B invested 60% more than that of A. Ratio of time period of investment
of A and B is 8 : 9.
Quantity I: A’s share of profit out of total profit of Rs. 25,200.
Quantity II; B’s share of profit out of total profit of Rs. 14,000.
(a) Quantity I > Quantity II (b) Quantity I < Quantity II
(c) Quantity I ≥ Quantity II (d) Quantity I ≤ Quantity II
(e) Quantity I = Quantity II or No relation
Solution: (e); Ratio of profit share of A and B
x×8 x×1.6×9
5 : 9
5
Quantity I: A’s profit = 25,200 × = 9000
14
9
Quantity II: B’s profit = 14,000 × = 9000
14
Quantity I = Quantity II
Over the years, questions that test a candidate’s basic algebraic knowledge have been and are being asked. In these
questions, two algebraic equations are provided. The equation may be linear, quadratic or cubic. At times, we may come
across equations containing higher powers of variable, but they aren’t too difficult to solve.
These equations generally contain one variable. From the given couple of equations we need to find the values of
two variables. Usually one of the equations gives us the value of one of the variables and the other gives the value of the
other variable. After finding the values of two variables we compare them and select the correct option accordingly.
Example: I. 21x² – 22x + 5 = 0
II. 63y² – 54y + 11 = 0
Solution (a); I. 21x² – 22x + 5 = 0
⇒ 21x² – 15x – 7x + 5 = 0
⇒ 3x (7x – 5) – 1 (7x – 5) = 0
⇒ (3x – 1) (7x – 5) = 0
1 5
⇒ 𝑥 = 3 or 7
II. 63y² – 54y + 11 = 0
⇒ 63y² – 21y – 33y + 11 = 0
⇒ 21y (3y – 1) – 11 (3y – 1) = 0
⇒ (21y – 11) (3y – 1) = 0
11 1
⇒𝑦= or
21 3
∴ no relation can be established.

108 Adda247 Publications For More Study Material


Visit: adda247.com
Cracker Book for Bank (IBPS | SBI | RRB PO | Clerk) Mains Exams

Practice Exercise Based on new Pattern

PART A (a) Quantity I > Quantity II


(b) Quantity I < Quantity II
1. B is thrice as efficient as C. B and C can compete a (c) Quantity I ≥ Quantity II
work together in 45/2 days. A takes 50% more days (d) Quantity I ≤ Quantity II
than the days taken by A and B to complete the same (e) Quantity I = Quantity II or no relation
work together. 5. Quantity I – Veer takes two times more than the time
Quantity 1: No. of days taken by fastest among them taken by Sameer to complete a work. Sameer and veer
to complete the work alone. started to work alternatively starting with Sameer on
Quantity 2: Time taken by A and C to complete the first day and Veer on second day. If both completed
work together. work in 36 days, then find in how many days Sameer
and Veer will complete the work together, if both
(a) Quantity I > Quantity II
work with their double efficiency?
(b) Quantity I < Quantity II
(c) Quantity I ≥ Quantity II Quantity II – Tap P can fill a swimming pool in 8 days
and tap Q can fill the same swimming pool in 24 days,
(d) Quantity I ≤ Quantity II
while tap R can empty the swimming pool in 16 days.
(e) Quantity I = Quantity II or No relation If all three taps are opened together in swimming
2. Quantity I→ The profit earned by selling an item (in pool, then in how much time swimming pool will
2 filled completely?
Rs) if the difference b/w the SP and the CP is 117 % (a) Quantity I > Quantity II
3
of 600. (b) Quantity I < Quantity II
Quantity II→ The cost price of an article (in Rs) if the (c) Quantity I ≥ Quantity II
selling price of the article is 1000 Rs and he got 25% (d) Quantity I ≤ Quantity II
(e) Quantity I = Quantity II or no relation
profit after selling the item.
(a) Quantity I > Quantity II 6. Quantity I: P alone can do a piece of work in 72 days
(b) Quantity I < Quantity II and Q alone can do same work in 48 days, R is 50%
more efficient than Q then find in how many days
(c) Quantity I  Quantity II
total work will be completed if all three work on
(d) Quantity I = Quantity II alternate days starting with P and ending with R ?
(e) No relation
Quantity II: A alone can do a task in 48 days B is 25%
3. The largest possible right circular cylinder is cut out less efficient than A and C can do the same task in 40
from a wooden cube of edge 7 cm. days lesser than B does. Then find in how many days
Quantity I: volume of the cube left over after cutting all three will complete the task together?
(a) Quantity I > Quantity II
out the cylinder
(b) Quantity I < Quantity II
Quantity II: Surface area of cube remained after (c) Quantity I ≥ Quantity II
cutting out the cylinder. (d) Quantity I ≤ Quantity II
Note: compare the magnitudes of both quantities. (e) Quantity I = Quantity II or no relation
(a) Quantity I > Quantity II 7. Quantity I: A bucket carried one dozen of mangoes.
(b) Quantity I < Quantity II One third of mangoes become bad, if three mangoes
(c) Quantity I ≥ Quantity II taken out from the bucket at random. Then what is
(d) Quantity I ≤ Quantity II probability that one mango out of three mangoes
(e) Quantity I = Quantity II or No relation picked up is good?
Quantity II: Find the probability of selecting a face
4. Quantity I- ‘Area of rectangle’ Sum of circumference card from a well shuffled pack of card given that 2
of circle and perimeter of rectangle is 220 cm. and additional Joker card also counts as a face card?
area of circle is 1386 sq. cm. If length of rectangle is (a) Quantity I > Quantity II
1
33 3 % more than radius of circle. (b) Quantity I < Quantity II
Quantity II- ‘Perimeter of square’ Circumference of (c) Quantity I ≥ Quantity II
a circle is 132 cm and area of circle is 710 sq. cm more (d) Quantity I ≤ Quantity II
than area of square (e) Quantity I = Quantity II or no relation

109 Adda247 Publications For More Study Material


Visit: adda247.com
Cracker Book for Bank (IBPS | SBI | RRB PO | Clerk) Mains Exams

8. Quantity I – A train running at the speed of 72 km/hr 11. Quantity I – What is value of (A + B) if 25% of 48% of
passed a man in 14 sec and a platform in 32 sec. Find 55% of A is 1188 and 21% of 35% of B is equal to 5
the length of platform ? more than square of 32.
Quantity II – A train moves at the speed of 108 Quantity II – Ratio of male population to female
km/hr, passes a platform and a bridge in 15 sec and population of a town in the year 2016 was 8 : 9. If in
18 sec respectively. If the length of platform is 50% of the year 2017 male population increased by 25 % and
length of bridge, then find the length of train. female population increased by 40% as compared to
(a) Quantity I > Quantity II previous year so total population of town in the year
(b) Quantity I < Quantity II 2017 was 45200. What was the total population of
town in 2016.
(c) Quantity I ≥ Quantity II
(a) Quantity I > Quantity II
(d) Quantity I ≤ Quantity II
(b) Quantity I < Quantity II
(e) Quantity I = Quantity II or no relation (c) Quantity I ≥ Quantity II
9. Quantity I — Difference between the speeds of P and (d) Quantity I ≤ Quantity II
Q if 2 places A and B are 60 km apart. P and Q start (e) Quantity I = Quantity II or no relation
from A at same time & meet 1st time at a place 12 km 12. Quantity I – A man have two bucket which contains
from B & they reach A after immediate return from B. some fruits, first bucket contains 5 mango and 7
The speed of slower person is 48 km/hr. banana and second bucket contains 4 mango and 5
banana. Also man have a pack of six card with
Quantity II — Average speed of train if a distance of numbering 1 - 6, if a card is taken out and it is even
600 km is to be covered in 2 parts. In 1 st phase 120 number, then he choose a fruit from first bucket but if
km is travelled by train and rest by car and it took card is odd number, then man choose a fruit from
total of 8 hrs, but if 200 km is covered by train and second bucket. Find the probability of choosing a
rest by car it takes 20 min more. mango.
(a) Quantity I > Quantity II Quantity II – In egg tray carried two dozen of eggs.
(b) Quantity I < Quantity II One third of eggs are rotten, if three eggs taken out
(c) Quantity I ≥ Quantity II from the tray at random. What is probability that two
(d) Quantity I ≤ Quantity II eggs out of three eggs picked up is good?
(e) Quantity I = Quantity II or No relation (a) Quantity I > Quantity II
(b) Quantity I < Quantity II
10. Quantity I — Value of 𝑥, if ABCD is a rectangle and
(c) Quantity I ≥ Quantity II
AB= 10 unit, AD= 6 unit
(d) Quantity I ≤ Quantity II
(e) Quantity I = Quantity II or no relation
13. ‘A’ can complete a work alone in 12 days. A and B
2
together can complete the work in 6 3 days. C takes 5
more days than B to complete the work alone. C is
50% more efficient than D.
Quantity II — value of 𝑦, if volume of the cone is 16𝜋 Quantity I= ‘x’: A, B and C worked for alternate days
unit3 starting from A and finish the work in ‘x’ days
Radius = 4 unit
Quantity II= ‘y’: B and C worked together for 6 days
and left and then D completes the remaining work. ‘y’
is total time taken to complete the work.
(a) Quantity I > Quantity II
(b) Quantity I < Quantity II
(c) Quantity I ≥ Quantity II
(d) Quantity I ≤ Quantity II
(e) Quantity I = Quantity II or No relation
(a) Quantity I > Quantity II
(b) Quantity I < Quantity II 14. Quantity I= ‘x’: A shopkeeper mark up an article 40%
(c) Quantity I ≥ Quantity II above its cost price. At the time of sale, he gave 20%
(d) Quantity I ≤ Quantity II discount. By this, shopkeeper earn Rs 168. ‘x’ is the
(e) Quantity I = Quantity II or No relation difference between mark up price and cost price.

110 Adda247 Publications For More Study Material


Visit: adda247.com
Cracker Book for Bank (IBPS | SBI | RRB PO | Clerk) Mains Exams

Quantity II= ‘y’ : Satish invested Rs.12000 in scheme 18. ‘A’, ‘B’ and ‘C’ together can complete a work in 48 days
‘A’ at 12% S.I P.A. for 4 years and Rs.25,000 in scheme if they work alternatively. ‘A’ is 25% more efficient
1
B at 11% S.I P.A for 2 year. ‘y’ is the difference than ‘B’ who is 33 3 % less efficient than ‘C’.
between the interest earn from both schemes. Quantity I: Difference between days taken by ‘A’
(a) Quantity I > Quantity II alone and ‘C’ alone to complete the work.
(b) Quantity I < Quantity II Quantity II: Days in which ‘A’, ‘B’ and ‘C’ together can
(c) Quantity I ≥ Quantity II complete half of the work.
(d) Quantity I ≤ Quantity II (a) Quantity I > Quantity II
(e) Quantity I = Quantity II or No relation (b) Quantity I < Quantity II
15. ABCD is a rectangle and O is the midpoint of the line (c) Quantity I ≥ Quantity II
EF and diagonal AC. AO=20cm and AB=24cm (d) Quantity I ≤ Quantity II
(e) Quantity I = Quantity II or No relation
19. X started from a point A towards point B. After 2
hours. Y started from B towards A. By the time X
travelled one-fifth of the total distance, Y had also
travelled the same. Y’s speed is thrice of that of X’s
speed.
Quantity I: Difference in time (in hours) taken by X
and Y to reach their respective destinations.
Quantity I: Area of shaded region Quantity II: 12 hours
Quantity II: 98 cm² (a) Quantity I = Quantity II or No relation
(a) Quantity I > Quantity II (b) Quantity I ≤ Quantity II
(b) Quantity I < Quantity II (c) Quantity I ≥ Quantity II
(c) Quantity I ≥ Quantity II (d) Quantity I < Quantity II
(d) Quantity I ≤ Quantity II (e) Quantity I > Quantity II
(e) Quantity I = Quantity II or No relation 20. 5X men can complete a work in
𝑋
days while 2Y men
2
16. Quantity I= ‘x’: Veer distributed Rs.80,000 among his 4𝑌
can complete same work in days.
wife, two daughter and three sons. His wife got 300% 5
Quantity I: Value of ‘Y+20’.
more than the amount every son got. If 30% of the
Quantity II: Value of ‘1.25X'
total amount given to the sons then ‘x’ is the amount
(a) Quantity I ≥ Quantity II
every daughter got (b) Quantity I = Quantity II or No relation
Quantity II= ‘y’: Yogesh start a business with Rs (c) Quantity I > Quantity II
10800. After 4 months veer join him with Rs 37800. (d) Quantity I < Quantity II
After 2 months Amit join them with Rs 28800. After 1 (e) Quantity I ≤ Quantity II
year they got total Rs 54880 as profit. ‘y’ is the share
of Yogesh in profit. 21. Total surface area of a cylinder is 200% more than
(a) Quantity I > Quantity II that of its curved surface area. Volume of cylinder is
(b) Quantity I < Quantity II 2156 𝑐𝑚3
(c) Quantity I ≥ Quantity II Quantity I: Volume of cone whose base radius and
(d) Quantity I ≤ Quantity II height is same as that of radius and height of cylinder
respectively.
(e) Quantity I = Quantity II or No relation
Quantity II: Volume of hemisphere whose radius is
17. Quantity I: Profit share of ‘A’ out of total annual profit
same as that of radius of cylinder.
of Rs. 56,500. A, B and C enter into a partnership. ‘A’
(a) Quantity I > Quantity II
invests Rs. 4000 for the whole year, ‘B’ puts in Rs.
6000 at the first and increasing to Rs. 8000 at the end (b) Quantity I = Quantity II or No relation
of 4 months, whilst C puts in at first Rs. 8000 but (c) Quantity I ≥ Quantity II
withdraw Rs. 2000 at the end of 9 months. (d) Quantity I < Quantity II
Quantity II: Amount which when lend on C.I. at 20% (e) Quantity I ≤ Quantity II
interest being compounded annually for 3 years, gives 22. When two ships A & B are travelling in opposite
total interest equal to Rs.9100 direction crosses each other in 2 second. Ship B is
(a) Quantity I > Quantity II 50% faster than ship A in still water. On a particular
(b) Quantity I ≥ Quantity II day, ship A was ahead of ship B by 9 meters and both
(c) Quantity I < Quantity II of them were travelling in downstream. if Ship B
(d) Quantity I = Quantity II or No relation overtook Ship A in 11 seconds. ‘X’ is the speed of ship
(e) Quantity I ≤ Quantity II A and if length of ship B is 54 meters.
111 Adda247 Publications For More Study Material
Visit: adda247.com
Cracker Book for Bank (IBPS | SBI | RRB PO | Clerk) Mains Exams

Quantity I: 16 m/s 26. Quantity I – Center of semicircle B is O1 and radius is


Quantity II: Value of ‘X’ in m/s 28 cm. find area of shaded figure. BO1 is
(a) Quantity I = Quantity II perpendicular to AC
(b) Quantity I ≥ Quantity II
(c) Quantity I ≤ Quantity II
(d) Quantity I > Quantity II
(e) Quantity I < Quantity II
23. Quantity I= 𝒙 Quantity II – Length of a rectangle is two times of
side of square, while ratio between side of square to
breadth of rectangle is 8 : 7. If perimeter of rectangle
is 28 cm more than perimeter of square, then area of
rectangle.
(a) Quantity I > Quantity II
(b) Quantity I < Quantity II
(o is the center of the circle) (c) Quantity I ≥ Quantity II
Quantity II = 𝒚 (d) Quantity I ≤ Quantity II
(e) Quantity I = Quantity II or no relation
27. 5y² + 21y + 18 = 0 and
16(𝑥+2) ÷ 4(𝑥+3) = 64(𝑥+3) × 4(𝑥+4)
Quantity I: Value of y.
Quantity II: Value of x.
(a) Quantity I > Quantity II
(a) Quantity I ≥ Quantity II (b) Quantity I < Quantity II
(b) Quantity I > Quantity II (c) Quantity I ≥ Quantity II
(c) Quantity I ≤ Quantity II (d) Quantity I ≤ Quantity II
(d) Quantity I = Quantity II (e) Quantity I = Quantity II or no relation
(e) Quantity I < Quantity II
28. Quantity I: Find the cost of B per kg if a product C is
24. ‘B’ kg of Rs. 36 per/kg rice that mixed with 8 kg of Rs. made of two ingredients A and B in the proportion of
42 per/kg rice, in order to earn profit of 10% while 2 : 5. The price of A is three times that of B. The
selling the mixture at Rs. 44 per/kg. overall cost of C is Rs. 5.20 per kg including labour
Quantity I: Value of ‘B+20’. charges of 80 paisa per kg.
Quantity II: Value of ‘6B'
Quantity II: Find the cost of manufacturing each
25. Quantity I –Vessel A contains (Q + 36) liter mixture of article if a manufacturer estimates that on inspection
milk & water in the ratio of 7 : 2, while vessel B 12% of the articles he produces will be rejected. He
contains (2Q + 42) liter mixture of milk & water in the accepts an order to supply 22,000 articles at Rs. 7.50
ratio of 2 : 3. If 40% & 46% of mixture from vessel A each. He estimates the profit on his outlay including
and B taken out respectively, then remaining mixture the manufacturing of rejected articles, to be 20%.
in vessel B is 150% of remaining mixture in vessel A. (a) Quantity I > Quantity II
Find the total initial quantity of milk in mixture of (b) Quantity I < Quantity II
vessel A & vessel B together? (c) Quantity I ≥ Quantity II
(d) Quantity I ≤ Quantity II
Quantity II – Two vessels contains mixture of mango
(e) Quantity I = Quantity II or no relation
juice and orange juice in the ratio of 5 : 3 and 5 : 4
respectively. If 40 liter mixture from first vessel taken 29. Quantity I: Value of x
out and mixed in second vessel, so new ratio of mango
juice and orange juice in second vessel becomes 25 :
19. Find initial quantity of mixture in second vessel?
(a) Quantity I > Quantity II
(b) Quantity I < Quantity II
(c) Quantity I ≥ Quantity II
Quantity II: Value of y
(d) Quantity I ≤ Quantity II (20)3 +32 ×103 +25×50
(e) Quantity I = Quantity II or no relation = 𝑦+1
54

112 Adda247 Publications For More Study Material


Visit: adda247.com
Cracker Book for Bank (IBPS | SBI | RRB PO | Clerk) Mains Exams

(a) Quantity I > Quantity II (a) Quantity I > Quantity II


(b) Quantity I < Quantity II (b) Quantity I < Quantity II
(c) Quantity I ≥ Quantity II (c) Quantity I ≥ Quantity II
(d) Quantity I ≤ Quantity II (d) Quantity I ≤ Quantity II
(e) Quantity I = Quantity II or No relation (e) Quantity I = Quantity II or no relation

30. Quantity I: What is probability of getting two red 34. Quantity I – A shopkeeper has two articles A & B.
Marked price of article B is 20% more than marked
balls when they are drawn from a bag which contains
price of article A, shopkeeper sold article A at 25%
3 Red balls, 5 blue balls and 4 white balls. discount and article B at 20% discount. He made 20%
Quantity 2: Value of x. 2
1 2 1 1 3 11 loss on article A and 6 % 𝑝𝑟𝑜𝑓𝑖𝑡 on article B. If total
3
5 +5 −5 = +5 −
2 7 14 𝑥 14 2 loss of shopkeeper was Rs. 765, then find marked
(a) Quantity I > Quantity II price of article B?
(b) Quantity I < Quantity II
(c) Quantity I ≥ Quantity II Quantity II – A shopkeeper gives a discount of 24%
(d) Quantity I ≤ Quantity II on marked price of Shirt and cost price of Jeans is
(e) Quantity I = Quantity II or No relation 25% more than selling price of Shirt. If shopkeeper
sold Jeans at 10 % profit and selling price of Jeans was
31. Total surface area of cube and sphere are equal. Rs. 1140 more than selling price of Shirt , then find
Quantity I: Volume of cube ÷ √𝜋 the cost price of article Jeans ?
Quantity II: volume of sphere (a) Quantity I > Quantity II
(a) Quantity I > Quantity II (b) Quantity I < Quantity II
(b) Quantity I < Quantity II (c) Quantity I ≥ Quantity II
(c) Quantity I ≥ Quantity II (d) Quantity I ≤ Quantity II
(d) Quantity I ≤ Quantity II (e) Quantity I = Quantity II or no relation
(e) Quantity I = Quantity II or No relation 35. Quantity I – A bag contains four green balls, three red
32. Figure I and II shows two semicircles of radius 14 cm balls and five blue balls. If three balls taken out at
with O and P as their centres respectively. random what is probability of at least one ball is
green and at least one ball is blue color.
Quantity II – There are five red toys and six green
toys in a cartoon. What will be the probability of
selection of four toys which contains at least two
Quantity I: Area of shaded region in figure I green toys?
Quantity II: Area of triangle in figure II (a) Quantity I > Quantity II
(a) Quantity I > Quantity II
(b) Quantity I < Quantity II
(b) Quantity I < Quantity II
(c) Quantity I ≥ Quantity II
(c) Quantity I ≥ Quantity II
(d) Quantity I ≤ Quantity II (d) Quantity I ≤ Quantity II
(e) Quantity I = Quantity II or No relation (e) Quantity I = Quantity II or no relation

33. Quantity I – A can complete a task in 24 days and B 36. Quantity I – Six years ago ratio between age of Rohit
can do same task in 18 days. another two persons C & and Prakash was 7 : 8, while six years hence ratio
1 1 1
D complete 583 % of the same task in 7 days and between 6 𝑡ℎ of Rohit age and 3 𝑟𝑑 of Prakash age will
efficiency of D is 40% more than that of C. Find in be 9 : 20. The age of Rohit two years hence will be.
how many days A, B and D will be complete the task
together? Quantity II – Ratio between age of A, B and C is 16 : 9
: 7. Three years hence average of all three age will be
Quantity II – Ankit is 60% less efficient than Satish
35 years. The age of A two years hence will be?
and complete a piece of work in 22.5 days. Ankit and
(a) Quantity I > Quantity II
Saatish start work together and after 4.5 days both
(b) Quantity I < Quantity II
left the work, if veer complete the remaining of work
(c) Quantity I ≥ Quantity II
in 4.5 days then find in how many days the whole
(d) Quantity I ≤ Quantity II
work will be completed, if all three work together ?
(e) Quantity I = Quantity II or no relation
113 Adda247 Publications For More Study Material
Visit: adda247.com
Cracker Book for Bank (IBPS | SBI | RRB PO | Clerk) Mains Exams

37. A diagram is given below. AC is the diameter of circle. (a) Quantity I > Quantity II
AD || BC and ∠𝐴𝐶𝐷 = 90° ∠𝐴𝐶𝐵 ≤ 45° (b) Quantity I < Quantity II
(c) Quantity I = Quantity II or No relation
(d) Quantity I ≥ Quantity II
(e) Quantity I ≤ Quantity II
41. Quantity I: ‘x’ : A and B can do a work in 24 days
together, in which 40% of work is done by B. C who is
Quantity I: -. ∠𝐴𝐷𝐶 25% more efficient than A, do the same work alone in
Quantity II: - 45° ‘x’ days.
(a) Quantity I > Quantity II
(b) Quantity I < Quantity II Quantity II: ‘y’ : P, Q and S work for x, x+1 and x+2
(c) Quantity I ≥ Quantity II days and their efficiency are in the ratio of x : x+1 :
(d) Quantity I ≤ Quantity II x+2 respectively. If P got 16 Rs for his work, then ‘y’ is
(e) Quantity I = Quantity II or No relation the total money earned by all three in Rs.. (Compare
38. Quantity I: ‘x’: There is an interview conducted by Magnitude of x and y)
HCL. Out of total applicants six are female. Probability (a) Quantity I > Quantity II
of selecting two candidates such that at least one is (b) Quantity I ≥ Quantity II
7
male is . ‘x’ is total number of males. (c) Quantity I = Quantity II or No relation
8
(d) Quantity I < Quantity II
Quantity II: ‘y’ : A, B and C together start a business. (e) Quantity I ≤ Quantity II
Ratio between investment of A and B is 15 : y while
ratio between investment of B & C is 9 : 11. A left 42. Quantity I:’x’ -. Two circles are concentric with center
business after 8 months of starting of business and B ‘O’. Their radii are 8cm and 10cm respectively. ‘B’ and
left before 7 months of completion of year. Out of total ‘C’ are the points of contact of two tangents drawn
profit of Rs. 2080, A got Rs. 900. from bigger circle to smaller circle from point ‘A’ lying
(a) Quantity I > Quantity II on bigger circle. ‘x’ is area of quadrilateral ABOC
(b) Quantity I < Quantity II
formed in 𝑐𝑚2
(c) Quantity I ≥ Quantity II
(d) Quantity I ≤ Quantity II Quantity II: - ‘y’ - The lengths of two parallel sides of
(e) Quantity I = Quantity II or No relation a trapezium are 6 cm and 8 cm. If the height of the
39. Quantity I: ‘x’: Two mutually perpendicular chords trapezium be 6 cm, then its area is ‘y’ 𝑐𝑚2
PQ and EF meet at a point S inside a circle. PS = 12 cm, (a) Quantity I > Quantity II
SQ = 8 cm and FS = 6 cm. ‘x’ is the area of circle. (b) Quantity I < Quantity II
Quantity II: ‘y’: In a circle, two parallel chords are (c) Quantity I ≥ Quantity II
drawn on opposite sides of diameter. Distance (d) Quantity I ≤ Quantity II
between these chords is 14cm. Length of one chord is (e) Quantity I = Quantity II or No relation
16cm while length of another chord is 12cm. ‘y’ is the
43. Quantity I: ‘x’ -. Kishan invested Rs. 20,000 in a bank
area of circle
offering 22% p.a. at Simple interest. After 2 years he
(a) Quantity I > Quantity II
(b) Quantity I < Quantity II withdraws his money and invested in another bank
(c) Quantity I ≥ Quantity II which is offering ‘x%’ p.a. at compound interest. After
(d) Quantity I ≤ Quantity II 3 years, interest earned by him is Rs.1350 less than
(e) Quantity I = Quantity II or No relation amount invested by him in this bank.
Quantity II: - ‘y’ – Bhavya sell a diary at Marked price
40. Quantity I: ‘x’ :A dishonest shopkeeper sales his good 5
by weighting 15% less and cheat his wholesaler by and earns 85 7 % profit while if he gives 'y%’ discount
giving 15% less amount. ‘x’ is his approximate total on Marked price then he will earn ‘y%’ profit.
profit % if he sells it at cost price. (a) Quantity I > Quantity II
Quantity II: ‘y’ : A dishonest milkman, add 20 litre (b) Quantity I < Quantity II
water in pure milk, after selling half of mixture. He
(c) Quantity I ≥ Quantity II
again added 20 liter of water in remaining mixture
(d) Quantity I ≤ Quantity II
now milk and water ratio in mixture becomes 4 : 3. ‘y’
is his overall profit % on selling total mixture. (e) Quantity I = Quantity II or No relation

114 Adda247 Publications For More Study Material


Visit: adda247.com
Cracker Book for Bank (IBPS | SBI | RRB PO | Clerk) Mains Exams

44. Quantity I —‘x’ : B alone can do the work in ‘x’ days. (a) Quantity I > Quantity II
A can complete a work in 5 more days than B while A (b) Quantity I < Quantity II
does the same work in 9 more days than C. If A and B (c) Quantity I ≥ Quantity II
can complete the whole work in same time as time (d) Quantity I ≤ Quantity II
taken by C alone to do the whole work. (e) Quantity I = Quantity II or No relation

Quantity II —‘y’ : ‘y’ is the days taken by 8 men and 48. Quantity I: ‘x’: Sum of ‘𝒙′ consecutive number is
7 4040. First number is 11.
14 women to reap 12 𝑝𝑎𝑟𝑡 of 360-hectare land by Quantity II: ′𝒚′: Sum of ′𝒚′ consecutive even numbers
working 7 hrs per day if 6 men and 10 women can is 7120. First number is 10.
5 (a) Quantity I > Quantity II
reap part of the land in 15 days by working 6 hrs
12 (b) Quantity I < Quantity II
per day. It is also given that work of 2 men is equal to (c) Quantity I ≥ Quantity II
that of 3 women. (d) Quantity I ≤ Quantity II
(a) Quantity I > Quantity II (e) Quantity I = Quantity II or No relation
(b) Quantity I < Quantity II
(c) Quantity I ≥ Quantity II 49. Quantity I: ‘x’: A man invested 𝑹𝒔. ′𝒙′ in two scheme
(d) Quantity I ≤ Quantity II which offers 40% S.I. for 2 years and 44% S.I. for 5
years respectively. Difference between interest
(e) Quantity I = Quantity II or No relation
earned from both scheme is 18,900.
45. Quantity I — ‘x’ : ‘x’ is the difference between the Quantity II: ′𝒚′: Interest earned in a scheme is ′𝑦′
speeds of P and Q. Distance between A and B are 60 which offers 12% rate of interest for 3 years on a sum.
km. P and Q start from A at same time & meet 1st time 30% of the sum is 12000.
at a place 12 km from B. They return to A immediately (a) Quantity I > Quantity II
after reaching B. The speed of slower person is 48 (b) Quantity I < Quantity II
km/hr. (c) Quantity I ≥ Quantity II
(d) Quantity I ≤ Quantity II
Quantity II —‘y’ : ‘y’ is the average speed of train if a
(e) Quantity I = Quantity II or No relation
distance of 600 km is to be covered in 2 parts. In 1st
phase 120 km is travelled by train and rest by car and 50. The largest sphere that can fit in a cube of edge 7 cm.
it took total of 8 hrs, but if 200 km is covered by train Quantity I: ‘x’: ‘x’ is the volume of vacant space left
and rest by car it takes 20 min more. over after putting the sphere.(𝑖𝑛 𝑐𝑚3 )
(a) Quantity I > Quantity II Quantity II: ′𝒚′: ’y’ is the surface area of the sphere.
(b) Quantity I < Quantity II (in 𝑐𝑚2 )
(c) Quantity I ≥ Quantity II (a) Quantity I > Quantity II
(b) Quantity I < Quantity II
(d) Quantity I ≤ Quantity II
(c) Quantity I ≥ Quantity II
(e) Quantity I = Quantity II or No relation (d) Quantity I ≤ Quantity II
46. Quantity I: ‘x’ : ‘𝒙′ is the number of days taken by A (e) Quantity I = Quantity II or No relation
alone to complete a work. A and B together can PART B
complete the same work in 16 days while A and C can
1
complete the same work in 15 3 days. Ratio of Directions (Q1-5): In each of these questions, two
equations I. and II. are given. You have to solve both the
efficiency of B and C is 3 : 4. equations and give answer
Quantity II: ′𝒚′: P, Q and R takes ‘y’ days to complete 1. I. 3𝑥+5 . 92𝑥−4 = 95𝑥−14
a work. Ratio of efficiency of P, Q and R are 2 : 3 : 4 II. 2𝑦 2 − 15𝑦 − 28 = 3𝑦 2 − 23𝑦 − 13
respectively and P and Q together takes 18 days to (a) x < y
complete that work. (b) x ≥ y
(a) Quantity I > Quantity II (c) No relation can be established between x and y.
(b) Quantity I < Quantity II (d) x ≤ y
(c) Quantity I ≥ Quantity II (e) x > y
(d) Quantity I ≤ Quantity II
25 15
(e) Quantity I = Quantity II or No relation 2. I. − +2=0
x² x
40 13
47. Quantity I: ‘x’ : A article is sold at 100 Rs. profit II. + 1=
y² y
whose cost price is ‘Rs.𝒙’. If article is sold at 20%
(a) x < y
more than its S.P. then S.P becomes 2𝑥.
(b) x ≥ y
Quantity II: ′𝒚′ : Article is sold at y% profit whose (c) No relation can be established between x and y.
M.P. is 400% above its C.P. and discount given on M.P. (d) x ≤ y
is 60%. (e) x > y
115 Adda247 Publications For More Study Material
Visit: adda247.com
Cracker Book for Bank (IBPS | SBI | RRB PO | Clerk) Mains Exams

3. I. 2(y+4) . 12(2y+4) = 9(y+2) . 16(y+4) Directions (11-15): In each of these questions, two
II. x 2 – 10x + 24 = 0 equations I. and II. are given. You have to solve both the
(a) x ≤ y equations and give answer
(b) x > y (a) if x>y
(c) No relation can be established between x and y. (b) if x≥y
(d) x < y (c) if x<y
(e) x ≥ y (d) if x ≤y
48 14 (e) if x = y or no relation can be established between x and
4. I. x2 − x
+1= 0 y.
45 1
II. + =2 11. I. 𝑥 2 − 5𝑥 + 4 = 0
y² y
II. 𝑦 2 + 5𝑦 + 6 = 0
(a) No relation can be established between x and y.
(b) x ≤ y 12. I. 2𝑥 2 − 𝑥 − 15 = 0
(c) x < y II. 3𝑦 2 − 23𝑦 + 42 = 0
(d) x > y
13. I. 𝑥 2 − 15𝑥 + 54 = 0
(e) x ≥ y
II. 𝑦 2 + 15𝑦 − 54 = 0
5. I. (𝑥 − 4)2 = 9
14. I. 𝑥 2 + 14𝑥 + 40 = 0
II. (2𝑦 + 3)2 = 25
II. 𝑦 2 − 5𝑦 − 24 = 0
(a) No relation can be established between x and y.
(b) x < y 15. I. 𝑥 2 − 225 = 0
(c) x ≤ y II. 𝑥 2 + 𝑦 2 = 306
(d) x ≥ y
Directions (16-20): In each of these questions, two
(e) x > y
equations I. and II. are given. You have to solve both the
Directions (6-10): In each of these questions, two equations and give answer
equations I. and II. are given. You have to solve both the (a) if x>y
equations and give answer (b) if x≥y
(a) if x>y (c) if x<y
(b) if x≥y (d) if x ≤y
(c) if x<y (e) if x = y or no relation can be established between x and
(d) if x ≤y y.
(e) if x = y or no relation can be established between x
16. I. 2x² – 7x + 6 = 0
and y.
II. 3y² – 19y + 28 = 0
6. I. 5x² + 3x – 36 = 0
17. I. x² – 13x + 36 =0
II. 2y² – 13y + 20 = 0
II. 3y² – 19y + 30 = 0
7. I. x² – 7x + 12 = 0
18. I. x³– 120 = 96
II. 2y² – 11y + 15 = 0 25
II. y – =0
8. I. 2x² + 11x + 15 = 0 𝑦

II. 2y² + 9y + 10 =0 19. I. x² – 2x – 48 = 0


9. I. 3x² + 7x – 40 = 0 II. y² – 15y + 54 = 0
II. 5y² – 29y + 42 = 0
20. I. 2x² + 25x + 72 = 0
10. I. 3x² – 23x + 42 = 0
II. 3y² + 22y + 40 = 0
II. 2y² – 19y + 45 = 0

Solutions

1. (a); Suppose B takes ‘x’ days, 1 1 1


+ =
3𝑦 30 2𝑦
Then C will take ‘3x’ days
1 1 1 Or, y = 5
+ = Hence, A will take 3y = 3 × 5 = 15 days.
3𝑥 𝑥 22.5
Or, x = 30 Quantity I = 15
Now, Let A and B together take ‘2y’ days, 90
Quantity II = 6+1 = 90/7
Then A will take ‘3y’ days, Quantity I > Quantity II
116 Adda247 Publications For More Study Material
Visit: adda247.com
Cracker Book for Bank (IBPS | SBI | RRB PO | Clerk) Mains Exams
117×3+2
2. (b); Quantity I→ × 600 = 706 Rs Quantity II—
300
25𝑥
Quantity II→ 𝑥 + 100 = 1000
100
𝑥 = 125 × 1000 = 800 Rs
∴ Quantity I < Quantity II
3. (b); Quantity I: Volume of cube left When all three opened together
7 2 Total work in one day = (6 + 2 – 3) = 5 units
= 73 − 𝜋 (2) × 7 48 3
22 49×7
Required time = 5
= 9 5 days
= 343 − ×
7 4 Quantity I < Quantity II
= 343 − 269.5
6. (a); Quantity I -
= 73.5 cm3
Quantity II: Surface area of cube left
7 2 7
= 6 × 72 − 2. 𝜋 (2) + 2𝜋 (2) 7
= 294 − 77 + 154
150
= 371 cm2 R efficiency = 3 × 100 = 4.5 w⁄d
Quantity II > Quantity I 3 days work = (P + Q + R)
= 2 + 3 + 4.5 = 9.5 work
4. (a); Quantity I – 2πr + 2 (ℓ + b) = 220 cm
Required days = 9.5 × 15 = 142.5 work
πr² = 1386 sq. cm Remaining work = 144 – 142.5 = 1.5
1386×7
r2 = 22
1.5
Next day by P = 2 = 4
3

r = 21 cm Total days = 15 × 3 + = 45 days


3 3

Length of rectangle 4 4
4
= 21 × 3 = 28 cm Quantity II. - A : B = 100 : 75 = 4 : 3
22
Total work = 4 × 48 = 192
2× × 21 + 2(28 + b) = 220 B alone =
192
= 64 days
7
3
132 + 56 + 2b = 220 C alone = 64 – 40 = 24 days
32 192
b= = 16 cm C efficiency = = 8 w⁄d
2 24
Area of rectangle = (28 × 16) = 448 cm 192
(A + B + C)together =
(4+3+8)
192 4
Quantity II–2πr = 132 = = 12 days
132×7 15 5
r= 2×22
So, Quantity I > Quantity II
r = 21 cm 7. (b); Quantity I - No. of selection of 3 mango out of
πr² – a² = 710 one dozen mangoes = ¹²C₃ = 220
22 No. of selection of 2 bad mango out of 4 bad
7
× 21 × 21 – a2 = 710
mangoes = ⁴C2 = 6
a² = 1386 – 710 No. of selection of a good mango from 8 good
a² = 676 cm mangoes = 8
a = 26 cm 8×6 12
Required probability = 220 = 55
Perimeter of square = 4 × 26 = 104 cm
Quantity II - Face card = 12
Quantity I > Quantity II Joker = 2
14 7
5. (b); Quantity I— Let Veer take 3𝑥 days and Sameer Probability = =
54 27
take 𝑥 days. So, Quantity I < Quantity II
Efficiency of Veer and Sameer be 𝑥 unit/day and
8. (e); Quantity I - Let length of train be L meter
3𝑥 units/day respectively
ATQ—
Total work = 3𝑥 × 18 + 𝑥 × 18 = 72x units 5
If both do with double efficiency Speed = 72 × = 20 m/sec
18
L
Then, 20 =
72𝑥 14
= (3𝑥×2+2𝑥) = 9 days L = 280 meter

117 Adda247 Publications For More Study Material


Visit: adda247.com
Cracker Book for Bank (IBPS | SBI | RRB PO | Clerk) Mains Exams

Let length of platform be P meter B = 19600


P+280 (A + B) = 18000 + 19600 = 37600
20 = 32
P = 640 – 280 Quantity II— Let male and female population of
P = 360 meter town in 2016 be 8x and 9x respectively
125 140
𝐐𝐮𝐚𝐧𝐭𝐢𝐭𝐲 𝐈𝐈 – Let’s length of train is X meter and 8𝑥 × 100 + 9𝑥 × 100 = 45200
63𝑥
length of bridge and platform are 2L and L meter 10𝑥 + = 45200
5
respectively 113𝑥 = 226000
5
Speed of train = 108 × 18 = 30 m⁄s 𝑥 = 2000
When train passed the bridge Total population of town in 2016 = 16000 +
Distance covered = X + 2L 18000
X + 2L = 540 …(i) = 34000
When train passes the platform Quantity I > Quantity II
Distance covered = X + L 12. (a); Quantity-I : Probability of choosing Mango
X + L = 450 …(ii) 1 5 1 4 31
= × + × =
From (i) and (ii) 2 12 2 9 72
L = 90 meter Quantity-II : Total no. of cases to choose 3 egg =
X = 360 meter ²⁴C₃
So, Quantity I = Quantity II Favourable cases = ¹⁶C₂ × ⁸C₁
16C 8
2 × C1 15
Probability = =
9. (b); 24 C
3 253
Quantity I > Quantity II
13. (e); A can complete work in= 12 days
3 1 9−5 4
‘B’ 1 day work is = − = =
Let P is faster than Q 20 12 60
60
60
Then P covers 72 km distance in the same time as B can complete the work in = = 15 days
4
Q covers 48 km distance C can complete the work in ‘20’ days
Ratio of the speed = 72 : 48 = 3 : 2 20×150
D can complete the work in 100 = 30 days
48
∴ Speed of faster train i.e., P = 2 × 3 = 72 km/hr
Quantity I: ‘A + B + C three days’ work
Quantity 1→ Difference between P and Q 1 1 1 5+4+3 12 1
= + + = = =
12 15 20 60 60 5
= 72 – 48 = 24 km/hr.
‘A + B + C’ can complete the work in total ’15 days
Let speed of train = T km/hr
Let speed of car = C km/hr Quantity II: Remaining work for D
120 480 6 6 60−24−18 3
∴ + = 8 ………..(i) = 1 − 15 − 20 = 60
= 10
𝑇 𝐶
200 400 1 3 30×3
+ 𝐶 = 8 3 ………(ii) of work is completed by D in
𝑇 10 10
On solving (i) and (ii) = 9days
T = 60 km/hr Total time = 6 + 9 = 15 days
∴ Quantity I < Quantity II Quantity I = Quantity II
6 14. (a); Quantity I: let C.P = 100
10. (b); Quantity I → 2 = 3
1
Mark Up price = 140
And, 3 𝜋𝑟 2 ℎ = 16𝜋 80
Selling price = 140 × 100 = 112
ℎ=3 ATQ,
∴ 𝑦 = √42 + 32 = √16 + 9 12 → 168
𝑦=5 x = 40 → 560
∴ Quantity I < Quantity II x = 560
25 48 55
11. (a); Quantity I— 𝐴 × 100 × 100 × 100 = 1188 Quantity II: Interest from scheme A
12000×12×4
1188×100×100×100 = = 5760
A= 100
25×48×55
Interest from scheme B
A = 18000 25000×11×2
21 35 = = 5500
𝐵× × = (32)2 + 5 100
100 100 y= 5760 – 5500 = 260
21 25
𝐵× × = 1024 + 5 x>y
100 100
1029×100×100 Quantity I > Quantity II
𝐵= 21×25

118 Adda247 Publications For More Study Material


Visit: adda247.com
Cracker Book for Bank (IBPS | SBI | RRB PO | Clerk) Mains Exams

15. (b); Quantity I: 19. (d); Let the speed of X be x kmph. Distance travelled
OF=√202 − 12²=√400 − 144 = √256 = 16 by X in 2 hours = 2x km.
1
Area of shaded region Suppose X takes ‘t’ hours to travel 5th of the
Area of ∆AFE – Area of ∆ AFO distance AB.
1 1 1
= × 12 × 32 − × 12 × 16 Y would take (t–2) hours to travel th of the
2 2 5
= 192 – 96 = 96 cm² distance AB.
Quantity II > Quantity I As Y’s speed is thrice that of X’s speed.
𝑡−2 1
16. (a); Quantity I: Total amount sons got = ⇒ t=3
𝑡 3
30 1𝑡ℎ
= 100 × 80000 = 24000 of the distance AB = 3x km.
24000 5
Each son got = = 8000 AB =15x km
3
400 15𝑥
Wife got = 8000 × = 32000 Time taken by x to cover 15x km = = 15
100 𝑥
Total amount daughters got hours
15𝑥
80,000 – 24000 – 32000 = 24000 Time taken by Y to cover 15x km = 3𝑥
24000
Each daughter got = 2 = 12000 = 5 ℎ𝑜𝑢𝑟𝑠.
x = 12,000 ∴ Difference in the time = 10 hours.
Quantity II: Ratio of investment or profit Quantity I : Difference in the time = 10 hours.
= 10800 × 12 : 37800 × 8 : 28800 × 6 Quantity II : 12 hours
=3:7:4 Quantity I < Quantity II
54880
Share of Yogesh in profit = ×3 20. (c); Total work = 5𝑋 × 2 = 2𝑌 ×
𝑋 4𝑌
14
5
y = 11760 𝑋2 16 𝑋 4
x>y ⇒ 2= ⇒ =
𝑌 25 𝑌 5
Quantity I > Quantity II Let X = 4a and Y = 5a
Quantity I: Y+20 = 5a+20
17. (c); ATQ,
Quantity II: 1.25X = 5a
Quantity I: Ratio of profit share
A : B : C
Quantity I > Quantity II
(4000×12) (6000×4)+(8000×8) (8000×9)+(6000×3) 21. (b); T.S.A of cylinder = 2𝜋𝑟(𝑟 + ℎ)
24 : 44 : 45 C.S.A of cylinder = 2𝜋𝑟 2
24
Share of A = 113 × 56,500 = Rs. 12,000 ATQ,
3 2𝜋𝑟(𝑟+ℎ)
Quantity II: 1
= 2𝜋𝑟2
20 3 ⇒ ℎ = 2𝑟
9100 + P = P (1 + 100)
𝑉𝑜𝑙𝑢𝑚𝑒 𝑜𝑓 𝑐𝑦𝑙𝑖𝑛𝑑𝑒𝑟 = 𝜋𝑟 2 ℎ = 2156
⇒ 9100 + P = 1.728P ⇒ 𝑟 = 7𝑐𝑚, ℎ = 14𝑐𝑚
or, P = Rs. 12,500 1 2156
Quantity I: Volume of cone = 𝜋𝑟 2 ℎ =
𝐐𝐮𝐚𝐧𝐭𝐢𝐭𝐲 𝐈𝐈 > 𝐐𝐮𝐚𝐧𝐭𝐢𝐭𝐲 𝐈 3 3
Quantity II: Volume of hemisphere
18. (e); ‘A’, ’B’ and ‘C’ alternatively can complete a work 2 2156
= 3 𝜋𝑟 3 = 3
in 48 days
⇒ ‘A’, ‘B’ and ‘C’ together can complete same Quantity I = Quantity II
work in 16 days 22. (e); Let speed of ship A in still water is 2x m/s , and
Ratio between efficiency of A, B and C is its length is ℓ m
5:4:6 And speed of ship B is 3x m/s and its length is 54
Ratio between days taken by A, B and C alone to and speed of water is Y m/s.
complete the same work is 12 : 15 : 10 ATQ, when both of them are travelling in
Let A, B and C alone can complete work in 12x, opposite direction.
15x and 10x days respectively. Downstream speed of ship A = (2x + Y) m/s
1 1 1 1
⇒ + + = [assume ship A is travelling in downstream and
12𝑥 15𝑥 10𝑥 16
⇒𝑥=4 ship B in upstream]
Quantity I: Required difference Upstream speed of ship B = (3x – Y) m/s
= (12 − 10)4 = 8 𝑑𝑎𝑦𝑠 Their relative speed = 2x + Y + 3x – Y = 5x m/s
Quantity II: A, B and C together can complete ATQ,
54+ℓ
same work in 16 days, so they can complete half =2 …(i)
5x
work in 8 days. Similarly when both are travelling in
Quantity I = Quantity II downstream their relative speed is
119 Adda247 Publications For More Study Material
Visit: adda247.com
Cracker Book for Bank (IBPS | SBI | RRB PO | Clerk) Mains Exams

= 3x + Y – 2x –Y= X m/s 26. (e); Quantity I–Area of semicircle – area of triangle


ATQ, 22 28×28 1
= 7 × 2 − 2 × 56 × 28
54+9+ℓ
𝑥
= 11 = 1232 – 784 = 448 cm²
63 + ℓ = 11x …(ii)
Quantity II – Ratio of Length, breadth of a
From (i)
ℓ = 10x – 54 rectangle & side of square = 16 : 7 : 8
put this value in eqn. …(ii) Let Length, breadth of a rectangle & side of
= 63 + 10x – 54 = 11x square be 16x cm, 7x cm & 8x cm respectively
x = 9 m/s ATQ –
Hence speed of ship A in still water = 2 × 9 = 18 2(16x + 7x) – 4 × 8𝑥 = 28 𝑐𝑚
m/s 46x – 32𝑥 = 28
Quantity I = 16 m/s x = 2 cm
Quantity II = 18 m/s Area of rectangle
Quantity II > Quantity I = (16× 2 ) × (2 × 7) = 448 cm2
Quantity I = Quantity II
23. (b); 𝑥 = 180 − (90 + 30)
= 90 − 30 = 60° 27. (c); Quantity I: 5y² + 21y + 18 = 0
𝑦 = 180 − (60 + 40) (angles subtended by same 5y² + 5y + 15y + 18 = 0
arc in the same segment are equal) Y (5y + 6) +3 (5y + 6) = 0
= 80° (y + 3) (5y + 6) = 0
∴ Quantity I < Quantity II –6
y = –3 or
3
24. (e); Selling mixture at Rs. 44/kg with 10% profit Quantity II: 16(x+2) ÷4(x+3) = 64(x+3)×4(x+1)
means, the actual price of mixture is Rs. 40/kg 24x+8–2x–6 = 26x+18+2x+2
Let B kg of Rs. 36/kg are mixed 8x + 20 = 2x + 2
Then ∴ x = –3.
36𝐵+8×42
= 40 ∴ Quantity I ≥ Quantity II
𝐵+8
36B + 336 = 40B + 320 28. (b); Quantity I: Let the price of B per kg be Rs. X.
4B = 16 Then, the price of A per kg = Rs. 3X
B=4 1kg of C contains 2/7 kg of A and 5/7 kg of B
Quantity I = B + 20 = 24 Price of 1 kg of C = (2/7) × 3X + (5/7)X
Quantity I = 𝟔 × 𝑩 = 𝟐𝟒
= (11/7)X
Quantity I = Quantity II
By the given condition, 11X/7
25. (b); Quantity I – = 5.20 – 0.80 = Rs. 4.40
ATQ – ⇒ X = 4.40 × (7/11) = Rs. 2.80
60 150 54
(Q + 36) × × = [(2Q + 42) × ] Hence the price of B per kg = Rs. 2.80.
100 100 100
3(60Q + 2160) = 2 (108Q + 2268) Quantity II: If 100 articles are manufactured
180Q + 6480 = 216Q +4536 then 12 will be rejected
36Q = 1944 Total selling price of 88 articles = 88 × 75
Q = 54 l 660×100
Total cost price = = 550
Total initial quantity of milk in mixture of vessel 120
A & vessel B cost of manufacturing per article = 5.5 Rs
7 2 Quantity II > Quantity I
= (54 + 36) × 9 + (2 × 54 + 42) × 5
= 70 + 60 = 130 l 29. (b); Quantity I: ∠CAD = 90° – ∠ACD = 62
(x) = ∠CBD = 90° – ∠CAD
Quantity II – x = 28°
ATQ – 8000+9000+1250
Let mixture of mango juice and orange juice in Quantity II: =𝑦+1
625
second vessel be 5y liter and 4y respectively y = 28.2
5𝑦+40×
5
25 5𝑦+25 25 Quantity II > Quantity I
8
3 = ⇒ =
4𝑦×40× 19 4𝑦+15 19 3C 1
8
19y + 95 = 20y + 75 30. (b); Quantity I: 12 2 =
C2 22
y = 20 l 1 1 2 1 3 1
Quantity II: 5 (2 + 2 + 7 − 14 − 14) = 𝑥
Initial quantity of mixture in second vessel = 20 1 1
× 9 = 180 𝑙 5 = 𝑥⇒𝑥 = 5
Quantity I < Quantity II Quantity II > Quantity I

120 Adda247 Publications For More Study Material


Visit: adda247.com
Cracker Book for Bank (IBPS | SBI | RRB PO | Clerk) Mains Exams
4
31. (b); Let side of cube and radius of sphere be a and r S. P. of article B = 120 × 5 = 96x
respectively. 5
C. P. of article A = 75 × = 93.75x
⇒ 6a² = 4πr² 4
15
2
C. P. of article B = 96x × = 90x
16
a = √3 π r ATQ—
(93.75x + 90x) – (75x + 96x) = 765
Quantity I: Volume of cube 183.75x – 171x = 765
=a×a×a 12.75x = 765
2 2 x = 60
=3 π × √3 π r 3 ÷ √𝜋 M.P. of article B = 60 × 120 = 7200 Rs.
3
=(2/3 )2 π r 3 Quantity II – MP of shirt = 100x Rs
76
4
SP of shirt = 100 × = 76𝑥 Rs
100
Quantity II : volume of sphere πr 3 5
3 Cost price of jeans = 76𝑥 × 4 = 95𝑥 Rs
∴ Quantity II > Quantity I 110
Selling price of jeans = 95𝑥 × 100
32. (a); Quantity I: Area of shaded region = 104.5x Rs
Area of circle – Area of triangle ATQ –
Base of triangle = 2 × 14 = 28 104.5x - 76x = 1140 Rs
=
22
×
14×14 1
– × 28 × 14 28.5x = 1140 Rs.
7 2 2 X = 40 Rs.
⇒ 112 cm2 Cost price of jeans = 95×40 = 3800 Rs.
Quantity II: Radius = 14 So, Quantity I > Quantity II
1 35. (b); Quantity I – Required cases = (1 green, 1 red, I
Area of triangle = 2 × 14 × 14 = 98 cm²
blue) or ( 2 green , 1 blue) or (1 green , 2 blue)
Quantity I > Quantity II 4C 3 5
1 × C1 × C1
4C 5
2 × C1
4C 5
1 × C2
33. (a); Quantity I – Total work = 72 units (Lcm of days = 12C + 12C + 12C
3 3 3
3 3 2 13
taken by A & B) = 11 + 22 + 11 =
72 22
Efficiency of A = = 3 𝑢𝑛𝑖𝑡𝑠/𝑑𝑎𝑦
24
72
Quantity II – Favorable case = (2G, 2R) or (3G,
Efficiency of B = = 4 𝑢𝑛𝑖𝑡𝑠/𝑑𝑎𝑦 1R) or 4G
18
Let efficiency of C = x unit/day ∴ Probability
6C 5 6C 5 6C
175 1 2 × C2 3 × C1
7x + 7(1.4x) = 72 × = 42 𝑢𝑛𝑖𝑡𝑠 = 11C + 11C + 11 4
3 100 4 4 C4
15×10 20×5 15
x = 2.5 units = + + 330
330 330
Efficiency of D = 1.4 × 2.5 = 3.5 𝑢𝑛𝑖𝑡𝑠 265 53
72 6 = 330 = 66
(A + B + C) together = (3+4+3.5) = 6 7 𝑑𝑎𝑦𝑠
So, Quantity I < Quantity II
Quantity II – Ratio of efficiency of Satish : 36. (e); Quantity I – Let age of Rohit and Prakash six
Ankit = 100x : 40x = 5x : 2x years ago be 7x years and 8x years respectively
Total work = 22.5 × 2x = 45x units ATQ –
𝟕𝒙+𝟏𝟐
ATQ, 𝟗
(Ankit + Satish) × 4.5 = 4.5 (5x + 2x)
𝟔
𝟖𝒙+𝟏𝟐 =
𝟐𝟎
𝟑
= 31.5x work 140x + 240 = 144x + 216
Remaining work = 45x– 31.5 x = 13.5x x = 6 years
13.5
Veer efficiency = 4.5 = 3 units⁄day Age of Rohit two years hence will be
45x = (7× 6 + 8) = 50 𝑦𝑒𝑎𝑟𝑠
(Ankit + Satish + Veer) =
(5x+2x+3x)
Quantity II – Let age of A, B and C be 16y, 9y
= 4.5 days and 7y respectively.
So, Quantity I > Quantity II ATQ –
34. (a); Quantity I – Let marked price of article (16y + 9y + 7y) = 35 × 3 − 3 × 3
A = 100x 32y = 96
y = 3 years
Marked price of article B = 120x
3
Age of A two years hence = 3 × 16 + 2 = 50 years
S. P. of article A = 100x × 4 = 75x So, Quantity I = Quantity II

121 Adda247 Publications For More Study Material


Visit: adda247.com
Cracker Book for Bank (IBPS | SBI | RRB PO | Clerk) Mains Exams

37. (c); ∠𝐴𝐵𝐶 = 90° (Semicircle property) OE = √125 = 5√5 cm


∠𝐶𝐴𝐵 + ∠𝐴𝐶𝐵 + ∠𝐴𝐵𝐶 = 180° 2
Area of circle = πr²= 𝜋(5√5)
∠𝐶𝐴𝐵 + ∠𝐴𝐶𝐵 = 90° 22
As ∠𝐴𝐶𝐵 ≤ 45° So, ∠𝐶𝐴𝐵 ≥ 45° = 125 ×
7
∠𝐴𝐶𝐷 + ∠𝐶𝐴𝐷 + ∠𝐴𝐷𝐶 = 180° = 392.86 cm ²
∠𝐶𝐴𝐷 + ∠𝐴𝐷𝐶 = 90° Quantity II:
But ∠𝐴𝐶𝐵 = ∠𝐶𝐴𝐷 (AD || BC)
∠𝐴𝐶𝐵 + ∠𝐴𝐷𝐶 = 90°
As ∠𝐴𝐶𝐵 ≤ 45° So, ∠𝐴𝐷𝐶 ≥ 45°
Quantity I ≥ Quantity II
38. (a); Quantity I: Let, no. of males = x
ATQ,
x 6 x(x–1) 7
2× × + (x+6)(x+5) =
x+6 x+5 8 Let AB and CD are the chord drawn on opposite
12x+x2 –x 7
=8 sides of diameter. Length of CD is 16 cm and
(x+6)(x+5)
length of AB is 12 cm.
88x + 8x² = 7 (x² + 5x + 6x + 30)
ATQ
x² + 11x – 210 = 0 𝐶𝐷 16
x² + 21x – 10x – 210 = 0 FC = = = 8𝑐𝑚
2 2
𝐴𝐵 12
x (x + 21) – 10 (x + 21) = 0 AE = = = 6𝑐𝑚
2 2
x = 10, –21 EF = 14 cm
Quantity II: Ratio between investment of A, B But AO = OC = radius of circle
and C is 135 : 9y : 11y 𝑂𝐴2 = 𝐴𝐸 2 + 𝑂𝐸 2 = 𝑂𝐶 2 = 𝐶𝐹 2 + 𝑂𝐹 2
Profit sharing of A, B and C is Let OE and OF be ‘a’ and ‘b’ respectively
135 × 8 : 9𝑦 × 5 : 11𝑦 × 12 𝑎 2 − 𝑏2 = 82 − 62 = 28
= 360 : 15𝑦 : 44𝑦 And 𝑎 + 𝑏 = 14
ATQ, So, 𝑎 = 8, 𝑏 = 6
900
=
360 Radius of circle = √62 + 82 = √36 + 64
2080 360+59𝑦
= √100 = 10𝑐𝑚
⇒ (360 + 59y) = 104 × 8 Area of circle = 𝜋(10)2 = 314.28 𝑐𝑚2
⇒ 59y = 472 Quantity I > Quantity II
⇒y=8
Quantity I > Quantity II 40. (b); Quantity I:
Let cost price is 1000 of 1000 gm. he pays to
39. (a); Quantity I: whole seller ⇒ 850 Rs. for 1000 gm.
Effective cost price = 0.85 Rs. per gm.
Now he sells 850 gm. instead of 1000 gm.
Effective cost price of 850 gm. = 0.85 × 850 =
722.5
1000 –722.5
Profit(x) = 722.5 × 100 = 38.40 ≈ 38%

As, PQ and EF are two line that intersect at S. Quantity II:


So, PS × SQ = ES × SF Let milkman have → q litre of milk
12 × 8 = ES × 6 Initially he add → 20 litre of water
𝑞
ES = 16 cm Sale half Remaining mixture = 2 + 10
From center O draw Again add 20 litre water
𝑞
OB ⊥ PQ and OA ⊥ EF 4
(12+8) So, 2
=
10+20 3
So, PB = PQ = 2 = 10 cm 40
Total profit(y) = 80 × 100 = 50%
BS = OA = 2 cm
EF = 16 + 6 = 22 cm Quantity II > Quantity I
22
EA = 2 = 11 cm 41. (d); Quantity I: If B do 40% of work thus A do 60% of
work
In triangle ∆𝑂𝐴𝐸
Ratio of efficiency of A and B = 3 : 2
OA² + EA² = OE²
Total work = 24 × (3 + 2) = 120 units
2² + 11² = OE² 3×125 15
OE² = 125 Efficiency of C = 100 = 4 units/day

122 Adda247 Publications For More Study Material


Visit: adda247.com
Cracker Book for Bank (IBPS | SBI | RRB PO | Clerk) Mains Exams
120 × 4 Let Cost price = 100x and profit % is y% after y%
Time taken by C = 15 = 32 days
discount
Quantity II: P work for → x day 1300𝑥
Efficiency → x ⇒ 100𝑥 + 𝑥𝑦 = (100 − 𝑦)
700
So, 1300𝑥
⇒ 100𝑥 + 𝑥𝑦 = 700 (100 − 𝑦)
x × x ⇒ 16 Rs.
⇒ 700 + 7𝑦 = 1300 − 13𝑦
x ⇒ 4 Rs.
Money earned by P, Q and R ⇒ 20𝑦 = 600 ⇒ 𝑦 = 30%
⇒ x² + (x + 1)² + (x + 2) ² = 4² + 5² + 6² Quantity II > Quantity I
⇒ 77 Rs. A B C
44. (b);
Quantity II > Quantity I Time 𝑥 + 5 𝑥 𝑥 − 4
1 1 1
42. (a); Quantity I ∴ 𝑥+5 + 𝑥 = 𝑥−4
𝑥 = 10
∴ Quantity I → 10 days
Given 2M = 3W
(8M+14W)×x×7 (6M+10W)×15×6
∴ 7 = 5
×360 ×360
12 12
171 2
𝑥= ⇒ = 13
13 13
OB = 8cm and OA = 10cm
2
As, AB and AC are tangents on smaller circle Quantity II → 13
13
⇒ ∠𝐴𝐵𝑂 = ∠𝐴𝐶𝑂 = 90° ∴ Quantity I < Quantity II
⇒ 𝐴𝐵 = √102 − 82 = 6
Area of quadrilateral ABOC 45. (b);
= 2 × 𝐴𝑟𝑒𝑎 𝑜𝑓 𝑡𝑟𝑎𝑖𝑛𝑔𝑙𝑒 𝐴𝐵𝑂
Area of triangle ABO
1 1
= 2 × 𝑂𝐵 × 𝐴𝐵 = 2 × 8 × 6 = 24𝑐𝑚2
Let P is faster than Q
Area of quadrilateral ABOC Then P covers 72 km distance in the same time as
= 2 × 24 = 48𝑐𝑚2 Q covers 48 km distance
Quantity II: Area of trapezium Ratio of the speed = 72 : 48 = 3 : 2
1 48
= (sum of parallel sides) × perpendicular ∴ Speed of P = × 3 = 72 km/hr
2 2
distance Quantity I→ Difference between P and Q = 72 –
1
= (6 + 8) × 6 = 42𝑐𝑚2 48 = 24 km/hr.
2
Quantity I > Quantity II Let speed of train = T km/hr
Let speed of car = C km/hr
43. (b); Quantity I 120 480
∴ + =8 ………..(i)
Interest earned by Kishan after 2 years 200
𝑇
400
𝐶
1
20,000×22×2 + 𝐶 = 83 ………(ii)
= 100
= 8800 𝑇
On solving (i) and (ii)
Amount invested by KIshan in another bank T = 60 km/hr
= 20,000 + 8800 = 28,800 ∴ Quantity I < Quantity II
ATQ,
3 46. (a); 𝑥: Let efficiency of A = 𝑎
𝑥
⇒ 28,800 [1 + ] = 28,800 + 28,800 and efficiency of B and C be 3𝑦 and 4𝑦
100
−1350 Now
3 1
𝑥
⇒ 28,800 [1 + 100] = 56,250 (𝑎 + 3𝑦) × 16 = 15 (𝑎 + 4𝑦)
3
𝑥 3 56,250 𝑎 = 20𝑦
⇒ [1 + 100] = 28,800 (20𝑦+3𝑦)×16
3
Time taken by A = = 18.4 days
𝑥 125 20𝑦
⇒ [1 + 100] = [ 64 ]
𝑦: Let efficiency of P, Q and R be 2𝑎, 3𝑎, 4𝑎
𝑥 5
⇒ [1 + ] = [ ] respectively
100 4
⇒ 𝑥 = 25% Total work = (2𝑎 + 3𝑎) × 18
5𝑎×18
Quantity II: Marked price is Required time = = 10 days
9𝑎
5
1857 % 𝑜𝑓 𝑡ℎ𝑒 𝑐𝑜𝑠𝑡 𝑝𝑟𝑖𝑐𝑒 𝑜𝑓 𝑎𝑟𝑡𝑖𝑐𝑙𝑒 Quantity I > Quantity II

123 Adda247 Publications For More Study Material


Visit: adda247.com
Cracker Book for Bank (IBPS | SBI | RRB PO | Clerk) Mains Exams

47. (a); 𝑥: PART B


Cost price = Rs. 𝑥
S.P. = 𝑥 + 100 1. (b); I. 3𝑥+5 . 92𝑥−4 = 95𝑥−14
S.P. after increasing it by 20% ⇒ 3𝑥+5 . 34𝑥−8 = 310𝑥−28
120
2𝑥 = 100 (𝑥 + 100) ⇒ 3𝑥+5+4𝑥−8 = 310𝑥−28
𝑥 = 𝑅𝑠. 150 ⇒ 35𝑥−3 = 310𝑥−28
𝑦: ⇒ 5𝑥 − 3 = 10𝑥 − 28
Let C.P. = Rs.100 ⇒ 5𝑥 = 25
So M.P. = Rs.500 ⇒ 𝑥=5
S.P. =
500×40
= 200 II. 2𝑦 2 − 15𝑦 − 28 = 3𝑦 2 − 23𝑦 − 13
100
200–100 ⇒ 𝑦 2 − 8𝑦 + 15 = 0
Profit % = 100 × 100 = 100% ⇒ 𝑦 2 − 3𝑦 − 5𝑦 + 15 = 0
Quantity I > Quantity II ⇒ 𝑦(𝑦 − 3) − 5(𝑦 − 3) = 0
⇒ (𝑦 − 5)(𝑦 − 3) = 0
48. (e); 𝑥 :
𝑎 = 11 ⇒ 𝑦 = 5 ,3
𝑑=1 ⇒x≥y
𝑛 25 15
𝑆𝑛 = [2𝑎 + (𝑛– 1)𝑑] 2. (d); I. − x +2=0
2 x²
𝑥
4040 = [2 × 11 + (𝑥– 1)1] ⇒2x 2 − 15x + 25 = 0
2
𝑥 = 80 ⇒2x 2 − 10x − 5x + 25 = 0
𝑦: 2x (x − 5) − 5(x − 5) = 0
𝑎 = 10 (2x − 5)(x − 5) = 0
5
𝑑=2 x = 2,5
So, 40 13
𝑦 II. +1=
7120 = 2 [2 × 10 + (𝑦– 1)2] y² y

𝑦 = 80 ⇒ y 2 − 13y + 40 = 0
Quantity I = Quantity II ⇒ y 2 − 8y − 5y + 40 = 0
⇒ y(y − 8) − 5(y − 8) = 0
49. (b); 𝑥 : (y − 5)(y − 8) = 0
Amount Invested = 𝑥 y = 5, 8
Time in first scheme = 2 years y ≥ x
Rate = 40%
𝑥×2×40 4𝑥 3. (e); 𝟐𝐲+𝟒 . 32y+4 . 42y+4 = 32y+4 . 22y+16
S.I. = =
100 5 𝒚 + 4 + 4𝑦 + 8 = 4𝑦 + 16
Time in 2 scheme = 5 years
nd
𝐲=4
Rate = 44% 𝒙𝟐 – 10 + 24 = 0
𝑥×5×44 11𝑥
S.I. = 100 = 5 𝒙𝟐 – 6𝑥– 4𝑥 + 24 = 0
11𝑥 4𝑥 𝒙(𝑥– 6)– 4(𝑥– 6) = 0
Difference = 5 – 5 = 18,900
(𝑥– 4)(𝑥– 6) = 0
𝑥 = Rs. 13,500
𝒙 = 4, 6
𝑦:
𝒙≥𝑦
𝑦 = Interest earned
12000 48 14
P = 30 × 100 = 40,000 4. (d); I. − x +1=0

40000×3×12
Interest = = 14,400 ⇒ x 2 − 14x + 48 = 0
100
Quantity II > Quantity I ⇒ x 2 − 8x − 6x + 48 = 0
⇒ x(x − 8) − 6(x − 8) = 0
50. (a); 𝑥 : ⇒ (x − 8)(x − 6) = 0
Edge of cube = 7 cm x = 8, 6
7 45 1
So, largest sphere radius = cm II. + = 2
2 y² y
Volume of vacant space ⇒ 2y 2 − y − 45 = 0
4 22 7 7 7
=7×7×7−3× 7 ×2×2×2 ⇒ 2y 2 − 10y + 9y − 45 = 0
490 ⇒ 2y(y − 5) + 9(y − 5) = 0
= 3 cm³
⇒ (2y + 9)(y − 5) = 0
𝑦 : Surface area of sphere = 4πr² 9
22 7 7
= 4 × 7 × 2 × 2 = 154 𝑐𝑚2 𝐲 = 5, −
2
Quantity I > Quantity II 𝐱 > y

124 Adda247 Publications For More Study Material


Visit: adda247.com
Cracker Book for Bank (IBPS | SBI | RRB PO | Clerk) Mains Exams

5. (d); I. (𝑥 − 4)2 = 9 10. (e); I. 3x² – 23x + 42 = 0


⇒ 𝑥 − 4 = ±3 3x² – 9x – 14x + 42 = 0
⇒ 𝑥 = 7, 1 3x( x – 3) – 14(x – 3) =0
II. (2𝑦 + 3)2 = 25 (3x – 14) (x – 3) = 0
⇒ 2𝑦 + 3 = ±5
x =3, 14/3
⇒ 𝑦 = 1, −4
II. 3x² – 19y + 45 = 0
⇒x≥y
2y² – 10y – 9y + 45 = 0
6. (c); I. 5x² + 3x – 36 = 0 2y (y – 5) – 9 (y – 5) =0
5x² + 15x – 12x – 36 = 0 (2y – 9) (y – 5) = 0
5x (x + 3) – 12 (x + 3) = 0
y=9/2, 5
(5x – 12) (x +3) = 0
x = 12/5, –3 No relation can be established between
𝑥 𝑎𝑛𝑑 𝑦
II. 2y² – 13y + 20 = 0
2 y ² – 8 y – 5 y + 20 = 0 11. (a); I. 𝑥 2 − 5𝑥 + 4 = 0
2 y (y – 4) – 5(y – 4) = 0 𝑥 2 − 4𝑥 − 𝑥 + 4 = 0
(2 y – 5) (y – 4) = 0 𝑥(𝑥 − 4) − 1(𝑥 − 4) = 0
y = 5/2, 4 (𝑥 − 1)(𝑥 − 4) = 0
y>x 𝑥 = 1, 4
7. (b); I. x² – 7x + 12 = 0
II. 𝑦 2 + 5𝑦 + 6 = 0
x² – 4x – 3x + 12 = 0
x(x – 4) –3 (x – 4) = 0 𝑦 2 + 3𝑦 + 2𝑦 + 6 = 0
(x – 3) (x – 4) = 0 𝑦(𝑦 + 3) + 2(𝑦 + 3) = 0
x = 3, 4 (𝑦 + 2)(𝑦 + 3) = 0
𝑦 = −2, −3
II. 2y² – 11y + 15 = 0
2y² – 6y – 5y + 15 = 0 𝑥>𝑦
2y(x – 3) – 3 (y – 3) = 0 12. (d); I. 2𝑥 2 − 𝑥 − 15 = 0
(2y – 5) (y – 3) = 0 2𝑥 2 − 6𝑥 + 5𝑥 − 15 = 0
y = 5/2, 3
2𝑥(𝑥 − 3) + 5(𝑥 − 3) = 0
x≥y
(2𝑥 + 5)(𝑥 − 3) = 0
8. (d); I. 2x² + 11x + 15 = 0 −5
𝑥= ,3
2x² + 6x + 5x + 15 = 0 2
2x (x + 3) +5 (x + 3) = 0 II. 3𝑦 2 − 23𝑦 + 42 = 0
(2x + 5) (x + 3) = 0
3𝑦 2 − 14𝑦 − 9𝑦 + 42 = 0
x = –5/2, –3
II. 2y² + 9y + 10 = 0 𝑦(3𝑦 − 14) − 3(3𝑦 − 14) = 0
2y² + 4y + 5y + 10 = 0 (𝑦 − 3)(3𝑦 − 14) = 0
14
2y (y + 2) +5 (y + 2) =0 𝑦 = 3,
3
(2y + 5) (y + 2) = 0 𝑦≥𝑥
Y = –5/2, –2
y≥x 13. (a); I. 𝑥 2 − 15 + 54 = 0
9. (c); I. 3x² + 7x – 40 = 0 𝑥 2 − 9𝑥 − 6𝑥 + 54 = 0
3x² + 15x – 8x – 40 = 0 𝑥(𝑥 − 9) − 6(𝑥 − 9) = 0
3x (x + 5) – 8x – 40 = 0 (𝑥 − 6)(𝑥 − 9) = 0
(3x – 8) (x + 5) = 0 𝑥 = 6, 9
x = 8/3, –5 II. 𝑦 2 + 15𝑦 − 54 = 0
II. 5y² – 29y + 42 = 0 𝑦 2 + 18𝑦 − 3𝑦 − 54 = 0
5y – 14y – 15 y + 42 =0 𝑦(𝑦 + 18) − 3(𝑦 + 18) = 0
y(5y – 14) – 3 (5y – 14) = 0
(𝑦 + 18)(𝑦 − 3) = 0
(y – 3) (5y – 14) = 0
y = 3, 14/5 𝑦 = −18, 3
y>x 𝑥>𝑦

125 Adda247 Publications For More Study Material


Visit: adda247.com
Cracker Book for Bank (IBPS | SBI | RRB PO | Clerk) Mains Exams

14. (c); I. 𝑥 2 + 14𝑥 + 40 = 0 II. 3y² – 19y + 30 = 0


𝑥 2 + 10𝑥 + 4𝑥 + 40 = 0 3y² – 10y – 9y + 30 = 0
𝑥(𝑥 + 10) + 4(𝑥 + 10) = 0 y(3y – 10) – 3 (3y – 10)=0
(𝑥 + 4)(𝑥 + 10) = 0 (y – 3) (3y – 10) = 0
𝑥 = −4, −10 y = 3,
10
3
II. 𝑦 2 − 5𝑦 − 24 = 0 x>y
𝑦 2 − 8𝑦 + 3𝑦 − 24 = 0
𝑦(𝑦 − 8) + 3(𝑦 − 8) = 0 18. (a); I. x³ = 120 + 96 = 216
(𝑦 + 3)(𝑦 − 8) = 0 x=6
𝑦 = −3, 8 II. y² – 25 = 0
𝑦>𝑥 y² = 25
y = ±5
15. (e); I. 𝑥 2 − 225 = 0
x>y
𝑥 2 = 225
𝑥 = ±15 19. (e); I. x² – 2x – 48 = 0
x² – 8x + 6x – 48 = 0
II. 𝑥 2 + 𝑦 2 = 306
x(x – 8) +6 (x – 8) =0
225 + 𝑦 2 = 306
(x + 6) (x – 8) = 0
𝑦 2 = 306 − 225
x = 8, –6
𝑦 2 = 81
𝑦 = ±9 II. y² – 15y + 54 = 0
No relation can be established between x and y² – 9y – 6y + 54 = 0
y. y(y – 9) –6 (y – 9) = 0
16. (c); I. 2x² – 7x + 6 = 0 (y – 6) (y – 9) = 0
2x² – 4x – 3x + 6 = 0 y = 6, 9
2x (x –2) –3 (x – 2) = 0 No relation can be established between x and y
(2x – 3) (x – 2) =0
3
20. (c); I. 2x² + 25x + 72 = 0
𝑥 = 2 𝑜𝑟 2 2x² + 16x + 9x + 72 = 0
II. 3y² – 19y + 28 = 0 2x (x + 8) +9 (x + 8) = 0
3y² – 12y – 7y + 28 = 0 (2x + 9) (x + 8) = 0
3y (y – 4) –7 (y – 4) = 0 9
x = −2 , –8
(3y – 7) (y – 4) = 0
7 II. 3y² + 22y + 40 = 0
y=3,4
y>x 3y² + 10 y + 12y + 40 = 0
y (3y + 10) + 4 (3y + 10) = 0
17. (a); I. x² – 13x + 36 =0
x² – 9x – 4x + 36 =0 (y + 4) (3y + 10) = 0
x (x – 9) – 4 (x – 9) =0 10
y = – 4, –
(x – 4) (x – 9) = 0 3

x = 4, 9 y>x

126 Adda247 Publications For More Study Material


Visit: adda247.com
Chapter
Data Interpretation
13
What is data interpretation: When data is organized into tables and charts it is done with the purpose of making it meaningful.
The objective of data interpretation is to assess whether a student can understand bars and charts and Answer some questions
based on them. This act of organizing and interpreting data to get meaningful information under a given set of conditions is
Data interpretataion.

About data interpretation: This is the calculation intensive portion, it consists of a myrid of graph. charts and tables and
analyze data. The key to crack this area is to quickly Identify the key pieces of information that you will require to work on.

Basic key that will help you to solve this topic:


• Calculation
• Square
• cube
• table
• BODMAS
• Percentage
• Profit and loss
• Ratio and proportion
• Average

Types of Data Interpretation:


• Data table
• Line graph
• Pie charts
• Bar graph
• Mixed graph
• Line with pie chart
• Table with Bar
• Table with Line, etc.
• Radar graph
• Triangular graph
• Case study (Puzzle)
• Venn Diagram
• Table format
Approach for data interpretation:
• First you look carefully at the table or graph and the direction. Note the years to which, the data refers to and the units.
Sometimes the figures may be given in thousands. While the Answer may be millions Resulting in mistakes.
• The level of approximation that can be done is assessed from the choices. If the answer is wide, time should not be
wasted in working out exact figures. If the choice ‘none of the above exists, a close approximation may be required’.
• Read the question carefully, it will give an indication as to which row and column should be seen. A carefull reading
of the question will reveal exactly what is to be done and the units in which the answer is required.
• There may be one or two very large question requiring calculations. Attempt these at the last.
• Revise bar charts, table and line graphs before attempting D.I. question remember that the D.I. section is a scoring one
and also time saving.
Data Table: The numbers in the bracket give the maximum marks in each subject.
The following table gives the percentage of marks obtained by seven students in six different subjects in an examination. Study it and
answer the question given below it.
Cracker Book for Bank (IBPS | SBI | RRB PO | Clerk) Mains Exams

Students Subject (Maximum Marks)


Maths Chemistry Physics Geography History Computer
(150) (130) (120) (100) (60) Science (40)
Golu 90 50 90 60 70 80

Mithi 100 80 80 40 80 70

Suraj 90 60 70 70 90 70

Gapplu 80 65 80 80 60 60

Mahi 80 65 85 95 50 90

Khushi 70 75 65 85 40 60

Sheetal 65 35 50 77 80 80

1. What are the average marks obtained by all the seven students in physics? (Round off two digits after decimal)
(a) 77.26 (b) 89.14 (c) 91.37 (d) 96.11
2. The number of students who obtained 60% or above marks in all subjects is
(a) 1 (b) 2 (c) 3 (d) 4
3. What was the aggregate of marks obtained by suraj in all the six subjects ?
(a) 409 (b) 419 (c) 429 (d) 449
4. In which subject is the overall percentage the best?
(a) Maths (b) Chemistry (c) Physics (d) History
5. What is the overall percentage of sheetal
(a) 52.5% (b) 55% (c) 60% (d) 63%
1
Solution 1: é( 90%of 120 ) + ( 80%of 120 ) + ( 70%of 120 ) + ( 80%of 120 ) + ( 85%of 120 ) + ( 65%of 120 ) + ( 50%of 120 ) ùû

1
= é( 520%of 120 ) ùû

624
= = 89.14
7
Solution 2: From the table, it is clear tha that Suraj & Gapplu have 60% or more marks in each of the six subjects.
Solution 3: Aggregate marks obtained by suraj
= [(90% of 150)+(60% of 130)+(70% of 120)+(70% of 100)+(90% of 60)+(70% of 40)]
= [135 + 78 + 84+ 70 + 54 + 28] = 449
Solution 4: We shall find the best overall % with respect to each subject: Mathematics
Solution 5: Aggregate marks obtained by sheetal.
= [(65% of 150)+(35%of 130)+(50% of 120)+(77% of 100) + (80% of 60)+(80% of 40)] = 360
Maximum marks = 600

360
Overall % = ´ 100 = 60%
600

128 Adda 247 Publications For More Study Meterial


Visit: adda247.com
Cracker Book for Bank (IBPS | SBI | RRB PO | Clerk) Mains Exams

Line Graph:
Answer the question based on the given line graph
Following line graph shows the ratio of export to import of company A and company B over the year
2
Company A
1.75

1.5 Company B
1.25

.75

.5

.25

0
Years 2005 2006 2007 2008 2009 2010
1. In how many of the given years were the exports more than the imports for company A?
(a) 2 (b) 3 (c) 4 (d) 5
2. If the imports of company A in 2007 were increased by 40%. What would be the ratio of exports to the increased imports?
(a) 1.50 (b) 1.25 (c) 1.75 (d) 1.35
3. If the exports of company B in 2008 was Rs 237 Crore, what was the amount of imports in that years ?
(a) 189.6 (b) 188.8 (c) 184.6 (d) 182.8
4. In 2005, the export of company A was double that of company B. If the imports of company A during the year was 180 crore.
What was the approximate amount of imports of company B during that year?
(a) 190 crore (b) 210 crore (c) 225 crore (d) 200 crore
5. In which year were the exports of company A minimum proportionate to its imports.
(a) 2008 and 2009 (b) 2009 (c) 2008 (d) 2005
Solution 1: required ratio, more than 1 for the years; 2005, 2006 and 2007,ie; for 3 years
Solution 2: In 2007, for company A the ratio of exports to import = 175 : 100
Let exports of company A = 175 x Þ and imports of company A = 100x
New imports of company = 140% of 100x = 140x
175x
There for requied ratio = = 1.25
140x
Solution 3: Let import of company B in 2008 = x
237 237
Then, = 1.25 Þ x = Þ 189.6 crore
x 1.25
Solution 4: In 2005 for company A Þ Export = 1.75 × 180 crore
1.75 ´ 180 Ex
in 2005 for company B export = = 157.5 crore, But = .75
2 Im

157.5
Import of company B = = 210 crore
.75
Solution 5: 2008 and 2009, A option.

129 Adda 247 Publications For More Study Meterial


Visit: adda247.com
Cracker Book for Bank (IBPS | SBI | RRB PO | Clerk) Mains Exams

Pie Charts or Circle Graphs:


Distribution of candidate who were enrolled in MBA and the candidate (out of those enrolled) who passed the exam in different
institutes

X X
16% P 12% P

V 22% V 18%

12% 15%
Q Q
T T
15% 17%
8% 9%
S R S R

17% 10% 16% 13%

Total number of candidates Total number of candidates


enrolled = 8550 who passed the exam = 5700

1. What percentage of candidates passed the exam from institute T out of the total no. of candidate enrollled from the same
institute?
(a) 50% (b) 52.5% (c) 75% (d) 80%
2. Which institute has the highest % of candidates passed to the candidates enrolled?
(a) Q (b) R (c) V (d) T
3. The no. of candidates passed from institute S and P together exceeds the no. of candidates enrolled from institutes T and R
together by?
(a) 228 (b) 279 (c) 399 (d) 407
4. What is % of candidates passed to the candidate enrolled for institutes Q and R together ?
(a) 68% (b) 80% (c) 74% (d) 65%
5. What is the ratio of candidates passed to the candidates enrolled from institute P ?
(a) 9 : 11 (b) 14 : 17 (c) 6 : 11 (d) 9 : 17
9%of 5700
Solution 1: Required % = ´ 100 = 75%
8%of8550
Solution 2: The % of candidates passed to candidates enrolled can be determined for each institute as under.
17% of 5700 13%of 5700
For Q Þ ´ 100 = 75.56% ; For R Þ ´ 100 = 86.67%
15% of 8550 10% of 8550

9%of 5700 15%of 5700


For T Þ ´ 100 = 75% ; For V Þ ´ 100 = 83.33%
8% of 8550 12% of 8550
So the highest of these is 86.67% corresponding to institutes R.
Solution 3: Required Difference = [(16% + 18%) of 5700] – [(8% + 10%) of 8550] = 1938 – 1539 = 399
30%of 5700
Solution 4: ´ 100 = 80%
25% of 8550

18% of 5700 6
Solution 5: = = 6 : 11
22%of 8550 11

130 Adda 247 Publications For More Study Meterial


Visit: adda247.com
Cracker Book for Bank (IBPS | SBI | RRB PO | Clerk) Mains Exams

Bar Graph: Percentage profit earned by two companies X & Y over the given years.
Income - Expenditure
Pr ofit% = ´ 100
Expenditure
Company X
70
65
Company Y
60
60
55 55
50 50 50 50
50
45 45
Profit%

40
40
35

30

20

2006 2007 2008 2009 2010 2011


1. The income of two company X and Y in 2010 were in the rato of 3 : 4. respectively. What is the respective ratio of their
expenditures in 2010?
(a) 7 : 22 (b) 14 : 19 (c) 15 : 22 (d) 27 : 35
2. If the expenditure of company Y in 2007 was Rs 220 crore, what was its income in 2007?
(a) Rs. 312 crore (b) Rs. 297 crore (c) Rs. 283 crore (d) Rs. 275 crore
3. If the expenditures of company X and Y in 2006 were equal and the total income of the two companies in 2006 was Rs 342
crore, what was the total profit of the two company together in 2006?
(a) Rs. 240 crore (b) Rs. 171 crore (c) Rs. 120 crore (d) Rs. 102 crore
4. The expanditure of company X in the year 2008 was Rs 200 crore and the income of company X in 2008 was the same as its
expenditure in 2011. The income of company X in 2011 was?
(a) Rs. 465 crore (b) Rs. 385 crore (c) Rs. 335 crore (d) Rs. 295 crore
5. If the income of two company were equal in 2009, then what was the ratio of expenditure of company X to that of company
Y in 2009?
(a) 6 : 5 (b) 5 : 6 (c) 11 : 6 (d) 16 : 15
165%of E 1 3 E 15
Solution 1: = Þ Therefore, 1 = = 15 : 22
150%of E 2 4 E 2 22
Solution 2: 220 + 35% of 220 Þ 220 + 77 = 297 crore
Solution 3: Let the expenditure of each companies x and y in 2006 be 100
342
Then we have 100 + 40 + 100 + 45 = 342 crore Þ Required profit = ×85 = 102 crore
285
Solution 4: Expenditure of Company X in year 2008 is 200 crore, Then income of company X in year 2008
= 200 + 55% of 200 = 310 crore
Hence, the expenditure of company X in year 2011 is 310 crore,
Therefore, the required income of X in year 2011 = 310 + 50% of 310 = 465 crore
150 160
Solution 5: X= Y
100 100

x 160 16
be the incomes of two companies in 2009; = = = 16 : 15
y 150 15

131 Adda 247 Publications For More Study Meterial


Visit: adda247.com
Cracker Book for Bank (IBPS | SBI | RRB PO | Clerk) Mains Exams

Practice Exercise Based on new Pattern

Direction (1–5): Table given below shows total number of students in two sections of five different schools, percentage
distribution of students in these different sections (X & Y) and also ratio between boys to girls in each section. Read the
table carefully and answer the questions:
Percentage of Ratio between boys: Ratio between boys :
Schools Total students students in girls in section girls in section
Section ‘X’ ‘X’ ‘Y’
LPS 450 48% 2:1 4:5
CMS 360 55% 7:2 1:2
DPS 420 60% 5:4 11 : 10
SKD 300 55% 1:2 2:1
Loyola 250 50% 3:2 2:3
1. Total number of boys in section ‘X’ from CMS & Loyola together are what percent of total girls in section ‘Y’ from
DPS & SKD together?
(a) 183.2 % (b) 180.2 % (c) 188.2 %
(d) 189.2 % (e) 185.2%
2. Find the ratio between total number of girls in section ‘Y’ from CMS & SKD together to total number of boys in
section ‘X’ from DPS & SKD together?
(a) 65 : 51 (b) 51 : 65 (c) 51 : 67
(d) 51 : 62 (e) 51 : 49
3. Out of total number of boys in section ‘X’ from D.P.S school ratio between number of boys who got first, second and
third division is 1 : 2 : 4. Find total number of boys who got first and third division together in section ‘X’ from
D.P.S school?
(a) 40 (b) 60 (c) 80
(d) 100 (e) 120
4. Find the difference between average number of girls in section ‘X’ from SKD & Loyola together and average
number of boys in section ‘Y’ from LPS & CMS together?
(a) 5 (b) 4 (c) 3
(d) 2 (e) 1
5. Total girls in section ‘Y’ from CMS & LPS together are how much more than total boys in section ‘X’ from both DPS
& SKD together?
(a)33 (b) 43 (c)53
(d) 63 (e) 45

Direction (6 – 10): Table given below shows total number of applicants, who have applied for CDS exam in the five
years. Percentage of applicants appeared in exam, percentage of applicants qualified in tier I and tier II. Line graph
shows percentage of applicants got selection in CDS exam. Read the data carefully and answer the questions:

Appeared % of applicants % of students


Years Total Applicants applicants qualified in qualified in
in tier I tier I tier II
2013 2400 85% 25% 40%
2014 3200 75% 30% 25%
2015 4800 90% 20% 25%
2016 7500 80% 25% 20%
1
2017 8400 60% 12 % 40%
2

Note – All applicants qualified in tier I appeared for tier II.

132 Adda247 Publications For More Study Material


Visit: adda247.com
Cracker Book for Bank (IBPS | SBI | RRB PO | Clerk) Mains Exams

50

(Percentage)
40

30

20
2013 2014 2015 2016 2017

6. Number of applicants, who have got the final selection in the year 2017, is how many percent less than number of
applicants, who have got the final selection in the year 2014?
2 2 2
(a) 22 9 % (b) 24 9 % (c) 28 9 %
2 2
(d) 18 % (e) 21 %
9 9

7. Find the difference between number of applicants, who have got final selection in the years 2013 & 2016 together
and number of applicants, who have got final selection in the year 2014 & 2015 together?
(a) 5 (b) 8 (c) 3
(d) 6 (e) 11
8. Find the ratio between applicants qualified in tier II but did not get final selection in the year 2017 to applicants
qualified in tier I but did not qualify for tier II in the year 2014?
(a) 7 : 20 (b) 7 : 22 (c) 7 : 19
(d)7 : 18 (e)7 : 16
9. Total applicants appeared for tier I in the year 2017 is what percent more than total applicants appeared for tier II
in the year 2015?
1 1 1
(a) 498 % (b) 438 % (c) 485 %
3 3 3
1 1
(d) 483 % (e) 487 %
3 3

10. Find the total number of applicants, who did not qualify in tier I in the year 2014, 2016 & 2017 together?
(a) 10390 (b) 10590 (c) 10390
(d) 10290 (e) 10190
Direction (11 – 15): Given below pie graph shows total number of tickets of ‘Sanju’ movie sold by five multiplex on
opening days, while bar graph shows percentage distribution of three different type of class ticket sold by each store.
Read given data and answer the questions:

Total sold tickets - 12500

A
12%
E
36% B
18%

D C
10% 24%

133 Adda247 Publications For More Study Material


Visit: adda247.com
Cracker Book for Bank (IBPS | SBI | RRB PO | Clerk) Mains Exams

Jubali Silver Gold


100
90
80
70
60

Percentage
50
40
30
20
10
0
A B C D E
Multiplex
11. Total Jubali class ticket sold by multiplex C & E together is what percent more than total Gold class ticket sold by
multiplex A & C together?
1 1 1
(a) 105 % (b) 102 % (c) 100 %
7 7 7
1 1
(d) 977 % (e) 107 7 %

12. Find difference between total Silver class ticket sold by multiplex A, C & E together and total silver gold class ticket
sold by multiplex B, C & E together?
(a) 400 (b) 200 (c) 650
(d) 600 (e) 450
13. Find sum of average number of Jubli class ticket sold by multiplex B & C and average number of Gold class ticket
sold by multiplex C & E?
(a) 2075 (b) 2175 (c) 2275
(d) 2375 (e) 2205
14. Total Silver class tickets sold by multiplex A & D is what percent less than total Gold class ticket sold by B & E?
4 4 4
(a) 42 9 % (b) 40 9 % (c) 38 9 %
4 4
(d) 44 9 % (e) 36 9 %

15. The ratio of price of one Gold ticket in multiplex D, C & A is 10 : 6 : 7, and all three multiplex earned Rs. 40000 from
the selling of all Gold Class tickets. Find difference between price of one Gold ticket in Multiplex A and D?
(a) 10 Rs. (b) 15 Rs. (c) 5 Rs.
(d) 20 Rs. (e) 25 Rs.
Direction (16 – 20): Given below table shows total number of five different brands of mobiles sold by a store on
Sunday. Also given percentage of 2G mobiles and ratio between 3G and 4G mobile phone sold out of total number of sold
mobiles. Read the table carefully and answer the following questions:

Brand Number of total sold mobiles Percentage of 2G mobiles Ratio of 3G : 4G mobiles


Samsung 750 20% 2:3
Lava 840 25% 3:4
MI 1050 30% 1:4
Huwai 960 15% 1:2
Oppo 640 35% 1:3

134 Adda247 Publications For More Study Material


Visit: adda247.com
Cracker Book for Bank (IBPS | SBI | RRB PO | Clerk) Mains Exams
2
16. Total number of Samsung mobiles sold by store on Saturday is 3 𝑟𝑑 of the number of same brand of mobile sold on
Sunday. If total number of 3G Samsung mobile sold on Saturday is 40 less than that of total number of 3G Samsung
mobile sold on Sunday, then find total number of 4G Samsung mobile sold on Sunday is what percent more/less
than total number of 4G Samsung mobile sold on Saturday (on Saturday store sold only 3G and 4G mobile)?
(a) 25% (b) 20% (c) 10%
(d) 15% (e) 5%

17. Total number of MI mobile sold on Monday is equal to average number of Huwai & Oppo mobile sold on Sunday. If
ratio between number of MI 2G, 3G and 4G mobiles sold on Monday is 4 : 5 : 7, then find the ratio between total
number of MI 3G mobile sold on Monday to total number of Lava 3G mobile sold on Sunday?
(a) 25 : 29 (b) 25 : 27 (c) 25 : 36
(d) 25 : 31 (e) 25 : 33

18. If ratio of total number of Oppo 4G mobiles sold by store on Sunday, Monday and Tuesday is 6 : 4 : 7, then find
total number of Oppo 4G mobile sold by store on Monday & Tuesday together is what percent less/more than the
total number of Samsung & Lava 4G mobile sold by store on Sunday together?
5 5 5
(a) 18 9 % (b) 16 9 % (c) 24 9 %
5 5
(d) 20 % (e) 12 %
9 9

19. Find the difference between average numbers of Samsung & Lava 2G mobile and average numbers of Huwai &
Oppo 3G mobile sold by store on Sunday?
(a) 12 (b) 16 (c) 10
(d) 8 (e) 4

20. If total number of Vivo mobile sold by store on Sunday is 56 more than total number of 4G Lava & Huwai mobile
sold by store on Sunday, then find total number of Oppo mobile phone sold by store on Sunday is what percent
more/less than total number of Vivo mobile sold by store on Sunday ?
1 1 1
(a) 35 % (b) 38 % (c) 33 %
3 3 3
1 1
(d) 39 % (e) 41 %
3 3

Direction (21- 25): Table given below shows total number of tickets which were estimated to be sold in five stadiums.
Out of total tickets sold, percentage of A1 tickets sold, and ratio between A2 and A3 is given. Study the data carefully and
answer the following questions.
Stadiums Tickets which can be sold A1 tickets sold (in %) A2 : A3
Nehru 1800 15% 9:8
Patel 2600 25% 4:5
Eden 2200 20% 5:3
Kaloor 2000 20% 7:5
Raipur 2700 30% 3:4
Note: - 200 tickets remain unsold in each stadium
Tickets which can be sold = A1 tickets + A2 tickets + A3 tickets + Unsold tickets
21. Total A1 and A3 tickets sold in Patel stadium is what percent more than total number of A 1 and A2 tickets sold in
Kaloor stadium?
2 2
(a) 163 % (b) 50% (c) 663 %
1
(d) 25% (e) 333 %

135 Adda247 Publications For More Study Material


Visit: adda247.com
Cracker Book for Bank (IBPS | SBI | RRB PO | Clerk) Mains Exams

22. Total A2 tickets sold in Nehru stadium is how much less than A3 tickets sold in Raipur stadium?
(a) 220 (b) 240 (c) 260
(d) 280 (e) 300
23. Find the ratio between total A3 tickets sold in Eden and Kaloor stadium together to total A2 tickets sold in Patel
and Eden stadium?
(a) 3 : 2 (b) 2 : 3 (c) 20 :23
(d) 23 : 20 (e) 8 : 9
24. Find the average number of total A1 tickets sold by all five stadiums together?
(a) 514 (b) 490 (c) 470
(d) 450 (e) 430
25. A1 and A2 tickets sold in Nehru stadium is what percent more/less than A1 and A2 tickets sold in Raipur stadium?
(a) 36% (b) 64% (c) 56.25%
(d) 43.75% (e) 28%
Direction (26-30): Given below bar graph shows percentage distribution of weekly income of four different Bollywood
movies. Table shows total income of these four movies and percentage of income earning by these movies from foreign.
Read the data carefully and answer the questions.

First week Second week Third week


100

80

60

40

20

0
A B C D

Movies Total income % of income from foreign


(in crores)
A 210 26 ⅔%
B 180 25%
C 200 20%
D 240 30%
26. What is the ratio between income of movie B in second week to income of movie C in third week?
(a) 13 : 5 (b) 13 : 3 (c) 27 : 5
(d) 27 : 10 (e) 27 : 13
27. Out of these four movies, which movie beard highest percentage drop in its income in second week with respect to
first week?
(a) A (b) B (c) C
(d) D (e) None of these
28. What is the ratio of income of movie A from foreign to the income of movie C from foreign?
(a) 8 : 5 (b) 8 : 7 (c) 7 : 5
(d) 28 : 27 (e) 7 : 8
29. If ratio of foreign income of B of given three weeks is 1 : 2 : 2 respectively. Then find the income earn by movie B in
second week in India?
(a) 36 crores (b) 18 crores (c) 26 crores
(d) 30 crores (e) 32 crores

136 Adda247 Publications For More Study Material


Visit: adda247.com
Cracker Book for Bank (IBPS | SBI | RRB PO | Clerk) Mains Exams

30. Income of movie D in second week and income of movie C in third week together is approximately what percent
more or less then total income of movies A and B of second week?
(a) 55 (b) 78 (c) 62
(d) 44 (e) 38

Directions (31-35): Study the following graph carefully and answer the question. The following bar graph shows the
budget allocation (in crore) for education, sport and Health care from year 2018-22.
Education Sport Health care
250
220 220
200
200 190 190
170 170
160 160
Budget (in crore)

150
150
125 125 130 130

105
100

50

0
2018 2019 2020 2021 2022
Year
31. What percent is the average budget of sport during 2018 to 2022 than that of average budget of Education
throughout all the years? (approximately)
(a) 84% (b) 88% (c) 87%
(d) 90% (e) 94%
32. In 2022, budget allocated for sport for male to female are in the ratio of 3 : 2 and it proposed that budget allocated
for males to be increased by 25% in 2023 compared to previous year for sport then, what is the percentage
increase in sport budget allocation for 2023 with no change in sport budget allocated for female in 2023 as
compared to previous year?
(a) 20% (b) 18% (c) 15%
(d) 12% (e) 10%
33. Find the ratio between total budget allocated for Education from year 2018 to 2020 together to total budget
allocated for Health care from year 2020 to 2022 together ?
(a) 17 : 29 (b) 18 : 29 (c) 21 : 29
(d) 27 : 29 (e) 17 : 27
34. If in 2019 budget allocated for only three sports i.e. Cricket, Badminton and Hockey in the ratio of 5 : 4 : 4
respectively and budget allocated for Education in 2022 for Urban and rural area in the ratio of 5 : 6 respectively.
Then find budget allocated for Education for rural area in 2022 how much more than budget allocated for Cricket
& Badminton together in 2019 ?
(a) 30 cr (b) 35 cr (c) 40 cr
(d) 45 cr (e) 25 cr
35. Find the sum of average of budget allocated for Health care throughout all the year 2018-22 and average of budget
allocated for Education throughout all the year 2018-2022 ?
(a) 336 (b) 338 (c) 340
(d) 342 (e) 348

137 Adda247 Publications For More Study Material


Visit: adda247.com
Cracker Book for Bank (IBPS | SBI | RRB PO | Clerk) Mains Exams

Direction (36 – 40): Given below table shows the percentage of number of cars sold by four different motor companies,
percentage of number of total returned cars out of total number of sold cars in two successive years(2016, 2017) . Read
the graph carefully and answer the following questions.
Note (I) Total number of manufactured cars = number of Unsold cars + number of Sold cars
(II) Total actual number of cars sold = Total number of sold cars – number of returned cars
Percentage of Percentage of Percentage of Percentage of
number of cars number of cars number of cars number of cars
sold in 2016 sold in 2017 returned in 2016 returned in 2017
TATA 80% 75% 15% 10%
HYUNDAI 65% 70% 20% 15%
MARUTI-SUZUKI 80% 85% 12.5% 17.5%
HONDA 75% 70% 15% 22.5%
36. The ratio between total number of cars manufactured by TATA and HONDA in the year 2016 is 6 : 7 and the total
actual number of cars sold by HONDA is 4590 units more than that of TATA in the same year. If total number of
cars returned to HYUNDAI in the year 2017 is 1920 units less than total number of cars returned to TATA in 2016
and total number of cars returned to MARUTI SUZUKI in the year 2017 is 4830 units more than total number of
cars returned to HONDA in 2016, then find the difference between total number of manufactured cars by
HYUNDAI and MARUTI SUZUKI in the year 2017?
(a) 36,000 (b) 42,000 (c) 44,000
(d) 40,000 (e) 32,000
37. Total number of cars manufactured by TATA in the year 2017 is 40% more than that of total number of cars
manufactured by MARUTI SUZUKI in the year 2016 and total number of cars returned to TATA in 2017 is 120
units more than total number of cars returned to MARUTI SUZUKI in the year 2016. If total number of cars
returned to HONDA in the year 2017 is 2010 units more than total number of cars returned to MARUTI SUZUKI in
the year 2016, then find total number of cars manufactured by HONDA in the year 2017?
(a) 24,000 (b) 20,000 (c) 28,000
(d) 18,000 (e) 16,000
38. The ratio between total number of cars manufactured by HYUNDAI, MARUTI SUZUKI & HONDA in the year 2016 is
2 : 3 : 4 and average number of cars returned to these three companies in the same year is 4040 units. If total
number of cars manufactured by HYUNDAI, MARUTI SUZUKI & HONDA in the year 2017 is increased by 20%, 25%
and 12.5% respectively over that of the previous year, then find the average number of cars manufactured by
HYUNDAI, MARUTI SUZUKI & HONDA in the year 2017?
(a) 42,400 (b) 42,800 (c) 42,600
(d) 42,000 (e) 41,600
39. The ratio between total number of cars manufactured by HYUNDAI & HONDA in the year 2016 is 8 : 9 and the ratio
of number of cars manufactured by HYUNDAI & HONDA in the year 2016 to 2017 is 2 : 3 and 3 : 5 respectively. If
total actual number of cars sold by HYUNDAI & HONDA together in the year 2016 is 39590 units then find total
number of cars manufactured by HONDA in the year 2017 is what percentage more/less than total number of cars
manufactured by HYUNDAI in the same year?
(a) 20% (b) 15% (c) 10%
(d) 25% (e) 35%
40. The ratio between total number of cars manufactured by TATA, HYUNDAI & HONDA in the year 2017 is 7 : 6 : 8
and total actual number of cars sold by these companies in the year 2017 is 75810 units, then find total number of
cars manufactured by TATA, HYUNDAI & HONDA together in the year 2017?
(a) 1,24,000 (b) 1,25,000 (c) 1,20,000
(d) 1,26,000 (e) 1,36,000
138 Adda247 Publications For More Study Material
Visit: adda247.com
Cracker Book for Bank (IBPS | SBI | RRB PO | Clerk) Mains Exams

Direction (41 – 45): Pie-chart shows the distribution of total bikes manufactured by five different companies. Also
Table shows percentage ‘150 CC’ and ‘200 CC’ & total bike sold by these five companies. Study the data carefully and
solve the following questions

Hero
Yamaha 15%
22%

Bajaj
TVS 25%
18%

Honda
20%

150CC 200CC Total sold


Companies
Sold bikes Sold bikes bikes
Hero 35% 65% —
Bajaj 40% 60% —
Honda — — 80%
TVS 75% — —
Yamaha 70% — 75%
Note All companies manufactured only two types of bike ‘150 CC’ & ‘200 CC’. Some values are missing you have to
calculate it according to questions.
Total bikes sold of any company= Total 150 CC bikes sold + Total 200 CC bikes sold
41. Total ‘200 CC’ Honda bikes sold by store is 25% more than total ‘150 CC’ bikes sold by same store. If difference
between ‘200 CC’ & ‘150 CC’ bike sold by Honda is 640 then find total ‘200 CC’ bike sold by Yamaha?
(a) 1785 (b) 1782 (c) 1780
(d) 1787 (e) 1791
42. If Hero & Bajaj sold 80% & 90% of total manufactured bikes respectively and difference between total ‘200 CC’
bikes sold by these two companies is 4104, then find total ‘150 CC’ bike sold by Bajaj is what percent more than
total ‘150 CC’ bike sold by Hero?
10 10 2
(a) 88 21 % (b) 92 21 % (c) 94 7 %
2 10
(d) 114 7 % (e) 96 21 %

43. If Tvs sold 80% of total manufactured bikes and difference between total ‘150 CC’ bikes sold by Tvs and Yamaha is
450, then find bike manufactured by Hero & Bajaj together?
(a) 28000 (b) 26000 (c) 32000
(d) 24000 (e) 36000

44. If Hero, Bajaj & Tvs sold 75%, 80% & 90% of total manufactured bikes respectively and total number of ‘150 CC’
bikes sold by these three companies is 23124, then find the total bike sold by Honda?
(a) 15568 (b) 16164 (c) 17162
(d) 15360 (e) 17172
45. If total 84000 bikes manufactured by all five companies and ratio between ‘150 CC’ to ‘200 CC’ bike manufactured
by Honda is 3 : 2, then find total ‘200 CC’ bike sold by Yamaha is what percent less than total ‘150 CC’ bike
manufactured by Honda?
(a) 56.75% (b) 54.75% (c) 62.75%
(d) 59.75% (e) 58.75%

139 Adda247 Publications For More Study Material


Visit: adda247.com
Cracker Book for Bank (IBPS | SBI | RRB PO | Clerk) Mains Exams

Directions (46-50): Given below are two pie-charts which shows the percentage distribution of employees in Adda247
who travel to their office in Gurgoan by two different means i.e. by metro and by cab on different days of week. First pie
chart shows data for Metro and second pie chart shows data for Cab

Saturda Saturda
y Monday y
15% 20% 10% Monday
25%

Friday Friday
(50/3) 20%
% Tuesday
(40/3)
%
Tuesday
(70/3)
Thursda % Thursda
y y Wed.
10% Wed. (20/3) 25%
15% %

Note: 1. Ratio of total employees travelling to Gurgaon by metro to by cab is 5 : 4.


2. If difference of persons travelling by metro and by cab on Saturday is 420.
46. If number of employees travelling on Sunday by cab decreases by ‘Y’ with respect to employees travelling on
Friday by cab and number of employees travelling on Sunday by metro is twice than that of travelling on Friday by
metro . Then find value of ‘Y’?( Given that total employees on Sunday is 75% of total employees on wed.)
(a) 1280 (b) None of these (c) 1385
(d) 1415 (e) 1255
47. What is the difference of number of employees travelling by metro on Wednesday and Thursday together and
number of employees travelling by cab on same days together?
(a) 50 (b) 20 (c) None of these
(d) 40 (e) 60
48. If total fare per person travelling by metro & cab are Rs.120, then ratio of total amount spent on Thursday by all
employees travelling by cab to total amount spent by all employees on same day travelling by metro? (given that
ratio of fare per person travelling by metro to by cab is 5 : 7)
(a) 21 : 23 (b) 56 : 73 (c) None of these
(d) 56 : 75 (e) 53 : 73
49. If total fare on Monday by all employees travelling by metro is Rs. 48000. Then find the total fare on same day by
all employees travelling by cab? (if per person fare for each employee is same)
(a) Rs. 48000 (b) Rs. 42000 (c) Rs. 56000
(d)None of these (e) Rs. 26000
50. Total number of employees travelling by metro on Friday and Saturday together is approximately what percent
more or less than number of employees travelling by cab on same days together?
(a) 48% (b)28% (c) 42%
(d) 38% (e) 32%
Directions (51-55): Table given below shows number of students appeared in preliminary examination of an exam
‘ASK’ in six different cities and percentage of students failed in preliminary, mains and in only reasoning. Those students
who cleared the Preliminary examination can give the Mains examination. In Mains examination of ASK, there are only
two subjects i.e. Reasoning and Quant. Student has to clear both the subjects to pass in Mains examination of ‘ASK’.

140 Adda247 Publications For More Study Material


Visit: adda247.com
Cracker Book for Bank (IBPS | SBI | RRB PO | Clerk) Mains Exams

Students Failed in Students Failed in Students Failed in


Students appeared in
Preliminary exam Mains examination only Reasoning
Preliminary exam
(in %) (in %) (in %)
A 96,000 25% 87.5% 25%
B 80,000 40% 75% 37.5%
2
C 1,20,000 35% 80% 16 %
3
D 1,60,000 15% 75% 25%
E 1,25,000 28% 70% 40%
1
F 72,000 33 3 % 68.75% 25%

51. Total number of students who failed only in Quant in City ‘A’ is three times of total number of students passed the
mains exams in city ‘B’ while total number of students who failed in both subjects in City ‘B’ is 3000 less then total
number of students who failed is both subjects in city ‘A’. Find number of students who failed only in Quant in city
‘A’ is what percent more than number of students who failed in Reasoning in city ‘B’?
(a) 250% (b) 50% (c) 150%
(d) 200% (e) 100%
52. In city D, total number of students who failed in Reasoning is ‘x’ more than total number of students who failed in
only Quant. If total number of students who passed in mains exam in city D is ‘x’ then find the ratio between total
number of students who failed in Reasoning to total number of students who failed in only Quant in city ‘D’.
(a) 1 : 3 (b) 3 : 1 (c) 1 : 1
(d) 1 : 2 (e) 2 : 1
53. Total number of students who failed in Quant in city E is how much more than total number of students who failed
in Quant in city F?
(a) 3000 (b) 6000 (c) 9000
(d) 12000 (e) 15000
54. Ratio between total number of students who failed in Quant in city ‘B’ to total number of students who failed in at
most one subject in city ‘C’ is 30 : 91. Find what percent of students failed in both subject in city ‘C’ in mains exam?
(a) 25% (b) 15.5% (c) 17.5%
(d) 19.5% (e) 30%
55. Total number of students who failed in at most one subjects in city F is 24000 less than total number of students
who failed in at least one subject in city A. Find total number of students who failed in both subjects in city ‘F’ is
how much less than total number of students who failed in only reasoning in city ‘A’.
(a) 3000 (b) 6000 (c) 9000
(d) 12000 (e) 15000

Direction (56-60): - A school has four hostels in which there are two sections one for boys and one for girls. Line chart
given below shows increase/decrease in total number of students in 2013 in these hostels as compare to 2012(previous
year). Table given below shows ratio between number of boys to number of girls in these hostels in 2013. Study the data
carefully and answer the following questions

300

250 Hostel Boys : Girls


270 Aravali 2:3
200 Nilgiri 4:1
150 Udaygiri 5:3
Shivalik 4:7
100

50
Aravali Nilgiri Udaygiri Shivalik

141 Adda247 Publications For More Study Material


Visit: adda247.com
Cracker Book for Bank (IBPS | SBI | RRB PO | Clerk) Mains Exams

56. In Aravali, number of girls in 2013 is 20% more than that in 2012. If change in the number of girls in Aravali is
25% greater than change in number of boys in Aravali then find the Number of boys in Aravali in 2013.
(a) 900 (b) 600 (c) 500
(d) 750 (e) 800
57. Number of students is increases and decreases in Aravali and Nilgiri in 2013 respectively as compare to 2012.
Number of girls who left Aravali joins Nilgiri (No other change in Number of girls) such that ratio between
increase and decreases in Number of boys in Aravali and Nilgiri in 2013 as compare to 2012 is 16 : 15, then find
the change in Number of boys in Nilgiri?
(a) 350 (b) 320 (c) 300
(d) 420 (e) 400
58. In 2012, Number of boys in Udaygiri and Shivalik are equal and Number of girls in Shivalik is 540 more than the
Number of girls in Udaygiri. If in 2013, Number of boys in both hostels are equal then find the Number of total
student in Shivalik, If In 2013 strength of both hostels increases as compare to previous year
(a) 1650 (b) 1200 (c) 960
(d) 1500 (e) 900
59. In 2012 and in 2013 ratio of total strength of Udaygiri to that of Nilgiri is 4 : 5 and 5 : 8 respectively. Find the
strength of Nilgiri in 2012 if strength of Udaygiri decreases
(a) 1600 (b) 1200 (c) 1250
(d) 1400 (e) 1750
60. In2013, strength of all hostels increases as compare to 2012. If total strength of hostels in 2012 is 1030, then find
the Number of boys in Aravali in 2013 is how much more than the Number of boys in Udaygiri in 2013, given that
ratio of strength of Aravali, Nilgiri, Udaygiri and Shivalik is 2 : 3 : 2 : 3 respectively
(a) 80 (b) 100 (c) 120
(d) 90 (e) 10

Solutions
55 2 50 2
1. (a); Total boys in section ‘X” from CMS & Loyola 300× × +250× × 110+50
55 7 50 3 = 100 3
=
100 5
= 80
= 360 × × + 250 × × 2 2
100 9 100 5 Average number of boys in section ‘Y’ from LPS &
= 154 + 75 = 229 CMS
Total girls in section ‘Y’ from DPS & SKD (100–48) 4 (100–55) 1
(100 –60) 10 (100–55) 1 450× × +360× ×
= 420 × × + 300 × × = 100 9 100 3
100 21 100 3 2
104+54
= 80 + 45 = 125 = = 79
229 2
Required percentage = × 100 = 183.2% Required difference = 80 – 79 = 1
125

2. (b); Total girls in section ‘Y’ from CMS & SKD 5. (b); Total girls in section ‘Y’ from CMS & LPS together
(100–55) 2 (100 –55) 1 (100–48) 5 (100–55) 2
= 360 × 100 × 3 + 300 × 100 × 3 = 450 × 100 × 9 + 360 × 100 × 3
= 108 + 45 = 153 = 130 + 108 = 238
Total boys in section ‘X’ from DPS & SKD Total boys in section ‘X’ from DPS & SKD
60 5 55 1 60 5 55 1
= 420 × 100 × 9 + 300 × 100 × 3 = 420 × × + 300 × ×
100 9 100 3
= 140 + 55 = 195 = 140 + 55 = 195
153
Required ratio = = 51 : 65 Required difference = 238 – 195 = 43
195

3. (d); Total number of boys in section ‘X’ from D.P.S 6. (a); Applicants got final selection in the year 2017=
60 1 40 25
school 8400 × × × × = 63
100 8 100 100
60 5
= 420 × 100 × 9 = 140 Applicants got final selection in the year 2014
75 30 25 45
Required number of students = 140 ×
5 = 3200 × 100 × 100 × 100 × 100 = 81
7 81−63
= 100 Required percentage = 81
× 100
18 2 2
4. (e); Average number of girls in section ‘X’ from SKD = 81 × 1100 = 9 × 100 = 22 9 %
& Loyola
142 Adda247 Publications For More Study Material
Visit: adda247.com
Cracker Book for Bank (IBPS | SBI | RRB PO | Clerk) Mains Exams
18 30 24 20
7. (d); Total applicants got final selection in the year’s = 12500 × 100 × 100 + 12500 × 100 × 100 +
2013 & 2016 together 36 40
85 25 40 25
12500 × 100 × 100
= 2400 × × × × + 7500 ×
100 100 100 100 = 675 + 600 + 1800 = 3075
80 25 20 30
× 100 × 100 × 100 Required difference = 3675 – 3075 = 600
100
= 51 + 90 = 141 13. (b); Average number of jubali ticket sold by Multiplex
Total applicants got final selection in the year B&C
18 40 24 35
2014 & 2015 together 12500× × +12500× ×
75 30 25 45 = 100 100 100 100

= 3200 × × × × + 4800 × 900+1050


2
100 100 100 100
90 20 25 25 = = 975
100
× 100 × 100 × 100 2
Average number of gold class ticket sold by
= 81 + 54 = 135
multiplex C & E
Required difference = 141 – 135 = 6 24 20 36 40
12500× × +12500× ×
8. (a); Applicants qualified in tie II but did not get final = 100 100 100 100
2
600+1800
selection in the years 2017 = = 1200
60 1 40 (100−25) 2
= 8400 × 100 × 8 × 100 × 100 = 189 Required sum = 975 + 1200 = 2175
Applicants qualified in tier but did not qualify for
14. (d); Total silver class ticket sold by multiplex A & D
tier II in the year 2014 12 50 10 50
75 30 (100−25) = 12500 × × + 12500 × ×
= 3200 × × × = 540 100 100 100 100
100 100 100 = 750 + 625 = 1375
189
Required ratio = 540 = 7 : 20 Total Gold glass ticket sold by B & E
18 30 36 40
9. (d); Total appeared applicants for tier I in the year = 12500 × 100 × 100 + 12500 × 100 × 100
60
2017= 8400 × 100 = 5040 = 675 + 1800 = 2475
2475−1375
Total appeared applicants for tier II in the year Required percentage = × 100
2475
2015 1100 4
90 20 = 2475 × 100 = 44 9 %
= 4800 × 100 × 100 = 864
5040−864 15. (b); Let price of one Gold ticket in multiplex D, C and
Required percentage = × 100
4176 1
864 A be Rs.10x, Rs6x and Rs7x respectively.
= × 100 = 483 % ATQ,
864 3
Total earning = 40000
10. (b); Total number of applications did not qualify in 10 10 24
tier I in the year 2014, 2016 and 2017 12500 × × × 10x + 12500 × ×
100 100 100
75 (100−30) 80 20 12 30
= 3200 × × + 7500 × × × 6x + 12500 × 100 × 100 × 7x
100 100 100 100
(100−25) 60 (100−12.5)
+ 8400 × 100 × 1250x + 3600x + 3150x = 40000
100 100
40000
= 1680 + 4500 + 4410 = 10590 x= 8000
11. (e); Total Jubali class ticket sold by multiplex C & E x = 5 Rs
24 35 36 25
= 12500 × 100 × 100 + 12500 × 100 × 100 Required difference = 5 × 10 – 5 × 7
= 1050 + 1125 = 2175 = 15 Rs.
Total gold class ticket sold by multiplex A & C
12 30 24 20 16. (b); Total Samsung mobile sold by store on Saturday
= 12500 × 100 × 100 + 12500 × 100 × 100 2
= 750 × 3 = 500
= 450 + 600 = 1050
Required percentage =
2175−1050
× 100 Total Samsung 3G mobile sold on Saturday
1050 80 2
1 = 750 × 100 × 5 – 40 = 240 – 40 = 200
= 107 7 %
Total Samsung 4G mobile sold on Saturday
12. (d); Total silver class ticket sold by multiplex A, C = 500 – 200 = 300
and E
12 50 36 35 Total Samsung 4G mobile sold on Sunday
= 12500 × 100 × 100 + 12500 × 100 × 100 + 80 3
24 45
= 750 × × = 360
100 5
12500 × 100 × 100 360–300
Required percentage = × 100
= 750 + 1350 + 1575 = 3675 300
60
Total gold class ticket sold by multiplex B, C & E = 300 × 100 = 20%

143 Adda247 Publications For More Study Material


Visit: adda247.com
Cracker Book for Bank (IBPS | SBI | RRB PO | Clerk) Mains Exams

17. (b); Total MI mobile sold on Monday 23. (b); A3 tickets sold in Eden and Kaloor stadium
=
960+640
=
1600
= 800 together
2 2 80 3 80 5
MI 3G mobile sold on Monday = 2000 × 100 × 8 + 1800 × 100 × 12
= 800 × (4+5+7) = 250
5 = 600 + 600 = 1200
A2 tickets sold in Patel and Eden stadium
250 250
Required ratio = 75 3 = = 25 : 27 together
840× × 270 75 4 80 5
100 7
= 2400 × 100 × 9 + 2000 × 100 × 8
18. (d); Total Oppo 4G mobile sold on Monday and = 800 + 1000 = 1800
Tuesday 1200 2
Required ratio = 1800 = 3
640×65 3 (4+7)
= 100
×4× 6
= 572
24. (c); Total number of A1 tickets sold by all five
Total Samsung & Lava 4G mobile sold by store on stadiums together
Sunday 15 25
750×80 3 75 4 = 1600 × 100 + 2400 × 100 + 2000
= × + 840 × × 20 20 30
100 5 100 7 × 100 + 1800 × 100 + 2500 × 100
= 360 + 360 = 720
720 –572 = 240 + 600 + 400 + 360 + 750 = 2350
Required percentage = × 100 2350
720 Required average = 5 = 470
148 5
= 720 × 100 = 20 9 %
25. (a); A1 and A2 tickets sold in Nehru stadium
15 85 9
19. (d); Average number of Samsung & Lava 2G mobile = 1600 × + 1600 × × = 960
100 100 17
sold by store on Sunday A1 and A2 tickets sold in Raipur stadium
20 25 30 70 3
(750× +840× )
= 100 100
=
150+210
= 180 = 2500 × 100 + 2500 × 100 × 7 = 1500
2 2 1500−960
Average number of Huwai & Oppo 3G mobile Required % = × 100 = 36%
1500
sold by store on Sunday
85 1 65 1 26. (d); Income of movie B in second week
960× × +640× × 272+104 30
= 100 3 100 4
= = 188 = × 180 = 54 crores
2 2 100
Required difference = 188 – 180 = 8 Income of movie C in third week
10
= 100 × 200 = 20 crores
20. (c); Total Vivo mobile sold by store on Sunday
75 4 85 2 Required ratio = 27 : 10
= (840 × × + 960 × × ) + 56
100 7 100 3
27. (d); Income of movie A in first week
= (360 + 544) + 56 = 960 50
960–640 = × 210 = 105 crores
Required percentage = 960
× 100 100
320 1 Income of movie A in second week
= × 100 = 33 % 30
960 3 = 100 × 210 = 63 crores
42
21. (e); Total A1 and A3 tickets sold in Patel stadium = % drop = 105 × 100 = 40%
25 75 5
2400 × 100 + 2400 × 100 × 9 Similarly,
= 600 + 1,000 = 1600 Income of movies B in first week
40
Total number of A1 and A2 tickets sold in Kaloor = 180 × 100 = 72 crore
stadium Income of movies B in second week
20 80 7 30
= 1800 × 100 + 1800 × 100 × 12 = 180 × =54 crore
100
18
= 360 + 840 = 1200 % drop = × 100 = 25%
72
1600−1200
Required % = × 100 Income of movie C in first week
1200 60
400 1 = 200 × 100 = 120 crores
= 1200 × 100 = 33 3 %
Income of movie C in second week
30
22. (d); A2 tickets sold in Nehru stadium = 200 × 100 = crores
85 9 60
= 1600 × 100 × 17 = 720 % drop = 120 × 100 = 50%
A3 tickets sold in Raipur stadium 70
Income of movie D in week 1 = 100 × 240
70 4
= 2500 × × = 1,000 = 168 crore
100 7
Required difference = 1000 − 720 = 280
144 Adda247 Publications For More Study Material
Visit: adda247.com
Cracker Book for Bank (IBPS | SBI | RRB PO | Clerk) Mains Exams
10 160+190+170+190+220 930
income of movie D in week 2 = 100 × 240 = 5
= 5 = 186
= 24 crore Average of budget allocated for Education in
144 5 year 2018-22
% drop = × 100 = 85 %
168 7 105+125+130+200+220 780
Hence D is the answer. = 5
= 5 = 156
80 Required sum = 186 + 156 = 342
28. (c); Foreign income of movie A = × 210
300
36. (e); Let, total number of manufactured cars by TATA
= 56 crores
20 & HONDA in the year 2016 be 6x and 7x units
Foreign income of movie C = × 200 respectively.
100
= 40 crores ATQ,
Required ratio = 7 : 5 75 (100−15) 80 (100−15)
7𝑥 × 100 × 100 − 6𝑥 × 100 × 100
25
29. (a); Total income of B from foreign = × 180 = 4590
100 75 85 80 85
= 45 crores 7𝑥 × × − 6𝑥 × × = 4590
100 100 100 100
Foreign income in second week 357𝑥 102𝑥
− = 4590
2 80 25
= 5 × 45 crores = 18 crores 1785𝑥−1632𝑥
= 400
= 4590
Income earn by movie B in second week in India 4590×400
30 𝑥 = 153 ⇒ x = 12,000 units
= × 180 – 18 = 36 crores
100
Total number of cars returned to HYUNDAI in
30. (c); Second week income of movie D 80 15
2017 = (12000 × 6) × × − 1920
10 100 100
= 100 × 240 = 24 crore = 8640 – 1920 = 6720
10
Third week income of movie C = × 200 Total number of cars returned to MARUTI
100
SUZUKI in 2017
= 20 crore 75 15
Second week income of movie A = (12000 × 7) × 100 × 100 + 4830
30
= × 210 = 63 crore = 9450 + 4830 = 14,280 units
100
Let total number of cars manufactured by
Second week income of movie B
30 HYUNDAI in 2017 be x units
= × 180 = 54 crore 70 15
100
117–44 73
𝑠𝑜, 𝑥 × 100 × 100 = 6720
Required % = × 100 = 6720×100×100
117 117 𝑥= ⇒ x = 64,000 units
70×15
31. (e); Average budget for sport Let total number of cars manufactured by
125+130+150+170+160 735
= = 5 = 147 MARUTI SUZUKI in 2017 be y units
5
so,
Average budget for Education 85 17.5
105+125+130+200+220 780 𝑦 × 100 × 100 = 14280
= 5
= 5 = 156
14280×100×100
147 𝑦= ⇒ y =96,000 units
Required% = 156 × 100 = 94.23 ≈ 94% 85×17.5
Required difference = 96000 – 64000
32. (c); Budget allocated for sport for male in 2022 = 32,000 units
3
= 160 × = 96 𝑐𝑟. 37. (c); Let total number of cars manufactured by
5
MARUTI SUZUKI in the year 2016 be 100x units
Budget allocated for sport for male in 2023
5 So, total number of cars manufactured by TATA
= 96 × 4 = 120 cr in 2017 be 140x units
24
Required increase = × 100 = 15% ATQ,
160 75 10 80 12.5
33. (b); Required ratio =
105+125+130
=
360 140𝑥 × 100 × 100 − 100𝑥 × 100 × 100 = 120
170+190+220 580 21𝑥
= 18 ∶ 29 − 10𝑥 = 120 ⇒ x = 240
2
Total number of cars returned to HONDA in 2017
34. (a); Budget allocated for Cricket and Badminton 80 12.5
together in 2019 = (240 × 100) × 100 × 100 + 2010
(5+4)
= 130 × (5+4+4) = 90 cr = 2400 + 2010 = 4410 units
Let total number of cars manufactured by
Budget allocated for Rural Education in 2022 HONDA in year 2017 be x units
6
= 220 × = 120 cr ATQ,
11 70 22.5
Required difference = 120 – 90 = 30 cr 𝑥 × 100 × 100 = 4410
4410×100×100
35. (d); Average of budget allocated for Health care in 𝑥= ⇒ x = 28,000 units
70×22.5
the year 2018-22
145 Adda247 Publications For More Study Material
Visit: adda247.com
Cracker Book for Bank (IBPS | SBI | RRB PO | Clerk) Mains Exams

38. (c); Let total number of cars manufactured by 80 5 80 4


20x × 100 × 9 − 20𝑥 × 100 × 9 = 640
HYUNDAI, MARUTI SUZUKI & HONDA in 2016 be 80𝑥 64𝑥
− 9 = 640 ⇒ x = 360
2y, 3y and 4y respectively. 9
ATQ, Total ‘200 CC’ bike sold by Yamaha
22 75 (100−70)
2𝑦×
65 20 80 12.5
× +3𝑦× ×
75 15
+4𝑦× × = 36000 × 100 × 100 × 100 = 1782
100 100 100 100 100 100
3
= 4040
13𝑦 3𝑦 9𝑦 42. (d); Let total bike manufactured by all five companies
+ + = 12120
50 10 20 = 100x
101y = 1212000 Total ‘150 CC’ bike sold by Bajaj
y = 12,000 = 100x ×
25
×
90
×
40
= 9x
Required average 100 100 100
120 125 112.5 Total ‘150 CC’ bike sold by Hero
(12000×2)× +(12000×3)× +(12000×4)× 15 90 35
= 100 100 100
= 100x× × × = 4.2x
3 100 100 100
28800+45000+54000 127800 9𝑥−4.2𝑥
= = = 42,600 Required percentage = × 100
3 3 4.2𝑥
2
39. (d); Let total number of cars manufactured by = 1147 %
HYUNDAI & HONDA in the year 2016 be 8x and 43. (d); Let total bike manufactured by all five companies
9x units respectively. = 100x
ATQ, ATQ –
65 (100−20) 75 (100−15) 75 70 80 75
8𝑥 × × + 9𝑥 × × 22x × 100 × 100 − 18𝑥 × 100 × 100 = 450
100 100 100 100
= 39590 11.55x − 10.8x = 450
104𝑥 459𝑥
+ 80 = 39590 0.75x = 450 ⇒ x = 600
25 Total bike manufactured by Hero & Bajaj
3959𝑥
400
= 39590 ⇒ x = 4,000 units (15+25)
= 60000 × 100 = 24000
Total number of cars manufactured by HONDA in
36000 44. (d); Let total bike manufactured by all five companies
2017 = 3 × 5 = 60,000
= 100x
Total number of cars manufactured by HYNDUAI 75 35 80 40
32000 15x × × + 25𝑥 × × +
in 2017 = × 3 = 48,000 100 100 100 100
2 90 75
Required percentage =
60000−48000
× 100 18𝑥 × 100 × 100 = 23124
48000
12000 3.9375x + 8x + 12.15x = 23124
= × 100 = 25%
48000 x = 960
40. (d); Let total number of cars manufactured by TATA, Total bike sold by Honda
HYUNDAI & HONDA in the year 2017 is 7y , 6y & = 96000 ×
20
×
80
= 15360
8y units respectively 100 100

ATQ – 45. (e); Total ‘200 CC’ bike manufactured by Honda


75 100−10 70 100−15
7𝑦 × 100 × 100 + 6𝑦 × 100 × 100 + = 84000 ×
20
×
3
= 10080
70 100−22.5 100 5
8𝑦 × × = 75810 Total ‘150 CC’ bike sold by Yamaha
100 100
189𝑦 357𝑦 217𝑦 22 75 (100−70)
40
+
100 50
+ = 75810 = 84000 × × × = 4158
100 100 100
945𝑦+714𝑦+868𝑦
200
= 75810 Required percentage =
10080−4158
× 100
10080
2527y = 75810 × 200 ⇒ y = 6,000
= 58. 75%
Total number of cars manufactured by TATA,
HYUNDAI & HONDA together in the year 2017 Solutions (46-50): Let total employees travelling by
= 7 × 6000 + 6 × 6000 + 8 × 6000 metro & by Cab be 5𝑥 & 4𝑥 respectevely.
= 42000 + 36000 + 48000 = 1,26,000 15 10
5𝑥 × x 100 – 4𝑥 × x 100 = 420
41. (b); Let total bike manufactured by all five companies 75𝑥 40𝑥
= 100x − = 420 ⇒ X=1200
100 100
So, total bike manufactured by Honda ∴ Total employees travelling by metro = 1200 x 5
= 20x = 6000
Ratio between total ‘200 CC’ & ‘150 CC bikes sold & total employees travelling by cab = 1200 x 4
by Honda = 5 : 4 = 4800

146 Adda247 Publications For More Study Material


Visit: adda247.com
Cracker Book for Bank (IBPS | SBI | RRB PO | Clerk) Mains Exams

46. (c); Total employees travelling on Friday by metro = 12.5


= 72000 × 100 = 9000
50
300
x 6000 = 1000 In city B,
Total employees travelling on Sunday by cab = Number of students appeared in mains exam =
20
x 4800 – y =960 – y 60
100 80,000 × 100 = 48000
ATQ,
960 – y + 2 x 1000 Number of students passed in mains exam
25
=
75 15
[ × 6000 +
25
× 4800] = 48000 × = 12000
100 100 100 100
2960 – y = .75 x 2100 Total number of students who failed only in
Y = 2960 - 1575 = 1385 quant in city A = 3 × 12000 = 36000
47. (b); No. of employees travelling by metro on Total number of students who failed in both
Wednesday & Thursday together. subjects in city ‘A’
15 10 25
= × 6000 + × 6000 = 72000 – 9000 – 36000 – 72000 ×
100 100 100
= 900 + 600 = 1500 = 9000
No. of employees travelling by Cab on same days Total number of students who failed in both
together. exam in city ‘B’ = 9000 – 3000
25 20
= × 4800 + × 4800 = 6000
100 300
= 1200 + 320 = 1520 Total number of students who failed in
Required difference = 1520 – 1500 = 20 37.5
Reasoning = 48000 × + 6000
100
48. (d); Fare per person travelling by metro
5 = 18000 + 6000 = 24000
= × 120 = Rs. 50 36000−24000
12 Required % = × 1000 = 50%
Fare per person travelling by cab . 24000
7
= × 120 = 𝑅𝑠 70 Vein diagram for failed students:-
12
20
×4800×70
Require ratio = 300
10 = 56 : 75
×6000×50
100

49. (a); Total employees travelling by metro on Monday


20
= 100
× 6000 = 1200
48000
∴ Per person fare = = 𝑅𝑠. 40
1200
∴ Total fare on Monday of all employees
travelling by cab
25
= × 4800 × 40 = 𝑅𝑠. 48000
100

50. (e); Total No. of employees travelling by metro on


Friday & Saturday together
50 15
= 300 × 6000 + 100 × 6000
= 1000 + 900 = 1900 52. (e); In city D,
Total no. of employees travelling by cab on same Number of students appeared in mains exam =
85
days together 1,60,000 × 100 = 1,36,000
20 10
= 100 × 4800 + 100 × 4800 Number of students passed in mains exam
25
= 960 + 480 = 1440 = 1,36,000 × 100 = 34,000 = 𝑥
1900−1440
Required Percentage = × 100 Total number of students who failed in reasoning
1440
approximately 32% = 34,000 + x
Total number of students who failed in Quant =
51. (b); In city A, 75
Number of students appeared in mains exam = 1,36,000 × − 34,000 = 68,000
100
75
96000 × 100 = 72000 Let Total number of students who failed in only
Number of students passed in mains exam Quant = a

147 Adda247 Publications For More Study Material


Visit: adda247.com
Cracker Book for Bank (IBPS | SBI | RRB PO | Clerk) Mains Exams

And = 26,000
Let Let Total number of students who failed in Total number of students who failed in Quant
both subjects = b 65 80 50
= 120000 × 100 × [100 − 300] = 49,400
ATQ,
Total number of students who failed in both
𝑎 + 𝑏 = 68,000
exams = 49,400 − 26,000 = 23,400
And 34,000 + 𝑏 = 𝑎 + 34,000 23,400
So 𝑎 = 𝑏 = 34,000 Required % = 78000 × 100 = 30%
Total number of students who failed in
Reasoning = 34,000 + 34,000 = 68,000
Total number of students who failed in only
Quant = 34,000
68,000 2
Required Ratio = 34,000 = 1

55. (c); Total number of students who failed in atleast


one subject in city ‘A’
75 87.5
= 96,000 × 100 × 100 = 63,000
Total number of students who failed in at most
one subject in city ‘F’ = 63,000 − 24,000 =
53. (b); Total number of students who failed in Quant in 39,000
city ‘E’ = Total number of students who failed in Total number of students who failed in only
Mains exam – Total number of students who Quant in city ‘F’
failed in only Reasoning 2 31.25+25
= 39,000 − 72,000 × 3 × 100 = 12,000
72 70−40
= 1,25,000 × 100 × 100 = 27,000 Total number of students who failed in both
Total number of students who failed in Quant in subjects
city ‘F’ = Total number of students who failed in 2 68.75−25
= 72,000 × 3 × 100 − 12,000
Mains exam – Total number of students who
failed in only Reasoning = 21,000 − 12,000 = 9,000
2 68.75−25 Total number of students who failed in only
= 72,000 × 3 × 100 = 21,000 reasoning in city ‘A’
Required difference = 27,000 − 21,000 = 96,000 ×
75
×
25
= 18,000
= 6,000 100 100
Required difference = 18,000 − 9,000
54. (e); Total number of students who failed in Quant in = 9,000
60 75−37.5
city ‘B’ = 80,000 × 100 × 100
= 18,000
Total number of students who failed in at most
18,000
one subject in city ‘C’ = 30 × 91
= 54,600
In city ‘C’
Total number of students who failed in at most
one subject in city ‘C’ = Total number of students
who failed in Reasoning only + Total number of
students who failed in only Quant + Total
number of students who passed in both subjects
Total number of students who passed in both
65 20
subjects = 1,20,000 × 100 × 100
56. (b); Let Number of girls and boys in Aravali in 2012
= 15,600
Total number of students who failed in only be x and y respectively.
65 50 In 2013
Reasoning = 1,20,000 × ×
100 300 Number of girls = 1.2 x
= 13,000 Change in girls = 0.2x
Total number of students who failed in only 0.2𝑥
Change in boys is = 125 × 100 = 0.16x
Quant = 54600 − 15,600 − 13,000

148 Adda247 Publications For More Study Material


Visit: adda247.com
Cracker Book for Bank (IBPS | SBI | RRB PO | Clerk) Mains Exams

Total increment = 0.2x + 0.16x x + (y + 540) + 300 = 55a … (ii)


0.36x = 270 Solving (i) and (ii)
270×100
x = 36 ⇒ x = 750 a = 30
Number of boys in Aravali in 2013 Shivalik strength = 55a = 55 × 30 = 1650
2
= × 1.2 × 750 = 600 59. (e); Let in 2012
3
Total strength of Udaygiri and Nilgiri be 4x and
57. (c); Let Number of Girls in Aravali be x. 5x respectively
Let Number of girls who left Aravali and joins And total strength of Udaygiri and Nilgiri in 2013
Nilgiri is ‘a’
be 5y and 8y respectively
Total strength of Aravali increases which means
ATQ,
change in number of boys is
4x – 5y = 150 … (i)
= 270 + a
Two cases formed
Total strength of Nilgiri decreases, which means
Either
change in number of boys in Nilgiri is = 250 + a
270+𝑎 16 8y – 5x = 250 ... (ii)
250+𝑎
= 15 ⇒ a = 50 Or 5x – 8y = 250 ... (iii)
Number of boys increases in Nilgiri On solving (i) and (iii) it does give-ve integer
= 250 + 50 = 300 value
58. (a); Let Number of boys in Udaygiri in Shivalik in So, on solving (i) and (ii)
2012 = x x = 350 ⇒ y = 250
Let Number of Girls in Udaygiri in 2012 = y Strength of Nilgiri in 2012 = 1750
So, Number of Girls in Shivalik = y + 540 60. (d); Total strength in 2013 = 1030 + 970
In 2013, = 2000
Let Number of boys in Udaygiri and Shivalik = Number of boys in Aravali in 2013
20a =
2000
×2×
2
= 160
10 5
⇒ Number of Girls in Shivalik = 35a
Number of boys in Udaygiri in 2013
And, Number of Girls in Udaygiri = 12a 2000 5
ATQ = × 2 × = 250
10 8
x + y + 150 = 32a … (i) Required Difference = 250 – 160 = 90

149 Adda247 Publications For More Study Material


Visit: adda247.com
Cracker Book for Bank (IBPS | SBI | RRB PO | Clerk) Mains Exams

Chapter
Arithmetic Data Interpretation
14
BEST APPROACH TO SOLVE THE QUESTIONS

INTRODUCTION: DI stands for DATA INTERPRETATION. Interpretation means representing a data in a sensitive way
after analyzing it well. In all the banking exam or particularly quant section of bank, DI plays vital role. The basic DI
consists of pie table, bar graph, numerical value table or graph.
Look at this example:
900
800
700
Number of mobiles

600
500
400
300
200
100
0
Mon Tue Wed Thu Fri Sat

Instead of telling ‘number of mobiles sold’ for each day it is represented in the graph. So that it is more visible and
interested for a reader in this form.
Common form of DI are ratios, percentages or average.

ARITHMETIC DI
Arithmetic DI means representing arithmetic equations and conditions in bar graphs, line graph, pie graph or in any
other form of DI. The questions on arithmetic DI also cover major portion in mains of any banking exam. So it is
necessary to understand the actual concept. Easy way to understand this is through an example.

Total Road construct = 3600 meters

E
A
46°
72°

D
90°
B
80°

C
72°
In this pie chart, portion of ‘Road construct’ is given for 5 different companies. For eg. Company A constructs
72
360
× 3600 = 720 meters road. Now to make question more complicated different individual for each team may be
given. Let us consider through a bar graph.

150 Adda247 Publications For More Study Material


Visit: adda247.com
Cracker Book for Bank (IBPS | SBI | RRB PO | Clerk) Mains Exams

32
28
24

No. of Labour in 1 team


20
16
12
8
4
0
A B C D E
Name of teams
Here we have considered number of individuals in each team. Now they may ask you questions like “How many meters 1
member of team A constructs”. It is definitely easy one. We just had to do
720
= 40 meters.
18
or to make it more complicated they may ask you
“How many day will 8 members of team C will take to construct 3600 meters road?”
Do it yourself.
HINT: Calculate road constructed by an individual of team C in 1 day as we have calculated previously. Then calculate
how many days 8 men will take to construct given road.
A key point to solve any arithmetic DI is that to understand what had been given in DI. Then consider these as an
individual separate questions from arithmetic, also these questions are way easier. Now let us consider another
example:

25% Series 1
20%
20%
15%
15% 12%
10%
10%
5%
5%

0%
A B C D E
Look carefully, here a % graph is given and nothing have been mentioned.
Examiner may mention it “% of distance covered by these (A, B, C, D & E) individuals in 1 hour and total distance is 100
km” OR
“ % of work done by individuals in given time” OR
“ Profit/loss % earned on selling these items for shopkeeper”
“ Interest rate given by a bank to different individuals” or many other things.
Point is arithmetic DI is not as hard as student consider it in exams, it is always easier than actual arithmetic questions
that are asked in exam. Before solving a question, we must exactly know the information he had given. For eg. Consider
that this graph shows profit % for selling these items. In question, if examiner ask to calculate actual selling price, then
he must mention the CP or some other clue in exam.

151 Adda247 Publications For More Study Material


Visit: adda247.com
Cracker Book for Bank (IBPS | SBI | RRB PO | Clerk) Mains Exams

Example: Profit earned on selling item C is Rs. 160, what is amount of profit earned for item D, if both items have
same selling price?
Solution: This is a typical DI question. Now look, if we know that 20% means Rs 160 we can calculate actual CP of
item C. (Profit % is always calculated on CP).
100
CP OF ITEM C = 160 × = 800.
20
It means SP of item C is Rs. 960
Also SP of item D is Rs. 960. And then we can calculate CP and amount of profit of item D.

Practice Exercise Based on new Pattern

Direction (1 - 4): Given below bar graph shows number of hours taken by six person to complete a task individually.
Read the data carefully and answer the questions:

50

40

30

20

10

0
Neeraj Aniket Saurabh Gopal Veer Sameer

1. Neeraj, Saurabh and Gopal start working together but due to bad health the efficiency of Neeraj and Gopal
1 1
decreases by 12 % and 33 % respectively. Then find in how many hours task will be completed by all three?
2 3
1 1 1
(a) 11 hours (b) 10 hours (c) 12 hours
4 4 4
1 1
(d) 9 4 hours (e) 13 4 hours
2. Veer and Sameer started working together on another task, where Sameer work with 25% less efficiency. Veer and
Sameer work for y hours and remaining work complete by Aniket in (y + 1) hours, if ratio of work done by Veer
and Sameer together and by Aniket alone is 2 : 1, then in how many hours Neeraj will complete same task alone?
1 1 1
(a) 15 hours (b) 13 hours (c) 17 hours
2 2 2
1 1
(d) 11 hours (e) 9 hours
2 2

3. If Divyaraj can do 50 % more work in one hour as Neeraj can do in one hour, while Manish can do 25% less work
in one hour as Aniket can do in one hour. Saurabh started working alone and after some time he left and remaining
work completed by Divyaraj & Manish together in 11.75 hours more than time for which Saurabh worked. Then
find total time in which work is completed?
1 1 1
(a) 14 hours (b) 12 hours (c) 10 hours
4 4 4
1 1
(d) 16 4 hours (e) 84 hours

4. Neeraj, Aniket, Veer and Sameer worked together on a work in first hour, while Saurabh & Gopal together
destroyed the same work in second hour. If this work continues till the total work completed. Find how many
hours required to complete the whole work?
270 270 270
(a) 22 hours (b) 20 hours (c) 18 hours
401 401 401
270 270
(d) 24 hours (e) 16 hours
401 401

152 Adda247 Publications For More Study Material


Visit: adda247.com
Cracker Book for Bank (IBPS | SBI | RRB PO | Clerk) Mains Exams

Direction (5 - 9): The given bar-graph shows the number of marbels of different colors which are contained by two
different bags X and Y. Read the data carefully and answered the following questions.

X Y
7
6
5
4
3
2
1
0
Yellow Red Black Blue

5. If two marbels are picked at random from bag Y then what is the probability that both marbels are blue in color?
21 19 63
(a) (b) (c)
190 190 190
11 10
(d) 95 (e) 99

6. If two marbels are picked at random from bag X then what is the probability that both are yellow in color?
3 4 7
(a) (b) (c)
53 51 59
5 8
(d) (e)
51 59

7. If two marbels are picked at random from bag Y then what is the probability that one is red and one is black in
color?
1 6 7
(a) (b) (c)
19 95 95
8 13
(d) 95 (e) 190

8. If two marbels are picked at random from bag X then what is probability that either both are black or both are blue
in color?
1 4 9
(a) 15 (b) 105 (c) 115
3 1
(d) 37 (e) 17

9. If three marbels are picked at random from bag Y then what is the probability that one is black, one is blue and one
is yellow in color?
17 49 53
(a) 95 (b) 285 (c) 381
13 47
(d) 95 (e) 360

153 Adda247 Publications For More Study Material


Visit: adda247.com
Cracker Book for Bank (IBPS | SBI | RRB PO | Clerk) Mains Exams

Directions (10-14): Given below pie chart (I) shows percentage distribution of total filled quantity of water in seven
tanks. Line graph shows time (minutes) taken by seven pipe A, B, C, D, E, F and G in filling tanks P, Q, R, S, T, U and V
respectively. Give the answer of the question according to given data :

65
Total quantity of seven tanks = 1500 ℓ
60
P 55
V

Time (in minutes)


15% 50
12%
45
U Q 40
18% 12%
35
R 30
T 8% 25
15% S 20
20% A B C D E F G
Pipes

10. A man has a tank which has quantity of water equal to the quantity of water in P and T together. He opened pipe B
and D together for filling his tank. After X minutes man closed both pipes and opened pipe C and E for filling
remaining tank. If tank filled in next (x + 15) minutes then find how many liter of water filled by B and D together ?
(a) 160 l (b)180 l (c)150l
(d)120l (e) 144 l
11. For filling tank S, man opened pipe D which fills the tank with 20% more efficiently. After X minutes man close the
pipe D and opened pipe C and G together. Pipe C fills the tank with 25% more efficiently. If pipe C and G fills
remaining tank in 15 minutes then find the value of X ?
(a) 12 minutes (b) 10 minutes (c) 15 minutes
(d) 8 minutes (e) 18 minutes
12. Quantity of another tank K is 120% of total quantity of all seven tank. For filling the bigger tank man opened pipes
alternatively in such a way that three pipe B, D and G together for first minute, pipe A alone for second minute and
pipe F and C together for third minutes. Find the total time required in filling the tank alternatively?
(a) 151 7 minutes (b)141 7 minutes (c)151 minutes
9 9
(d)150 minutes (e) 170 minutes
13. A man have a tank M, which has equal quantity of water as tank S and U together . For filling tank M man opened
17
pipe A, C and G together for some minutes man thought tank will be fully filled, but it takes 7 19 minutes more
because of a leak. Find in what time leak will empty the fully filled tank?
(a) 3hr (b) 3 2 hr (c)2 1 hr
3 6
1
(d) 3 1 hr (e) 4 3 hr
6
14. A man have a tank of 570 liter. Man opened three pipes B, D and C together for filling the tank but pipe C fills the
tank at 175% of its initial Efficiency. There is a leak at a height of 2/3 from the bottom of tank ,which empty the
tank with half of the efficiency of what B, D and C filling the tank. Man noticed the leak after 32 minutes and closed
all pipes and leak. Find how many liter of tank remain unfilled?
(a) 90l (b) 76l (c) 78l
(d) 85l (e) 96l

154 Adda247 Publications For More Study Material


Visit: adda247.com
Cracker Book for Bank (IBPS | SBI | RRB PO | Clerk) Mains Exams

Directions (15 − 𝟏𝟗): Given below the table shows Investment of five person, time and share of profit. Some data are
missing, calculate that according to the question and answer the questions –

Person Investment (Rs) Time (months) Profit share (Rs.)


P — 12 65700
Q 32000 — —
R — — —
S 24000 — —
T — — 24300
15. P, Q and S invested in a business together, investment of P is 75% of investment of S and all three invested for
same time. If Q and S gets extra 12% and 15% of total profit respectively and remaining profit is distributed
according to their share, then find the total profit.
(a) 270000 Rs. (b) 370000 Rs. (c) 375000 Rs.
(d) 275000 Rs. (e) 325000 Rs.
16. Person Q and R invested in the ratio of 4 : 3 and ratio between time period of Investment Q to R is 5 : 3. If both
person agree that 65% of the total profit should be divided equally and remaining profit is to be divided into ratio
of their capital. If Q gets 10010 Rs. more to R. then find the total profit share of R ?
(a) 27765 Rs. (b) 28190 Rs. (c) 42497 Rs.
(d) 34307 Rs. (e) 32695 Rs.
17. Q started a business with his investment, after some month S came to joined with him and invest his amount in
business. At the end of 3 year, the ratio of profit of Q to S is 3 : 2. Find after how many month S joined the business?
(a) 4 months (b) 6 months (c) 3 months
(d) 5 months (e) 6 months
18. If T received Rs. 24300 as profit out of the total profit of Rs. 40500 which T and R earned at the end of one year. If
T invested Rs. 81000 for 9 months, whereas R invested his amount for the whole year, what was the amount
invested by R ?
(a) 30500 Rs. (b) 40500 Rs. (c) 35500 Rs.
(d) 32500 Rs. (e) 40050 Rs.
19. If Q and R invested into the ratio of 8 : 9 and R and S Invested into the ratio of 3 : 2. At the end of the year if they all
got a total profit of 37030 Rs. then find the share of profit of Q, R and S individually ?
(a) 14490 Rs, 9660 Rs, 12880 Rs. (b) 9660 Rs, 17710 Rs, 11005 Rs.
(c) 12880 Rs, 14490 Rs, 9660 Rs. (d) 12434 Rs, 13594 Rs, 11006 Rs.
(e) None of these
Direction (20- 23): Data about investments of different persons is given below. Study the data and solve the questions
carefully.
→ Investment of Neeraj is 50% more than investment of Aman while investment of Neeraj is 25% less than that of
Sandeep.

Total Investment = 40,000

Rakesh
, 30%
Saurabh,
40%

Sandeep,
30%

155 Adda247 Publications For More Study Material


Visit: adda247.com
Cracker Book for Bank (IBPS | SBI | RRB PO | Clerk) Mains Exams

20. Neeraj and Saurabh started a business together. After 8 months of business they invested Rs 1000 per month for
every month. If annual profit is Rs 8000 then find Saurabh’s profit out of total profit?
(a) Rs 2950 (b) Rs 4000 (c) Rs 5050
(d) Rs 4450 (e) Rs 4850
21. Aman and Rakesh started a business together. Rakesh left the business ‘x’ months before the completion of year
while Aman increased his investment by Rs 2000 after 8 months and then after 2 months more he increased his
investment by Rs 2000 again. If ratio between profit share of Aman and Rakesh is 7 : 8 then Rakesh worked for
how many months?
(a) 8 months (b) 4 months (c) 6 months
(d) 2 months (e) 10 months
22. Sandeep and Neeraj started a business together. After 8 months, Neeraj is replaced by another partner, Satish
whose investment is Rs 4000 more than investment of Saurabh. If Satish worked for 3 months, then find the profit
share of Sandeep if annual profit is Rs 11,500.
(a) Rs 2500 (b) Rs 3000 (c) Rs 4500
(d) Rs 5000 (e) Rs 6000
23. Aman invested his amount in a scheme which after 20% p.a. at C.I. for 2 years while Saurabh invested his amount
in a car whose value will depreciate at 20% p.a. every year. Find the total value of amount Aman and Saurabh will
have after 2 years.
(a) Rs. 31,680 (b) Rs. 18,880 (c) Rs. 26,880
(d) Rs. 16,320 (e) Rs. 22,880
Direction (24 – 25): Given below table shows quantity of four different liquid in four different vessels. Read the data
carefully and answer the questions.

Vessels Milk (ml) Water (ml) Orange Juice (ml) Mango juice (ml)
P 240 — 144 —
Q 160 80 — 210
R — 144 192 —
S 160 — — 120

24. In vessel P quantity of water is 50% of quantity of milk in same vessel, while quantity of mango juice in vessel P is
6.25% less than quantity of Orange juice in vessel R. In vessel S quantity of water is 50% more than quantity of
milk in same vessel and quantity of Orange juice in vessel S is 40 ml less than quantity of water in same vessel. If
114 ml mixture from vessel P and 180 ml from vessel S taken out and mixed in vessel T, then find percentage of
orange juice in vessel T?
25 25 25
(a) 29 147 % (b) 25 147 % (c) 27 147 %
25 25
(d) 23 147 % (e) 21 147 %
1
25. Quantity of milk in vessel R is 33 3 % less than quantity of water in same vessel and quantity of orange juice in
vessel Q is 25% less than quantity of milk in same vessel. For making 1086 ml mixture solution mixture of vessel Q
& R mixed together, if ratio of quantity of total mango juice in vessel R to total quantity of four liquid in vessel R is
7 : 43, then find the ratio between total mango juice in resulting mixture to total given mixture?
49 49 49
(a) 161 (b) 151 (c) 181
49 49
(d) (e)
121 111

156 Adda247 Publications For More Study Material


Visit: adda247.com
Cracker Book for Bank (IBPS | SBI | RRB PO | Clerk) Mains Exams

Direction (26-30): - Table given below shows details of 3 types of item of a garment shop MRP, discount% (on MRP)
and certain offer.
Shopkeeper have shoes, Jeans & T-Shirts in 2 types of each.
S₁, S₂; J₁, J₂; and T₁, T₂ represents the types of two types of shoes, jeans and T-shirts respectively.

Items Shoes Jeans T-shirts


Types S₁ S₂ J₁ J₂ T₁ T₂
MRP (Rs) 1200 1647 1200 — 400 600
1
Discount 0% 11 9 % 0% 25% 10% 0%
Shop offer      

Note: Shop offer: Buy 2 get 2 free


 → Offer given  → Offer not given
26. If a person wants to buy T-shirts, what is minimum average price/t-shirts he could get?
(a) Rs. 400 (b) Rs. 360 (c) Rs. 300
(d) Rs.600 (e) Rs. 500
27. If cost price of J₂ for shopkeeper is Rs. 1000. Find MRP of J₂ if he earns a profit of 20% on selling a single piece.
(a) Rs. 1600 (b) Rs. 1800 (c) Rs. 1500
(d) Rs. 1700 (e) None of these
2
28. Find the cost price of S₂ if he earns a profit of 14 % on C.P.
7
(a) Rs. 1464 (b) Rs. 1098 (c) Rs. 1281
(d) Rs. 1379 (e) None of these
29. A boy buys 5 pieces of T₂, 4 pieces of J₁, & 1 piece of S₁ including offer. Find the amount paid by him (in Rs.)
(a) 6000 (b) 7200 (c) 4800
(d) 3600 (e) 5400
30. If a person buys 4 pieces of T₂, 2 pieces of T₁ & 4 pieces of J₁ including offer. Find overall discount% obtained by
him.
(a) 40% (b) 44% (c) 42%
(d) 46% (e) 48%
Directions (31-35): Given below line graph shows the distance travelled by car A in given time intervals and table
shows ratio of average speed of car A to average speed of car B in same time intervals. Both car started at 12:00 Noon.

90

80

70

60

50

40

30
12:00 – 1:00 1:00 – 2:00 2:00 – 3:00 3:00 – 4:00 4:00 – 5:00

157 Adda247 Publications For More Study Material


Visit: adda247.com
Cracker Book for Bank (IBPS | SBI | RRB PO | Clerk) Mains Exams

Time Internal Ratio


12:00 – 1:00 6:5
1:00 – 2:00 14 : 11
2:00 – 3:00 6:7
3:00 – 4:00 4:3
4:00 – 5:00 4:7

31. What is average speed of car A during whole journey?


(a) 63 km/hr (b) 65 km/hr (c) 62 km/hr
(d) 70 km/hr (e) 68 km/hr
32. If both cars started from same point at same time, what is difference between distance travelled by them till 3: 00
PM.
(a) 20 km (b) 15 km (c) 24 km
(d) 16 km (e) 12 km
33. Average speed of B between 2: 00 PM to 4: 00 PM is what % more or less than average speed of A in same time
interval?
1 2 4
(a) 7 7 % (b) 14 7 % (c) 28 7 %
5
(d) 35 % (e) None of these
7

34. If B have to cover a distance of 540 km till 9 : 00 PM, even after having a rest of 45 minutes between 5 : 00 to 5 : 45
PM. What should be his average speed for 4 hours between 5:00 PM to 9:00 PM.
1 4
(a) 60 km/hr (b) 59 km/hr (c) 58 km/hr
7 7
(d) 58 ¾ km/hr (e) 56 ⅔ km/hr
35. If A consumes 1liter petrol for every 15 km, and if A is 25% more economic than B. What is the ratio of petrol
consumed by them till2:00PM?
(a) 91 : 92 (b) 93 : 97 (c) 104 : 105
(d) 106 : 107 (e) 103 : 106
Directions (36-40): The following line graph shows principal amount(in 10000) submitted by 5 persons in a scheme
which offers simple interest and table shows the rate of interest (ROI) per annum obtained by these persons, time (in
years) for which they invested their amount and third column shows ratio of amount obtained after they withdraw their
money to that of interest obtained.
Some figures shown are missing. Calculate them according to question.

12
11
10
9
8
7
6
5
4
3
A B C D E

158 Adda247 Publications For More Study Material


Visit: adda247.com
Cracker Book for Bank (IBPS | SBI | RRB PO | Clerk) Mains Exams

Persons ROI Time A:I


A — 5 8:3
B 16% — 41 : 16
C — 6 —
D 8% 5 —
E 15% — 29 : 9

36. What is the time period in months for which B invested his money?
(a) 60 (b) 48 (c) 64
(d) 72 (e) None of these
37. If the ratio of amount obtained by A to that of C is 4: 3. Find the ROI obtained by C.
(a) 8% (b) 12% (c) 8.5%
(d) 10% (e) 12.5%
38. If a new person Vikas deposit half of the money invested by A, for 3 more years than C, calculate his ROI, if SI
obtained by him is same as SI obtained by D.
1 1 1
(a) 11 % (b) 9 % (c) 12 %
9 11 3
2
(d) 14 % (e) None of these
7

39. What is the ratio of time period for which E invested to that of B?
(a) 1 : 3 (b) 7 : 2 (c) 5 : 4
(d) 4 : 5 (e) 3 : 4
40. If after drawing his whole money, D invested 50% of this amount in CI for 2 years at ROI/annum of 10% and
remaining of the money he kept with him. Calculate total money with him after 2 years.
(a) Rs. 169400 (b) Rs. 84700 (c) Rs. 154700
(d) Rs. 167400 (e) Can’t be determined
Directions (41-44): Bar-graph shows the percentage distribution of distance covered in upstream and downstream by
Rahul on different dates.

Upstream distance=6000 km Downstream distance=2000 km


32
28
24
20
16
12
8
4
0
1st Aug. 2nd Aug. 3rd Aug. 4th Aug. 5th Aug.

Table given below shows the speed of current in km/hr on different dates.
Date speed of current (in km/hr)
1st Aug. 10
2nd Aug. —
3rd Aug. 4
4th Aug. —
5th Aug. 3

159 Adda247 Publications For More Study Material


Visit: adda247.com
Cracker Book for Bank (IBPS | SBI | RRB PO | Clerk) Mains Exams
1
41. If time taken by Rahul to swim upstream on 1st August. is 61 3 hr. more than time taken by him to swim
downstream on same date, then find the speed of Rahul in still water?
(a) 16 km/hr (b)30 km/hr (c) 25 km/hr
(d) 20 km/hr (e) None of these
42. If speed of Rahul in still water on 2nd Aug. is 8 km/hr and speed of Rahul in still water on 3rd Aug. is 25% more
2
than his speed on 2nd Aug. and time taken by him to travel upstream on 2nd August is 166 % more than time taken
3
by him to travel downstream on same date, then find the seven times of the speed of current on 2nd Aug.?
(a) 8 km/hr (b) 6 km/hr (c) 4 km/hr
(d)None of these (e) 2 km/hr
43. Speed of current on 4th Aug. is 50% of downstream speed of Rahul on 5th August, then time to cover 128 km
upstream on 4th Aug. is ? [Given that time of Rahul to cover upstream distance on 5th August is 80 hr and assuming
that speed of Rahul in still water is same on both days]
1 2
(a) 12 hr (b) 6 hr (c) 10 hr
3 3
(d) 21 hr (e)None of these
44. If ratio of speed of current on 2nd August to 4th August is 2 : 3 & speed of Rahul on both days are same and time in
upstream on 4th August. is 24 times the time taken in downstream on 2th August then find the speed of Rahul in
still water if speed of current on 3rd August is 50% of speed of current on 2nd Aug.(approximately)?
(a) 21 km/h (b) 23km/h (c) None of these
(d) 25km/h (e) 20km/h
Directions (45-48): The following bar-graph shows the ratio of speed of three different trains Rajdhani exp., Shatabadi
exp. and Duranto exp. on five different days of a week. And line graph shows the number of coaches attached to Rajdhani
express on different days of week. Length of each train is the sum of the length of all coaches and length of engine.
Rajdhani Exp Shatabadi Exp Duranto Exp
7

0
Monday Tuesday Wednesday Friday Sunday

30

25

20

15

10
Monday Tuesday Wednesday Friday Sunday
Length of each coaches for every train = 15 m
And length of engine for every train = 20 m
160 Adda247 Publications For More Study Material
Visit: adda247.com
Cracker Book for Bank (IBPS | SBI | RRB PO | Clerk) Mains Exams

45. On Tuesday, Rajdhani exp. crosses a tunnel of some length in 12.5 sec while Duranto express having length 25%
more than that of Rajdhani exp. crosses the same tunnel on the same day in 11.9 sec. Then find the time taken by
Rajdhani exp. and Shatabadi exp. on the same day to cross each other if they are running in opposite direction and
the length of Shatabadi exp. is 10 m less than that of Rajdhani exp.
(a) 8 sec (b) 9 sec (c) 10sec
(d) 11 sec (e) 12 sec
46. On Sunday, Duranto exp. having length 110 m less than that of Rajdhani exp. crosses Rajdhani exp. travelling in
8
opposite direction in 8 sec then find the approximate time taken by Shatabadi exp. to cross Duranto exp. on the
9
same day when both are running in the same direction and length of the Shatabadi exp. is 20% less than that of
Rajdhani exp.
(a) 79 sec (b) 81 sec (c) 75 sec
(d) 84 sec (e) 72 sec
47. On Friday, the speed of Rajdhani exp. was 90 km/hr and on the same day, Shatabadi exp. started from station P
and going to Q while Duranto started from station Q and going to P. If speed of Shatabadi is increased by 10% after
first hour and 20% after second hour whereas the speed of Duranto is increased by 20% after first hour and 25%
after second hour then they meet in 3 hours. Find the distance between P and Q.
(a) 947 km (b) 952 km (c) 955 km
(d) 957 km (e) 960 km
48. Duranto exp. having length 28% more than that of Rajdhani exp. on Monday crosses a platform of same length in
12.8 sec. Then find the difference between average speed of shatabadi and Rajdhani and the speed of Duranto exp.
on Monday.(in km/hr)
(a) 32.8 (b) 59.4 (c) 42.6
(d) 54.2 (e) 58.8
Directions (49-53): Pie chart given below shows length of six different trains and table given below shows ratio
between speed of six trains on three different days. Study the data carefully and answer the following questions.
Total length = 1600m

F, 8%
A,
16%
E,
22% B,
18%
D,
12%
C,
24%

Train Speed on Monday : Speed on Tuesday : Speed on Wednesday


A 2 : 3 : 2
B 3 : 4 : 5
C 4 : 6 : 5
D 4 : 4 : 7
E 6 : 9 : 5
F 4 : 5 : 3
49. On Wednesday, train ‘B’ crosses train ‘D’ coming from opposite direction in 6 seconds. If speed of train ‘B’ on
Monday is 97.2 km/hour then in how much time train ‘F’ can cross train ‘D’ on Monday if train ‘D’ is coming from
opposite direction and speed of train ‘F’ on Monday is 20 m/sec. (2 Marks)
(a) 6 seconds (b) 8 seconds (c) 10 seconds
(d) 12 seconds (e) 14 seconds

161 Adda247 Publications For More Study Material


Visit: adda247.com
Cracker Book for Bank (IBPS | SBI | RRB PO | Clerk) Mains Exams

50. Train ‘C’ start from Delhi on Monday at 7:00 p.m and reach Kanpur on next day at 3:00 pm. In return journey on
Tuesday, train ‘C’ start from Kanpur at 6 : 00 pm and reach Jaipur which is 180 km ahead of Delhi at 5 : 48 pm on
Wednesday. Find the time taken by train ‘C’ to cross a pole on Monday? (2 Marks)
(a) 8 seconds (b) 16 seconds (c) 19.2 seconds
(d) 14.4 seconds (e) 28.8 seconds
51. On Monday, train ‘A’ takes 2.5 hours more to cover 900 km distance than train ‘C’. If train ‘A’ can cross a platform
of length 128 in 12.8 seconds on Tuesday then find in how much time (in seconds) train ‘C’ can cross two poles 66
m apart from each other on Tuesday? (2 Marks)
(a) 12 seconds (b) 16 seconds (c) 20 seconds
(d) 24 seconds (e) 30 seconds
52. Ratio between speed of train ‘E’ to train ‘F” on Monday is 3 : 2. On Tuesday train ‘E’ cross train ‘F’ running in same
direction in 24 seconds then find the time in which train ‘E’ can overtakes train ‘F’ on Wednesday? (2 Marks)
(a) 48 seconds (b) 24 seconds (c) 12 seconds
(d) 36 seconds (e) 60 seconds
53. Ratio between time taken by train ‘B’ to train ‘D’ to cross a pole on Monday is 1 : 1. The time taken by train ‘B’ to
cross a pole on Wednesday is what percent more/less than time taken by train ‘D’ to cross a pole on Monday? (2
Marks)
(a) 30% (b) 40% (c) 50%
(d) 60% (e) 70%
Direction (54-58): Table given below shows profit percentage earned on selling two different items X and Y and
discount percentage offered by five retailers on these items. Mark price of each article sold by each retailer is same while
cost price of article for each retailor may vary. Study the data carefully & answer the following question
Items→ X Y
Retailor
Profit % Discount % Profit % Discount %

A 20% – – 15%
B – 26.5% 20% 32.5%
C 25% – 60% 24%
D 20% 34% – 37%
E 35% 46% – 28%
54. On article ‘X’, 28% discount is offered by ‘A’. If selling price of article ‘Y’ sold by ‘A’ is Rs 312 more than selling
price of article ‘X’ sold by ‘A’ then find profit percent earned by ‘A’ on selling article ‘Y’ given that average of cost
price of both article for ‘A’ is Rs 1520. (2 Marks)
(a) 18.5% (b) 22.5% (c) 27.5%
(d) 32.5% (e) 37.5%
1
55. Discount % on article ‘X’ offered by ‘C’ is 15% while profit % of article ‘Y’ sold by ‘E’ is 33 %. If difference between
3
cost price of article ‘Y’ sold by ‘B’ and ‘E’ together is Rs 216 more than cost price of article ‘X’ sold by ‘C’ and ‘E’
together then find the cost price of article ‘Y’ sold by ‘C’? (2 Marks)
(a) Rs. 4940 (b) Rs. 3420 (c) Rs. 3800
(d) Rs. 4180 (e) Rs. 4560
56. Find the ratio between cost price of article ‘X’ for ‘B’ to cost price of article ‘Y’ for ‘D’. If profit % of article ‘X’ sold by
‘B’ and profit % of article ‘Y’ sold by ‘D’ is 68% and 20% respectively. (2 Marks)
(a) 2 : 3 (b) 5 : 6 (c) 1 : 2
(d) 5 : 8 (e) 5 : 7
57. Selling price of article ‘X’ sold by ‘E’ is same as cost price of article ‘Y’ for ‘E’. Find the profit % earned by ‘E’ on
selling article ‘Y’? (1 Mark)
2
(a) 15% (b) 16 % (c) 25%
3
1 2
(d) 33 3 % (e) 41 3 %

162 Adda247 Publications For More Study Material


Visit: adda247.com
Cracker Book for Bank (IBPS | SBI | RRB PO | Clerk) Mains Exams

58. Seller A marked article ‘X’ 100% above its cost price. If on selling both article he earns total 25% profit, then find
selling price of article ‘X’ sold by ‘A’ is what percent less than cost price of article ‘Y’ for ‘A’. (2 Marks)
1 1
(a) 10% (b) 9 % (c) 8 %
11 3
(d) 20% (e) 15%
Directions (59-63): Line graph given below shows the distance between Delhi to five different cities in kilometer and
Table given below shows the speed of five different cars in km/hr
3500

3000
Distance from Delhi (in km)

2500

2000

1500

1000

500

0
City A City B City C City D City E

Cars Speed (in kmph)


P 40
Q —
R 60
S —
T 75

NOTE: - Some data is missing you have to calculate according to question.


59. Time taken by car ‘P’ to travel from city ‘E’ to Delhi and then Delhi to city ‘B’, is equal to the time taken by car ‘R’ to
travel from Delhi to city ‘A’ and then city ‘A’ to city ‘B’. Find the distance between city ‘A’ and city ‘B’.
(a) 5650 km (b) 5750 km (c) 5450 km
(d) 5550 km (e) 5320 km
60. Find the approximate time car ‘T’ takes to reach city ‘E’ from city ‘A’ if city ‘A’ and city ‘E’ is north and east
direction of Delhi respectively.
(a) 24 hours (b) 27 hours (c) 20 hours
(d) 36 hours (e) 42 hours
61. Car Q and Car S start from Delhi for city B and city C respectively and they reached in equal time. If Car Q and Car S
starts from city B and city D respectively at same time and move towards each other, then time taken by car Q to
cross car S is what percent of the time taken by car Q to reach city B from Delhi. Distance between city B and city D
is 1500 km.
(a) 25% (b) 20% (c) 30%
(d) 40% (e) 50%
62. A thief runs in a car S from Delhi to city E and after 6 hours of running, a policeman started to catch him in a car R.
3
Due to this, thief increases the speed of his car by 100%. By this, the policeman is able to catch him at 5th of the
distance of city E from Delhi. Find the initial speed of car ‘S’.
(a) 15 km/hr (b) 27 km/hr (c) 20 km/hr
(d) 25 km/hr (e) 40 km/hr
163 Adda247 Publications For More Study Material
Visit: adda247.com
Cracker Book for Bank (IBPS | SBI | RRB PO | Clerk) Mains Exams

63. Car P and Car Q start from Delhi for city A. Car Q first reaches at city A and meets car P in between the way, 200 km
from city ‘A’. Find after how much time they will meet second time after first time meeting if they continue their to
and fro motion.
(a) 24 hours (b) 15 hours (c) 16 hours
(d) 25 hours (e) 20 hours
Direction (64-68): - Line chart given below shows time taken by five different persons to complete a work ‘M’ alone.
Ratio of efficiency of all five persons remain same throughout any work. Study the data carefully and answer the
following questions.

70

60

50

40

30

20

10

0
Abhi Bhavya Neeraj Satish Veer

64. All five starts working together to complete work ‘X’. ‘Veer’ left after 8 days. Work done by ‘Bhavya’ is same as
work done by ‘Neeraj’ while ‘Abhi’ and ‘Neeraj’ worked for same time. ‘Satish’ worked for ‘y’ days. If ‘Bhavya’,
‘Neeraj’ and ‘Satish’ together can complete work ‘X’ in 24 days then find the value of ‘y’ if Bhavya worked for
starting 10 days.
(a) 7 days (b) 9 days (c) 11 days
(d) 13 days (e) 15 days
65. Abhi and Neeraj together can complete work ‘Z’ in (A + 42) days while Bhavya and Satish together can complete
work ‘Z’ in (A + 15) days. All start the work Z such that ratio between work done by Abhi, Bhavya and Veer is 1 : 2 :
3, while ratio between days, Neeraj, Satish and Veer worked is 2 : 2 : 1. Find how many days ‘Bhavya’ worked.
(a) 10 days (b) 15 days (c) 20 days
(d) 30 days (e) 40 days
66. All five persons started together to complete work ‘Y’. Veer worked for starting 6 days and left the work. After 3
days more both Bhavya and Satish left too. Remaining 40% work should be completed by Abhi and Neeraj
together but ‘Abhi’ left after ‘x’ days. Remaining work is completed by ‘Neeraj’ in ‘z’ days. If ‘z – x = 3’, then number
of days for which ‘Neeraj’ worked is what percent more than number of days for which ‘Abhi’ worked.
1 2
(a) 33 % (b) 50% (c) 66 %
3 3
(d) 75% (e) 100%
67. Abhi, Bhavya and Neeraj together starts to do work ‘M’. After 7 days ‘Neeraj’ left and after 3 days more ‘Abhi’ and
‘Bhavya’ left. Remaining work is completed by Satish and Veer working alternatively in ‘y’ days. If ‘y’ is integer
then find ‘Veer’ worked for how many days?
(a) 3 days (b) 4 days (c) 5 days
(d) 6 days (e) Cannot be determined
68. Abhi, Bhavya and Satish starts working together to complete work ‘M’. After 5 days, Bhavya and Satish replaced by
Neeraj and Veer. After 5 more days Abhi left the work. After 1 more day Veer left too. Neeraj worked for total ‘x’
days. In other case Abhi and Bhavya starts working together to complete ‘M’. After 4 days both are replaced by
Veer. Veer worked for 5 days and replaced by Satish who worked for 8 days. Remaining work is completed by
Neeraj in ‘y’ days. Find (y − x)2 ?
(a) 25 (b) 36 (c) 49
(d) 64 (e) 81

164 Adda247 Publications For More Study Material


Visit: adda247.com
Cracker Book for Bank (IBPS | SBI | RRB PO | Clerk) Mains Exams

Solutions
Solutions (1-4): Total work = 5040 units (LCM of days = 375 units
taken by all) 24
Total work in 24 hours = 375 × 2
5040
Efficiency of Neeraj = 35 = 144 units/hour = 4500 unit
5040 On 25th hour remaining work by Neeraj, Aniket,
Efficiency of Aniket = 30
= 168 units/hour
5040 Veer and Sameer together
Efficiency of Saurabh = = 112 units/hour 5040 −4500 270
5040
45 = 802
= 401 hours
Efficiency of Gopal = = 315 units/hour 270
16 Required time = 24 hours
5040 401
Efficiency of Veer = = 210 units/hour
24
5040 5. (a); Total marbels in bag Y = 20
Efficiency of Sameer = = 280 units/hour 7C 7×6 21
18
7 Required probability = 20C2 = 20×19 = 190
1. (a); New efficiency of Neeraj = 144 × 8 2

= 126 units/hour 6. (d); Total marbels in bag X = 18


6C 6×5 5
2
New efficiency of Gopal = 315 × 3 Required probability = 18C2 = 18×17 = 51
2
= 210 units/hour
5040 1 7. (c); Total marbels in bag y = 20
Required time = = 11 hours 2C 7
1 × C1 7
(126+112+210) 4 Required probability = 20C =
3 2 95
2. (c); New efficiency of Sameer = 280 ×
4
= 210 units/hour 8. (e); Total marbels in bag X = 18
3C 4C
ATQ – Required probability = 18 2 + 18 2
C2 C2
(210+210 )(y) 2
= =
3
+
4×3
=
9
=
1
168(y+1) 1
9×17 18×17 153 17
420y = 336y + 336
420y – 336y = 336 9. (b); Total marbels in bag Y = 20
7C 7 4
1 × C1 × C1 49
y = 4 hour Required probability = 20C =
3 285
Total work = 420 × 4 + 168 × 5
= 2520 units 10. (b); Quantity of tank P and T together
2520 (15+15)
Neeraj will complete alone in = = 1500 × = 450 ℓ
144 100
1
= 17 hours Efficiency of pipe B and D
2 12
1500×
3. (d); Divyaraj work in one hour = 144 × 1.5 Pipe B = 100
= 4.5 ℓ⁄m
40
20
= 216 units 1500×
75 Pipe D = 100
= 7.5 ℓ⁄m
40
Manish work in one hour = 168 ×
100 Efficiency of pipe C and E
= 126 units 1500×
8

Let Saurabh work for n hours and Divyaraj & Pipe C = 30


100
= 4 ℓ⁄m
Manish work for (n + 11.5) hours 15
1500 
ATQ – Pipe E = 100 = 5l / m
N × 112 + (216 + 126)(n + 11.75) 45
= 5040 According to question
112n + 342n = 5040 – 4018.5 = 12X + 9(X + 15) = 450
1021.5
n = 454 = 21X = 450 – 135
315
n = 2.25 hours =X= = 15 minutes
21
Total time = (2.25 + 2.25 + 11.75 ) Liter of water filled by pipe B and D together
1
= 16 4 hours = 15 (4.5 + 7.5) = 15 × 12 = 180ℓ

4. (d); Total work completed by Neeraj, Aniket, Veer 11. (c); Quantity of tank S
20
and Sameer together in one hour = 1500 × = 300ℓ
100
= (144 + 168 + 210 + 280) = 802 units New efficiency of Pipe D
In Second hour total work destroyed by Saurabh 300 6
= 40 × 5 = 9 ℓ⁄m
& Gopal together
Efficiency of pipe C and G
= −(315 + 112) = − (427) 8 12
1500× 1500×
Total work in 2 hours = 802 – 427 = 100
× 125 + 100
30 30

165 Adda247 Publications For More Study Material


Visit: adda247.com
Cracker Book for Bank (IBPS | SBI | RRB PO | Clerk) Mains Exams

= 5 + 6 = 11 l/ m Leak at a height of 2/3 from the bottom of tank


According to question = 9X + 11 × 15 = 300 2
= 570 × = 380 ℓtr
= 9X = 300 – 165 = 9X = 135 3
135 380 ℓ filled without leak
X = 9 = 15 minutes 380
= 19 = 20 minutes
12. (a); Given, total quantity of all seven tank Given, man notice after 32 minutes
= 1500ℓ Tank filled in 32 minutes = (20 + 12) minutes
Quantity of bigger tank K 1
120 = 380 + 12 × (19 – 19 × ) = 380 + 114
= 1500 × 100 = 1800ℓtr 2
= 494
Efficiency of pipe B, D and G together
1500×
12
1500×
20
1500×
12 Remain = 570 – 494 = 76 ℓ
= 100
+ 100
+ 100
40 40 30 15. (b);
= 4.5 + 7.5 + 6 = 18 ℓ/m Q S P
15
1500× 75
Efficiency of pipe A = 100
= 9 ℓ⁄m 32000 : 24000 : 24000 × 100
25
Efficiency of Pipe F and C 16 : 12 : 9
18 8
1500× 1500× Let total profit = 100X
= 60 100 + 30 100 12
Q extra profit share = 100X × = 12X
= 4.5 + 4 = 8.5 ℓ/m 100
15
According to question S extra profit share = 100X × = 15X
100
Tank filled in three minutes alternatively Remaining profit = 100X – (12X + 15X)
= (18 + 9 + 8.5) = 35.5 litre = 73X
In (50 × 3) minutes = 35.5 × 50 = 1775 litre 73X
Next by (B + D + G) = 18 ℓtr/m Share of P = (16+12+9) × 9 = 65700
After 151 minutes remain quantity of bigger tank 657X
= = 65700
37
= 1800 – (1775 + 18) = 7 ℓtr 65700×37
Tank K, total filled in = 151 +
7 X= = 3700
657
9
7 Total profit = 3700 × 100 = 370000 Rs.
= 151 9 minutes
16. (e); Ratio of investment by Q & R = 4 : 3
13. (d); Total quantity of tank M = (total quantity of tank ⇒ 4x : 3x
S + U together) Ratio of time period = 5 : 3 ⇒ 5y : 3y
20 18
= 1500 × + 1500 × Ratio of profit share between Q & R
100 100
= 300 + 270 = 570ℓ = 4x × 5y ∶ 3x × 3y = 20xy ∶ 9xy
Efficiency of pipe A, C and G together = 20a ∶ 9a [say]
15 8 12
1500× 1500×
= 25 100 + 30 100 + 30 100
1500× ATQ,
20a – 9a = 100%
= 9 + 4 + 6 = 19 ℓ/m 10010
(A + C + G) filled tank M together in a= = 910
11
570 Now 35% profit = 29a = 29 × 910
= 19 = 30 minutes
65
But according to question — 65% profit = 29 × 910 × 35
17 65 29×910×13
tank 7 minutes more due to leak % profit = 2×7 = 24505
19 2
leak empty quantity of water in 30 minutes R’s total profit share = 24505 + 9a
17
= (A + C + G) filled in 7 19 minutes = 24505 + 8190 = 32695
17 150
= 19 × 7 19 = 19 × 19
= 150 ℓtr 17. (a); Lets S joined the business after X months
150 According to question
Efficiency of leak = 30 = 3 ℓ⁄m
32000×36 3
Leak empty tank M in 24000×(36 –X)
=2
570 36
= 3 = 190 minutes x = = 4 months
9
1
= 3 6 hr 18. (b); Profit share of R = 30500 – 24300
= 16200 Rs.
14. (b); Given, total quantity of tank = 570 ℓ Lets R invested X Rs. for 12 months
Efficiency of pipe B, D and E together
12 20 8 ATQ -
1500× 1500× 1500× 175 81000×9 24300 81000×3 3
= 40 100 + 40 100 + 30
100
× 100 = 16200 = 4X = 2
X×12
= 4.5 + 7.5 + 7 = 19 ℓ/m X = 40500 Rs.
166 Adda247 Publications For More Study Material
Visit: adda247.com
Cracker Book for Bank (IBPS | SBI | RRB PO | Clerk) Mains Exams

19. (c); Q : R = 8 : 9 Ratio of profit share of Sandeep, Neeraj and


R:S=3:2 Satish
Q:R:S=8:9:6
Total profit = 37030
Q’s share
8 11500
× 37030 = 12880 Sandeep’s profit share = (12+6+5) × 12
23
R’s share = Rs 6000
9
× 37030 = 14490 23. (b); Investment of Aman
23 30 75 100
S’s share = 40,000 × 100 × 100 × 150 = 6,000
6 40
× 37030 = 9660 Investment of Saurabh = 40,000 × 100
23
= 16,000
40
20. (c); Investment of Saurabh = 40,000 × Total amount they have after 2 years
100 120 120 80 80
= 16,000 = 6000 × 100 × 100 + 16000 × 100 × 100
30 75 = 8,640 + 10,240 = 18,880
Investment of Neeraj = 40,000 × ×
100 100 50
= 9,000 24. (b); Total quantity of in vessel P = 240 ×
100
Ratio of profit share of Neeraj to Saurabh = 120 ml
Quantity of Mango juice in vessel P
93.75
= 192 × = 180 ml.
100
150
Quantity of water in vessel S = 160 ×
100
= 240 ml
Quantity of Orange juice in vessel S
= 240 – 40 = 200 ml
Ratio of (Milk : Water : Orange juice : Mango
juice) in vessel P
= 240 : 120 : 144 : 180 = 20 : 10 : 12 : 15
Ratio of (milk : Water : Orange juice : Mango
8000 juice) in vessel S
Saurabh’s profit = 59+101 × 101
8000
= 160 : 240 : 200 : 120 = 4 : 6 : 5 : 3
= 160
× 101 = 5050 Total orange juice in vessel T
12 5
21. (a); Investment of Rakesh = 40,000 ×
30 = 114 × (20+10+12+15) + 180 × (4+6+5+3)
100
= 12,000 = 24 + 50 = 74 ml
74
Investment of Aman Required % = (114+180) × 100
30 75 100 74 25
= 40,000 × 100 × 100 × 150 = 6,000 = 294 × 100 = 25 147 %
Ratio of profit share of Aman to Rakesh 2
Aman : Rakesh 25. (c); Quantity of milk in vessel R = 144 × 3
6000  8 + 8000  2 + 10,000  2 : 12000(12 − x) = 96 ml
Quantity of Orange juice in vessel Q
84000 : 12000(12 − x) 3
= 160 × 4 = 120 ml
7 : (12 − x)
Total solution in vessel Q
ATQ, = (160 + 80 + 120 + 210) = 570 ml
7 7
= ⇒ x = 4 months Total solution in vessel R = 1086 – 570
12−x 8
Rakesh worked for (12-x) = 12 -4 = 516 ml
= 8 months Let mango juice and total solution in vessel R be
8n and 43 n respectively
30
22. (e); Investment of Sandeep = 40000 × 100 ATQ—
= 12000 96 + 144 + 192 + 7n = 43n
30 75 36n = 432
Investment of Neeraj = 40000 × × n = 12
100 100
= 9000 Total mango juice in vessel R = 12 × 7
40
Investment of Satish = 40000 × 100 + 4000 = 84 ml
210+84 294 49
= 20000 Required ratio = 1086 = 1086 = 181

167 Adda247 Publications For More Study Material


Visit: adda247.com
Cracker Book for Bank (IBPS | SBI | RRB PO | Clerk) Mains Exams

26. (c); Minimum average is obtained when he will buy Also, we know that ratio of distance travelled by
2T₂ t- shirts. two objects in same time is same as ratio of their
Price for 2 shirts = 600 × 2 = Rs. 1200 speed.
He will get 4 t-shirts in this price D S
1200 ∴ DA = SA
Price/t-shirt = = Rs. 300 B B
4
NOTE: If he buys T₁ t-shirt, price will be Distance travelled by B between 12:00 to 1:00
90
400 × 100 = Rs. 360 PM
60 6
= ⇒ DB =50 km
DB 5
27. (a); 20% profit means he is selling it in
20 Similarly distance travelled by B between 1:00
1000 (1 + 100) = Rs. 1200
PM – 2:00 PM
This amount is obtained after 25% discount 70 14
4 = ⇒ DB = 55 km
Hence MRP = 1200 × = Rs. 1600 DB 11
3
Distance travelled between 2:00 – 3:00 PM
1
11 8 60 6
28. (c); SP of S₂ = 1647 [1– 100 ] = 1647 [9]
9
DB
= 7 = DB = 70 km

= 183 × 8 = Rs. 1464 Required difference = 190 – [50 + 55 + 70]


ATQ, = 15 km
2
14
CP [1 + 1007 ] = Rs. 1464 33. (a); Average speed of A between 2:00 to 4:00 pm
60+80
7 = = 70 km/hr
CP = 1464 × = Rs. 1281 2
8 Average speed of B between 2:00 to 3:00 pm
29. (e); Price paid for 5 pieces of T₂ is actually price paid 7
= × 60=70 km/hr
6
for
Similarly between 3:00 to 4:00 pm
2 pieces of T₂ + 1 piece of T₂. 3
As he will get 2 – t-shirts (T₂) free after buying = 80 × = 60 km/hr
4
two = 2 × 600 + 600 = 1800 Average speed of B between 2:00 to 4:00 pm
Similarly price for 4 pieces of J₁ = 2 × 1200 60+70
= = 65 km/hr
1 piece of S₁ = 1 × 1200 = Rs. 1200 2
70–65
Total = Rs. 1800 + Rs. 2400 + Rs. 1200 = Rs. 5400 Required answer = ( ) × 100
70
1 1
30. (d); Actual price of 4 pieces of T₂ = 4 × 600 = × 100 = 7 %
14 7
= 2400 Rs.
Actual price of 2 pieces of T₁ = 2 × 400 34. (d); Distance covered by B till 5:00 PM is
= 800 rs. (60 × ) + (70 ×
5 11 7
) + (60 × ) + (80 × ) +
3
Actual price of 4 pieces of J₁ = 4 × 1200 6 14 6 4
7
= Rs 4800 (40 × )
4
Total actual price is Rs (2400 + 800 + 4800) = Rs = 50 + 55 + 70 + 60 + 70 = 305 km
8000 Remaining distance = 540 – 305 = 235 km
Price paid for 4 pieces of T₂ = 2 × 600 235
= Rs. 1200 Required average = 4
= 58¾ km/hr
90
For 2 pieces of T₁ = 400 × (100) × 2 35. (c); A consumes 1 litre/15 km
= Rs. 720 B consume 15 × 125
100
For 4 pieces of J₁ = 2 × 1200 = Rs. 2400 4
Discount amount obtained 15 × = 1 litre/12 km
5
= (8000 – 4320) Rs. = 3680 Distance travelled by A till 2:00 PM = 60 + 70 =
3680
D% = 8000 × 100 = 46% 130 km
130 26
Total distance Petrol consumed = = litre
31. (c); Average speed = Total time
15 3
60+70+60+80+40 310 Distance travelled by B till 2:00 PM = 50 + 55 =
= = = 62 km/hr
5 5 105 km
105 35
32. (b); Distance travelled by A till 3 PM is = 60 + 70 + 60 Petrol consumed = = litre
12 4
= 190 km 26 35
Now note that if A is travelling a distance of 60 Required Ratio = 3
∶ 4
km in 1 hour, then its speed is also 60 km/hr. ⇒ 104: 105
168 Adda247 Publications For More Study Material
Visit: adda247.com
Cracker Book for Bank (IBPS | SBI | RRB PO | Clerk) Mains Exams

36. (b); Let the amount obtained by B is 41x and S.I. 40. (c); Amount obtained by D after drawing his whole
obtained is 16x. money
Therefore, principal is 41x – 16x = 25x = 100000 +
100000×8×5
= Rs. 140000
= Rs75000 100

⇒ x = 3000/– Now he invested 50% of this = 70000


Therefore, SI obtained 16x = 16 × 3000 Amount obtained after two years from this
110 2
= 4800/- = 70000 × (100) = Rs. 84700
Using formulae
PRT 48000×100 Total amount with him = Rs. 84700 + Rs.70000
100
= 48000 ⇒ T = 16×75000 = Rs. 154700
= 4 years = 48 months 41. (d); Upstream distance travelled on 1st Aug.
12
37. (d); If we assume amount of A 8x and SI obtained 3x, = × 6000 = 720km.
100
then we can calculate principal of A (5x) = Rs. Downstream distance travelled on 1st Aug.
80000 16
= × 3000 = 320 km
x = 16000 100
and amount obtained by A = 16000 × 8 Let speed of Rahul in still water be x km/hr.
= 128000 ATQ.
720 320 184
3
∴ amount obtained by C = × 128000 = x−10 = x+10 + 3
4
= Rs. 96000 On solving
Let ROI obtained by C is m%/annum and from x = 20km/hr.
line graph, we know that Principal submitted by ∴ speed of Rahul in still water = 20Km/hr.
him is Rs 60000. 42. (a); Speed of Rahul in still water on 3rd Aug.
Then, 125
60000(m)×6
= × 8 = 10Km/hr.
100
= Rs. (96000 – 60000) = 36000
100 ATQ.
m = 10% 8
×600 8
12
×2000
100
8−x
= 3 × 1008+x
38. (a); Time for which Vikas invested is 6 + 3
Solving we get
= 9 years.
80000 x = 8/7 km/hr.
Sum deposit by him = 2 = Rs. 40000 ∴ seven times Speed of Current = 8 Km/hr
100000×8×5
SI obtained by D = 100
= Rs. 40000 43. (c); Let speed of Rahul in still water be x Km/hr.
40000×R×9 32
∴ ROI of Vikas = = 40000 ×6000
100 ∴ 80 = 100x−3
100 1
R= 9
= 11 9 % x = 27Km/hr.
Downstream speed on 5 August =27 + 3
39. (e); Let amount obtained by E is 29y & SI obtained is
= 30 km/h
9y
Speed of current on 4th Aug.
∴ 29y – 9y = 20y = 50000 50
[∵ 50000 is principal] = (27+3) × = 15 Km/hr.
100
⇒ y = 2500 Time to cover 128 Km up stream on 4th Aug.
Interest obtained = 2500 × 9 = Rs. 22500 128 2
= = 10 hr.
50000×15%×T 27−15 3
∴ 100
= 22500
22500 44. (e); Let speed of current on 2nd Aug. & 4th Aug. be 2x
T= = 3 years. km/hr & 3xkm/hr.
500×15
Similarly, Let speed Rahul in still water be y km/hr.
For B principal is Rs. 75000 ATQ.
(41–16) = 25 units = Rs. 75000 If speed of current on 3 August is 4 km/h then
75000
⇒ 16 units (SI) = × 16 = Rs. 48000 2x = 8
25
∴ time for which B invested Therefore
75000×16×T 3x = 12
= 48000
100 Now,
48×100 12 28
T= = 4% ×2000 ×6000
75×16 24×100y+8 = 2x × 100y−12
Required ratio = 3 : 4

169 Adda247 Publications For More Study Material


Visit: adda247.com
Cracker Book for Bank (IBPS | SBI | RRB PO | Clerk) Mains Exams

7y + 56 = 24y − 288 48. (b); On Monday,


y = 20.22 km/h Length of Rajdhani exp. = 15 × 17 + 20
Approximately 20km/h = 275 m
45. (c); On Tuesday, Length of Duranto exp. = 275 × 1.28
Length of Rajdhani exp → 24 × 15 + 20 = 352 m
= 380 m Let the speed of Rajdhani, shatabadi and Duranto
Let the speed of Rajdhani exp on Tuesday be 8 x exp. on Monday be 3x m/s, 4x m/s, 5x m/s
m/s respectively.
Speed of shatabadi exp = 7x m/s ATQ,
352+352
Speed of duranto exp = 10 x m/s 5x
= 12.8 ⇒ 704 = 64x ⇒ x = 11
Let the length of tunnel be y m Required difference = 5x −
7x 3
= ×11×
18
2 2 5
ATQ,
380+y = 59.4 km/hr
= 12.5
8x 18
⇒ 380 + y = 100x 49. (b); Length of train B = × 1600 = 288m
100
12
⇒ 100x – y = 380 … (i) Length of Train D = 100 × 1600 = 192m
125
And, length of Duranto exp = 380 × Length of Train F =
8
× 1600 = 128m
100 100
= 475 m 5
475+y
Speed of train B on Monday = 97.2 × 18
= 11.9
10x = 27 m/sec
⇒ 475 +y = 119x Speed of train ‘B’ on Wednesday
⇒ 119x – y = 475 … (ii) =
27
× 5 = 45 m/sec
3
From (i) and (ii)
ATQ,
x = 5 m/s
380+370 750
288 + 192 = (45 +y)×6
Required time = 15×5
= 75
= 10 sec where y is the speed of train ‘D’ on Wednesday
⇒ y = 80 – 45 = 35 m/sec
46. (a); Length of Rajdhani on Sunday 35
= 29 × 15 + 20 = 455m Speed of train ‘D’ on Monday = 7
×4
Length of Duranto exp = 345 m = 20 m/sec
192+128
Let the speed of Rajdhani, shatabadi and Duranto Time required to cross train F = 20+20
exp be 6x m/s, 5x m/s and 4x m/s respectively. = 8 seconds
ATQ,
455+345 80 50. (e); Let speed of train ‘C’ on Monday, Tuesday and
= ⇒ x = 9 m/s
10x 9 Wednesday be 4x, 6x and 5x respectively.
80
Length of Shatabadi = 455× 100 = 364 m Train ‘C’ travel 5 hours on Monday and 15 hours
Required time =
364+345
≈ 79 sec on Tuesday.
9 ∴ Total distance = 5 × 4x + 15 × 6x = 110x
47. (d); Speed of Shatabadi exp on Friday On the same day i.e, Tuesday, train ‘C’ start from
= 150 km/hr Kanpur. It travels 6 hours on Tuesday and 17.8
Speed of Duranto exp on Friday hours on Wednesday.
= 120 km/hr ∴ total distance travel = 6 × 6x + 17.8 × 5x
Distance travelled by Shatabadi exp in 3 hours = 36x + 89x = 125x
= 150 + 150 × 1.10 + 150 × 1.1 × 1.2 ATQ, 125x = 110x + 180
= 513 km ⇒ 15x = 180 ⇒ x = 12
Distance travelled by Duranto exp in 3 hours speed of train ‘C’ on Monday
40
= 120 + 120 × 1.2 + 120 × 1.2 × 1.25 = 12 × 4 = 48 km/hour = m/sec
3
= 444 km Length of train ‘C’ =
24
× 1600 = 384
100
Total distance between P and Q 384
Required time = × 3 = 28.8 sec
= 513 + 444 = 957 km 40

170 Adda247 Publications For More Study Material


Visit: adda247.com
Cracker Book for Bank (IBPS | SBI | RRB PO | Clerk) Mains Exams

51. (a); Let, speed of train ‘A’ and train ‘C’ on Monday be ATQ,
288
‘4x’ and ‘4y’ respectively 1 3 4y 1
3x
192 =1 ⇒ × 3x = 1
ATQ, 4y
2
900 900 y 1
2.5 = 4x − 4y ⇒ x = 2 ⇒ x = 2y
1 1 Time taken by train ‘B’ on Wednesday to cross
2.5 = 225 [x − y] 288 57.6
pole = =
xy = 90 (y – x) 5x x
16 Time taken by train ‘D’ on Monday to cross a pole
length of train ‘A’ = 100 × 1600 = 256 192 96
= =
256+128 4y x
speed of train ‘A’ on Tuesday = 96 57.6
( − )×100
12.8
384 Required % = x x
96
= 12.8 = 30 m/sec x
38.4
⇒ Speed of train ‘A’ on Monday = × 100 = 40%
96
30
= 3
× 2 = 20 m/sec = 72 km/hr 54. (c); Let, marked price of both articles be 600x
72
⇒ 4x = 72 ⇒ x = 18 S.P. of article X = 600x × = 432x
100
xy = 90(y – x) 85
S.P. of article Y = 600x × = 510x
y = 5(y – 18) 100

⇒ y = 22.5 ATQ,
510x − 432x = 312 ⇒ 78x = 312
Speed of train ‘C’ on Monday = 4y
⇒x=4
= 4 × 22.5 = 90 km/hr 432×4
90 Cost price of article ′X ′ = 120 × 100
Speed of train ‘C’ on Tuesday = ×6
4 = 1440
= 135 km/hr = 37.5 m/sec Cost price of article ‘Y’ = 1520 × 2 – 1440
24
Length of train ‘C’ = 100 × 1600 = 384 = 3040 – 1440 = 1600
384+66 Profit % earned on selling article Y
Required time = = 12 seconds 510×4−1600 440
37.5 = × 100 = × 100 = 27.5%
1600 1600
22
52. (a); Length of train ‘E’ = 100 × 1600 = 352 55. (e); Let M.P. of each article sold by each seller be
8
Length of train ‘F’ = 100 × 1600 = 128 800x
800x×67.5
Let speed of train ‘E’ and train ‘F’ on Monday be Cost price of article Y sold by B = 120
6x and 4y respectively. = 450x
6x 3 x 1 Cost price of article Y sold by E
⇒ 4y = 2 ⇒ y = 1 800x×72
= 4×100 × 3 = 432x
Let speed of train ‘E’ on Tuesday = 9x 800x×85
So speed of train ‘F’ on Tuesday = 5y = 5x Cost price of article X sold by C = 125
ATQ, = 544x
800x×54
9x – 5x =
352+128
= 20 ⇒ 4x = 20 Cost price of article X sold by E = 135
24
= 320x
⇒x=5
ATQ,
Speed of train ‘E’ on Wednesday = 5 × 5
(450x + 432x) – 544x – 320x = 216
= 25 m/sec 882x – 864x = 216
Speed of train ‘F’ on Wednesday = 3 × 5 216
⇒ x = 18 = 12
= 15 m/sec
352+128 480 M.P. of each article = 800 × 12 = 9600
Required time = = 9600×76
25−15 10 Cost price of article Y sold by C = 160
= 48 seconds = Rs 4560
53. (b); Let, speed of train ‘B’ on Monday, Tuesday & 56. (b); Let M.P. of each article be 400x
Wednesday be 3x, 4x & 5x respectively. 400x×73.5
Cost price of article X sold by B = 168
And speed of train ‘D’ on Monday, Tuesday &
= 175x
Wednesday be 4y. 4y & 7y respectively. 400x×63
18 Cost price of article Y sold by D =
Length of train ‘B’ = 100 × 1600 = 288 120
12
= 210x
Length of train ‘D’ = 100 × 1600 = 192 Required ratio =
175x
=
5
210x 6

171 Adda247 Publications For More Study Material


Visit: adda247.com
Cracker Book for Bank (IBPS | SBI | RRB PO | Clerk) Mains Exams

57. (d); Let M.P. of each article = 400x Distance between city B and city D
400x×54 = 1500 km
S.P. of article X sold by E = 100 = 216x
1500 300
Cost price of article Y sold by E = 216x Time taken to cross each other = 5a = a
Selling price of article Y sold by E =
400x×72 Time taken by car Q to reach city B from Delhi =
100 3000 1000
= 288x 3a
= a
288x−216x 300×100
Profit % = 216x × 100 Required% = 1000
= 30%
72x 1
= × 100 = 33 % 62. (d);
1500×3
= 900 km
216x 3 5
58. (b); Let, M.P. of each article be 400x Time taken by car R to cover this distance
400x 900
C.P. of article X sold by A = 2 = 200x = = 15 hour
60
120 Let initial speed of car S = x km/hr
S.P of article X sold by A = 200x ×
100 So, ATQ
= 240x 6x + 15(2x) = 900
85
SP of article Y sold of A = 400x × 6x + 30x = 900
100
= 340x 36x = 900 ⇒ x = 25 km/hr
Let, CP of article Y sold by A = y 63. (e); Distance between Delhi and city A
ATQ, = 1000 km
125
(200x + y) × = 240x + 340x Distance covered by Car Q before first meeting =
100
580x 1200 km
(200x + y) = × 4 ⇒ y = 264x
5 Distance covered by Car P before first meeting =
264x−240x 1
Required % = × 100 = 9 % 800 km
264x 11
Speed of car P = 40 km/hr
59. (b); Distance travel by car P = 1500 + 3000 800
⇒ Time for first meeting = 40 = 20hr
= 4500 km 1200
4500 Speed of car Q = = 60km/hr
Total Time taken = 40 = 112.5 hour 20
When car P reaches city ‘A’ distance covered by
Time taken by car R from Delhi to City A 200
1000 50 car ‘Q’ = 40 × 60 = 300km
= 60 = 3 hours 500
50 Time taken by car ‘Q’ to reach Delhi =
Time taken from city A to city B = 112.5– 25
60
3
287.5 = hr
= 3
25
3
Distance covered by car ‘P’ in hour
Distance from between City A to city B 3
287.5 25 1000
= × 60 = 5750 km = 3 × 40 = 3 km
3
Distance between car ‘Q’ and car ‘P’
60. (a); 1000
= 1000 − 3 = 3
2000

2000
20
Time to meet = 3
60+40
= 3
hour
200 25 20
Total time = 40
+ 3
+ 3
= 20 hours
Distance between city A and city E 64. (d); Total work = 24 × (4a + 2a + 4a) = 240a
= √10002 + 15002 Bhavya worked for10 days
= √1000000 + 2250000 ⇒ Work done by ‘Bhavya’ = 40a
40a
= √3250000 = 500√13km ⇒ Time taken by ‘Neeraj’ = = 20 days
2a
Approximate time taken by car ‘T’ ⇒ ‘Neeraj’ and ‘Abhi’ worked for 20 days
500√13
= 75 ≈ 24 hours And ‘Veer’ worked for 8 days
61. (c); Let speeds of car Q and car S be x and y 20 × 3a + 10 × 4a + 20 × 2a + y × 4a +
respectively. 8 × 6a = 240a
ATQ— ⇒ 118a + y × 4a = 240a
240a−188a 52a
3000
⇒ x = y
2000 x
⇒ y=2
3 ⇒y= ⇒ y= = 13 days
4a 4a

Let speed of car Q and car S be 3a and 2a 65. (b); 5a × (A + 42) = 8a × (A + 15)
respectively ⇒ 5A + 210 = 8A + 120
⇒ A = 30
172 Adda247 Publications For More Study Material
Visit: adda247.com
Cracker Book for Bank (IBPS | SBI | RRB PO | Clerk) Mains Exams

Total work = 5a (30 + 42) = 5a(72) ‘Abhi’ worked for (9 +3) = 12 days
= 360a ‘Neeraj’ worked for (9 + x+ z) = 9 + 3 + 6
Let, Abhi, Bhavya and Veer worked for x, y and z = 18
18−12
days Required % = × 100
12
ATQ, 6
3x : 4y : 6z = 1 : 2 : 3 = 12 × 100 = 50%
Ratio between working days of Abhi, Bhavya and 67. (b); There are two possibilities
Veer First – Satish worked first
⇒x:y:z=2:3:3
Second – Veer worked first
Ratio between working days of Neeraj, Satish
When satish worked first
and Veer
remaining work
=2:2:1
Let working days of Abhi, Bhavya, Neeraj, Satish → 120a – 10(3a+4a) – 7(2a) = 36a
and Veer be 2m, 3m, 6m and 6m and 3m days Satish and Veer worked in 3 days = 30a
respectively 4th day satish’s worked = 4a
ATQ, Veer’s worked for = 1/3 day
3a × 2m + 4a × 3m + 2a × 6m + 4a × 6m + 6a × Now ‘y’ cannot be in fraction
3m = 360a 2nd case-
⇒ 72am = 360a ⇒ m = 5 When Veer worked first-
Bhavya worked for 5 × 3 = 15 days Veer and Satish worked for first 3 day
= 30a
66. (b); Veer, Bhavya and Satish worked for 6 days, 9
Remaining work = 6/6= 1 day
days and 9 days respectively. Total 60% of work
So Veer worked for 4 days.
completed by them
⇒ 60% of work = 4a × 9 + 4a × 9 + 6a × 6 68. (d); In first case
= 108a ATQ,
108a 10 5 x 5 6
⇒ Total work = × 5= 180a + + + + =1
3 40 30 60 30 20
‘Abhi’ worked for (9 + x) days & ‘Neeraj’ worked x
⇒ + =1 ⇒
53
= ⇒ x=7
x 7
for (9+ x + z) days and completed 40% of work 60 60 60 60

⇒ 72a = 3a (9 + x) + 2a(9 + x+ z) In second case


72 = 27 + 3x + 18 + 2x + 2z ATQ,
4 4 y 8 5 y 3
27 = 5x +2z … (i) 40
+ 30 + 60 + 30 + 20 = 1 ⇒ 60 + 4 = 1
And, y 1
⇒ = ⇒ y = 15
z–x=3 … (ii) 60 4
On solving (i) & (ii) (y − x) = (15 − 7)2 = 82 = 64
2

z = 6, x= 3

173 Adda247 Publications For More Study Material


Visit: adda247.com
Cracker Book for Bank (IBPS | SBI | RRB PO | Clerk) Mains Exams

Chapter
Data Sufficiency
15
BEST APPROACH TO SOLVE THE QUESTIONS

Data Sufficiency
These questions formerly asked in only MAINS Examination but now-a-days these questions were seen in
PRELIMINARY Examinations too. So, student can’t deny the importance of DATA SUFFICIENY Questions.
In these questions some data are given in form of statements and one question is asked. Student must choose that
statement/statements which is/are sufficient to solve the given question.
They consist of two types. In first type, two statements are given while in second type, three statements are given,
and student is expected to choose the statement/statements among the given statements which is/are required to solve
the question. Five questions of each type are given below to practice this topic.
Example1.
What is the length of train ‘A’?
A. Train A takes 8 second to cross a pole
B. Train A cross train B coming from opposite side in 8 second, speed of train B is 25 m/sec and length of train B is
half of train A.
C. Train A takes 10 sec to cross two pole 100 m apart.
(a) Only A and B together
(b) Only A and C together
(c) Any two of them
(d) Either B alone or A and C together
(e) All statements are required
Solution:
1. (c); Let length of train A = L
Speed of train A = x
L
From A = = 8
x
L+0.5L
From B = x+25
=8
L+100
From C = x
= 10
From any of these two-statement question be solved.
Example 2:

In how many days B and C together can complete work?


A. Per day efficiency of A, B and C is in the ratio 3 : 2 : 4
B. A and B together can complete the work in 7.2 days
C. Time taken by A, B and C alone to complete the work is in the ratio 4 : 6 : 3.
(a) Only A and B together
(b) Either A and B or B and C
(c) Any two of them
(d) Either B alone or A and C together
(e) All statements are required

174 Adda247 Publications For More Study Material


Visit: adda247.com
Cracker Book for Bank (IBPS | SBI | RRB PO | Clerk) Mains Exams

Solution:
2. (b);From A per day efficiency = 3 : 2 : 4
⇒ ratio of time taken by A, B and C alone to complete work = 4 : 6 : 3 same as (C)
From B) time taken by A and B together = 7.2 days
Either B and A or B and C questions can be solved
Example3:
What is the base radius of the cone?
A. Total surface area of the cone is 462 cm²
1078√3
B. Volume of the given cone is 𝑐𝑚3 and height = 7√3 cm
3
C. Ratio of height and radius of the given cone is √3 : 1
(a) Only A and B together
(b) Only A and C together
(c) Any two of them
(d) Either B alone or A and C together
(e) All statements are required
Solution:
3. (d) From A) T.S.A = πrℓ + πr² = 462 cm2
1078√3 h
From B) V = 3
= πr 2 3
And h = 7√3 cm
From C) h : r = √3 ∶ 1
Either B alone or A and C together, question can be solved.
Example 4:
Find the share of Satish in profit after one year among Satish, veer and Yogesh
A. Investment of Veer, Satish and Yogesh is in the ratio 6: 8: 9 and share of Yogesh in profit is Rs.900
B. Satish and Veer invested Rs.8000 and Rs.6000 for 9 months and one year respectively. Difference b/w profit earned
by Satish and Veer is Zero.
C. Satish, Veer and Yogesh invested for 9 months, 1 year and 8 months respectively and share of Veer in profit is 900.
(a) Either A and B or B and C
(b) Either A and B or A and C
(c) Any two of them
(d) Either B alone or A and C together
(e) Either A and C or B and C
Solution:
4. (e); Veer : Satish : Yogesh
From A and C) 6×12 : 8×9 : 9×8
72 : 72 : 72
1 : 1 : 1
Share of Veer = Share of Satish Rs 900
Satish : Veer
From (B and C) 8000×9 : 6000×12
1 : 1
Share of Veer share of Satish = Rs 900
Either A and C or B and C question can be solved.

175 Adda247 Publications For More Study Material


Visit: adda247.com
Cracker Book for Bank (IBPS | SBI | RRB PO | Clerk) Mains Exams

Example 5:
What will be the probability of choosing two fresh eggs simultaneously from tray A given that there are only two trays
i.e. tray A and tray B?
A. Tray B contains 30 Eggs in which fresh and rotten eggs in the ratio 7 : 3. Ratio of fresh eggs in Tray A and Tray B is
2:3
B. Tray A contains 20 Eggs in total out of which 14 are fresh and rest are rotten.
C. Tray B contains 30 Eggs and Tray A contains 20 Eggs respectively. Ratio of Rotten eggs and fresh eggs in Tray A is 7 :
3
(a) Either A or B
(b) Either B or C
(c) Any one of them
(d) A alone
(e) All statements are required

Solution:
5. (b); From A) Tray B = 30
Rotten eggs in Tray B = 9 eggs & fresh eggs in Tray B = 21 eggs
Tray A, Fresh eggs = 14 eggs
From B) Tray A = 20 eggs
Fresh eggs in Tray A = 14 Rotten eggs in Tray A = 6
From C)
Tray B = 30 eggs
Tray A = 20 Eggs
Rotten eggs in Tray A =6 eggs
Fresh eggs in Tray A = 14 eggs
From Either B or C question can be solved.

Practice Exercise Based on new Pattern

Directions (1-5): Given below in each question there two (a) Only statement I is sufficient
statements (I) and (II). You have to determine, which (b) Only statement II is sufficient
statement is sufficient to give the answer of question. Also (c) Statement I and II both together sufficient
there are five alternatives given, you have choose one (d) Either statement I or Statement II alone sufficient
alternative as your answer of the questions: (e) Neither statement I or statement II sufficient
1. What will perimeter of smaller rectangle? 3. What was profit shopkeeper made on article?
I. Ratio between length of smaller and larger I. Shopkeeper sold article on 5% discount in the Rs.
rectangle is 4 : 5 and breadth of both rectangle is of 3800.
equal. Difference between Perimeter of both II. If shopkeeper sold article on marked price, he
rectangle is 8 cm. would made a profit of 25%.
II. Breadth of both rectangle is equal to side of square, (a) Only statement I is sufficient
whose area is 196 cm2 . (b) Only statement II is sufficient
(a) Only statement I is sufficient (c) Statement I and II both together sufficient
(b) Only statement II is sufficient (d) Either statement I or Statement II alone sufficient
(c) Statement I and II both together sufficient (e) Neither statement I or statement II sufficient
(d) Either statement I or Statement II alone sufficient 4. What is speed of boat?
(e) Neither statement I or statement II sufficient I. Speed of boat in still water is two times more that
2. How many Students in college? speed of current.
II. Boat takes equal time to cover a distance
I. Ratio between girls to boy is 9 : 11.
downstream to 50% of that distance upstream.
II. Out of total girls in the college 20% are belongs to (a) Only statement I is sufficient
below eighteen years age group. Total girls belongs (b) Only statement II is sufficient
to below eighteen years age group are 9% of total (c) Statement I and II both
students in college (d) Either statement I or Statement II alone sufficient
(e) Neither statement I or statement II sufficient
176 Adda247 Publications For More Study Material
Visit: adda247.com
Cracker Book for Bank (IBPS | SBI | RRB PO | Clerk) Mains Exams

5. Find the amount invested at the rate of 10%? 8. Satish sold an article to Ayush at 20% profit. If Ayush
I. Total amount of Rs. 4500 invested in two different purchased article from Satish in Rs. 1440, then find
parts at the rate of 20% p.a. and 10% p.a. for two the profit percentage of Veer if Satish bought this
years. Simple Interest obtained from both parts are article form Veer?
equal. I. Veer sold the article on Rs. 240 more than its cost
II. A man invested an amount in two schemes A and B price to Satish.
in the ratio of 2 : 1 respectively. Scheme A offered II. If Veer sold article to Ayush on same price as
simple interest at the rate of 10% p.a. and Scheme Satish sold to Ayush, then he made overall profit of
B offered compound interest at the rate of 20% p.a. 50%.
and man got a total interest of Rs. 1260 after two
years from both scheme. Amount invested on SI is 9. If x : y = 11 : 9 and y : z = 3 : 4, then find (x + y) – 1.5z
same as amount invested at the rate of 10% in =?
statement I? I. Average of all three is two more than average of x
(a) Only statement I is sufficient and y.
1 1
(b) Only statement II is sufficient II. Sum of 9 11 % of x and 11 9 % of y is equal to
(c) Statement I and II both (36)0.5.
(d) Either statement I or Statement II alone sufficient
(e) Neither statement I or statement II sufficient 10. Ratio between length and breadth of rectangle ‘X’ is 7
: 4. Find area of a square ‘Y’?
Directions (6-10): The following questions are I. Length of rectangle ‘X’ is two times of radius of
accompanied by two statements (I) and (II). You have to circle, whose area is 616 cm2.
determine which statements(s) is/are II. Perimeter of rectangle ‘X’ is 20 cm more than
sufficient/necessary to answer the questions. perimeter of square ‘Y’.
(a) Statement (I) alone is sufficient to answer the
question but statement (II) alone is not sufficient to Directions (11-14): The following questions are
answer the questions. accompanied by three statements (A) or (I) and (B) or
(b) Statement (II) alone is sufficient to answer the (II), You have to determine which statement(s) is/are
question but statement (I) alone is not sufficient to sufficient/necessary to answer the questions.
answer the question. 11. What is the minimum passing percentage in a test?
(c) Both the statements taken together are necessary to I. Raman scored 25% marks in the test and Sunil
answer the questions, but neither of the statements
scored 288 marks which is 128 more than that of
alone is sufficient to answer the question.
Raman.
(d) Either statement (I) or statement (II) by itself is
II. Raman scored 64 marks less than the minimum
sufficient to answer the question.
passing marks.
(e) Statements (I) and (II) taken together are not
(a) Both I and II together are not sufficient
sufficient to answer the question.
(b) Both I and II together are needed.
6. Ratio between length of two trains is 4 : 3. What will (c) Only I alone is sufficient.
be difference between lengths of both trains? (d) Only II alone is sufficient.
I. Speed of larger trains and smaller train is 72 (e) Either I or II alone is sufficient
km/hr and 90 km/hr respectively. Both trains
28 12. Whose body weight is second highest among the five
cross each other in 3 𝑠𝑒𝑐, when running in
boys Arun, Vinay, Suraj, Raju and Pratap?
opposite direction. I. Average weight of Arun, Suraj and Vinay is 68 kg
II. Speed of smaller train is 90 km/hr and it can cross and average weight of Raju and Pratap is 72 kg.
a pole in 7.2 sec. Also Suraj is 78 kg. Raju is 68 kg and Vinay is 46 kg.
7. There are three men P, Q and R. Find the difference II. Average weight of Arun, Suraj, Vinay and Raju is 68
between time taken by P & Q together to complete a kg and also Suraj is 78 kg. Raju is 68 kg and Vinay
task and time taken by Q & R together to complete the is 46 kg. All of them have different weights.
same task? (a) Both I and II together are not sufficient
I. ‘R’ takes twice as much time as ‘Q’ and thrice as (b) Both I and II together are needed.
much time as ‘P’ takes alone. (c) Only I alone is sufficient.
II. If they all three works together work will be (d) Only II alone is sufficient.
completed in 4 days. (e) Either I or II alone is sufficient

177 Adda247 Publications For More Study Material


Visit: adda247.com
Cracker Book for Bank (IBPS | SBI | RRB PO | Clerk) Mains Exams

13. What is the population of the city A? (A) Median of these numbers is 18 while mode of
I. The ratio of the population of males and females in these numbers is 12 and 24.
city A is 27 : 23 and the difference between their 18. ‘X’ mark up an article 50% above its cost price. Find
population is 100000. the cost price of article.
II. The population of city A is 80% of that of city B. (A) ‘X’ gave Rs 60 discount on mark price and earn
The difference between populations of city A and 20% profit.
city B is 312500. (B) If ‘X’ gave two successive discounts of 10% each
(a) Both I and II together are not sufficient then he will earn Rs 43 as profit.
(b) Both I and II together are needed.
(c) Only I alone is sufficient. 19. Amit tells truth 2 times out of x times while Ankush
(d) Only II alone is sufficient. tells truth 5 times out of ‘y’ times. Find the product of
(e) Either I or II alone is sufficient ‘x’ and ‘y’
(A) Probability that Amit and Ankush contradict
14. How many students did participate in Singing? with each other on facts is 50%
I. The students who participated in dancing were (B) Probability that both tells truth is 20%
150% more than that who participated in Singing.
II. 150 students participated in dancing. Directions (20-24): The following questions are
(a) Both I and II together are not sufficient accompanied by two statements (A) and (B). You have to
(b) Both I and II together are needed. determine which statements(s) is/are
(c) Only I alone is sufficient. sufficient/necessary to answer the questions.
(d) Only II alone is sufficient. 20. O is center, find area of shaded region
(e) Either I or II alone is sufficient
Directions (15-19): The following questions are
accompanied by two statements A and B. You have to
determine which statements(s) is/are
sufficient/necessary to answer the questions.
(a) Statement A alone is sufficient to answer the question A. Length of AO is given.
but statement B alone is not sufficient to answer the B. AC is given in multiple of radius of semicircle.
questions. (a) Only A
(b) Statement B alone is sufficient to answer the question (b) Only B
but statement A alone is not sufficient to answer the (c) Either Only A or Only B
question. (d) A and B together
(c) Both the statements taken together are necessary to (e) A and B together are not sufficient
answer the questions, but neither of the statements
alone is sufficient to answer the question. 21. In a box three type of balls are there, Black, Red and
(d) Either statement A or statement B by itself is White. If no. of white balls is given then find out the
sufficient to answer the question. probability of getting one white ball.
(e) Statements A and B taken together are not sufficient A. Probability of getting one Red ball is given.
to answer the question. B. Probability of getting one black ball is given.
(a) Only A
15. An equilateral triangle is inscribed in a circle. What (b) Only B
will be the difference between the area of circle and (c) Either Only A or Only B
area of triangle? (d) A and B together
(A) Radius of circle is given (e) A and B together are not sufficient
(B) Sum of the perimeter of triangle and circle is
given. 22. What is the volume of the sphere?
A. Surface area of hemisphere is equal to the total
16. Is ‘n’ even or odd, if n is a natural number. surface area of the cylinder having radius and
(A) 2n + 1 is divisible by 3 height in ratio 3 : 4.
(B) 4n − 1 is divisible by 3 B. When we cut sphere into two hemi-sphere then
total surface area is equal to the area of a circle
17. Five positive naturals numbers are given which may
whose radius is 21 cm.
or may not be equal the sum of these numbers.
(a) Only A
(A) If numbers are arranged is ascending order then
(b) Only B
median is 18 while sum of first and last number
(c) Either Only A or Only B
and second and fourth number is equal
(d) A and B together
(e) A and B together are not sufficient
178 Adda247 Publications For More Study Material
Visit: adda247.com
Cracker Book for Bank (IBPS | SBI | RRB PO | Clerk) Mains Exams

23. PR is diameter of circle. Find, ∠QPO – ∠SRO = ? (a) Only III


(b) Only I and II or II and III.
(c) Only II
(d) All I, II and III
(e) None of these
27. How many students failed in class 11th ?
I. 400 Students passed in class 11th.
II. No. of students failed in class 11th is 20% of those
A. ∠SPO = 40° failed in class 12th.
B. ∠PQR + ∠QRO = 120° III. Ratio of student appeared to that of failed in class
11th is 5 : 3.
(a) Only A
(a) Only I and III
(b) Only B
(b) Only II
(c) Either Only A or Only B (c) Only I and II
(d) A and B together (d) All I, II and III
(e) A and B together are not sufficient (e) Cannot be answered even including all three
24. A shopkeeper gets a loss of 70 Rs. when he sold an statement
article at 20% discount on M.P. Find cost price of 28. What is the rate of interest on some amount?
Article. I. S.I. accrued in two years on same amount at same
A. % of mark up above cost price is equal to % rate of interest is Rs. 44000.
discount given on M.P. II. The amount after some years on S.I. is Rs. 154000.
B. When no discount is given, article sold at profit of III. Difference between the C.I. and S.I. earned in two
350 Rs. years on the same amount invested is Rs. 120.
(a) Only I and III
(a) Only A
(b) Only III
(b) Only B
(c) Only II and III
(c) Either Only A or Only B (d) Cannot be answered even including all statement
(d) A and B together (e) None of these
(e) A and B together are not sufficient
29. What is the sum of two number?
Directions (25-29): The following questions are I. The bigger no. is 6 more than the smaller no.
accompanied by three statements (I), (II), and (III). You II. 40% of smaller no. is equal to 30% of bigger no.
have to determine which statement(s) is/are sufficient III. The ratio b/w half of the bigger no. & one-third of
/necessary to answer the questions smaller no. is 2 : 1.
(a) Only II & III
25. What is distance between A and B ? (b) Only I & II
I. Two persons Amit and Abhi started (c) Any two of the three statement
simultaneously from A to B with their speed in (d) All statement is required
ratio 4 : 5. (e) I and II or I and III
II. Abhi reached reached Q one hour earlier than Directions (30-33): In the following questions three
Amit. statements either A, B and C or I, II and III are given. You
III. Difference between speed of Amit and Abhi is 20 have to use your knowledge of mathematics to answer
km/hr. which statement(s) is/are sufficient to answer the
(a) Only I and II. question.
(b) Only II and III
30. What is the cost price of wrist watch?
(c) All I, II and III A. Shopkeeper gives 20% discount on a speaker.
(d) Cannot be answered even including all three B. The marked price of wrist watch is 25% more than
statement that of speaker.
(e) None of these C. The shopkeeper earns a profit of 10% after selling
26. What is the area of rectangle ? the speaker.
(a) Any two of them are sufficient
I. If ratio of length and breadth of the rectangle is 3 :
(b) All the three statements are required
2. (c) Only C is sufficient.
II. Circumference of a circle is 440 m and breadth of (d) Even using all statements, answer cannot be
rectangle is 1/7 th of diameter of circle. found
III. If length is 50% more than breadth. (e) Only A is sufficient
179 Adda247 Publications For More Study Material
Visit: adda247.com
Cracker Book for Bank (IBPS | SBI | RRB PO | Clerk) Mains Exams

31. What is the total quantity of milk in final mixture of II. Time taken by Nitin alone to make 60 t-shirts is 9
milk and water after adding some milk ? days.
A. 68 liters of initial mixture has ratio of milk and III. The ratio of days taken by Vishal to cut cloth for
water as 11:6. 50 t-shirts and to stich 100 t-shirts is 4 : 25.
B. A certain amount of milk is added to this mixture (a) Only I and III together or only II and III together
to make the ratio of water to milk as 6:13. (b) All I, II and III together
C. Selling the mixture at a certain rate, 35 % profit is (c) Any two of the three
obtained. (d) Only I and III together
(a) Any two of them is sufficient (e) Either from I & II or I & III
(b) A and B together are sufficient
(c) All the three statements are required 35. Ratio of age of Aashish & Kullu is 4 : 5 and that of
(d) Answer cannot be found even using all the three Nikhil and Yash is 7 : 8. What is their average age after
statements 4 years.
(e) Only B and C are required I. Age difference of Nikhil and Aashish is 18 years.
II. Ratio of 12 years ago age of Aashish & Nikhil is 2 :
32. The speed of a train A and time taken by it to cross a
5 and Kullu & Yash is 1 : 2.
tunnel is known. Find length of tunnel.
III. 12 years later Aashish will be 150% of his present
A. Another train B is running in opposite direction to
A with a speed 40% more than A. age.
B. Train B crosses a platform X and a pole in 24 sec (a) Any two of I, II and III are required.
and 8 sec respectively before crossing the train A (b) Only I and II are required
and tunnel. (c) Only II and III are required
C. The ratio of length of train A and platform X is 3 : 5. (d) All I, II and III together are required
(a) Only A and B are sufficient. (e) II alone or from I and III together are required
(b) Only B and C are sufficient 36. Three athletes practice running on a circular track of
(c) Even using all the three statements answer 400 m. who runs fastest among them?
cannot be found. I. When A starts running clockwise and B starts
(d) All the three statements are required. running anticlockwise, they meet 2nd time at a
(e) Only B is sufficient 1
distance of 57 meter from starting point in
7
33. In how many ways 2 green balls can be chosen from 6
box ‘A’ which contains red, green and black balls? clockwise direction after 22 7 seconds.
A. Ratio of total number of balls in boxes A and B is 2 : II. B runs 5 m/s faster than C. A is not twice or more
3 and box A contains 5 green balls. fast than B and neither B is twice or more fast
B. Total balls in box B is 18 while ratio of red and than A or C.
black balls in box A is 3:4. III. When A, C and B, all runs in same direction, A and
C. Box ‘B’ contains 5 red 7 black and 6 green balls C only meet at starting point, while ratio of speed
only while ratio of red and green balls in box ‘A’ is of B to C is 3 : 2.
3:5. (a) Any one of them
(a) Statement A and either B or C are sufficient (b) Only I and II together are sufficient
(b) Any two statements are sufficient (c) Any two of the three together are sufficient
(c) All the three statements are required (d) None of the above
(d) Statement B and either A or C are sufficient (e) All the three statements are not sufficient.
(e) None of these
37. A man has 16 balls with him, in 3 colors i.e. Red, green
Directions (34-38): The following questions are and blue. Three balls are drawn at random, what is
accompanied by three statements (I), (II), and (III). You the probability that all are blue.
have to determine which statements(s) is/are I. The probability of drawing a blue ball is greater
sufficient/necessary to answer the questions. 1 5
than and less than . Also, the number of green
8 16
34. Making a new t-shirt, consists of two parts first is
balls he has is odd and number of red balls he has
cutting of cloth and second is stitching the cloth. In
is even.
how many days Nitin and Vishal together can make 3 1
100 t-shirts. II. Probability of drawing red ball is which is
8 16
I. Nitin can cut cloth for 20 t-shirts in a day and he is lesser than that of green balls.
20% slow as that of Vishal in cutting clothes and III. If a man lost one ball, probability of drawing a blue
25% faster than Vishal for stitching the cloth. ball is ⅓.

180 Adda247 Publications For More Study Material


Visit: adda247.com
Cracker Book for Bank (IBPS | SBI | RRB PO | Clerk) Mains Exams

(a) Only I and III or only II and III (a) Either A and B or B and C are sufficient to
(b) All I, II and III together answer the question
(c) Any one of them is sufficient (b) Either A and B or A and C are sufficient to
(d) Only I and III together sufficient answer the question
(e) None of the above (c) Either A and C or B and C are sufficient to
answer the question
38. A man bought two bats and 6 identical balls, he sold (d) A, B and C together are sufficient to answer the
all of them in a day, calculate his overall profit %. question
I. He sold one bat at a price of Rs. 600 and other at (e) Either only A or B and C together are sufficient
Rs. 420 and profit is 20% on both bats. Each ball to answer the question
is sold at 12% profit.
II. Profit earned from 6 balls is Rs. 36 and profit 41. What will be 26 times of 26th term of Arithmetic
earned from both bats is Rs. 170. progression (AP)?
III. Profit earned on each bat is 20% while cost price A. If eight times of the 8th term of that A.P. is equal to
of each ball is Rs. 50. 18th times the 18th term of that A.P.
B. If first term of AP is a prime number.
(a) Any one of them
C. If difference between two terms in A.P. is an odd
(b) Only I and II together are sufficient
number.
(c) Any two of the three together are sufficient
(a) Only statement ‘A’ alone is sufficient to answer
(d) None of the above
the question
(e) All three together are sufficient
(b) Only statement ‘B’ alone is sufficient to answer
Directions (39-42): The following questions are the question
accompanied by three statements A, B and C. You have to (c) Only statement ‘C’ alone is sufficient to answer
determine which statement(s) is/are necessary/sufficient the question
to answer the question. (d) Any of two statements are sufficient to answer
39. Find the area of the shaded region? the question
(e) All three together are sufficient to answer the
question
42. What is cost price of the article?
A. On selling the article at 119 Rs. shopkeeper earn
profit % equal to cost price of article.
2
B. If 14 % discount is given on the mark price, then
7
its selling price will be 102 Rs.
C. If article sold on profit percent half of the mark
A. Radius of bigger circle is given. price, then profit is 41.65 Rs.
B. Diagonal of square ABCD is given. (a) Either A and B or B and C are sufficient to
C. Difference between area of bigger circle and area of answer the question
smaller circle is given. (b) Either A and B or A and C are sufficient to
(a) Either A or B alone is sufficient to answer the answer the question
question (c) Either A and C or B and C are sufficient to
(b) Either B or C alone is sufficient to answer the answer the question
(d) A, B and C together are sufficient to answer the
question
question
(c) Either A or C alone is sufficient to answer the
(e) Either only A or B and C together are sufficient
question
to answer the question
(d) Any of A, B or C alone is sufficient to answer the
question 43. A joker has three identical boxes having same number
(e) Either only A or B and C together is sufficient to of balls but are of different colors. In each box ‘x’ balls
answer the question are red, 7 balls are blue and ‘y’ balls are green. Find
(𝒙 + 𝒚)?
40. Find the surface area of cone? A. Probability of choosing one ball which is either red
A. Ratio between height and radius of cone is 8 : 7. 𝟑
B. Radius of cone is half of the radius of hemisphere, or blue from any one of the box, is 𝟒.
which volume is 19404 cm³. B. Number of ways to select two balls from any one of
C. Radius of cone is 75% of radius of cylinder, which the box is 6.
total surface area of 2640 cm³ and ratio between C. Probability of choosing one ball which is either red
𝟗
height and radius of cylinder is 8 : 7. or green from any one of the box is 𝟏𝟔.

181 Adda247 Publications For More Study Material


Visit: adda247.com
Cracker Book for Bank (IBPS | SBI | RRB PO | Clerk) Mains Exams

(a) Only statement ‘A’ alone is sufficient to answer (c) Statement ‘C’ and ‘A’ together are sufficient to
the question answer the question
(b) Only statement ‘B’ alone is sufficient to answer (d) Any of two statements are sufficient to answer
the question the question
(c) Only statement ‘C’ alone is sufficient to answer (e) Either ‘A’ and ‘B’ together or ‘B’ and ‘C’
the question together
(d) Any of two statements are sufficient to answer 47. In how much time Bhavya can row 64km in a stream
the question to reach a destination ‘A’ and return back?
(e) Both A and C are required together A. Time taken by Bhavya to reach destination ‘A’ is
twice than that of to return back.
44. Is (bc − ab) even integer or odd integer if a, b and c
B. Ratio between speed of boat in still water to speed
are integers? of boat in downstream is 3 : 4
A. a, b and c are in arithmetic progression. C. Upstream speed of boat is 4kmph less than
B. Set {a,b,c} is co-prime and a,b and c are downstream speed.
consecutive integers. (a) Either statement ‘A’ and ‘B’ together or ‘B’ and
C. a, b and c all three are prime numbers. ‘C’ together are sufficient
(a) Either statement ‘A’ or ‘B’ alone is sufficient to (b) Either statement ‘A’ and ‘C’ together or ‘B’ and
answer the question ‘C’ together are sufficient
(b) Either statement ‘B’ or ‘C’ alone is sufficient to (c) Either statement ‘A’ and ‘B’ together or ‘A’ and
answer the question ‘C’ together are sufficient
(c) Either statement ‘C’ or ‘A’ alone is sufficient to (d) Any of two statements are sufficient to answer
answer the question the question
(d) Any of two statements are sufficient to answer (e) All three are required to solve the question
the question
(e) Any one of them is sufficient to answer the 48. A, B and C together can complete work ‘X’ in 5 days.
question Find in how many day ‘C’ alone can complete work
‘X’?
45. Find the area of a rhombus? A. A and B together can complete work ‘X’ in 15 days
A. The ratio of the diagonals of the rhombus is 3 : 4. if both worked with half of their efficiencies.
B. The perimeter of the rhombus is equal to that of a B. B, who is 25% more efficient than ‘C’, takes 8 days
square whose diagonal length is 20√2m. more than ‘A’ to complete work alone
C. The difference between the squares of the C. ‘A’ is 25% less efficient than ‘B’.
diagonals of rhombus is 448 m2. (a) Either statement ‘A’ alone or ‘B’ alone is
(a) Statement ‘A’ and ‘B’ together are sufficient to sufficient to answer the question
answer the question (b) Either statement ‘B’ alone or ‘C’ alone is
(b) Statement ‘B’ and ‘C’ together are sufficient to sufficient to answer the question
answer the question (c) Either statement ‘C’ alone or ‘A’ alone is
(c) Statement ‘C’ and ‘A’ together are sufficient to sufficient to answer the question
answer the question (d) Any of two statements are sufficient to answer
(d) Any of two statements are sufficient to answer the question
the question (e) Any one of them is sufficient to answer the
(e) Any one of them is sufficient to answer the question
question 49. Find the sum of ‘x’ and ‘y’ ?
46. What will be the sum of the ages of father and the son A. H.C.F and L.C.M of ‘x’ and ‘y’ is 21 and 3003
after five years? respectively.
B. Difference between x and y is 42
A. Father’s present age is twice son’s present age C. Both ‘x’ and ‘y’ are multiple of 21.
B. After ten years the ratio of father’s age to the son’s (a) Either statement ‘A’ and ‘B’ together or ‘B’ and
age will become 12 : 7. ‘C’ together are sufficient
C. Five years ago the difference between the father’s (b) Either statement ‘A’ and ‘C’ together or ‘B’ and
age and son’s age was equal to the son’s present ‘C’ together are sufficient
age. (c) Either statement ‘A’ and ‘B’ together or ‘A’ and
(a) Statement ‘A’ and ‘B’ together are sufficient to
‘C’ together are sufficient
answer the question
(b) Statement ‘B’ and ‘C’ together are sufficient to (d) Any of two statements are sufficient to answer
answer the question the question
(e) All three are required to solve the question

182 Adda247 Publications For More Study Material


Visit: adda247.com
Cracker Book for Bank (IBPS | SBI | RRB PO | Clerk) Mains Exams

50. Find the area of shaded region if radius of each circle C. Area of triangle is approximately 310% more than
is same? area of a circle.
(a) Either statement ‘A’ or ‘B’ alone is sufficient to
answer the question
(b) Either statement ‘B’ or ‘C’ alone is sufficient to
answer the question
(c) Either statement ‘C’ or ‘A’ alone is sufficient to
answer the question
(d) Any of two statements are sufficient to answer
the question
A. Area of circle is given. (e) Any one of them is sufficient to answer the
B. Perimeter of triangle is given. question

Solutions

1. (c); From I. 4. (e); From I,


Lets length of larger rectangle and smaller Let speed of boat in
rectangle be 5x and 4x respectively. still water x km/hr and speed of current y km/hr
2(5x+b) – 2(4x+b) = 8 x = 3y …(1)
Form II. From II,
Breadth of rectangle = side of square ′𝑑′
Let bot cover ‘d’ distance downstream and
Side of square = 14 cm 2

From I & II we get distance in upstream


d
10x – 8x = 8 d
= 2

1
=
1
x = 4cm 3y+y 3y−y 4y 4y

perimeter of smaller rectangle So, From I & II both not sufficient to answer of the
= 2 (4×4+14) = 60 cm question–
So, I and II both together sufficient to give answer 5. (d);From I,
2. (e); From I, Let amount invested on 20% is x Rs and on 10%
Lets number of girls and boys be 9x and 11x is (4500–x) Rs.
respectively 𝑥 × 20 × 2 (4500– 𝑥) × 2 × 10
=
From II, 100 100
Let total students 100x 40x = 90000 – 20x
Girls below eighteen years group 60x = 90000
9
= 100x ×100 = 9x x = 1500 Rs.
9𝑥 amount invested on 10% = (4500–1500)
Given 20 × 100 = 45x
= 3000 Rs.
Boys : girl = (100x–45x) : 45x = 11 : 9
From I and II we get same equation but we can’t From II,
get the answer. Lets man invested Rs 3x
So, Neither I nor II both sufficient to give answer Equivalent CI of two years on 20%
of question. 20×20
= 20 + 20 + 100
= 44%
3. (c); Form I, ATQ –
Selling price of article = 3800 Rs. 20 44
3800 2𝑥 × + 𝑥× = 1260
Marked price of article = 95 × 100 = 4000 100 100
108x = 126000
From II
4000 x = 1500 Rs.
Cost price = 125 × 100 = 3200
Amount invested on 10% = 2 × 1500 = 3000 𝑅𝑠.
From I & II So, Either statement I alone or statement II alone
Profit of shopkeeper = 3800 – 3200
sufficient is to give answer of question
= 600 Rs.
183 Adda247 Publications For More Study Material
Visit: adda247.com
Cracker Book for Bank (IBPS | SBI | RRB PO | Clerk) Mains Exams

6. (d);Let length of two trains be 4x meter and 3x meter 𝑎=3


So, we can determine (x + y) – 1.5z from II also
From I,
5 (4𝑥+3𝑥)3 So, Either statement I or Statement II alone
(90 + 72) × = sufficient
18 28
21x = 1260
10. (c); Let length and breadth of rectangle be 7x and 4x
x = 60 meters
respectively
Required difference = 60 × 4 − 60 × 3
= 60 meters From I,
Given, πr2 = 616
From II, 616×7
5 3𝑥 r2 =
22
90 × = r = 14 cm
18 7.2
3x = 180 meters length of rectangle = 14 × 2 = 28 𝑐𝑚
180
Length of larger train = 4x = × 4 = 240 breadth of rectangle =
28
× 4 = 16 𝑐𝑚
3 7
Required difference = 60 meters From I and II together,
So, either I or II alone sufficient to give answer of
2( l + b) – 4a = 20
question.
2 (28 + 16) – 4a = 20
7. (c); From I and II together 4a = 88 – 20
Let P, Q and R takes 2x days, 3x days and 6x days a = 17 cm
respectively
Area of Square = (17)2
So efficiency of P, Q and R is 3x, 2x and x unit/day
Total work = 4 × (3𝑥 + 2𝑥 + 𝑥) = 24x = 289 cm2
So, we can determine required difference with I So, Statement I and II both together sufficient
and II together 11. (b);If the max marks of exam = x
x
8. (d);Given, Cost price of article for Satish Raman = 4
1440 x
= 120 × 100 = 1200 Rs. ⇒ = 288 − 128 = 160
4
From I, x = 640
Cost price of article for Veer = 1200 – 240 ∴ Minimum passing marks
= 960 Rs. = 160 + 64 = 224
240 224
Profit percentage of Veer = 960 × 100 Required %= 640 × 100 = 35%
= 25% 12. (c); From I,
From II, A + S+ V = 3 × 68 = 204 kg
Cost price of article for Veer = 1440 ×
100 R + P = 144 kg
150 A ⇒ 204 – 46 – 78 = 80 kg
= 960
P ⇒ 144 – 68 = 76 kg
Veer profit percentage
1200−960 S = 78 kg
= × 100 = 25%
960
So, either I or II alone sufficient to give answer of 13. (e); From I
question. 27x − 23x = 100000
9. (d);x : y : z = 11 : 9 : 12 x = 25000
Let x, y and z be 11a, 9a and 12a respectively Population of city A = 50x
From I, = 1250000
11a + 9a + 12a 11a + 9a
− =2 From II
3 2
32a Population of city B = x
− 10a = 2 4x
3 Population of city A = 5
𝑎=3
4x
So, x− = 312500
(x + y) – 1.5z = (11 × 3 + 9 × 3)–1.5×3×12 = 6 5
x = 1562500
From II, 4
1 1
11𝑎 × + 9𝑎 × = (62)0.5 Population of city A = 5 × 1562500
11 9
2𝑎 = 6 = 1250000
184 Adda247 Publications For More Study Material
Visit: adda247.com
Cracker Book for Bank (IBPS | SBI | RRB PO | Clerk) Mains Exams

14. (b);From I and II 2


19. (b);Probability that Amit tells truth = 𝑥
Students participating in dance = 150 𝑥−2
Students who participate in singing Probability that Amit tells lie = 𝑥
150×100 5
= 250 = 60 Probability that Ankush tells truth= 𝑦
𝑦−5
15. (d);Form A → Let radius of circle = r Probability that Ankush tells lie =
𝑦
2 𝑦−5 𝑥−2 5 1
Then side of equilateral ∆ = √3𝑟 From A→ 𝑥 × + ×𝑦=2
𝑦 𝑥
Area of equilateral triangle and circle can be find 2 5 1
out and required difference can also be find out From B → 𝑥 × 𝑦 = 5
easily. ⇒ 𝑥𝑦 = 50
From B → Let radius of circle =r Hence only B is sufficient to answer the question.
⇒ side of equilateral triangle = √3 r 20. (d);Given
Sum of peremeter of triangle and circle is given. AO + OC = OB
By this value of ‘r’ can be find out and after this From A = AO → find
required difference can be find out easily. Hence, From B → AC = x (AO)
either only A or only B is sufficient to answer the When ‘x’ multiple
question From A & B together area of triangle and Area of
16. (a); From A → 2𝑛 + 1 is divisible by 3 semi circle find out.
⇒ n =1, 3, 5, 7…. Required area
π(AO)2
⇒ n = odd = − √S(S – AO). (S – OC). (S – AC)
2
From B) → 4𝑛 − 1 is divisible by 3 S=
AO+OC+AC
⇒ n =1, 2, 3……. 3

⇒ ‘n’ can be even as well as odd ∴ A & B together sufficient to answer the question
Hence, Only A is sufficient to answer the question 21. (d);Given no. of white ball
17. (b);From A → Let numbers are a, b, c, d, e in Let → a
x
ascending order From A let probability → y
⇒ Median = 18 = c Let no. of red ball → px, total balls → py
And a+e=b+d From B → Let probability = t
𝑠

From B → let number are a, b, c, d, e is ascending


Let no. of black ball = qs, total balls = qt
order.
From A & B
Median = 18 = c
Mode = 12 and 24 px + a + qs = qt = py
This means two number are 12 and two number we know the values of x, y, s, t and a so we can
are 24 find the value of p and q
a a
⇒ Numbers are 12, 12, 18, 24, 24 So probability of white ball found = or
𝑞𝑡 py
Required sum 12 + 12 + 18 + 24 + 24 = 90 ∴ A & B together sufficient to answer the question
Hence, Only B is sufficient to answer the question.
22. (b);Only B is sufficient to answer the question
18. (d);Let CP = 200x When we cut sphere into hemisphere total
⇒ MP = 300x surface area of two hemisphere
From A → SP = 300x – 60 Total surface area of two hemisphere
120
200𝑥 × = 300𝑥 − 60 3πr² + 3πr² = π × 21 × 21
100 r = find out
⇒ 60𝑥 = 60
So, volume of sphere can be find out.
⇒ CP = 200x = 200
From B → Two successive discount 23. (d);Given
10 × 10 ∠PQR = ∠PSR = 90° {Angle of diameter}
= 10 + 10 − = 19%
100 A → ∠SRO = 180°– 90°(∠PSR) – 40°(∠ SPO)
81
S.P =200x + 43 = 300𝑥 × 100 ∠SRO = 50°
200x + 43 = 243x B→ ∠PQR + ∠QRO = 120°
x=1 ∠QRO = 120°– 90 = 30°
CP = 200x = 200 ∠QPO = 60°
Hence, either alone A or alone B is sufficient to From B & A together, required difference can be
answer the question. find out.

185 Adda247 Publications For More Study Material


Visit: adda247.com
Cracker Book for Bank (IBPS | SBI | RRB PO | Clerk) Mains Exams

24. (c); From A→ Discount % = 20% = Mark up% From III


PR2
If cost price is 100x then Markup price 120x and Difference = 1002
selling price is → 96x PR2
= 120
So ATQ, 1002
100x – 96x = 70 From I and III R can be found.
70 29. (e); Let the smaller no. is x & bigger no. is y.
C. P = 100x = × 100x = 1750
4x
From I
From B→ Let mark up price is → 100x y=x+6
Then selling price is → 80x From II,
40 30
ATQ, ×𝑥 = ×𝑦
100 100
100x – 80x = 350 + 70 From III,
𝑦
20x = 420
2 =2
100x = 2100 𝑥 1
80x = 1680 3
⇒ 3y = 4x
C.P. → 1680 + 70 = 1750
∴ from I and II or I and III
So Either A or B alone required.
30. (d);From statement A,
Solutions (25-29) Let MP of speaker = Rs. 100
25. (c); From I, II & III ∴ SP = 80 rupees
Let speed of Amit and Abhi be 4x and 5x km/hr From B,
respectively. MP of wrist watch = 1.25 M.P of speaker
5x – 4x = 20 = Rs. 125 (From st. A)
∴ x = 20 km/hr From C,
100 800
Let distance be d km C. P. of speaker = 80 × = (from st. A)
d d 110 11
– =1 Here, there is no information about S.P. and Profit
80 100
∴d=
80×100
= 400 km of watch. So, answer cannot be found.
20
31. (b);From A,
26. (b);From I and II 11
Let length and breadth be 3x and 2x Milk = 68 × = 44 ℓ
17
6
2πr = 440 [r → radius of circle] Water = 68 × = 24 ℓ
17
r = 70 m From B, let x ℓ milk is added to the mixture.
∴ breadth = 10 m 24 6
= 13 ⇒ x = 8 ℓ
& length = 15 m 44+x
∴ Area = 10 × 15 = 150 m² ∴ Total milk in final mixture = 44 + 8 = 52 ℓ
Statement I and III are same. 32. (d);From st. A,
27. (a); From I Speed of train A = P m/sec (Given)
140 14P
Passed = 400 Speed of train B = of P =
100 10
From III From st. B+A,
Let appeared & Passed student be 5x and 3x Length of platform
14P 14P
respectively 24 × 10 – 10 × 8
14P
2x = 400 ⇒ x = 16 × 10 = 22.4 P
= 200 From C, Length of train A
∴ failed = appeared – passed 3
= 5 × 22.4P (with help of st. A and B together)
= 1000 – 400 3
∴ Length of tunnel = Pt(given) – × 22.4P
= 600 5

33. (e); With help of statement A only, required number


28. (a); From I of ways can be found out.
PRT(2)
= 44000
100 34. (e); From statements (I)
PR = 2200000 Nitin can cut cloth for 20 t-shirts in a day
5
From II ∴ Vishal can cut cloth for 20 × = 25 t-shirts in a
PRT 4
P + 100 = 15400 day

186 Adda247 Publications For More Study Material


Visit: adda247.com
Cracker Book for Bank (IBPS | SBI | RRB PO | Clerk) Mains Exams

And if Vishal can stich 4x t-shirts in a day, Nitin Hence their age can be calculated. Also their
will stich 5x t-shirts. average age after 4 years.
It can’t be solved further. Therefore, statement (II) alone is efficient to
From statement (II) solve question.
We only can obtain time taken by Nitin to build From (III) statement
4𝑥+12 3
100 t shirts which is15 days 4𝑥
= 2 ⇒ x = 6, present age of Aashish is 24
Nothing more can be obtained from (II) alone. years as that of Kullu is 30 years.
From statement (III) Nothing more can be calculated
Question can’t be solved from this statement Now combining statement (III) & (I)
alone. We will get x = y = 6.
So, by using (I) & (II) Hence question can be solved.
Nitin will take 3 days to cut cloth for 60 t-shirts, Therefore, this question can be solved by
Hence 6 days to stich them. statement (II) alone or combining (I) and (III).
Therefore, Nitin stiches 10 t-shirts per day 36. (b);
And Vishal stiches 8t-shirts per day. (From ratio
obtained in (I))
Total time required for Nitin alone to make 100 t-
shirts
100 100
= + = 15 days
20 10
Total time required for Vishal alone to make 100
t-shirts
100 100
= 25 + 8 = 4 + 12.5 = 16.5 days Let us draw a circular track of 400 meters with 4
point U, V, W and X at anti-clockwise distance of
Hence time can be calculated for both. 100m, 200 m and 300 meter.
Using statement (I) and (III) From statement (i)
160
Days required by Vishal to cut cloth for 50 They meet second time in seconds that is
7
t-shirts is 2 days somewhere between U and X.
∴ Days required by Vishal to stich 50 Therefore their sum of speed [as both were
t-shirts in 12.5 days travelling in opposite direction] is
800×7
Hence days required for both Nitin and Vishal can = = 35 ms¯¹
160
be calculate, [first we will calculate, days required As they meet between U and X.
individually and then for both of them] There are two chances, either A have only covered
Hence question can be solved through I & II or I & 1
a distance of 57 meter or total distance of
III. 7
1
(400 + 57 7) meter, hence, we can’t determine
35. (e); From Question,
Let age of Aashish and Kullu is 4x and 5x and that who is more faster.
of Nikhil & Yash is 7y and 8y. From statement (II)
From (I) statement It is clearly given that B is faster than C, but we
can’t determine who is faster between A and C.
7y – 4x = 18 …eqn. (A)
From statement III
It can’t be solved further.
A and C only meet at starting points means either
From (II) statement
4𝑥–12 2 their speed is co-prime of each other or one of
7𝑦–12
=5 their speed is integral multiple of other and ratio
⇒ 20x – 60 = 14y – 24 of speed of B and C is given.
⇒ 10x – 30 = 7y – 12 Now combining statement (i) and statement (ii)
It is given that B doesn’t run faster at twice or
⇒ 10x – 7y = 18 …eq. (B) more speed. Hence chances that A only travelled
5x–12 1 1
8y–12
=2 (400 + 57 7) meter distance, hence faster than B,
⇒ 10x – 24 = 8y – 12 and it is given that B runs faster than C in
⇒ 5x – 12 = 4y – 6 statement (ii).
5x – 4y = 6 …eqn. (C) Hence question can be answered from statement
(i) and (ii)
Solving eqn. (B) & eqn. (C) We can’t answer question from (i) and (iii) or (ii)
We will get or (iii), as we don’t know who is faster among A
x=y=6 and C.
187 Adda247 Publications For More Study Material
Visit: adda247.com
Cracker Book for Bank (IBPS | SBI | RRB PO | Clerk) Mains Exams

37. (c); From statement …(I) We can say that Rs. 170 is 20% of total CP of but,
2
Probability is greater than ⅛ i.e. and less than hence SP and CP of both bats can be calculated.
16
5 CP of balls is given in statement (III), Hence
. Therefore, he could either have 3 balls or 4
16 question can be answered from (II) and (III).
balls. Now sum of green balls and red balls is odd.
Hence, we can answer question from any two
[Odd + Even = Odd]. 16 is even.
Therefore, blue balls must be odd. [Odd + Odd = statements.
Even]. 39. (d);
Hence there are 3 blue balls.
3
(II) Probability of drawing red ball is
8
Therefore, he has 6 red balls.
3 1 7
Probability of drawing green ball = 8 + 16 = 16
He has 7 green balls.
∴ No. of blue balls he has = 16 – (6 + 7) = 3
Hence question can be answered.
(III) Let he have x balls. If one ball is lost, By joining the centers of all four circles, a square
probability of drawing a blue ball is is formed which contain 4 quadrant of circles
which together makes a complete circle.
Area of shaded region = Area of square(EFGH) -
Area of bigger circle – Area of smaller circle
Let radius of smaller circle and bigger circle be ‘r’
and ‘R’ respectively
Which gives us x = 3 Then relation between ‘r’ and ‘R’ is
Hence question can be answered.
𝑟 = 𝑅(√2 − 1)
Question can be answered from either
A → Radius of bigger circle is given
statement alone.
By this side of square, radius of smaller circle can
38. (c); From (I) be find out. So, area of shaded region can be find
Total SP of bats = 1020 out
5
Therefore, CP of bats = 1020 × = Rs. 850 B → Diagonal of square is given
6
Profit % on ball = 12% By this, side of square then radius of bigger circle
And more can’t be calculated. and then radius of smaller circle can be find out
From (II) After that shaded region can be find out
Profit on 1 ball = Rs.6
C → by using this we can find out the radius of the
Profit on selling both bats = Rs. 170
both circles and side of square.
From (III)
Cost price of each ball = Rs.50 𝜋𝑅2 − 𝜋𝑟 2 = 𝑔𝑖𝑣𝑒𝑛
Profit earned on each bat = 20% Relation between R and r is:-
From (I) and (II) 𝑟 = 𝑅(√2 − 1)
Total SP of bats = Rs. 1020 Hence, Any of A, B or C alone is sufficient to
And CP of bats = Rs. 850 answer the question
Total profit of 36 on balls is 12% of total CP of
balls. 40. (b);From A,
36 Given 𝑟 ∶ ℎ = 7 ∶ 8
Hence profit on each ball is 12% which is
6 From B
= Rs. 6 Radius of hemisphere
Radius of cone =
Hence CP = Rs. 50/ ball 2
2
And question can be answered. Volume of hemisphere = 3 𝜋𝑟 3
From (I) and (III) 2
𝜋𝑟 3 = 19404
3
Question can be answered, as we can calculate 19404 × 3 × 7
both CP and SP of both balls & bat. 𝑟3 =
22 × 2
From (II) and (III) 𝑟 3 = 9261
Total profit earned is Rs. 206 [from (II)] 𝑟 = 21 cm

188 Adda247 Publications For More Study Material


Visit: adda247.com
Cracker Book for Bank (IBPS | SBI | RRB PO | Clerk) Mains Exams

From A & B together — From B,


21 As all boxes are identical and all colored balls are
Radius of cone = 2 cm
10.5 identical then number of ways to choose balls is 6
Height of cone = 7 × 8 = 12 cm ⇒ 𝑅𝑅, 𝐺𝐺, 𝐵𝐵, 𝑅𝐺, 𝑅𝐵, 𝐺𝐵
So from A and B we can determine the surface Where GG = Green | Green
area of cone. By this value of x and y cannot be find out
From C
From A and C,
ATQ,
Given, radius of cone : height of cone = 7 : 8 𝑥+𝑦 9
Ratio between height and radius of cylinder = 8 : 7 = ⇒x+y=9
𝑥+𝑦+7 16
2πr (r + h) = 2640 cm³ So, only statement ‘C’ is sufficient to give answer
2π7x (8x + 7x) = 2640 of question.
x = 2 cm
75 44. (a); From A,
Radius of cone = 7 × 2 × 100 = 10.5 cm a, b and c are in A.P
10.5
Height of cone = × 8 = 12 cm Let common difference = ‘d’ and
7
So, from A and B or from A and C, we can ⇒ a =b−d
And c = b + d
determine the surface area of cone
(bc − ab) = b2 + bd − b2 + bd = 2bd
41. (a); From A, ⇒ (bc − ab) is divisble by 2
8𝑡8 = 18𝑡18 From B,
We have to find 26𝑡26 Set {a, b, c} is coprime
8(𝑎 + 7𝑑) = 18(𝑎 + 17𝑑) ⇒ all three have ′1′ as common multiple
8𝑎 + 56𝑑 = 18𝑎 + 306𝑑 And they are consecutive numbers
10𝑎 + 250𝑑 = 0 So , either b is even or a and c are both even.
(𝑎 + 25𝑑) = 0 In both cases-
So 26th term of = (𝑎 + 25𝑑) = 0 b(c-a)= even
26 times = 26 × 0 = 0 From C
From B, All three are prime numbers but we can’t say
No result determines whether in these prime numbers 2 is taken or not.
From C And if 2 is taken then b = 2 or not. SO, in some
No result determines cases (bc − ab) is divisble by 2 and in some cases
So, only statement A sufficient to give answer of (bc − ab) is not divisble by 2. Correct conclusion
question. cannot be inferred from statement ‘C’ alone
So, either statement ‘A’ or ‘B’ alone is sufficient to
42. (e); From A: give answer of question.
Let cost price = profit percent = x
So, 45. (d);From A,
𝑥×(100+𝑥) diagonal = 3x, 4x
= 119 Rs.
100 From B,
x² + 100x = 11900 4 a = 80, a = 20 ,𝑑12 + 𝑑22 = 1600
x² + 100x – 11900 = 0 From C,
x = 70 Rs. (4𝑥)2 − (3𝑥)2 = 448
From B: x = 8, diagonal = 24, 32
Selling price = 102 Rs. 1
102 Area = 2 × 24 × 32 = 384 𝑚2
M.P. ⇒ 6 × 7 = 119 Rs.
So, using any two of the three statement we can
From C: find area of rhombus
If M.P. ⇒ 119 Rs.
Let cost price = x Rs. 46. (e); Let age of father = 𝑥
Age of son = 𝑦
From B and C together:
𝑥 119
From A,
100
× 2
= 41.65 Rs. 𝑥 = 2𝑦
x = 70 Rs. From B,
𝑥+10 12
Either only A or B and C together. 𝑦+10
= 7
From C,
43. (c); From A,
𝑥 − 𝑦 = 𝑦, 𝑥 = 2𝑦
ATQ,
𝑥+7 3 So, Either A and B together or B and C together
𝑥+𝑦+7
= 4 ⇒ 3y – x = 7 are sufficient to answer the question

189 Adda247 Publications For More Study Material


Visit: adda247.com
Cracker Book for Bank (IBPS | SBI | RRB PO | Clerk) Mains Exams

47. (b);Distance = 64 km From C


5
From A a=b
4
Ratio between time to reach destination A and to So, either statement ‘A’ alone or ‘B’ alone is
return back = 2 : 1 sufficient to solve the question.
Upstream to downstream Speed ratio
1 49. (c); From A,
= ∶ 1⇒1:2
2 𝑥 × 𝑦 = 21 × 3003 = 21 × 21 × 11 × 13
From this we can find out the ratio between speed L.C.M is 21 ⇒ Minimum value of ‘x’ or ‘y’ should
of boat in still water to speed of stream be 21
=3:1 From B,
From B (𝑥 − 𝑦) 𝑜𝑟 (𝑦 − 𝑥) = 42
Ratio of speed of boat in still water to speed of From C
boat in downstream = 3 : 4 Both x and y are multiple of 21 ⇒ L.C.M of x and y
From this we can find out the ratio between speed is 21.
of boat in still water to speed of stream Hence, either statement ‘A’ and ‘B’ together or ‘A’
=3:1 and ‘C’ together are sufficient
From C 50. (a); In the figure Triangle ABC is an equilateral
Difference between upstream speed to triangle
downstream speed = 4 km/hr And relation between radius or circle (r) and side
By using ‘A’ and ‘C’ together or ‘B’ and ‘C’ together, of triangle (a) is given below
answer can be find out. 𝑟 1
𝑎
= 2(1+√3)
48. (a); Let A, B and C can complete the work ‘X’ in a, b From A,
and c days Area is given ⇒ radius is given ⇒ side of triangle
Respectively can be find out ⇒ Area of shaded region can be
1
⇒ + + =
1 1 1 find out
a b c 5
From B,
From A Perimeter of triangle is given ⇒ radius of circle
1 1 1
2a
+ 2b = 15 can be find out ⇒ Area of shaded region can be
From B find out
4 From C
b = 5c It is a basic condition of this figure and anything
and cannot be concluded from this statement
b=a+8 Hence, either statement ‘A’ or ‘B’ alone is sufficient.

190 Adda247 Publications For More Study Material


Visit: adda247.com
Cracker Book for Bank (IBPS | SBI | RRB PO | Clerk) Mains Exams

Chapter

16 Caselets
BEST APPROACH TO SOLVE THE QUESTIONS

A Caselet is another way of providing data, more often, explicitly. Unlike various graphical methods of data
representation, a caselet requires to be solved completely before arriving at the answers to the questions that follow it.
Usually, in a caselet, data is provided through statements that imply the logic or relation of various components of data.
Some caselets are based on Set Theory and concepts of Set theory are applied to them. All other caselets are based on
basic mathematical understanding.
A sample example of a caselet is as follows:

‘PO’ exam in Lucknow is 60% more than candidates who are appearing for same exam in Agra. Number of
candidates appearing for ‘PO’ exam in Allahabad is 48% of total candidates appearing for ‘PO’ exam. Ratio between total
candidates appearing for ‘PO’ exam and ‘Clerk’ exam is 5 : 9. Difference between candidates appearing for ‘PO’ exam in
Lucknow and Allahabad is 400. Ratio between candidates appearing for ‘PO’ exam to ‘Clerk’ exam in Agra and Lucknow
is 5 : 12 and 8 ; 9 respectively.
Let total candidates appeared for ‘PO’ exam and ‘Clerk’ exam is 500x and 900x
48
Number of candidates appearing for ‘PO’ exam in Allahabad = 100 × 500x = 240x

Remaining candidates appeared for ‘PO’ exam in Agra and Lucknow together = 500x – 240x = 260x
Let total candidates appeared for ‘PO’ exam in Agra = y
Then total candidates appeared for ‘PO’ exam in Lucknow = 1.6y
ATQ,
y + 1.6y = 260x ⇒ y = 100x
12
total candidates appeared for ‘Clerk’ exam in Agra = 100x × 5
= 240x
9
total candidates appeared for ‘Clerk’ exam in Lucknow = 160x × = 180x
8

total candidates appeared for ‘Clerk’ exam in Allahabad = 900x – 240 – 180x = 480x

EXAM AGRA LUCKNOW ALLAHABAD


PO 100x 160x 240x
CLERK 240x 180x 480x

Now ATQ
240x – 160x = 400
400
⇒𝑥= 80
=5

EXAM AGRA LUCKNOW ALLAHABAD


PO 500 800 1200
CLERK 1200 900 2400

191 Adda247 Publications For More Study Material


Visit: adda247.com
Cracker Book for Bank (IBPS | SBI | RRB PO | Clerk) Mains Exams

Practice Exercise Based on new Pattern

Directions (1-5): Study the given passage carefully and 6. Number of people who did not watch any movie are
answer the questions. what percent less than number of people who
Rahul, Sandy and Sati invested in ratio 2 : 3 : 4. After 4 watching only Ravan, only PK and only 3 Idiots
months Sandy added Rs. 1500 more in his investment and together? (approx.)
Rahul withdrew Rs. 800 from his investment. After six (a) 55% (b) 50% (c) 65%
months more Sati invested half of the investment done by (d) 62% (e) 70%
Rahul in first four months and Sandy invested 50% more
7. Find the ratio between people who watching only
than the investment done by Sati in first 10 months. Rahul
invested same as investment done by Sandy in first four Ravan and only PK together to people who watching
months. Ratio of profit of Sati to total profit at the end of only Ravan & 3 idiots and only PK & 3 idiots
year is given as 125 : 376. together ?
(a) 16 : 5 (b) 16 : 7 (c) 16 : 13
1. Profit of Sandy is approximately what percent of total
(d) 16 : 11 (e) 76 : 43
profit?
(a) 64% (b) 48% (c) 72% 8. Out of total people watching 3 idiots only, ratio
(d) 68% (e) 42% between male to female is 4 : 1 and that of people
2. What is the difference between profit share of Rahul watching Ravan only is 5 : 3. Find total number of
and Sandy if total profit is Rs.37,600? males , who watching 3 idiots and Ravan only ?
(a) 12,000 (b) 16,400 (c) 18,500 (a) 436 (b) 428 (c) 440
(d) 22,900 (e) 20,000 (d) 438 (e) 442
3. Veer have 250% more than initial investment of Sati 9. People watching only Ravan & PK are what percent
for a year. Find total interest earned by him if he more than people watching only Raven & 3 idiots?
invested his amount in a scheme which offers 20% (a) 50 % (b) 45% (c) 55%
p.a. for 2 years? (d) 40% (e) 60%
(a) Rs. 1400 (b) Rs. 1500 (c) Rs. 1540
(d) Rs.1600 (e) Rs.1640 10. People who are watching only 3 Idiots, only PK and
only 3 Idiots and PK both is how much more than
4. What is the average of profit share of Sandy and Sati
out of total profit of Rs. 37,600? people who are watching Ravan?
(a) 18,220 (b) 18,250 (c) 16,420 (a) 252 (b) 192 (c) 672
(d) None of these (e) 12,490 (d) 488 (e) 272
5. If initial investment of Bhavya is one-third of initial Directions (11-15): Given below is the data about
investment of Rahul, Sandy and Sati together then students appeared in two exams i.e., A and B in six
find the difference between initial investment of different years i.e., 2011 to 2016. Total students appeared
Bhavya and Rahul. in both exam in 2016 is 8000 while in 2013 it is 5800.
(a) Rs.1000 (b) None of these (c) Rs.750 Average number of students appeared in exam B in 2011
(d) Rs.500 (e) Rs.250 and 2013 is 3100 and is in the ratio 18 : 13. Students
1
Directions (6-10): There are 2000people live in a village. appeared in exam A in year 2015 is 33 % more than
3
Out of 2000 people some people are watching ‘3 idiots’
students appeared in exam B in same year. Total students
‘Ravan’ and ‘PK’ on Sunday and some are not watching
appeared in 2016 is 25% more than total students
any movie. Number of people did not watch any movie is
equal to number people watching ‘PK’ only. Number of appeared in 2011. Students appeared in exam A in 2016 is
26
people who are watching ‘Ravan’ is 720. Ratio between 62 % more than students appeared in exam B is 2015.
27
number of people watching ‘3 Idiots’ and ‘Ravan’ only to Ratio of total students appeared in 2016 & 2014 is 16 : 13.
number of people watching ‘Ravan’ and ‘PK’ only is 2 : 3. Total number of students appeared in exam A in all six
Number of people watching ‘3 Idiots ’ and ‘PK’ both is half years is 21,100. Students appeared in exam B in 2011 is
of number of people watching only ‘PK’. Number of people
same as student appeared in exam A in 2015.Students
watching ‘3 idiots’ only is 50% more than number of
appeared in exam A in 2012 is 700 more than that of
people watching ‘Ravan’ only. Number of people watching
all the three movies is 4% of the total number of people in students appeared in same exam in 2014. Students
the village. Number of people watching ‘Ravan’ only is appeared in exam B in 2014 is 1200 less than that of in
same as number of people watching ‘Ravan’ and ‘PK’ only. same exam in 2012.

192 Adda247 Publications For More Study Material


Visit: adda247.com
Cracker Book for Bank (IBPS | SBI | RRB PO | Clerk) Mains Exams

11. In which year total students appeared in both exam is 20. How many students appeared in exam X or in exam Z?
3rd highest?
(a) 2012 (b) 2014 (c) 2016 (a) 240 (b) 360 (c) 500
(d) 2011 (e) Other than the given options (d) 680 (e) 760
12. What is the respective ratio between students Directions (21-25): Number of songs recorded by Sonu
appeared in exam A in year 2011, 2012 and 2014 Nigam in 2008 is 300. And number of songs recorded by
together to the students appeared in exam B in year
Sandeep and Arijit Singh in 2010 are in ratio 5 : 4. Total
2013, 2014 and 2016 together?
number of songs recorded in 2009 is 250% more than
(a) 95 : 97 (b) 99 : 97 (c) 98 : 97
(d) 99 : 95 (e) None of these songs recorded by Arijit Singh in 2009. Total songs
recorded by Sandeep in all three years is 650. Average of
13. Find the difference between average number of 4
songs recorded in 2010 is th of songs recorded by Arijit
students appeared in exam A and average of students 3
2
appeared in exam B in starting four years? Singh in 2008. Songs recorded by Arijit Singh in 2009 is 5th
(a) 250 (b) 225 (c) 215 of songs recorded by Sonu Nigam in 2008. Total songs
(d) 200 (e) None of these recorded in 2008 is 6 times of songs recorded by Arijit
14. Students appeared in exam A in 2013 is how much Singh in 2009 and songs recorded by Sandeep in 2008 is
less than students appeared in exam B in 2012? 25% less than songs recorded by Arijit Singh in same
(a) 1400 (b) 1000 (c) 1100 year. Ratio of songs recorded by Sonu Nigam and Sandeep
(d) 1200 (e) 1300 in 2009 is 2 : 3.
15. Total number of students appeared in both exam in 21. Number of songs recorded by Arijit Singh in all years
2012 is what percent more than total number of is what percent more/less than total songs recorded
students appeared in both exam in 2011?
in 2008 ?
(a) 25.25% (b) 28.25% (c) 31.25% 2 7 2
(d) 34.25% (e) 37.25% (a) 12 4 % (b) 17 9 % (c) 21 3 %
2
Directions (16-20): There are 1000 students in a college. (d) 27 % (e) None of these
5
Out of 1000 students some appeared in exams ‘X’, ‘Y’ and
22. What is the difference of average of total number of
‘Z’ while some not. Number of student not appeared in
any exam is equal to number of students appeared in songs recorded by Sandeep in 2008 and 2009 and
exam ‘Z’ only. Number of students appeared in exam ‘Y’ is average number of songs recorded by Sonu Nigam
360. Ratio of number of students appeared in exam ‘X’ and and Arijit Singh in 2010?
‘Y’ only to number of students appeared in exam ‘Y’ and ‘Z’ (a) 155 (b) 165 (c) None of these
only is 2 : 3. Number of student appeared in exam ‘X’ and (d) 150 (e) 145
‘Z’ both is half of number of students appeared in only
23. What is the ratio of total number of songs recorded in
exam ‘Z’. Number of students appeared in exam ‘X’ only is
2010 to total number songs recorded in 2011, if total
50% more than number of students appeared in ‘Y’ only.
Number of students appeared in all the three exam is 4% number of songs recorded in 2011 is 50% more than
of the total number of students in the college. Number of total number of songs recorded in 2009 ?
students appeared in ‘Y’ exam only is same as number of (a) 32 : 17 (b) None of these (c) 21 : 19
students appeared in ‘Y’ and ‘Z’ only. (d) 33 : 19 (e) 32 : 21
16. How many students appeared in at least two exams? 24. Average number of songs recorded by Sonu Nigam is
(a) 240 (b) 260 (c) 300 approximately what percent more/less than average
(d) 360 (e) 500 number of songs recorded in 2009?
17. How many students appeared in two exams only? (a) 104% (b) None of these (c) 124%
(a) 280 (b) 220 (c) 340 (d) 114% (e) 110%
(d) 300 (e) 260 25. Average number of songs recorded by all in all three
18. How many students appeared in at most two exams? years and is how much more than average number of
songs recorded by Sandeep and Arijit Singh in 2009?
(a) 240 (b) 260 (c) 300 (a) None of these
(d) 500 (e) 960 (b) 525
19. How many students not appeared in exam Y? (c) 650
(a) 440 (b) 360 (c) 540 (d) 550
(d) 640 (e) None of these (e) 450
193 Adda247 Publications For More Study Material
Visit: adda247.com
Cracker Book for Bank (IBPS | SBI | RRB PO | Clerk) Mains Exams

Directions (26-30): Study the following passage & Directions (31-35): Data regarding investment of three
answer the questions that follows. different persons in three different schemes is given
below. Study the data carefully and answer the following
There are three schools in a town named A, B & C. In questions.
each school students knows only Hindi, Only English and
both language. → Out of total amount invested by ‘Rahul’ in all the three
schemes, 25% is invested in scheme ‘X’. Remaining
Total number of student in school A is 1600. The no. amount is invested in scheme ‘Y’ and ‘Z’ equally.
of student who know only Hindi in school C is 40% of the
→ ‘Veer’s’ investment in scheme ‘X’ is 37.5% less than that
total students in school A. And total students who know of ‘Anurag’s’ in same scheme. Total amount invested by
only single language in school A are equal to student who ‘Rahul’, ‘Veer’ and ‘Anurag’ in all the three schemes is in
knows both language in same School and ratio of student the ratio of 4 : 5 : 6. Ratio between amount invested by
who know only Hindi to only English in school A is 2 : 3. ‘Veer’ in scheme ‘Y’ to in ‘Z’ is 3 : 2. Amount invested by
Student who know both language in all three schools is ‘Veer’ in scheme ‘Y’ is 80% more than that in scheme ‘X’.
50% more than student who knows only English in school
→ Amount invested by Anurag in scheme ‘Y’ and ‘Z’
B. Student who knows only Hindi in all schools is equal to
together is Rs. 20,000 more than the amount invested by
the total students in school A except student who knows
‘Rahul’ in both the same schemes together. Amount
only English. Ratio of students who know only English in invested by Anurag in scheme ‘Y’ is 200% more than that
school B to C is 1 : 4 and average of student who know in scheme ‘Z’.
only English in all the school is 460. Student who know
both language in school C is half of student who know 31. Rate of interest on scheme ‘X’, ‘Y’ and ‘Z” is 10%, 20%
3 and 30% p.a. at Simple interest. Find total interest
only Hindi in school A. Total student in school B is th of earned by Rahul after 2 years.
4
total student from all the school who knows only Hindi. (a) Rs. 44,000 (b) Rs. 41,000 (c) Rs. 34,000
(d) Rs. 30,000 (e) Rs. 24,000
26. Total student in school A who know only one
language is what percent more/less than total student 32. Veer and Anurag both invested in scheme ‘X’. After 8
in school B who knows both language? months Veer withdraw his total amount while Anurag
(a) 50% withdraw his total amount after 12 months. If total
(b) 60% profit of both is Rs. 7,650 then find the profit share of
(c) 70% Veer.
(a) Rs. 2,250 (b) Rs. 3,150 (c) Rs. 4050
(d) None of these
(d) Rs. 4950 (e) Rs. 6750
(e) 45%
33. Total amount invested in scheme ‘Y’ by all three
27. What is ratio of total student who knows only English together is what percent more than total amount
in school B and C together to total student in school invested by all three in scheme ‘Z’ all together?
B? (a) 50.25% (b) 56.75% (c) 62.75%
(a) 3 : 2 (b) 7 : 5 (c) None of these (d) 68.75% (e) 72.25%
(d) 5 : 3 (e) 15 : 14
34. Find the ratio between amount invested by Veer on
28. What is sum of average of student who knows only scheme ‘X’ to amount invested by Rahul in scheme ‘Z’?
3 (a) 4 : 3 (b) 1 : 1 (c) 2 : 3
English in all the school and 4th of student who knows
(d) 5 : 8 (e) 5 : 6
only single language in school B and C together.
(a) 1735 (b) 1335 (c) 1220 35. Scheme ‘Z’ offers 20% p.a. at Compound interest. Find
(d) 1525 (e) 1785 interest earned by Veer is what percent more than
interest earned by Anurag in that scheme?
29. 40% of total student in school B is what percent (a) 75% (b) 50% (c) 100%
more/less than half of student who know only Hindi (d) 150% (e) 200%
in school A ?
Direction (36-40): Data given below about five vessels
(a) 140% (b) 110% (c) 90%
M, N, O, P & Q contains mixture of different liquid read the
(d) None of these (e) 130%
data carefully and answer the questions.
30. What is difference of average of total student in all Vessel M contains mixture of milk and water, vessel N
three schools and average of student who knows both contains mixture of water and orange juice, vessel O
language in school B and C ? contains mixture of orange juice and Vodka, vessel P
(a) 920 (b) 840 (c) 890 contains mixture of Vodka and apple juice and vessel Q
(d) 990 (e) None of theses contains mixture of apple juice and Scotch.
194 Adda247 Publications For More Study Material
Visit: adda247.com
Cracker Book for Bank (IBPS | SBI | RRB PO | Clerk) Mains Exams
Mixture of milk and water in vessel M in the ratio of participated for national championship in 2005. Ratio of
7 : 2 and quantity of water in vessel N equal to quantity of total players participated in 2006 to 2004 is 15 : 13. Total
milk in vessel M. Mixture of orange juice and vodka into number of players participated for state championship in
the vessel O in the ratio of 5 : 3 and total quantity of all six years is 20000. Players participated for national
mixture in vessel O is 30 liter more than total quantity of championship in 2001 is same as players participated for
mixture in vessel N. state championship in 2005. Players participated for state
2 championship in 2002 is 800 more than that of players
Total quantity of mixture in vessel Q in 66 % of total
3 participated for same championship in 2004. Players
quantity of mixture in vessel M. Total quantity of mixture participated for National level championship in 2004 is
in vessel O is 50% more than total quantity of mixture in 1200 less than that of for same Championship in 2002.?
vessel P.
40. What is the respective ratio between players
Ratio of orange juice in vessel N and O is 7 : 15, while participated for state level championship in year
ratio of vodka in vessel O and P is 3 : 2. Total quantity of 2001, 2002 and 2004 together to the players
Scotch in vessel Q is 30 liter and total quantity of water in participated for national level championship in year
vessel M is 40 liter. Quantity of orange juice in vessel N is 2003, 2004 and 2005 together?
70 liters. (a) 1145:493 (b) 1154:441 (c) 1154:439
36. If 72 𝑙 of mixture from vessel M and 36 mixture taken (d) 439:1154 (e) 1105:439
out from vessel N and mixed in another vessel G, then
41. Find the difference between average number of
what will be ratio of milk, `water and orange juice in
players participated for state level championship and
vessel G?
average of players participated for national level
(a) 15 : 5 : 8 (b) 14 : 10 : 3 (c) 14 : 7 : 3
championship for starting four years?
(d) 15 : 8 : 3 (e) 14 : 10: 7
(a) 1900 (b) 1920 (c) 1820
37. 36 liter of mixture from vessel Q taken out and mixed (d) 1780 (e) 1940
in a vessel C, which contains 21 liter scotch, what
quantity of apple juice should mixed in vessel C to 42. Players participated for state level championship in
obtain ratio of apple juice and scotch 2 : 1: 2003 is how much more than players participated for
(a) 36 (b) 33 (c) 38 national level championship in 2002?
(d) 37 (e) 32 (a) 930 (b) 940 (c) 950
(d) 960 (e) 970
38. If vessel N and O mixed together in a big vessel Z and
22.5 liter of mixture taken out from vessel Z, find Direction (43-47): Given below statistical data of top five
quantity of Vodka in 22.5 liter of mixture? wheat producer states of India in the three successive
(a) 4.5 liter (b) 9 liter (c) 2.25 liter years. Read the data carefully and answer the question
(d) 1.5 liter (e) 3.6 liter given below.
39. If 64 liter of mixture from vessel O, 48 liter of mixture Year 2015– Uttar Pradesh and Punjab together
from P and 60 liter of mixture from vessel Q taken out produced 15860 thousand ton, while Punjab production
and mixed together in vessel A, find the ratio of was 1180 ton less than that of Uttar Pradesh. Madhya
orange juice, vodka, apple juice and scotch in vessel Pradesh produced 25% less than Uttar Pradesh, Haryana
A? produced 690 thousand ton less than that of Madhya
(a) 40 : 42 : 75 : 25 Pradesh. Production of Rajasthan was 70% of total
(b) 40 : 25 : 75 : 52 production of Haryana.
(c) 40 : 52 : 75 : 15
Year 2016– Production of Punjab decreased by 20%
(d) 30 : 52 : 75 : 15
compare to previous year, while production of Haryana
(e) 40 : 42 : 75 : 15
increased by 800 thousand ton compare to previous year.
Directions (40-42): Given below is the data about 1
Madhya Pradesh produced 33 3 % more than previous
players participated for state level and National level
year, while Rajasthan produced 2390 thousand ton more
championship in six different years i.e., 2001 to 2006.
wheat than previous year. All five states produced total
Total players participated for both Championship in 2006
36612 thousand ton wheat in 2016.
is 6000 while in 2003 is 4800. Total number of players
participated for national level championship in 2001 and Year 2017– Uttar Pradesh and Haryana together
2003 is 3600 and is in ratio of 11 : 7. Player participated produced 16520 thousand ton, while Uttar Pradesh
for state championship in 2005 is 25% more than player production was 2840 ton more than that of Haryana.
participated for national Championship in same year. Punjab produced 628 thousand ton more wheat than
Total players participated in 2006 is 20% more than total previous year and Madhya Pradesh produced 360
players participated in 2001. Players participated for thousand ton more wheat than previous year. All five
1 states produced total 38200 thousand ton wheat in 2017.
state Championship in 2006 is 62 2 % more than players

195 Adda247 Publications For More Study Material


Visit: adda247.com
Cracker Book for Bank (IBPS | SBI | RRB PO | Clerk) Mains Exams

43. Total wheat produced by Uttar Pradesh is how much 2013– Total number of Candidates selected from
more than total wheat produced by Madhya Pradesh 1
Uttar Pradesh is 33 3 % more than that of total number of
in the all three given years?
(a) 3550 thousand ton Candidates selected from same state in previous year,
(b) 3750 thousand ton while total number of Candidates selected from Kerala is
(c) 3650 thousand ton 30 less than average number of Candidates selected from
(d) 3250 thousand ton Kerala & Maharashtra in the year 2012. Total number of
(e) 3150 thousand ton Candidates selected from Maharashtra is 55 more than
44. Total wheat produced by Madhya Pradesh in the year that of total number of Candidates selected from same
2016 & 2017 together is what percent more than total state in previous year and total number of candidates
wheat produced by Haryana in the year 2015 & 2016 selected from Madhya Pradesh is equal to total number of
together? candidates selected from Kerala. Total number of
38 38 38
(a) 40 % (b) 44 % (c) 36 % Candidates selected from these five states in the year
61 61 61
38 38 2013 is 1130.
(d) 42 61 % (e) 48 61 %
2014– Total number of candidates selected from
45. Find the difference between average quantity of Uttar Pradesh and Maharashtra is 40 more and 25 less
wheat produced by Uttar Pradesh, Madhya Pradesh
than total number of candidates selected from Uttar
and Haryana in the year 2015 and average quantity of
wheat produced by Madhya Pradesh, Haryana and Pradesh and Maharashtra in previous year respectively.
Rajasthan in the year 2017? Average number of candidates selected from Uttar
(a) 360 thousand ton Pradesh, Kerala and Maharashtra is 280, while average
(b) 400 thousand ton number of candidates selected from Madhya Pradesh &
(c) 420 thousand ton Bihar is 195. Total number of selected candidates from
(d) 450 thousand ton Bihar is 20 more than that of Madhya Pradesh.
(e) 470 thousand ton
48. Total number of candidates selected from Kerala &
46. Find the ratio between total wheat produced by Maharashtra together in the year 2014 is what
Rajasthan & Punjab together in the year 2017 to total percent more/less than total number of candidates
wheat produced by Haryana & Rajasthan together in selected from Uttar Pradesh in the year 2012 & 2014
the year 2016?
together?
(a) 160 : 161 (b) 150 : 161 (c) 170 : 171
(d) 160 : 169 (e) 160 : 171 (a) 10% (b) 20% (c) 15%
(d) 25% (e) 5%
47. Find the total wheat produced by Rajasthan and Uttar
Pradesh in the year 2016 and 2017 together? 49. If total number of candidates selected from Karnataka
(a) 30700 thousand ton in the year 2014 is 50% more than total number of
(b) 30800 thousand ton candidates selected from Madhya Pradesh in the year
(c) 31700 thousand ton 2013, then find average number of candidates
(d) 32400 thousand ton selected from Karnataka, Uttar Pradesh, Madhya
(e) 33400 thousand ton Pradesh & Bihar in the year 2014?
Direction (48–51): Given below data is about the (a) 225 (b) 255 (c) 205
number of candidates who got final selection into (d) 215 (e) 200
UPSC exam from five different states (Uttar Pradesh, 50. Find the ratio between total number of candidates
Kerala, Maharashtra, Madhya Pradesh and Bihar) in selected from Kerala to total number of selected
three different successive years. Read the data candidates from Bihar in the given three years?
carefully and answer the questions: (a) 63 : 50 (b) 71 : 50 (c) 61 : 50
2012– Total number of Candidates selected from (d) 67 : 50 (e) 69 : 50
Uttar Pradesh are 20% more than total number of
51. Total number of candidates selected from
Candidates selected from Kerala and total number of
Maharashtra, Madhya Pradesh and Bihar together in
selected Candidates from these two states are 440.
the year 2013 is what percent more/less than total
Average number of Candidates selected from Uttar
Pradesh, Kerala and Maharashtra is 220 and total number number of candidates selected from Madhya Pradesh
of Candidates selected from Madhya Pradesh is 70 less in the year 2012 & 2013 together?
10 10 10
than that of from Maharashtra. Total number of (a) 92 % (b) 90 % (c) 94 %
11 11 11
Candidates selected from these five states in the year 10
(d) 91 11 %
10
(e) 95 11 %
2012 is 930.

196 Adda247 Publications For More Study Material


Visit: adda247.com
Cracker Book for Bank (IBPS | SBI | RRB PO | Clerk) Mains Exams

Directions (52-56): Neeraj have some toys which are in of 180 m length is twice than that time taken by same
the form of different structures. These are cylindrical, train to cross a pole. Train F can cross a platform double
conical, spherical. Other than solid conical structure, all of its length in 36 seconds. Speed of train ‘E’ is 20% more
two are of both types i.e., hollow as well as solid. than speed of train ‘C’. Train ‘A’ can cross a platform of
→ Volume of a conical toy is three times of the volume of 100 m length in same time in which train ‘F’ can cross a
pole.
a solid cylindrical toy while radius of a solid spherical toy
is half than that the radius of a conical toy. Outer radius of → Train ‘B’ can cross two poles 60 m apart from each
hollow cylindrical toys is same as radius of solid spherical other in 24 seconds. Train ‘B’ can cross train ‘C’ in 10
toy while average of outer radius and inner radius of seconds if they move towards each other. Time taken by
hollow cylindrical toys is equal to radius of solid train ‘D’ to cross a platform of 300 m length is same time
cylindrical toy. Height of cylindrical, conical and hollow in which train ‘C’ can cross a pole.
cylindrical toys is same i.e, 14c.m
Given that
→ Number of solid spherical toys is 20% of total number
of toys Neeraj have. Number of hollow spherical toys is a:b→5:3
150% more than number of conical toys. Ratio between d:f→4:3
𝑃𝐴 ∶ 𝑃𝑏 → 2 ∶ 5
number of solid cylindrical toys to number of conical toys
is 3 : 2. Total number of hollow cylindrical toys is 40% of 𝑙𝑑 ∶ 𝑙𝑒 → 1 ∶ 2
total number of toys Neeraj have and also ‘20’ more than 57. Train ‘C’ and Train ‘D’ enter in a tunnel from opposite
the total number of solid spherical toys Neeraj have. sides at same time. Find the length of tunnel if both
→ Volume of a hollow spherical toy is 33,957 cm² whose trains completely take exit from the tunnel at same
inner radius is half of its outer radius. Volume of a hollow time?
spherical toy is 5.25 time of volume of conical toy. (a) 900 m (b) 1200 m (c) 1500 m
52. Find the total space taken by all solid spherical toys? (d) 1800 m (e) 2100 m
(in cm³) 58. Find the time in which train ‘B’ can cross a platform of
(a) 97020 (b) 48510 (c) 72765 same as length as of train ‘F’?
(d) 14553 (e) 24255 (a) 50 seconds (b) 52 seconds (c) 54 seconds
(d) 56 seconds (e) 58 seconds
53. Find the number of conical toys Neeraj have?
(a) 40 (b) 20 (c) 15 59. Train ‘A’ starts from station Rewari and move
(d) 12 (e) 8 towards Gurgaon. After 4 hours Train ‘F’ starts from
station Rewari and move towards Gurgaon. After how
54. Find the curved surface area of one hollow cylindrical
toy? (in cm²) much time Train ‘F’ will overtake train ‘A’ if length of
(a) 616 (b) 1232 (c) 924 both trains is taken as negligible?
(a) 5 hours after start of Train ‘F’
(d) 462 (e) 1386
(b) 5 hours after start of train ‘A’
55. Find the ratio between outer radius of hollow (c) 9 hours after start of train ‘F’
spherical toy to radius of solid cylindrical toy? (d) 9 hours after start of train ‘A’
(a) 4 : 1 (b) 3 : 2 (c) 3 : 1 (e) Both (a) and (d)
(d) 4 : 3 (e) 2 : 1
60. Time taken by train ‘A’ to cross a platform of length
56. Volume of one hollow cylindrical toy is how much 250 m length is what percent less than time taken by
more then volume of one cylindrical toy?(in cm3 ) train ‘D’ to overtake train ‘F’ when they move in same
(a) 4312 (b) 3234 (c) 2696 direction?
(d) 2156 (e) 1078 (a) 37.5% (b) 62.5% (c) 50%
2 2
Direction (57-61): - Data about speed and length of six (d) 166 % (e) 266 %
3 3
different trains is given below. Study the data carefully
61. Train ‘C’ starts from Gurgaon station while train ‘D’
and answer the following question.
starts from Delhi station at same time. Both starts to
Speed of six trains A, B, C, D, E and F are a, b, c, d, e move towards each other. After how much time trains
and f respectively while time taken by six trains to cross a will completely cross each other if distance between
pole is 𝑃𝐴 , 𝑃𝑏 , 𝑃𝑐 , 𝑃𝑑 , 𝑃𝑒 and 𝑃𝑓 respectively. Length of six Delhi and Gurgaon is 30km?
trains A, B, C, D, E and F are 𝑙𝑎 , 𝑙𝑏 , 𝑙𝑐 , 𝑙𝑑 , 𝑙𝑒 and 𝑙𝑓 (a) 208 seconds
respectively (b) 218 seconds
(c) 223 seconds
→ Train C can cross a pole in double time in which train E (d) 228 seconds
can cross a pole. Time taken by train D to cross a platform (e) 238 seconds
197 Adda247 Publications For More Study Material
Visit: adda247.com
Cracker Book for Bank (IBPS | SBI | RRB PO | Clerk) Mains Exams
Directions (62-66) Satish, Abhi and Bhavya are three 62. Abhi and Bhavya both invested same amount they
persons who each invested some amount in three invested in scheme ‘Q’ in two different schemes i.e, S 1
different schemes (P, Q and R). Data tells about amount and S2 respectively. S1 and S2 offers 20% p.a at C.I and
invested and time of investment by them. Study the data 25% p.a at S.I respectively. Find the difference
carefully & answer the following questions. between interest earned by both after 2 years?
(a) 36,800 (b) 41,900 (c) 47,000
→ Amount invested by Satish in scheme ‘P’ is half than
(d) 52,100 (e) 57,200
that amount invested by Bhavya in same scheme. Abhi
invested Rs 80,000 in scheme ‘P’ and time for which he 63. If Bhavya earned Rs. 28,980 profits from scheme ‘R’
invested in scheme ‘P’ is 5 months more than time for which is 125% more than profit earned by him from
which Satish invested in same scheme. Ratio of amount scheme ‘P’, then find total profit earned by Satish
invested by Satish and Bhavya in Scheme ‘R’ is same as from scheme ‘P’ and ‘R’ together?
ratio of amount invested by Satish and Bhavya in scheme (a) 33,948 (b) 26,588 (c) 30,268
‘P’. Abhi invested 20,000 less in scheme ‘R’ than that of (d) 22,908 (e) 37,628
amount invested by Bhavya in scheme ‘R’.
64. Find interest earned by Satish if scheme ‘Q’ offers S.I.
→ Time of investment of Abhi and Bhavya is same in at the rate of 15% p.a?
scheme ‘Q’. Amount invested by Abhi in scheme ‘Q’ is 96% (a) Rs26500 (b) Rs29000 (c) Rs28000
of amount invested by Satish in scheme ‘Q’. Bhavya (d) Rs25000 (e) Rs27500
invested Rs 1,00,000 more than Abhi in scheme ‘Q’. Ratio
65. Amount invested by Bhavya in scheme ‘R’ is what
of amount invested by Satish in scheme ‘P’ to scheme ‘Q’ is
3 : 5. Ratio of amount invested by Bhavya in scheme ‘P’ to percent more than amount invested by Satish in
scheme ‘Q’ is 15 : 22. scheme ‘P’?
→ Out of total profit earned from scheme ‘Q’ Bhavya got (a) 140% (b) 100% (c) 144%
50%. Ratio of profit share of Satish and Abhi is 3 : 4 in (d) 50% (e) 20%
scheme ‘R’ while time of investment of Satish and Abhi is
scheme ‘R’ is 4 : 3. Bhavya invested for 10 month in 66. If Abhi doubles his investment in scheme ‘P’ than
scheme ‘R’. before, then find what percent of decrement is seen in
Satish got 25% of total profit both in scheme ‘R’ as well as Bhavya’s profit percentage?
4
in scheme ‘P’. Time of investment of Satish in scheme ‘P’ is (a) 20% (b) 40% (c) 28 %
7
2 months less that of in scheme ‘R’. Satish invested for 16 6 1
months in scheme ‘Q’ (d) 42 % (e) 57 %
7 7

Solutions
Solutions (1-5): Let investment of Rahul, Sandy and Sati 3. (c); Investment of Veer
be 2x, 3x and 4x respectively. 350
= 4 × 250 × 100 = 3500
Ratio of profit
Interest earned by Veer
Rahul : Sandy : Sati 20 2
= 3500 [1 + 100] − 3500 = 1540
2x × 4 : 3x × 4 : 4x × 10
+(2x – 800)×6 +(3x+1500)× 6 +(5x × 2) 4. (b); Required average
+(5x–800)× 2 (9x +1500) × 2 240+125 37600
30x – 6400 : 48x + 12000 : 50x = 2
× 376
=18,250
2x+3x+4x
ATQ, 5. (e); Investment of Bhavya = = 3x
50x 125 3
= ⇒ x = 250 = 3 × 250 = Rs.750
128x+5600 376
Ratio of profit share of Rahul, Sandy and Sati is Required profit = 750 - 2× 250
1100 ∶ 24000 ∶ 12500 → 11 ∶ 240 ∶ 125 = 750 – 500 = 250
240
1. (a); Required percentage = 376 × 100 Solutions (6–10): Total people = 2000
Let total people watching only ‘PK’ = a
= 63.829% ≃ 64%
Total number of people watching ‘Ravan’ = 720
2. (d); Required difference Ratio between number of people watching only ‘3 Idiiots’
and ‘Ravan’ to number of people watching only ‘Ravan’
240−11
= × 37600 = 22,900 and ‘PK’ = 2 : 3
376

198 Adda247 Publications For More Study Material


Visit: adda247.com
Cracker Book for Bank (IBPS | SBI | RRB PO | Clerk) Mains Exams
𝑎
Number of people watching ‘3 Idiot’ and ‘PK’ = 2 10. (b); Required difference
Number of people watching all three movies = 360 + 184 + 368 – 240 – 160 – 80 – 240
= 2000 ×
4
= 80 = 912 – 720 = 192
100
Solutions (11-15): Total students appeared in 2016
Number of people watching ‘Ravan’ only = number of
= 8000
people watching ‘Ravan’ and ‘PK’ = 3x
Total students appeared in 2013 = 5800
Number of people = watching ‘3 Idiots’ and ‘Ravan’ only =
2 Total students appeared in exam B is 2011 & 2013
× 3x = 2x
3 = 6200
Number of people watching ‘3 Idiots’ only Total students appeared in exam B in 2011
150 6200
= Number of people watching ‘Ravan’ only × 100 = × 18 = 3600
31
3
= 3𝑥 × = 4.5x Total students appeared in exam B in 2013
2 6200
= 31 × 13 = 2600
Total students appeared in exam A in 2013
= 5800 – 2600 = 3200
8000
Total students appeared 2011 = 125 × 100 = 6400
Total students appeared in exam A in 2011
= 6400 – 3600 = 2800
8000
Total students appeared in 2014 = 16
× 13= 6500
Students appeared in exam B in 2011
Now, 3x + 3x + 2x + 80 = 720 = Students appeared in exam A in 2015 = 3600
8x = 720 – 80 Students appeared in exam B in 2015
640
x= ⇒ x = 80 3600
= 4 × 3 = 2700
8

Now, Students appear in exam A in 2016


a 1700
12.5x + a + 2 + a = 2000 - 80 = [1 + ] × 2700 = 4400
2700
5a
= (1920 – 1000) ⇒ a = 368 Students appear in exam B in 2016 = 8000 – 4400
2
= 3600
Let, student appeared in exam A in 2014 = x
student appeared in exam A in 2012 = x + 700
⇒ x + x + 700 + 2800 + 3200 + 3600 + 4400
= 21,100
2x = 6400 ⇒ x = 3200
Students appeared in exam A in 2014 = 3200
Students appeared in exam A in 2012
= 3200 + 700 = 3900
6. (d); People watching only Ravan, only PK and only 3 Students appeared in exam B in 2014
Idiots = 6500 – 3200 = 3300
= 240 + 360 + 368 = 1000 Students appeared in exam B in 2012
Required percentage = 3300 + 1200 = 4500
968 −368
= 968 × 100 = 62% A B Total
240+368 2011 2800 3600 6400
7. (e); Required ratio = = 76 : 43
160+184 2012 3900 4500 8400
8. (d); Total number of males , who watching 3 idiots 2013 3200 2600 5800
and Ravan only 2014 3200 3300 6500
4 5
= 360 × 5 + 240 8 = 438 2015 3600 2700 6300
2016 4400 3600 8000
9. (a); Required percentage =
240 −160
× 100 Total 21,100 20,300
160
80
= 160 × 100 = 50% 11. (b); According to table its in 2014.

199 Adda247 Publications For More Study Material


Visit: adda247.com
Cracker Book for Bank (IBPS | SBI | RRB PO | Clerk) Mains Exams
2800 + 3200 + 3900 17. (e); Students appeared in two exams only
12. (d); Required ratio = 2600 + 3300 + 3600
9900 99 = 80 + 60 + 120 = 260
= =
9500 95
18. (e); Students appeared in atmost two exams
13. (b); Average students appeared in exam A in starting = 180 + 120 + 200 + 60 + 80 + 120 + 200
four years = 960
2800+3900+3200+3200
= = 3275 19. (d); Student not appeared in exam Y
4
Average students appeared in exam B in starting = 1000 – 360 = 640
four years 20. (d); Students appeared in exam X or in exam Z
3600+4500+2600+3300
= = 3500 = 180 + 60 + 40 + 80 + 200 + 120 = 680
4
Required difference = 225. Solutions (21-25):
14. (e); Required difference = 4500 – 3200 = 1300 No. of songs recorded by Sonu Nigam in 2008 = 300
8400 –6400
No. of songs recorded by Arijit Singh in 2009
15. (c); Required % = 6400
× 100 2
= × 300 = 120
2000 5
= × 100 = 31.25% Total number of songs recorded in 2008
6400
= 6 × 120 = 720
Solutions (16-20): Total students = 1000 Let number of songs recorded by Arijit Singh in 2008 be 𝑥.
Let, students appear in exam Z only = a ∴𝑥+
75
𝑥 = (720– 300)
Total students appeared in exam Y = 360 100
∴ 𝑥 = 240
Ratio of number of students appeared in exam X and Y
∴ No. of songs recorded by Arijit Singh and Sandeep in
only to students appeared in exam Y and Z only = 2 : 3 2008 are 240 & 180 respectively
Students appeared in exam X and Z both = a/2 Total no. of songs recorded in 2009
Number of students appeared in all three exams 350
4 = × 120 = 420
100
= 100 × 1000 = 40
No. of songs recorded by Sonu Nigam in 2009
Number of students appeared in Y exam only 2
= 5 × (420– 120) = 120
= No. of students appeared in Y and Z only = 3x
∴ No. of songs recorded by Sandeep in 2009 = 180
Number of students appeared in exam X and Y only
2 Total no. of songs recorded in 2010
= × 3x = 2x 4
3 = 3 × × 240 = 960
3
No. of songs recorded by Sandeep in 2010 = 650 – 180 –
180 = 290
No. of songs recorded by Arijit Singh in 2010
290
= × 4 = 232.
5
No. of songs recorded by Sonu Nigam in 2010
= 960 – 290 – 232 = 438

Years
Now, 2x + 3x + 3x + 40 = 360 ⇒ x = 40 2008 2009 2010
a
Singers
and, 12.5x + a + 2 + a = 1000
Sonu Nigam 300 120 438
5a
= 500 ⇒ a = 200
2 Arijit Singh 240 120 232
Sandeep 180 180 290
Total 720 420 960

21. (b); Required percentage


720–(240+120+232) 7
= 720
× 100 = 17 9 %

22. (a); Required difference


438+2321 180+180
=( )–( ) = 335 – 180 = 155
16 (c); Students appeared in atleast two exams 2 2
= 80 + 60 + 40 + 120 = 300

200 Adda247 Publications For More Study Material


Visit: adda247.com
Cracker Book for Bank (IBPS | SBI | RRB PO | Clerk) Mains Exams
960 1380 3
23. (e); Required ratio = 150 = 32 : 21 = 3
+ 4 [160 + 180 + 640 + 720]
420 ×
100
24. (a); Average number of songs recorded by = 460 + 1275 = 1735
40 1
Sonu Nigam ×840– ×320
300+120+438 858
29. (b); Required percentage = 100 1
2
× 100
×320
= = = 286 2
3 3 336–160
Average number of songs recorded in 2009. = × 100 = 110%
160
420
= 3 = 140 30. (d); Required difference
1600+840+1520 500+160
Required percentage =( )–( 2 )
3
286–140
= 140
× 100 ≃ 104% = 1320 – 330 = 990

25. (d); Required difference Solutions (31-35): Total amount invested by ‘Rahul’,
720+420+960 120+180 ‘Veer’ and ‘Anurag’ is in the ratio 4 : 5 : 6.
=( )–( ) Let total amount invested by Rahul, Veer and Anurag in all
3 2
= 700 – 150 = 550 the three schemes be 16x, 20x and 24x
Amount invested by Rahul in scheme ‘X”
Solutions (26-30): Total student in school A = 1600 25
Student who knows only Hindi in C = × 16𝑥 = 4𝑥
100
40 Amount invested by Rahul in scheme ‘Y’ or ‘Z’
= × 1600 = 640 16𝑥−4𝑥
100
= = 6𝑥
Let student who knows only Hindi & only English in 2
school A be 2𝑥 and 3𝑥 respectively. Let Amount invested by Veer in scheme Y and Z be 3y and
2y respectively
∴ Total student who knows both language in school A = 5𝑥
Then amount invested by Veer in scheme X is
ATQ, 3𝑦
= 180 × 100 = 3
5𝑦
10𝑥 = 1600 ⇒ 𝑥 = 160
∴ Student who knows only Hindi in A = 320 Ratio between amount invested by Veer in scheme ‘X’, ‘Y’
5𝑦
& only English in A = 480 and ‘Z’ = ∶ 3𝑦 ∶ 2𝑦 → 5 ∶ 9 ∶ 6
3
Both language in A = 800 Amount invested by Veer in scheme ‘X’, ‘Y’ and ‘Z’ is 5x, 9x
Student who knows only Hindi in B and 6x respectively.
= 800 + 320 – 640 – 320 = 160 Amount invested by Anurag in scheme ‘X’
Total student who knows Only English 5𝑥
= × 8 = 8𝑥
= 3 × 460 = 1380 5
Student who knows only English in B Amount invested by Anurag in scheme ‘Y’ and ‘Z’ together
1
= (1380– 480) = 180 = 24𝑥 − 8𝑥 = 16𝑥
5 Amount invested by Rahul in scheme ‘Y’ and ‘Z’ together =
Student who knows only English in C 12𝑥
= 900 – 180 = 720
320 ATQ,
Student who knows both language in C = 2 = 160
2 16𝑥 − 12𝑥 = 20,000 ⇒ 𝑥 = 5,000
Total student in school B = 1120 × = 840 Total amount invested by Anurag in scheme ‘Y’ and ‘Z’
4
∴ Student who knows both language in B together = 80,000
= 840 – 160 – 180 = 500 Let amount invested by Anurag in scheme ‘Z’ = ‘a’
Only Only Both Hindi Amount invested by Anurag in scheme ‘Y’ = 3𝑎
School Total ⇒ 𝑎 + 3𝑎 = 80,000 ⇒ 𝑎 = 20,000
Hindi English and English
A 320 480 800 1600 X Y Z
B 160 180 500 840 Rahul 20,000 30,000 30,000
C 640 720 160 1520 Veer 25,000 45,000 30,000
Total 1120 1380 1460 Anurag 40,000 60,000 20,000
(320+480)–500 31. (c); Required interest
26. (b); Required percentage = 500
× 100
20,000×10×2 30,000×20×2 30,000×30×2
300 = + +
= × 100 = 60% 100 100 100
500
= 4,000 + 12,000 + 18,000 = Rs. 34,000
180+720 900
27. (e); Required ratio = 840
= 840 = 15 : 14 32. (a); Ratio between profit share of Veer and Anurag
28. (a); Required sum Veer : Anurag
→ 25,000 × 8 ∶ 40,000 × 12 → 5 ∶ 12

201 Adda247 Publications For More Study Material


Visit: adda247.com
Cracker Book for Bank (IBPS | SBI | RRB PO | Clerk) Mains Exams

Profit share of Veer Vodka — — 90 60 —


5
= 17 × 7650 = Rs 2,250 Apple Juice — — — 100 90
Scotch — — — — 30
33. (d); Total amount invested in scheme ‘Y’
= 30,000 + 45,000 + 60,000 = 1,35,000 36. (b); In vessel G—
7
Total amount invested in scheme ‘Z’ Milk in 72 liter of mixture = 72 × = 56 liter
9
= 30,000 + 30,000 + 20,000 = 80,000 Water in 72 liter of mixture
1,35,000−80,000
Required % = × 100 2
= 72 × 9 = 16 liter
80,000
55,000 2
= × 100 = 68.75% Water in 36 liter of mixture =36 × =24 ltr.
80,000 3
25,000 5 Orange juice in 36 liter of mixture
34. (e); Required Ratio = = 1
30,000 6 = 36 × 3 = 12 liter
35. (b); Interest earned by Veer Ratio of milk : water : Orange juice in vessel G
20 2 = 56 : (16 + 24) : 12 = 14 : 10 : 3
= 30,000 × [1 + 100] − 30,000 = 13,200
3
Interest earned by Anurag 37. (b); Apple juice in vessel C = 36 × = 27 liter
4
20 2 Scotch in vessel C
= 20,000 × [1 + ] − 20,000 = 8,800
100 1
13,200−8,800 = 36 × + 21 = 9 + 21 = 30 liter
Required % = × 100 = 50% 4
8,800 Let 𝑥 liter of apple juice mixed
27+𝑥 2
Solutions (36 – 40): = =
30 1
Vessel M 𝑥 = 60 – 27 ⇒ 𝑥 = 33 liter
Water ⇒ 40 liters 38. (a); Ratio of water, Orange juice and Vodka vessel Z
40
So milk in M ⇒ × 7 = 140 liters = 140 : (70 + 150) : 90 = 14 : 22 : 9
2
Total quantity in vessel M = 180 liters Vodka in 22.5 liter of mixture
9 9
Total quantity quantity in vessel Q = 22.5 × (14+22+9) = 22.5 × 45 = 4.5 liter
180×2
= = 120 liters
3
Scotch in vessel Q = 30 liters 39. (e); Ratio of orange juice, Vodka, apple juice & scotch
Apple juice in vessel Q = 90 liters in vessel A
5 3 3 5
=[(64 × ) : (64 × + 48 × ) : (48 × +
In vessel N 3
8
1
8 8 8
Total quantity = 70 + 140 = 210 liter 60 × 4) ∶ 60 × 4]
Water = 140 liters = [ 40 : (24 + 18) : (30 + 45) : 15)
Orange juice = 70 liters = 40 : 42 : 75 : 15
In Vessel O Solutions (40-42): Total players participated in 2006
Total quantity = 210 + 40 = 240 liters = 6000
Orange juice in vessel O : Orange juice in vessel N Total players participated in 2003 = 4800
= 7 : 15 Total players participated for national level
70
Orange juice in vessel O = × 15 = 150 liters Championship in 2001 & 2003 = 3600
7
Vodoka in vessel = 240 – 150 = 90 liters Total players participated for National level
championship in 2001
In Vessel P 11
2
= 3600 × 18 = 2200
Total quantity = 66 3 % of total quantity of vessel P
Total players participated for national Level
2
= 3 × 240 = 160 championship in 2003
7
2 = 3600 × = 1400
Vodka = 3 of vodka in vessel O 18
2
Total player participated for state level championship in
90 × = 60 liters 2003 = 4800 – 1400 = 3400
3
Apple juice = 160 – 60 = 100 liters Total player participated in 2001
100
= 6000 × 120 = 5000
M N O P Q
Total player participated in 2001 for state level
Milk 140 — — — —
championship = 5000 – 2200 = 2800
Water 40 140 — — —
Total player participated in 2004
Orange juice — 70 150 — —

202 Adda247 Publications For More Study Material


Visit: adda247.com
Cracker Book for Bank (IBPS | SBI | RRB PO | Clerk) Mains Exams
6000 75
= 15 × 13 = 5200 = 8520 × 100 = 6390 thousands ton
Players participated for national championship in 2001 Production of wheat in Haryana
= Players participated for state championship in 2005 = 6390 – 690 = 5700 thousands ton
= 2200 Production of wheat in Rajasthan
Player participated for national championship in 2005 70
= 5700 × 100 = 3990 thousands ton
2200
= × 4 = 1760
5
Player participated for state level championship in 2006 2016 → Production of wheat in Punjab
80
5 13
= [1 + ] × 1760 = × 1760 = 220 × 13 = 2860 = 7340 × = 5872 thousands ton
100
8 8
Production of wheat in Haryana
Player participated for national championship in 2006 =
= 5700 + 800 = 6500 thousands ton
6000 – 2860 = 3140
Production of wheat in Madya Pradesh
Let players participated for state level championship in 4
2004 = X = 6390 × = 8520 thousands ton
3
Players participated for state level championship in 2002 Production of wheat in Rajasthan
= X + 800 = 3990 + 2390 = 6380 thousands ton
⇒ X + X + 800 + 3400 + 2800 + 2200 + 2860 =20000 Production of wheat in Uttar Pradesh
2X = 20000 – 12060 = 36612 – (5872 + 6500 + 8520 + 6380)
7940 = 9340 thousands ton
X = 2 ⇒ X = 3970
Player participated for state championship level in 2004 = 2017 → Production of wheat in Haryana
16520−2840
3970 = 2
= 6840 thousands ton
Player participated for state level championship in 2002 = Production of wheat in uttar Pradesh
3970 + 800 = 4770 = 6840 + 2840 = 9680 thousands ton
Players participated for national level championship in Production of Wheat in Punjab
2004 = 5200 – 3970 = 1230 = 5872 + 628 = 6500 thousands ton
Players participated for national level championship in Production of wheat in Madhya Pradesh
2002 = 1230 + 1200 = 2430 = 8520 + 360 = 8880 thousands ton
Production of wheat in Rajasthan
state level National Level = 38200 – (6840 – 9680 + 6500 + 8880)
Years Total
championship Championship = 6300 thousands ton
2001 2800 2200 5000 Production of wheat
2002 4770 2430 7200 States (thousands ton)
2015 2016 2017
2003 3400 1400 4800 Uttar Pradesh 8520 9340 9680
2004 3970 1230 5200 Punjab 7340 5872 6500
Madhya Pradesh 6390 8520 8880
2005 2200 1760 3960
Haryana 5700 6500 6840
2006 2860 3140 6000 Rajasthan 3990 6380 6300

40. (c); Required ratio 43. (b); Total wheat produced by Uttar Pradesh in all
2800+4770+3970 11540 three years
= 1400+1230+1760 = 4390 = 1154 : 439
= 8520 + 9340 + 9680
41. (b); Required difference = 27540 thousands ton
2800+4770+3400+3970 2200+2430+1400+1230 Total wheat produced by Madhya Pradesh in all
=( )–( )
4 4 three years
= 3735 – 1815 = 1920 = 6390 + 8520 + 8880
42. (e); Required difference = 3400 – 2430 = 970 = 23790 thousands ton
Required difference = 27540 – 23790
Solutions (43-47): = 3750 thousands ton
2015 → Production of wheat in Punjab 44. (d); Total wheat produced by Madhya Pradesh in
15860−1180
= = 7340 thousands ton year 2016 & 2017 together
2
Production of wheat in Uttar Pradesh = 8520 + 8880 = 17400 thousands ton
= 734 + 1180 = 8520 thousands ton Total wheat produced by Haryana in the year
Production of wheat in Madhya Pradesh 2015 & 2016 together = 5700 + 6500

203 Adda247 Publications For More Study Material


Visit: adda247.com
Cracker Book for Bank (IBPS | SBI | RRB PO | Clerk) Mains Exams

= 12200 thousands ton Total number of candidates selected from Uttar Pradesh =
Required% =
17400 –12200
× 100 320 + 40 = 360
12200 Total number of candidate selected from Maharashtra =
5200 38
= 12200 × 100 = 42 61 % 275 – 25 = 250
45. (e); Average quantity of wheat produced by Uttar Total number of candidate selected from Kerala
Pradesh, Madhya Pradesh and Haryana in the = 280 × 3 – (360 + 250) = 840 – 610 = 230
year 2015 Total number of candidate selected from Madhya Pradesh
8520+6390+5700 20610 and Bihar = 195 × 2 = 390
= 3
= 3 Total number of candidate selected from Madhya Pradesh
= 6870 thousands ton 390–20
= 2 = 185
Average quantity of wheat produced by Madhya
Pradesh, Haryana and Rajasthan in the year Total number of candidate selected from Bihar
2017 = 185 + 20 = 205
6840+6300+8880 22020
= 3
= 3
= 7340 thousands ton
Required difference = 7340 – 6870
= 470 thousands ton
6300+6500 12800
46. (a); Required ratio = =
6500+6380 12880
= 160 : 161
48. (b); Total number of candidates selected from Kerala
47. (c); Total Wheat produced by Rajasthan and Uttar & Maharashtra together in 2014
Pradesh in the year 2016 & 2017 together = = (250 + 230) = 480
(9340 + 6380) + (9680 + 6300) Total number of candidates selected from Uttar
= 31700 thousands ton Pradesh in 2012 & 2014 together
Solutions (48-51): = 240 + 360 = 600
600 –480
Required percentage = ×100 =20%
2012 → 600
Let total number of candidates selected from Kerala be 5𝑥 49. (b); Total number of candidate selected from
and from Uttar Pradesh be 6𝑥 Karnataka in the year 2014
150
ATQ— = 180 × 100 = 270
5𝑥 + 6𝑥 = 440 ⇒ 𝑥 = 40 Required average =
270+360+185+205
Total number of selected candidate from Uttar Pradesh = 1020
4

240 = = 255
4
Total number of selected candidate from Kerala = 200 (200+180+230)
Total number of candidate selected from Maharashtra 50. (c); Required ratio = (120+175+205)
= 220 × 3 – (240 + 200) = 660 – 440 = 220 610
= 500 = 61 : 50
Total number of candidates selected from Madhya
Pradesh = 220 – 70 = 150 51. (b); Total number of candidates selected from
Total number of candidates selected from Bihar Maharashtra, Madhya Pradesh and Bihar
= 930 – (240 + 200 + 220 + 150) = 930 – 810 = 120 together in 2013 = (275 + 180 + 175) =630
2013 → Total number of candidates selected from
Total number of candidates selected from Uttar Pradesh Madhya Pradesh in the year 2012 & 2013
4 together = (150 + 180) = 330
= 240 × 3 = 320 630 –330
Required percentage = 330 × 100
Total number of candidates selected from Kerala 300 1000 10
=
(200+220)
– 30 = 210 – 30 = 180 = 330 × 100 = 11
= 90 11 %
2
Total number of candidates selected from Maharashtra = Solution (52-56): Volume of hollow spherical toy
220 + 55 = 275 = 33,957𝑐𝑚2
Total number of candidates selected from Madhya Let Outer radius of hollow spherical toy = R
Pradesh = 180 𝑅
Inner radius of hollow spherical toy = 2
Total number of candidates selected from Bihar
= 1130 – (320 + 180 + 275 + 180) ATQ,
= 1130 – (955) = 175 4 𝑅 3
3
𝜋 (𝑅3 − ( 2 ) ) = 33,957
2014 →
⇒ R3 = 9261 ⇒ R = 21
204 Adda247 Publications For More Study Material
Visit: adda247.com
Cracker Book for Bank (IBPS | SBI | RRB PO | Clerk) Mains Exams

Outer radius of hollow spherical toy = 21cm Solution (57-61): Time taken by train ‘D’ to cross a
Inner radius of hollow spherical toy = 10.5 cm platform of 180 m length is twice than that time taken by
Volume of conical toy same train to cross a pole.
33957 14 ℓ +180 2ℓ
= 5.25 = 6468 = 𝜋(𝑟𝑎𝑑𝑖𝑢𝑠 𝑜𝑓 𝑐𝑜𝑛𝑒)2 × 3 ⇒ d = d ⇒ ℓd = 180m
d d
⇒ Radius of cone = 21 cm But, ℓd ∶ ℓe = 1 ∶ 2 ⇒ ℓe = 360m
Volume of solid cylindrical toy Train ‘F’ can cross a platform double of its length in 36
6468
= = 2156 seconds
3
= π (radius of cylinder)² × (height of cylinder) ⇒
ℓf +2ℓf
= 36 ⇒
ℓf
= 12 seconds
⇒ Radius of Cylinder = 7cm f f
21
Radius of Solid Spherical toy = 2 = 10.5𝑐𝑚 ⇒ Train ‘F’ can cross a pole in 12 seconds
Train ‘A’ can cross a platform of 100m length in same time
Outer Radius of hollow cylindrical toy = 10.5𝑐𝑚
Inner radius of hollow cylindrical toy in which train ‘F’ can cross a pole i.e, 12 seconds
ℓA +100
= 7 × 2 − 10.5 = 3.5 cm ⇒ = 12
a
Train ‘B’ can cross two poles 60 m apart from each other
in 24 seconds
ℓB +60
⇒ 24 = b
But a : b → 5 : 3
Let total number of toys = 100x And, PA : PB → 2 ∶ 5
ATQ, ℓa
2 ℓa b ℓa 2 2
Number of solid spherical toys = 20x a
ℓb ⇒ = × ⇒ = ⇒ ℓa = ℓb
5 a ℓb ℓb 3 3
Number of hollow cylindrical toys = 40x b

ATQ, And,
2(ℓA +100) (ℓb +60) 5 3ℓa
40𝑥 − 20𝑥 = 20 ⇒ 𝑥 = 1 = ⇒ 2(ℓa + 100) = ( + 60)
a b 3 2
Let Number pf conical toys = 2𝑦 15 3
6ℓa + 600 = ℓa + 300 ⇒ 300 = ℓa
Number of hollow spherical toys = 5𝑦 2 2
Number of solid cylindrical toys = 3𝑦 ⇒ ℓa = 200m

ATQ, And, ℓb = 300m


200+100 300
2𝑦 + 5𝑦 + 3𝑦 = 100 − 40 − 20 = 40 ⇒ 𝑦 = 4 a= = = 25 m/sec
12 12
25
Toy Number of Toys b = × 3 = 15 m/sec
5
Conical 8 Train ‘C’ can cross a pole in double time in which train ‘E’
Solid Cylindrical 12 can cross a pole.
Solid Spherical 20 ℓ
⇒ C= e
2ℓ
c e
Hollow Cylindrical 40
But,
Hollow Spherical 20
e = 1.2c
52. (a); Space taken by one solid spherical toy ℓ 2ℓ 5
⇒ c = e ⇒ ℓc = ℓe
c 1.2c 3
= Volume of one solid spherical toy
4 But ℓe = 360 m
= 3 𝜋(10.5)3 = 4851𝑐𝑚3
⇒ ℓc = 600m
Total space taken by solid spherical toys
Train ‘B’ can cross train ‘C’ in 10 seconds if they move in
= 20 × 4851 = 97020𝑐𝑚2
opposite direction
53. (e); Number of conical toys Neeraj have = 8 ℓ +ℓ 300+600
⇒ b10 c = b + c ⇒ 10 = 15 + c ⇒ 90 – 15 = C
54. (b); Curved surface area of one hollow cylindrical toy ⇒ c = 75 m/sec
= 2𝜋 × (3.5 + 10.5) × 14 = 1232 cm2 Time train by train ‘D’ to cross a platform of 300m is same
21 3 time in which train ‘C’ can cross a pole.
55. (c); Required Ratio = 7
=1 ℓ +300 ℓ 180+300 600
⇒ d = c⇒ =
d c d 75
56. (d); Volume of one hollow cylindrical toy 480
⇒d= = 60 m/sec
= 𝜋 × 14 × (10.52 − 3.52 ) = 4312 cm3 8
Required difference d:f→4:3
60
= 4312 − 2156 = 2156𝑐𝑚3 ⇒ f = 4 × 3 = 45 m/sec

205 Adda247 Publications For More Study Material


Visit: adda247.com
Cracker Book for Bank (IBPS | SBI | RRB PO | Clerk) Mains Exams
ℓf In scheme R,
f
= 12 ⇒ ℓf = 540 m
Bhavya invested for 10 months while Satish got 25% of
But profit out of total profit
e = 1.2c ⇒ e = 1.2 × 75 = 90 m/sec If ratio between Satish’s and Abhi’s profit share is 3 : 4,
3
Train Speed (in m/sec) Length (in m) then Bhavya’s profit share is 25 × 100 − 7 = 5
A 25 200 ⇒ Ratio of profit share of Satish, Abhi and Bhavya is
B 15 300 3 : 4 : 5.
C 75 600 Let Satish invested for ‘y’ month
D 60 180 1,80,000×10 5
= 3 ⇒ y = 12 month
E 90 360 90,000×𝑦
12
F 45 540 Abhi invested for = × 3 = 9 month
4
57. (c); Let length of tunnel is ‘x’ meter Time of investment of Satish in scheme ‘P’ is 2 months
Time taken by train ‘C’ to cross tunnel is same as less than that of in scheme ‘Q’
time taken by train ‘D’ to cross tunnel ⇒ Satish invested for 10 months in scheme ‘P’.
180+𝑥 600+𝑥
60
= 75 ⇒ 900 + 5𝑥 = 2400 + 4𝑥 Time of investment of Abi in scheme ‘P’ is 5 months more
𝑥 = 1500 𝑚 than time for Satish invested
300+540 840 ⇒ Abhi invested for 15 months in scheme ‘P’
58. (d); Required time = =
15 15 Let amount invested by Satish in scheme ‘Q’ is = 5b
= 56 seconds ⇒ Amount invested by Satish in scheme ‘P’ is
5𝑏
59. (e); Speed of train ‘A’ = 25m/sec = 90km /hr = × 3 = 3𝑏
5
Distance covered by train ‘A’ in 4 hours
And, Amount invested by Abhi in scheme ‘Q’ is
= 90 × 4 = 360 km 96
Let Train ‘F’ will overtake train ‘A’ in ‘x’ time = 5𝑏 × = 4.8𝑏
100
360×5
Then, 𝑥 = (45−25)×18 =5 hours Amount invested by Bhavya in scheme ‘Q’ is
Train ‘F’ meet train ‘A’ after 5 hours start of train = 4.8b + 1,00,000
‘F’ or after 9 hours start of train ‘A’ Amount invested by Bhavya in scheme ‘P’ is
= 3b × 2 = 6b
60. (b); Time taken by train ‘A’ to cross a platform of
200+250
250m length = 25 = 18 ATQ,
6𝑏 15
Time taken by train ‘D’ to overtake train ‘F’ = ⇒ 132b = 72b + 15,00, 000
4.8𝑏 + 1,00,000 22
180+540
= 60−45 = 48sec b= 25,000
48−18
Required % = × 100 = 62.5% Amount Invested
48
30,000+600+180 Scheme Satish Abhi Bhavya
61. (d); Required time = 75+60 P 75,000 80,000 1,50,000
=228 seconds Q 1,25,000 1,20,000 2,20,000
R 90,000 1,60,000 1,80,000
Solutions (62-66): Ratio of amount invested by Satish
and Bhavya in scheme ‘P’ and in ‘R’ is same that is 1 : 2 Let, Bhavya invested for ‘y’ month in scheme ‘P’.
Let Bhavya and Satish invested 2z and z in scheme ‘R’
ATQ,
⇒ Amount invested by Abhi in scheme ‘R’ is (2z – 20,000) 1 75,000×10
Ratio of profit share of Satish and Abhi in scheme ‘R’ is 3 : 4
= 75,000×10+80,000×15+1,50,000×𝑦 ⇒ y = 7 months
4 while time of investment of Satish and Abhi in scheme Time of investment of Abhi and Bhavya is same in scheme
‘R’ is 4 : 3. ‘Q’ and ratio between profit sharing of Bhavya to total
3 𝑧×4
⇒ = ⇒ 18z – 1,80,000 = 16z profit in scheme ‘Q’ is 1 : 2.
4 (2𝑧−20,000)×3
z = 90,000 Let, Abhi and Bhavya invested for ‘5a’ months and Satish
invested for ‘x’ month.
Amount Amount Amount
Scheme invested invested invested by ATQ,
by Satish by Abhi Bhavya 1 2,20,000×5𝑎
= 1,25,000×16+1,20,000×5𝑎
1
R 90,000 1,60,000 1,80,000

206 Adda247 Publications For More Study Material


Visit: adda247.com
Cracker Book for Bank (IBPS | SBI | RRB PO | Clerk) Mains Exams
12,880
⇒ 1,25,000 × 16 = 1,00,000 × 5a ⇒ 5a= 20 months = × 5 = 9,200
7
So Bhavya and Veer invested in scheme ‘Q’ for 20 months
Total profit earned by Satish from scheme ‘P’ and
‘R’ together
= 17,388 + 9,200 = 26,588
64. (d); Time of investment of Satish
20
= × 4 = 16 months
5
Interest earned by Satish
15 16
= 1,25,000 × × = 25,000
100 12
1,80,000−75,000
62. (e); Interest earned by Abhi 65. (a); Required % = 75,000
× 100
120 120 105
= 1,20,000 × × − 1,20,000 = 52,800 = 75
× 100 = 140%
100 100
Interest earned by Bhavya 66 (c); Ratio of profit sharing between Satihs, Abhi and
25
= 2,20,000 × 2 × = 1,10,000 Bhavya in scheme ‘P’ is 5 : 8 : 7
100
Required difference = 1,10,000 –52,800 Let total profit = 20
7
=57,200 Bhavya’s profit = 20 × 100
63. (b); Ratio of profit sharing between Satish, Abhi and = 35%
Bhavya in scheme ‘P’ and scheme ‘R’ is 5 : 8 : 7 When Abhi doubles his investment then ratio of
profit sharing between Satish, Abhi and Bhavya
and 3 : 4 : 5 respectively.
in scheme ‘P’ is 5 : 16 : 7
Profit earned by Satish from scheme ‘R’
28,980 Let total profit = 28
= × 3 = 17,388 7
5 Bhavya’s profit = 28 × 100 = 25%
Profit earned by Bhavya from scheme ‘P’ 35−25
28980 % Decrement in profit % ×100
= 225
× 100 = 12,880 4
35
=28 %
Profit earned by Satish from scheme ‘P’ 7

207 Adda247 Publications For More Study Material


Visit: adda247.com

You might also like